Условия равновСсия систСмы сил: Условия равновСсия ΠΏΡ€ΠΎΠΈΠ·Π²ΠΎΠ»ΡŒΠ½ΠΎΠΉ систСмы сил

Π‘ΠΎΠ΄Π΅Ρ€ΠΆΠ°Π½ΠΈΠ΅

Условия равновСсия ΠΏΡ€ΠΎΠΈΠ·Π²ΠΎΠ»ΡŒΠ½ΠΎΠΉ систСмы сил

Рассмотрим условия равновСсия ΠΏΡ€ΠΎΠΈΠ·Π²ΠΎΠ»ΡŒΠ½ΠΎΠΉ плоской ΠΈ пространствСнной систСм сил, Π²ΠΊΠ»ΡŽΡ‡Π°Ρ Ρ‚Ρ€ΠΈ основныС Ρ„ΠΎΡ€ΠΌΡ‹ ΠΈ частныС случаи равновСсия для систСм ΠΏΠ°Ρ€Π°Π»Π»Π΅Π»ΡŒΠ½Ρ‹Ρ… ΠΈ сходящихся сил:

Из основной Ρ‚Π΅ΠΎΡ€Π΅ΠΌΡ‹ статики слСдуСт, Ρ‡Ρ‚ΠΎ любая систСма сил ΠΈ ΠΌΠΎΠΌΠ΅Π½Ρ‚ΠΎΠ², Π΄Π΅ΠΉΡΡ‚Π²ΡƒΡŽΡ‰ΠΈΡ… Π½Π° Ρ‚Π²Π΅Ρ€Π΄ΠΎΠ΅ Ρ‚Π΅Π»ΠΎ, ΠΌΠΎΠΆΠ΅Ρ‚ Π±Ρ‹Ρ‚ΡŒ ΠΏΡ€ΠΈΠ²Π΅Π΄Π΅Π½Π° ΠΊ Π²Ρ‹Π±Ρ€Π°Π½Π½ΠΎΠΌΡƒ Ρ†Π΅Π½Ρ‚Ρ€Ρƒ ΠΈ Π·Π°ΠΌΠ΅Π½Π΅Π½Π° Π² ΠΎΠ±Ρ‰Π΅ΠΌ случаС Π³Π»Π°Π²Π½Ρ‹ΠΌ Π²Π΅ΠΊΡ‚ΠΎΡ€ΠΎΠΌ ΠΈ Π³Π»Π°Π²Π½Ρ‹ΠΌ ΠΌΠΎΠΌΠ΅Π½Ρ‚ΠΎΠΌ.

Если систСма ΡƒΡ€Π°Π²Π½ΠΎΠ²Π΅ΡˆΠ΅Π½Π°, Ρ‚ΠΎ ΠΏΠΎΠ»ΡƒΡ‡Π°Π΅ΠΌ условия равновСсия: R=0, MO=0. Из этих условий для пространствСнной систСмы сил получаСтся ΡˆΠ΅ΡΡ‚ΡŒ ΡƒΡ€Π°Π²Π½Π΅Π½ΠΈΠΉ равновСсия, ΠΈΠ· ΠΊΠΎΡ‚ΠΎΡ€Ρ‹Ρ… ΠΌΠΎΠ³ΡƒΡ‚ Π±Ρ‹Ρ‚ΡŒ ΠΎΠΏΡ€Π΅Π΄Π΅Π»Π΅Π½Ρ‹ ΡˆΠ΅ΡΡ‚ΡŒ нСизвСстных:

βˆ‘xiΒ =0, Β  Β  Β βˆ‘Mix=0;
βˆ‘yiΒ =0, Β  Β  Β βˆ‘Miy=0; Β Β  Β (1.20)
βˆ‘ziΒ =0, Β  Β  Β βˆ‘Miz=0
.

Π€ΠΎΡ€ΠΌΡ‹ условий равновСсия

ΠŸΠ΅Ρ€Π²Π°Ρ Ρ„ΠΎΡ€ΠΌΠ°

Для плоской систСмы сил (Π½Π°ΠΏΡ€ΠΈΠΌΠ΅Ρ€, Π² плоскости Oxy) ΠΈΠ· этих ΡƒΡ€Π°Π²Π½Π΅Π½ΠΈΠΉ ΠΏΠΎΠ»ΡƒΡ‡Π°ΡŽΡ‚ΡΡ Ρ‚ΠΎΠ»ΡŒΠΊΠΎ Ρ‚Ρ€ΠΈ:

βˆ‘xi=0;
βˆ‘yi=0;Β Β  Β (1.21)
βˆ‘MO=0
,

ΠΏΡ€ΠΈΡ‡Π΅ΠΌ оси ΠΈ Ρ‚ΠΎΡ‡ΠΊΠ° O, ΠΎΡ‚Π½ΠΎΡΠΈΡ‚Π΅Π»ΡŒΠ½ΠΎ ΠΊΠΎΡ‚ΠΎΡ€ΠΎΠΉ ΠΏΠΈΡˆΠ΅Ρ‚ΡΡ ΡƒΡ€Π°Π²Π½Π΅Π½ΠΈΠ΅ ΠΌΠΎΠΌΠ΅Π½Ρ‚ΠΎΠ², Π²Ρ‹Π±ΠΈΡ€Π°ΡŽΡ‚ΡΡ ΠΏΡ€ΠΎΠΈΠ·Π²ΠΎΠ»ΡŒΠ½ΠΎ. Π­Ρ‚ΠΎ пСрвая Ρ„ΠΎΡ€ΠΌΠ° ΡƒΡ€Π°Π²Π½Π΅Π½ΠΈΠΉ равновСсия.

Вторая Ρ„ΠΎΡ€ΠΌΠ°

УравнСния равновСсия ΠΌΠΎΠ³ΡƒΡ‚ Π±Ρ‹Ρ‚ΡŒ записаны ΠΈΠ½Π°Ρ‡Π΅:

βˆ‘xiΒ =0;
βˆ‘MA=0;Β Β  Β (1.22)
βˆ‘MB=0
.

Π­Ρ‚ΠΎ вторая Ρ„ΠΎΡ€ΠΌΠ° ΡƒΡ€Π°Π²Π½Π΅Π½ΠΈΠΉ равновСсия, ΠΏΡ€ΠΈΡ‡Π΅ΠΌ ось Ox Π½Π΅ Π΄ΠΎΠ»ΠΆΠ½Π° Π±Ρ‹Ρ‚ΡŒ пСрпСндикулярна Π»ΠΈΠ½ΠΈΠΈ, проходящСй Ρ‡Π΅Ρ€Π΅Π· Ρ‚ΠΎΡ‡ΠΊΠΈ A ΠΈ B.

Π’Ρ€Π΅Ρ‚ΡŒΡ Ρ„ΠΎΡ€ΠΌΠ°

βˆ‘MA=0;
βˆ‘MB=0;Β Β  Β (1.23)
βˆ‘MC=0
.

Π­Ρ‚ΠΎ Ρ‚Ρ€Π΅Ρ‚ΡŒΡ Ρ„ΠΎΡ€ΠΌΠ° ΡƒΡ€Π°Π²Π½Π΅Π½ΠΈΠΉ равновСсия, ΠΏΡ€ΠΈΡ‡Π΅ΠΌ Ρ‚ΠΎΡ‡ΠΊΠΈ A, B ΠΈ C Π½Π΅ Π΄ΠΎΠ»ΠΆΠ½Ρ‹ Π»Π΅ΠΆΠ°Ρ‚ΡŒ Π½Π° ΠΎΠ΄Π½ΠΎΠΉ прямой.Β 

ΠŸΡ€Π΅Π΄ΠΏΠΎΡ‡Ρ‚ΠΈΡ‚Π΅Π»ΡŒΠ½ΠΎΡΡ‚ΡŒ написания Ρ„ΠΎΡ€ΠΌ ΡƒΡ€Π°Π²Π½Π΅Π½ΠΈΠΉ равновСсия зависит ΠΎΡ‚ ΠΊΠΎΠ½ΠΊΡ€Π΅Ρ‚Π½Ρ‹Ρ… условий Π·Π°Π΄Π°Ρ‡ΠΈ ΠΈ Π½Π°Π²Ρ‹ΠΊΠΎΠ² Ρ€Π΅ΡˆΠ°ΡŽΡ‰Π΅Π³ΠΎ.

Π”Ρ€ΡƒΠ³ΠΈΠ΅ условия равновСсия

ΠŸΡ€ΠΈ дСйствии Π½Π° Ρ‚Π΅Π»ΠΎ плоской систСмы ΠΏΠ°Ρ€Π°Π»Π»Π΅Π»ΡŒΠ½Ρ‹Ρ… сил ΠΎΠ΄Π½ΠΎ ΠΈΠ· ΡƒΡ€Π°Π²Π½Π΅Π½ΠΈΠΉ исчСзаСт ΠΈ ΠΎΡΡ‚Π°ΡŽΡ‚ΡΡ Π΄Π²Π° уравнСния (рисунок 1.26, Π°):

βˆ‘xiΒ =0;
βˆ‘MO=0. (1.24)



Рисунок 1.26

Для пространствСнной систСмы ΠΏΠ°Ρ€Π°Π»Π»Π΅Π»ΡŒΠ½Ρ‹Ρ… сил (рисунок 1.26, Π±) ΠΌΠΎΠ³ΡƒΡ‚ Π±Ρ‹Ρ‚ΡŒ записаны Ρ‚Ρ€ΠΈ уравнСния равновСсия:

βˆ‘ziΒ =0;
βˆ‘Mix=0; (1.25)
βˆ‘Miy=0
.

Для систСмы сходящихся сил (Π»ΠΈΠ½ΠΈΠΈ дСйствия ΠΊΠΎΡ‚ΠΎΡ€Ρ‹Ρ… ΠΏΠ΅Ρ€Π΅ΡΠ΅ΠΊΠ°ΡŽΡ‚ΡΡ Π² ΠΎΠ΄Π½ΠΎΠΉ Ρ‚ΠΎΡ‡ΠΊΠ΅) ΠΌΠΎΠΆΠ½ΠΎ Π½Π°ΠΏΠΈΡΠ°Ρ‚ΡŒ Ρ‚Ρ€ΠΈ уравнСния для пространствСнной систСмы:

βˆ‘xiΒ =0;
βˆ‘yiΒ =0; (1.26)
βˆ‘ziΒ =0

ΠΈ Π΄Π²Π° уравнСния для плоской систСмы:

βˆ‘xiΒ =0;
βˆ‘yiΒ =0. Β  Β  Β Β  (1.27)

Π’ ΠΊΠ°ΠΆΠ΄ΠΎΠΌ ΠΈΠ· Π²Ρ‹ΡˆΠ΅ΠΏΡ€ΠΈΠ²Π΅Π΄Π΅Π½Π½Ρ‹Ρ… случаСв число нСизвСстных, Π½Π°Ρ…ΠΎΠ΄ΠΈΠΌΡ‹Ρ… ΠΏΡ€ΠΈ Ρ€Π΅ΡˆΠ΅Π½ΠΈΠΈ ΡƒΡ€Π°Π²Π½Π΅Π½ΠΈΠΉ, соотвСтствуСт числу записанных ΡƒΡ€Π°Π²Π½Π΅Π½ΠΈΠΉ равновСсия.

>> РавновСсиС систСмы сходящихся сил

Условия равновСсия Ρ‚Π²Π΅Ρ€Π΄ΠΎΠ³ΠΎ Ρ‚Π΅Π»Π° ΠΈ систСмы сил

Π’Π΅Ρ€ΠΌΠΈΠ½Ρ‹ «Ρ€Π°Π²Π½ΠΎΠ²Π΅ΡΠΈΠ΅ Ρ‚Π΅Π»Π°» ΠΈ «Ρ€Π°Π²Π½ΠΎΠ²Π΅ΡΠΈΠ΅ систСмы сил»

Π—Π΄Π΅ΡΡŒ ΠΌΡ‹ рассматриваСм условия, ΠΏΡ€ΠΈ ΠΊΠΎΡ‚ΠΎΡ€Ρ‹Ρ… Ρ‚Π²Π΅Ρ€Π΄ΠΎΠ΅ Ρ‚Π΅Π»ΠΎ находится Π² состоянии равновСсия. Под этим ΠΌΡ‹ ΠΏΠΎΠ΄Ρ€Π°Π·ΡƒΠΌΠ΅Π²Π°Π΅ΠΌ, Ρ‡Ρ‚ΠΎ Ссли Ρ‚Π΅Π»ΠΎ Π² Π½Π΅ΠΊΠΎΡ‚ΠΎΡ€Ρ‹ΠΉ ΠΌΠΎΠΌΠ΅Π½Ρ‚ Π²Ρ€Π΅ΠΌΠ΅Π½ΠΈ покоилось, Ρ‚ΠΎ ΠΎΠ½ΠΎ Π±ΡƒΠ΄Π΅Ρ‚ покоится ΠΈ Π² ΠΏΠΎΡΠ»Π΅Π΄ΡƒΡŽΡ‰ΠΈΠ΅ ΠΌΠΎΠΌΠ΅Π½Ρ‚Ρ‹ Π²Ρ€Π΅ΠΌΠ΅Π½ΠΈ, ΠΎΡ‚Π½ΠΎΡΠΈΡ‚Π΅Π»ΡŒΠ½ΠΎ Π½Π΅ΠΊΠΎΡ‚ΠΎΡ€ΠΎΠΉ ΠΈΠ½Π΅Ρ€Ρ†ΠΈΠ°Π»ΡŒΠ½ΠΎΠΉ систСмы отсчСта.

Об этом Ρ‚Π°ΠΊΠΆΠ΅ говорят ΠΊΠ°ΠΊ ΠΎΠ± условиях равновСсия систСмы сил. Под систСмой сил Π² статикС всСгда ΠΏΠΎΠ΄Ρ€Π°Π·ΡƒΠΌΠ΅Π²Π°ΡŽΡ‚ΡΡ силы, Π΄Π΅ΠΉΡΡ‚Π²ΡƒΡŽΡ‰ΠΈΠ΅ Π½Π° Π°Π±ΡΠΎΠ»ΡŽΡ‚Π½ΠΎ Ρ‚Π²Π΅Ρ€Π΄ΠΎΠ΅ Ρ‚Π΅Π»ΠΎ, ΠΈΠ»ΠΈ Π½Π° систСму, ΠΊΠΎΡ‚ΠΎΡ€ΡƒΡŽ, Π² соотвСтствии с ΠΏΡ€ΠΈΠ½Ρ†ΠΈΠΏΠΎΠΌ затвСрдСвания, ΠΌΠΎΠΆΠ½ΠΎ ΡΡ‡ΠΈΡ‚Π°Ρ‚ΡŒ Π΅Π΄ΠΈΠ½Ρ‹ΠΌ Ρ‚Π²Π΅Ρ€Π΄Ρ‹ΠΌ Ρ‚Π΅Π»ΠΎΠΌ. ВсС Π·Π°ΠΊΠΎΠ½Ρ‹ прСобразования сил относятся Ρ‚ΠΎΠ»ΡŒΠΊΠΎ ΠΊ силам, Π΄Π΅ΠΉΡΡ‚Π²ΡƒΡŽΡ‰ΠΈΠΌ Π½Π° ΠΎΠ΄Π½ΠΎ Ρ‚Π΅Π»ΠΎ. Под равновСсиСм систСмы сил подразумСваСтся ΡƒΡ€Π°Π²Π½ΠΎΠ²Π΅ΡˆΠ΅Π½Π½Π°Ρ систСма, ΠΊΠΎΡ‚ΠΎΡ€ΡƒΡŽ эквивалСнтными прСобразованиями ΠΌΠΎΠΆΠ½ΠΎ свСсти ΠΊ ΠΎΡ‚ΡΡƒΡ‚ΡΡ‚Π²ΠΈΡŽ сил, Ρ‚ΠΎ Π΅ΡΡ‚ΡŒ ΠΊ ΠΈΡ… Π²Π·Π°ΠΈΠΌΠ½ΠΎΠΌΡƒ ΡƒΠ½ΠΈΡ‡Ρ‚ΠΎΠΆΠ΅Π½ΠΈΡŽ. Π’ΠΎΠ³Π΄Π° Ссли систСма сил находится Π² равновСсии, Ρ‚ΠΎ ΠΎΠ½Π° эквивалСнтна ΠΎΡ‚ΡΡƒΡ‚ΡΡ‚Π²ΠΈΡŽ сил. Вакая систСма Π½Π΅ ΠΎΠΊΠ°Π·Ρ‹Π²Π°Π΅Ρ‚ Π½ΠΈΠΊΠ°ΠΊΠΎΠ³ΠΎ влияния Π½Π° Π΄Π²ΠΈΠΆΠ΅Π½ΠΈΠ΅ Ρ‚Π΅Π»Π°. И Ссли ΠΎΠ½ΠΎ Π²Π½Π°Ρ‡Π°Π»Π΅ покоилось, Ρ‚ΠΎ Π±ΡƒΠ΄Π΅Ρ‚ ΠΏΠΎΠΊΠΎΠΈΡ‚ΡŒΡΡ ΠΈ Π² ΠΏΠΎΡΠ»Π΅Π΄ΡƒΡŽΡ‰ΠΈΠ΅ ΠΌΠΎΠΌΠ΅Π½Ρ‚Ρ‹ Π²Ρ€Π΅ΠΌΠ΅Π½ΠΈ.

Π’Π΅Ρ€ΠΌΠΈΠ½ равновСсиС систСмы сил нСсколько отличаСтся ΠΎΡ‚ Ρ‚Π΅Ρ€ΠΌΠΈΠ½Π° равновСсиС Ρ‚Π²Π΅Ρ€Π΄ΠΎΠ³ΠΎ Ρ‚Π΅Π»Π°. Π Π°Π·Π»ΠΈΡ‡ΠΈΠ΅ связано с Ρ‚Π΅ΠΌ, Ρ‡Ρ‚ΠΎ силы, Π΄Π΅ΠΉΡΡ‚Π²ΡƒΡŽΡ‰ΠΈΠ΅ Π½Π° Ρ‚Π΅Π»ΠΎ ΠΌΠΎΠΆΠ½ΠΎ Ρ€Π°Π·Π±ΠΈΡ‚ΡŒ Π½Π° нСсколько систСм. НСкоторыС ΠΈΠ· этих систСм ΠΌΠΎΠ³ΡƒΡ‚ Π½Π°Ρ…ΠΎΠ΄ΠΈΡ‚ΡŒΡΡ Π² равновСсии, ΠΈ Π½Π΅ ΠΎΠΊΠ°Π·Ρ‹Π²Π°Ρ‚ΡŒ влияния Π½Π° Π΄Π²ΠΈΠΆΠ΅Π½ΠΈΠ΅. Π˜Ρ… ΠΌΠΎΠΆΠ½ΠΎ ΠΈΡΠΊΠ»ΡŽΡ‡ΠΈΡ‚ΡŒ. Π’ Ρ‚ΠΎΠΆΠ΅ врСмя ΠΌΠΎΠ³ΡƒΡ‚ ΡΡƒΡ‰Π΅ΡΡ‚Π²ΠΎΠ²Π°Ρ‚ΡŒ нСравновСсныС систСмы, приводящиС ΠΊ измСнСнию скорости двиТСния Ρ†Π΅Π½Ρ‚Ρ€Π° масс ΠΈ ΠΌΠΎΠΌΠ΅Π½Ρ‚Π° ΠΈΠΌΠΏΡƒΠ»ΡŒΡΠ° Ρ‚Π΅Π»Π°.

Однако, Ссли Π² систСму сил Π²ΠΊΠ»ΡŽΡ‡Π΅Π½Ρ‹ всС внСшниС силы, Ρ‚ΠΎ эти понятия ΡΠΎΠ²ΠΏΠ°Π΄Π°ΡŽΡ‚. Π”Π°Π»Π΅Π΅ ΠΌΡ‹ Π±ΡƒΠ΄Π΅ΠΌ Π³ΠΎΠ²ΠΎΡ€ΠΈΡ‚ΡŒ ΠΎΠ± условиях равновСсия Ρ‚Π²Π΅Ρ€Π΄ΠΎΠ³ΠΎ Ρ‚Π΅Π»Π°. Π­Ρ‚ΠΈ условия Π΅ΡΡ‚ΡŒ Ρ‚ΠΎ ΠΆΠ΅ самоС, Ρ‡Ρ‚ΠΎ условия равновСсия систСмы сил, Ссли ΠΏΠΎΠ΄ систСмой сил ΠΏΠΎΠ΄Ρ€Π°Π·ΡƒΠΌΠ΅Π²Π°Ρ‚ΡŒ всС внСшниС силы, Π΄Π΅ΠΉΡΡ‚Π²ΡƒΡŽΡ‰ΠΈΠ΅ Π½Π° Ρ‚Π΅Π»ΠΎ.

Основная Ρ„ΠΎΡ€ΠΌΠ° условий равновСсия

Для равновСсия Ρ‚Π²Π΅Ρ€Π΄ΠΎΠ³ΠΎ Ρ‚Π΅Π»Π° Π½Π΅ΠΎΠ±Ρ…ΠΎΠ΄ΠΈΠΌΠΎ ΠΈ достаточно, Ρ‡Ρ‚ΠΎΠ±Ρ‹ вСкторная сумма Π²Π½Π΅ΡˆΠ½ΠΈΡ… сил, Π΄Π΅ΠΉΡΡ‚Π²ΡƒΡŽΡ‰ΠΈΡ… Π½Π° Ρ‚Π΅Π»ΠΎ Ρ€Π°Π²Π½ΡΠ»Π°ΡΡŒ Π½ΡƒΠ»ΡŽ, ΠΈ Ρ‡Ρ‚ΠΎΠ±Ρ‹ вСкторная сумма ΠΌΠΎΠΌΠ΅Π½Ρ‚ΠΎΠ² этих сил, ΠΎΡ‚Π½ΠΎΡΠΈΡ‚Π΅Π»ΡŒΠ½ΠΎ ΠΏΡ€ΠΎΠΈΠ·Π²ΠΎΠ»ΡŒΠ½ΠΎ Π²Ρ‹Π±Ρ€Π°Π½Π½ΠΎΠ³ΠΎ Ρ†Π΅Π½Ρ‚Ρ€Π° Β C,Β  Ρ€Π°Π²Π½ΡΠ»Π°ΡΡŒ Π½ΡƒΠ»ΡŽ:
(1.1) Β  ;
(1.2) Β  .
Π”ΠΎΠΊΠ°Π·Π°Ρ‚Π΅Π»ΡŒΡΡ‚Π²ΠΎ ⇓

Π—Π΄Π΅ΡΡŒ внСшниС силы ΠΏΡ€ΠΈΠ»ΠΎΠΆΠ΅Π½Ρ‹ ΠΊ Ρ‚Π΅Π»Ρƒ Π² Ρ‚ΠΎΡ‡ΠΊΠ°Ρ… .

Если ΠΌΡ‹ Π²Ρ‹Π±Π΅Ρ€Π΅ΠΌ ΠΏΡ€ΡΠΌΠΎΡƒΠ³ΠΎΠ»ΡŒΠ½ΡƒΡŽ систСму ΠΊΠΎΠΎΡ€Π΄ΠΈΠ½Π°Ρ‚ Cxyz с Ρ†Π΅Π½Ρ‚Ρ€ΠΎΠΌ Π² Ρ‚ΠΎΡ‡ΠΊΠ΅ C, Ρ‚ΠΎ условия (1.1) ΠΈ (1.2) ΠΌΠΎΠΆΠ½ΠΎ Π²Ρ‹Ρ€Π°Π·ΠΈΡ‚ΡŒ Ρ‡Π΅Ρ€Π΅Π· ΠΏΡ€ΠΎΠ΅ΠΊΡ†ΠΈΠΈ сил ΠΈ ΠΌΠΎΠΌΠ΅Π½Ρ‚ΠΎΠ² Π½Π° оси этой систСмы. Π’ΠΎΠ³Π΄Π° ΠΌΡ‹ ΠΏΠΎΠ»ΡƒΡ‡ΠΈΠΌ ΡˆΠ΅ΡΡ‚ΡŒ ΡƒΡ€Π°Π²Π½Π΅Π½ΠΈΠΉ:
; Β  ; Β  ;
; Β  ; Β  .
Из этих ΡƒΡ€Π°Π²Π½Π΅Π½ΠΈΠΉ ΠΌΠΎΠΆΠ½ΠΎ ΠΎΠΏΡ€Π΅Π΄Π΅Π»ΠΈΡ‚ΡŒ ΡˆΠ΅ΡΡ‚ΡŒ нСизвСстных Π²Π΅Π»ΠΈΡ‡ΠΈΠ½, ΠΎΠΏΡ€Π΅Π΄Π΅Π»ΡΡŽΡ‰ΠΈΡ… Ρ€Π΅Π°ΠΊΡ†ΠΈΠΈ ΠΎΠΏΠΎΡ€ Ρ‚Π΅Π»Π°.

Π’Π°ΠΊΠΆΠ΅ ΠΌΡ‹ ΠΌΠΎΠΆΠ΅ΠΌ ΠΏΡ€ΠΎΠΈΠ·Π²ΠΎΠ»ΡŒΠ½Ρ‹ΠΌ ΠΎΠ±Ρ€Π°Π·ΠΎΠΌ Π²Ρ‹Π±Ρ€Π°Ρ‚ΡŒ Ρ‚Ρ€ΠΈ Π²Π΅ΠΊΡ‚ΠΎΡ€Π°, Π½Π΅ Π»Π΅ΠΆΠ°Ρ‰ΠΈΠ΅ Π² ΠΎΠ΄Π½ΠΎΠΉ плоскости, ΠΈ ΡΠΏΡ€ΠΎΠ΅ΠΊΡ‚ΠΈΡ€ΠΎΠ²Π°Ρ‚ΡŒ уравнСния (1.1) ΠΈ (1.2) Π½Π° ΠΈΡ… направлСния. Π’ Ρ€Π΅Π·ΡƒΠ»ΡŒΡ‚Π°Ρ‚Π΅ ΠΌΡ‹ Ρ‚Π°ΠΊΠΆΠ΅ ΠΏΠΎΠ»ΡƒΡ‡ΠΈΠΌ систСму ΠΈΠ· ΡˆΠ΅ΡΡ‚ΠΈ ΡƒΡ€Π°Π²Π½Π΅Π½ΠΈΠΉ.

Вторая Ρ„ΠΎΡ€ΠΌΠ° условий равновСсия

Условия равновСсия ΠΌΠΎΠΆΠ½ΠΎ Π·Π°ΠΏΠΈΡΠ°Ρ‚ΡŒ ΠΈ Π² Π΄Ρ€ΡƒΠ³ΠΈΡ… Ρ„ΠΎΡ€ΠΌΠ°Ρ…, ΠΊΠΎΡ‚ΠΎΡ€Ρ‹Π΅ ΠΌΠΎΠ³ΡƒΡ‚ ΠΎΠΊΠ°Π·Π°Ρ‚ΡŒΡΡ Π±ΠΎΠ»Π΅Π΅ ΡƒΠ΄ΠΎΠ±Π½Ρ‹ΠΌΠΈ ΠΏΡ€ΠΈ Ρ€Π΅ΡˆΠ΅Π½ΠΈΠΈ Π½Π΅ΠΊΠΎΡ‚ΠΎΡ€Ρ‹Ρ… Π·Π°Π΄Π°Ρ‡. Π’ΠΎΡ‚ вторая Ρ„ΠΎΡ€ΠΌΠ° условий равновСсия.

Для равновСсия Ρ‚Π²Π΅Ρ€Π΄ΠΎΠ³ΠΎ Ρ‚Π΅Π»Π° Π½Π΅ΠΎΠ±Ρ…ΠΎΠ΄ΠΈΠΌΠΎ ΠΈ достаточно, Ρ‡Ρ‚ΠΎΠ±Ρ‹ вСкторная сумма ΠΌΠΎΠΌΠ΅Π½Ρ‚ΠΎΠ² Π²Π½Π΅ΡˆΠ½ΠΈΡ… сил, Π΄Π΅ΠΉΡΡ‚Π²ΡƒΡŽΡ‰ΠΈΡ… Π½Π° Ρ‚Π΅Π»ΠΎ, ΠΎΡ‚Π½ΠΎΡΠΈΡ‚Π΅Π»ΡŒΠ½ΠΎ ΠΏΡ€ΠΎΠΈΠ·Π²ΠΎΠ»ΡŒΠ½Ρ‹ΠΌ ΠΎΠ±Ρ€Π°Π·ΠΎΠΌ Π²Ρ‹Π±Ρ€Π°Π½Π½Ρ‹Ρ… Ρ‚ΠΎΡ‡Π΅ΠΊ ΠΈ Ρ€Π°Π²Π½ΡΠ»Π°ΡΡŒ Π½ΡƒΠ»ΡŽ, ΠΈ Ρ‡Ρ‚ΠΎΠ±Ρ‹ сумма ΠΏΡ€ΠΎΠ΅ΠΊΡ†ΠΈΠΉ сил Π½Π° ΠΏΡ€ΠΎΠΈΠ·Π²ΠΎΠ»ΡŒΠ½ΠΎΠ΅ Π½Π°ΠΏΡ€Π°Π²Π»Π΅Π½ΠΈΠ΅, Π½Π΅ пСрпСндикулярноС Π½Π°ΠΏΡ€Π°Π²Π»Π΅Π½ΠΈΡŽ , Ρ€Π°Π²Π½ΡΠ»Π°ΡΡŒ Π½ΡƒΠ»ΡŽ:
(2.1) Β  ;
(2.2) Β  ;
(2.3) Β  .
Π”ΠΎΠΊΠ°Π·Π°Ρ‚Π΅Π»ΡŒΡΡ‚Π²ΠΎ ⇓

Если ΡΠΏΡ€ΠΎΠ΅ΠΊΡ‚ΠΈΡ€ΠΎΠ²Π°Ρ‚ΡŒ условия (2.1) ΠΈ (2.2) Π½Π° оси ΠΊΠΎΠΎΡ€Π΄ΠΈΠ½Π°Ρ‚, Ρ‚ΠΎ ΠΏΠΎΠ»ΡƒΡ‡ΠΈΠΌ Ρ‚Ρ€ΠΈ уравнСния (2.1), Ρ‚Ρ€ΠΈ уравнСния (2.2) ΠΈ ΠΎΠ΄Π½ΠΎ ΡƒΡ€Π°Π²Π½Π΅Π½ΠΈΠ΅ (2.3). ВсСго получаСтся сСмь ΡƒΡ€Π°Π²Π½Π΅Π½ΠΈΠΉ. Однако, ΠΊΠ°ΠΊ ΠΏΠΎΠΊΠ°Π·Π°Π½ΠΎ Π½ΠΈΠΆΠ΅, ΠΌΠ΅ΠΆΠ΄Ρƒ ΡˆΠ΅ΡΡ‚ΡŒΡŽ уравнСниями (2.1) ΠΈ (2.2) сущСствуСт ΠΎΠ΄Π½Π° линСйная Π·Π°Π²ΠΈΡΠΈΠΌΠΎΡΡ‚ΡŒ (см. «ЛинСйная Π·Π°Π²ΠΈΡΠΈΠΌΠΎΡΡ‚ΡŒ ΠΌΠΎΠΌΠ΅Π½Ρ‚ΠΎΠ² ΠΎΡ‚Π½ΠΎΡΠΈΡ‚Π΅Π»ΡŒΠ½ΠΎ Π΄Π²ΡƒΡ… Ρ‚ΠΎΡ‡Π΅ΠΊ ⇓»). Π’Π°ΠΊΠΈΠΌ ΠΎΠ±Ρ€Π°Π·ΠΎΠΌ, Π² условиях (2.1-3) имССтся 7-1=6 Π»ΠΈΠ½Π΅ΠΉΠ½ΠΎ нСзависимых ΡƒΡ€Π°Π²Π½Π΅Π½ΠΈΠΉ, ΠΈΠ· ΠΊΠΎΡ‚ΠΎΡ€Ρ‹Ρ… ΠΌΠΎΠΆΠ½ΠΎ ΠΎΠΏΡ€Π΅Π΄Π΅Π»ΠΈΡ‚ΡŒ ΡˆΠ΅ΡΡ‚ΡŒ нСизвСстных Π²Π΅Π»ΠΈΡ‡ΠΈΠ½.

Π’Ρ€Π΅Ρ‚ΡŒΡ Ρ„ΠΎΡ€ΠΌΠ° условий равновСсия

И Π½Π°ΠΊΠΎΠ½Π΅Ρ†, имССтся Ρ‚Ρ€Π΅Ρ‚ΡŒΡ Ρ„ΠΎΡ€ΠΌΠ° условий равновСсия.

Для равновСсия Ρ‚Π²Π΅Ρ€Π΄ΠΎΠ³ΠΎ Ρ‚Π΅Π»Π° Π½Π΅ΠΎΠ±Ρ…ΠΎΠ΄ΠΈΠΌΠΎ ΠΈ достаточно, Ρ‡Ρ‚ΠΎΠ±Ρ‹ вСкторная сумма ΠΌΠΎΠΌΠ΅Π½Ρ‚ΠΎΠ² Π²Π½Π΅ΡˆΠ½ΠΈΡ… сил, Π΄Π΅ΠΉΡΡ‚Π²ΡƒΡŽΡ‰ΠΈΡ… Π½Π° Ρ‚Π΅Π»ΠΎ, ΠΎΡ‚Π½ΠΎΡΠΈΡ‚Π΅Π»ΡŒΠ½ΠΎ ΠΏΡ€ΠΎΠΈΠ·Π²ΠΎΠ»ΡŒΠ½Ρ‹ΠΌ ΠΎΠ±Ρ€Π°Π·ΠΎΠΌ Π²Ρ‹Π±Ρ€Π°Π½Π½Ρ‹Ρ… Ρ‚ΠΎΡ‡Π΅ΠΊ ΠΈ , Π½Π΅ Π»Π΅ΠΆΠ°Ρ‰ΠΈΡ… Π½Π° ΠΎΠ΄Π½ΠΎΠΉ прямой, Ρ€Π°Π²Π½ΡΠ»Π°ΡΡŒ Π½ΡƒΠ»ΡŽ:
(3.1) Β  ;
(3.2) Β  ;
(3.3) Β  ;
(3.4) Β  .
Π”ΠΎΠΊΠ°Π·Π°Ρ‚Π΅Π»ΡŒΡΡ‚Π²ΠΎ ⇓

Если ΡΠΏΡ€ΠΎΠ΅ΠΊΡ‚ΠΈΡ€ΠΎΠ²Π°Ρ‚ΡŒ условия (3.1), (3.2) ΠΈ (3.3) Π½Π° оси ΠΊΠΎΠΎΡ€Π΄ΠΈΠ½Π°Ρ‚, Ρ‚ΠΎ ΠΏΠΎΠ»ΡƒΡ‡ΠΈΠΌ Ρ‚Ρ€ΠΈ уравнСния (3.1), Ρ‚Ρ€ΠΈ уравнСния (3.2) ΠΈ Ρ‚Ρ€ΠΈ уравнСния (3.3) – всСго Π΄Π΅Π²ΡΡ‚ΡŒ ΡƒΡ€Π°Π²Π½Π΅Π½ΠΈΠΉ. Как ΠΏΠΎΠΊΠ°Π·Π°Π½ΠΎ Π½ΠΈΠΆΠ΅, ΠΌΠ΅ΠΆΠ΄Ρƒ ΡˆΠ΅ΡΡ‚ΡŒΡŽ уравнСниями (3.1) ΠΈ (3.2) сущСствуСт ΠΎΠ΄Π½Π° линСйная Π·Π°Π²ΠΈΡΠΈΠΌΠΎΡΡ‚ΡŒ (см. «ЛинСйная Π·Π°Π²ΠΈΡΠΈΠΌΠΎΡΡ‚ΡŒ ΠΌΠΎΠΌΠ΅Π½Ρ‚ΠΎΠ² ΠΎΡ‚Π½ΠΎΡΠΈΡ‚Π΅Π»ΡŒΠ½ΠΎ Π΄Π²ΡƒΡ… Ρ‚ΠΎΡ‡Π΅ΠΊ ⇓»). Аналогичным ΠΎΠ±Ρ€Π°Π·ΠΎΠΌ, ΠΌΠ΅ΠΆΠ΄Ρƒ ΡˆΠ΅ΡΡ‚ΡŒΡŽ уравнСниями (3.1) ΠΈ (3.3) сущСствуСт Π΅Ρ‰Π΅ ΠΎΠ΄Π½Π° линСйная Π·Π°Π²ΠΈΡΠΈΠΌΠΎΡΡ‚ΡŒ. И Π½Π°ΠΊΠΎΠ½Π΅Ρ†, ΠΌΠ΅ΠΆΠ΄Ρƒ ΡˆΠ΅ΡΡ‚ΡŒΡŽ уравнСниями (3.2) ΠΈ (3.3) сущСствуСт Ρ‚Ρ€Π΅Ρ‚ΡŒΡ линСйная Π·Π°Π²ΠΈΡΠΈΠΌΠΎΡΡ‚ΡŒ. Π’ΠΎ Π΅ΡΡ‚ΡŒ, Π² условиях (3.1-3) имССтся Ρ‚Ρ€ΠΈ Π»ΠΈΠ½Π΅ΠΉΠ½Ρ‹Ρ… зависимости. Π’ΠΎΠ³Π΄Π° число Π»ΠΈΠ½Π΅ΠΉΠ½ΠΎ нСзависимых ΡƒΡ€Π°Π²Π½Π΅Π½ΠΈΠΉ Ρ€Π°Π²Π½ΠΎ 9–3=6. Π’Π°ΠΊΠΆΠ΅, ΠΊΠ°ΠΊ ΠΈ Π² ΠΏΡ€Π΅Π΄Ρ‹Π΄ΡƒΡ‰ΠΈΡ… Ρ„ΠΎΡ€ΠΌΠ°Ρ…, ΠΈΠ· этих ΡƒΡ€Π°Π²Π½Π΅Π½ΠΈΠΉ ΠΌΠΎΠΆΠ½ΠΎ ΠΎΠΏΡ€Π΅Π΄Π΅Π»ΠΈΡ‚ΡŒ ΡˆΠ΅ΡΡ‚ΡŒ нСизвСстных Π²Π΅Π»ΠΈΡ‡ΠΈΠ½.

Условия равновСсия плоского Ρ‚Π΅Π»Π°

Π’Π΅ΠΏΠ΅Ρ€ΡŒ рассмотрим ΠΏΠ»ΠΎΡΠΊΡƒΡŽ систСму, Π² ΠΊΠΎΡ‚ΠΎΡ€ΠΎΠΉ Ρ‚Π΅Π»ΠΎ ΠΌΠΎΠΆΠ΅Ρ‚ ΡΠΎΠ²Π΅Ρ€ΡˆΠ°Ρ‚ΡŒ Π΄Π²ΠΈΠΆΠ΅Π½ΠΈΠ΅ Ρ‚ΠΎΠ»ΡŒΠΊΠΎ вдоль ΠΎΠ΄Π½ΠΎΠΉ плоскости. ΠŸΡ€ΠΈ этом силы Ρ‚Π°ΠΊΠΆΠ΅ Π½Π°ΠΏΡ€Π°Π²Π»Π΅Π½Ρ‹ Π² этой плоскости. Π’ этом случаС ΠΌΡ‹ Π²Ρ‹Π±ΠΈΡ€Π°Π΅ΠΌ систСму отсчСта Ρ‚Π°ΠΊ, Ρ‡Ρ‚ΠΎΠ±Ρ‹ оси x ΠΈ y Π»Π΅ΠΆΠ°Π»ΠΈ Π² рассматриваСмой плоскости, Π° ось z Π±Ρ‹Π»Π° Π΅ΠΉ пСрпСндикулярна. Π’ΠΎΠ³Π΄Π° ΠΏΡ€ΠΈΠ²Π΅Π΄Π΅Π½Π½Ρ‹Π΅ Π²Ρ‹ΡˆΠ΅ Ρ„ΠΎΡ€ΠΌΡ‹ условий равновСсия ΡΠΎΡ…Ρ€Π°Π½ΡΡŽΡ‚ свой Π²ΠΈΠ΄. ΠŸΡ€ΠΈ этом z – ΠΊΠΎΠΌΠΏΠΎΠ½Π΅Π½Ρ‚Ρ‹ всСх сил Ρ€Π°Π²Π½Ρ‹ Π½ΡƒΠ»ΡŽ: , Π° Ρƒ ΠΌΠΎΠΌΠ΅Π½Ρ‚ΠΎΠ² сил ΠΎΡ‚Π»ΠΈΡ‡Π½Π° ΠΎΡ‚ Π½ΡƒΠ»ΡŽ Ρ‚ΠΎΠ»ΡŒΠΊΠΎ z – ΠΊΠΎΠΌΠΏΠΎΠ½Π΅Π½Ρ‚Π°: .

Π’Ρ‹ΠΏΠΈΡˆΠ΅ΠΌ условия равновСсия для плоской систСмы, расписав ΠΈΡ… ΠΏΠΎ ΠΊΠΎΠΌΠΏΠΎΠ½Π΅Π½Ρ‚Π°ΠΌ.

Основная Ρ„ΠΎΡ€ΠΌΠ° условий равновСсия
;
;
.

Вторая Ρ„ΠΎΡ€ΠΌΠ° условий равновСсия
;
;
.

Π’Ρ€Π΅Ρ‚ΡŒΡ Ρ„ΠΎΡ€ΠΌΠ° условий равновСсия
;
;
;
.

Π—Π΄Π΅ΡΡŒ Π²ΠΎ всСх Ρ„ΠΎΡ€ΠΌΠ°Ρ… имССтся ΠΏΠΎ Ρ‚Ρ€ΠΈ уравнСния, ΠΈΠ· ΠΊΠΎΡ‚ΠΎΡ€Ρ‹Ρ… ΠΌΠΎΠΆΠ½ΠΎ ΠΎΠΏΡ€Π΅Π΄Π΅Π»ΠΈΡ‚ΡŒ Ρ‚Ρ€ΠΈ нСизвСстных Π²Π΅Π»ΠΈΡ‡ΠΈΠ½Ρ‹.

Π”ΠΎΠΊΠ°Π·Π°Ρ‚Π΅Π»ΡŒΡΡ‚Π²ΠΎ условий равновСсия

Основная Ρ„ΠΎΡ€ΠΌΠ° условий равновСсия

ВсС Ρ„ΠΎΡ€ΠΌΡ‹ ⇑ Для равновСсия Ρ‚Π²Π΅Ρ€Π΄ΠΎΠ³ΠΎ Ρ‚Π΅Π»Π° Π½Π΅ΠΎΠ±Ρ…ΠΎΠ΄ΠΈΠΌΠΎ ΠΈ достаточно, Ρ‡Ρ‚ΠΎΠ±Ρ‹ вСкторная сумма Π²Π½Π΅ΡˆΠ½ΠΈΡ… сил, Π΄Π΅ΠΉΡΡ‚Π²ΡƒΡŽΡ‰ΠΈΡ… Π½Π° Ρ‚Π΅Π»ΠΎ Ρ€Π°Π²Π½ΡΠ»Π°ΡΡŒ Π½ΡƒΠ»ΡŽ, ΠΈ Ρ‡Ρ‚ΠΎΠ±Ρ‹ вСкторная сумма ΠΌΠΎΠΌΠ΅Π½Ρ‚ΠΎΠ² этих сил, ΠΎΡ‚Π½ΠΎΡΠΈΡ‚Π΅Π»ΡŒΠ½ΠΎ ΠΏΡ€ΠΎΠΈΠ·Π²ΠΎΠ»ΡŒΠ½ΠΎ Π²Ρ‹Π±Ρ€Π°Π½Π½ΠΎΠ³ΠΎ Ρ†Π΅Π½Ρ‚Ρ€Π° Β C,Β  Ρ€Π°Π²Π½ΡΠ»Π°ΡΡŒ Π½ΡƒΠ»ΡŽ:
(1.1) Β 
(1.2) Β 

Π”ΠΎΠΊΠ°Π·Π°Ρ‚Π΅Π»ΡŒΡΡ‚Π²ΠΎ

Для Π΄ΠΎΠΊΠ°Π·Π°Ρ‚Π΅Π»ΡŒΡΡ‚Π²Π° Π²ΠΎΡΠΏΠΎΠ»ΡŒΠ·ΡƒΠ΅ΠΌΡΡ Π·Π°ΠΊΠΎΠ½Π°ΠΌΠΈ двиТСния Ρ‚Π²Π΅Ρ€Π΄ΠΎΠ³ΠΎ Ρ‚Π΅Π»Π°. Они ΠΎΠΏΠΈΡΡ‹Π²Π°ΡŽΡ‚ΡΡ уравнСниями:
(1.3) Β  ;
(1.4) Β  .
Π—Π΄Π΅ΡΡŒ – ускорСниС Ρ†Π΅Π½Ρ‚Ρ€Π° масс Ρ‚Π΅Π»Π°; M – Π΅Π³ΠΎ масса; – ΠΌΠΎΠΌΠ΅Π½Ρ‚ ΠΈΠΌΠΏΡƒΠ»ΡŒΡΠ° Ρ‚Π΅Π»Π° ΠΎΡ‚Π½ΠΎΡΠΈΡ‚Π΅Π»ΡŒΠ½ΠΎ ΠΏΡ€ΠΎΠΈΠ·Π²ΠΎΠ»ΡŒΠ½ΠΎ Π²Ρ‹Π±Ρ€Π°Π½Π½ΠΎΠ³ΠΎ Ρ†Π΅Π½Ρ‚Ρ€Π° C; – внСшниС силы, Π΄Π΅ΠΉΡΡ‚Π²ΡƒΡŽΡ‰ΠΈΠ΅ Π½Π° Ρ‚Π΅Π»ΠΎ, ΠΏΡ€ΠΈΠ»ΠΎΠΆΠ΅Π½Π½Ρ‹Π΅ Π² Ρ‚ΠΎΡ‡ΠΊΠ°Ρ… .

ΠΠ΅ΠΎΠ±Ρ…ΠΎΠ΄ΠΈΠΌΠΎΡΡ‚ΡŒ.

ΠŸΡƒΡΡ‚ΡŒ Ρ‚Π΅Π»ΠΎ находится Π² состоянии покоя ΠΎΡ‚Π½ΠΎΡΠΈΡ‚Π΅Π»ΡŒΠ½ΠΎ Π²Ρ‹Π±Ρ€Π°Π½Π½ΠΎΠΉ ΠΈΠ½Π΅Ρ€Ρ†ΠΈΠ°Π»ΡŒΠ½ΠΎΠΉ систСмы ΠΊΠΎΠΎΡ€Π΄ΠΈΠ½Π°Ρ‚. Π’ΠΎΠ³Π΄Π°, Π² этой систСмС ΠΊΠΎΠΎΡ€Π΄ΠΈΠ½Π°Ρ‚, ΡΠΊΠΎΡ€ΠΎΡΡ‚ΡŒ всСх Ρ‚ΠΎΡ‡Π΅ΠΊ Ρ€Π°Π²Π½Π° Π½ΡƒΠ»ΡŽ. ΠžΡ‚ΡΡŽΠ΄Π°
, Β  .
ΠŸΠΎΠ΄ΡΡ‚Π°Π²Π»ΡΡ Π² (1.3) ΠΈ (1.4), ΠΏΠΎΠ»ΡƒΡ‡Π°Π΅ΠΌ (1.1) ΠΈ (1.2).
ΠΠ΅ΠΎΠ±Ρ…ΠΎΠ΄ΠΈΠΌΠΎΡΡ‚ΡŒ Π΄ΠΎΠΊΠ°Π·Π°Π½Π°.

Π”ΠΎΡΡ‚Π°Ρ‚ΠΎΡ‡Π½ΠΎΡΡ‚ΡŒ.

ΠŸΡƒΡΡ‚ΡŒ Π²Ρ‹ΠΏΠΎΠ»Π½ΡΡŽΡ‚ΡΡ условия равновСсия (1.1) ΠΈ (1.2). ΠŸΠΎΠ΄ΡΡ‚Π°Π²Π»ΡΡ ΠΈΡ… Π² уравнСния двиТСния (1.3) ΠΈ (1.4), ΠΏΠΎΠ»ΡƒΡ‡Π°Π΅ΠΌ:
;
.
ΠžΡ‚ΡΡŽΠ΄Π° ΠΏΠΎΠ»ΡƒΡ‡Π°Π΅ΠΌ, Ρ‡Ρ‚ΠΎ ΡΠΊΠΎΡ€ΠΎΡΡ‚ΡŒ двиТСния Ρ†Π΅Π½Ρ‚Ρ€Π° масс ΠΈ ΠΌΠΎΠΌΠ΅Π½Ρ‚ ΠΈΠΌΠΏΡƒΠ»ΡŒΡΠ° постоянны, Π½Π΅ ΠΌΠ΅Π½ΡΡŽΡ‚ΡΡ со Π²Ρ€Π΅ΠΌΠ΅Π½Π΅ΠΌ. ΠŸΡƒΡΡ‚ΡŒ Ρ‚Π΅ΠΏΠ΅Ρ€ΡŒ Π² Π½Π°Ρ‡Π°Π»ΡŒΠ½Ρ‹ΠΉ ΠΌΠΎΠΌΠ΅Π½Ρ‚ Π²Ρ€Π΅ΠΌΠ΅Π½ΠΈ Ρ‚Π΅Π»ΠΎ покоилось. Π’ΠΎΠ³Π΄Π° ΡΠΊΠΎΡ€ΠΎΡΡ‚ΡŒ двиТСния Π΅Π³ΠΎ Ρ†Π΅Π½Ρ‚Ρ€Π° масс ΠΈ ΠΌΠΎΠΌΠ΅Π½Ρ‚ ΠΈΠΌΠΏΡƒΠ»ΡŒΡΠ° Ρ€Π°Π²Π½Ρ‹ Π½ΡƒΠ»ΡŽ. А ΠΏΠΎΡΠΊΠΎΠ»ΡŒΠΊΡƒ ΠΎΠ½ΠΈ Π½Π΅ ΠΌΠ΅Π½ΡΡŽΡ‚ΡΡ со Π²Ρ€Π΅ΠΌΠ΅Π½Π΅ΠΌ, Ρ‚ΠΎ ΠΎΠ½ΠΈ Ρ€Π°Π²Π½Ρ‹ Π½ΡƒΠ»ΡŽ ΠΈ Π² ΠΏΠΎΡΠ»Π΅Π΄ΡƒΡŽΡ‰ΠΈΠ΅ ΠΌΠΎΠΌΠ΅Π½Ρ‚Ρ‹ Π²Ρ€Π΅ΠΌΠ΅Π½ΠΈ. Π’ΠΎ Π΅ΡΡ‚ΡŒ Ρ‚Π΅Π»ΠΎ остаСтся Π² состоянии покоя Π²ΠΎ всС ΠΌΠΎΠΌΠ΅Π½Ρ‚Ρ‹ Π²Ρ€Π΅ΠΌΠ΅Π½ΠΈ.

Бвойство Π΄ΠΎΠΊΠ°Π·Π°Π½ΠΎ.

Вторая Ρ„ΠΎΡ€ΠΌΠ° условий равновСсия

ВсС Ρ„ΠΎΡ€ΠΌΡ‹ ⇑ Для равновСсия Ρ‚Π²Π΅Ρ€Π΄ΠΎΠ³ΠΎ Ρ‚Π΅Π»Π° Π½Π΅ΠΎΠ±Ρ…ΠΎΠ΄ΠΈΠΌΠΎ ΠΈ достаточно, Ρ‡Ρ‚ΠΎΠ±Ρ‹ вСкторная сумма ΠΌΠΎΠΌΠ΅Π½Ρ‚ΠΎΠ² Π²Π½Π΅ΡˆΠ½ΠΈΡ… сил, Π΄Π΅ΠΉΡΡ‚Π²ΡƒΡŽΡ‰ΠΈΡ… Π½Π° Ρ‚Π΅Π»ΠΎ, ΠΎΡ‚Π½ΠΎΡΠΈΡ‚Π΅Π»ΡŒΠ½ΠΎ ΠΏΡ€ΠΎΠΈΠ·Π²ΠΎΠ»ΡŒΠ½Ρ‹ΠΌ ΠΎΠ±Ρ€Π°Π·ΠΎΠΌ Π²Ρ‹Π±Ρ€Π°Π½Π½Ρ‹Ρ… Ρ‚ΠΎΡ‡Π΅ΠΊ ΠΈ Ρ€Π°Π²Π½ΡΠ»Π°ΡΡŒ Π½ΡƒΠ»ΡŽ, ΠΈ Ρ‡Ρ‚ΠΎΠ±Ρ‹ сумма ΠΏΡ€ΠΎΠ΅ΠΊΡ†ΠΈΠΉ сил Π½Π° ΠΏΡ€ΠΎΠΈΠ·Π²ΠΎΠ»ΡŒΠ½ΠΎΠ΅ Π½Π°ΠΏΡ€Π°Π²Π»Π΅Π½ΠΈΠ΅, Π½Π΅ пСрпСндикулярноС Π½Π°ΠΏΡ€Π°Π²Π»Π΅Π½ΠΈΡŽ , Ρ€Π°Π²Π½ΡΠ»Π°ΡΡŒ Π½ΡƒΠ»ΡŽ:
(2.1) Β 
(2.2) Β 
(2.3) Β 

Π”ΠΎΠΊΠ°Π·Π°Ρ‚Π΅Π»ΡŒΡΡ‚Π²ΠΎ

ΠΠ΅ΠΎΠ±Ρ…ΠΎΠ΄ΠΈΠΌΠΎΡΡ‚ΡŒ.

ΠŸΡƒΡΡ‚ΡŒ Ρ‚Π΅Π»ΠΎ находится Π² состоянии равновСсия. Π’ΠΎΠ·ΡŒΠΌΠ΅ΠΌ ΠΏΡ€ΠΎΠΈΠ·Π²ΠΎΠ»ΡŒΠ½Ρ‹Π΅ Ρ‚ΠΎΡ‡ΠΊΠΈ ΠΈ ΠΈ Π²Ρ‹Π±Π΅Ρ€Π΅ΠΌ ΠΏΡ€ΠΎΠΈΠ·Π²ΠΎΠ»ΡŒΠ½Ρ‹ΠΉ Π²Π΅ΠΊΡ‚ΠΎΡ€ , Π½Π΅ пСрпСндикулярный прямой : Β  . Как ΡƒΠΆΠ΅ Π±Ρ‹Π»ΠΎ Π΄ΠΎΠΊΠ°Π·Π°Π½ΠΎ ΠΏΡ€ΠΈ Π²Ρ‹Π²ΠΎΠ΄Π΅ основной Ρ„ΠΎΡ€ΠΌΡ‹ условий равновСсия ⇑, Π²Ρ‹ΠΏΠΎΠ»Π½ΡΡŽΡ‚ΡΡ условия (1.1) ΠΈ(1.2):
(1.1) Β  :
(1.2) Β  .
ΠŸΠΎΡΠΊΠΎΠ»ΡŒΠΊΡƒ здСсь C – ΠΏΡ€ΠΎΠΈΠ·Π²ΠΎΠ»ΡŒΠ½Π°Ρ Ρ‚ΠΎΡ‡ΠΊΠ°, Ρ‚ΠΎ Π² качСствС Π½Π΅Π΅ возьмСм Ρ‚ΠΎΡ‡ΠΊΡƒ . Π’ Ρ€Π΅Π·ΡƒΠ»ΡŒΡ‚Π°Ρ‚Π΅ ΠΏΠΎΠ»ΡƒΡ‡ΠΈΠΌ (2.1):
(2.1) Β  .
Π”Π°Π»Π΅Π΅, Π² качСствС C возьмСм Ρ‚ΠΎΡ‡ΠΊΡƒ . ΠŸΠΎΠ»ΡƒΡ‡ΠΈΠΌ (2.2):
(2.2) Β  .
Π’Π΅ΠΏΠ΅Ρ€ΡŒ спроСктируСм ΡƒΡ€Π°Π²Π½Π΅Π½ΠΈΠ΅ (1.1) Π½Π° Π½Π°ΠΏΡ€Π°Π²Π»Π΅Π½ΠΈΠ΅ Π²Π΅ΠΊΡ‚ΠΎΡ€Π° . ΠŸΠΎΠ»ΡƒΡ‡ΠΈΠΌ (2.3):
(2.3) Β  .
Π­Ρ‚ΠΎ ΡƒΡ€Π°Π²Π½Π΅Π½ΠΈΠ΅ выполняСтся для Π»ΡŽΠ±Ρ‹Ρ… Π²Π΅ΠΊΡ‚ΠΎΡ€ΠΎΠ² . Π’ Ρ‚ΠΎΠΌ числС ΠΈ для Ρ‚Π΅Ρ…, Π½Π°ΠΏΡ€Π°Π²Π»Π΅Π½ΠΈΠ΅ ΠΊΠΎΡ‚ΠΎΡ€Ρ‹Ρ… Π½Π΅ пСрпСндикулярно : Β  .
ΠΠ΅ΠΎΠ±Ρ…ΠΎΠ΄ΠΈΠΌΠΎΡΡ‚ΡŒ Π΄ΠΎΠΊΠ°Π·Π°Π½Π°.

Π”ΠΎΡΡ‚Π°Ρ‚ΠΎΡ‡Π½ΠΎΡΡ‚ΡŒ.

ΠŸΡƒΡΡ‚ΡŒ Π²Ρ‹ΠΏΠΎΠ»Π½ΡΡŽΡ‚ΡΡ условия (2.1), (2.2) ΠΈ (2.3). Π”ΠΎΠΊΠ°ΠΆΠ΅ΠΌ, Ρ‡Ρ‚ΠΎ Ρ‚ΠΎΠ³Π΄Π° Ρ‚Π΅Π»ΠΎ Π±ΡƒΠ΄Π΅Ρ‚ Π½Π°Ρ…ΠΎΠ΄ΠΈΡ‚ΡŒΡΡ Π² состоянии равновСсия. Π’ΠΎΡΠΏΠΎΠ»ΡŒΠ·ΡƒΠ΅ΠΌΡΡ Π²Π΅ΠΊΡ‚ΠΎΡ€Π½Ρ‹ΠΌ ΡƒΡ€Π°Π²Π½Π΅Π½ΠΈΠ΅ΠΌ:
(2.4) Β  .
ΠŸΠΎΠ΄ΡΡ‚Π°Π²ΠΈΠΌ Π΅Π³ΠΎ Π² (2.1):

.
ΠŸΠΎΡΠΊΠΎΠ»ΡŒΠΊΡƒ ΠΈΠ· (2.2), Β  , Ρ‚ΠΎ Β  .
ΠžΡ‚ΡΡŽΠ΄Π°
(2.5) Β  ,
Π³Π΄Π΅ Ξ» – ΠΏΡ€ΠΎΠΈΠ·Π²ΠΎΠ»ΡŒΠ½Π°Ρ постоянная. Π£ΠΌΠ½ΠΎΠΆΠΈΠΌ это ΡƒΡ€Π°Π²Π½Π΅Π½ΠΈΠ΅ скалярно Π½Π° ΠΈ ΠΏΡ€ΠΈΠΌΠ΅Π½ΠΈΠΌ (2.3):
(2.6) Β  .
По ΡƒΡΠ»ΠΎΠ²ΠΈΡŽ, . ΠŸΠΎΡΡ‚ΠΎΠΌΡƒ .
Π’ΠΎΠ³Π΄Π°, Ρ‡Ρ‚ΠΎΠ±Ρ‹ Π²Ρ‹ΠΏΠΎΠ»Π½ΡΠ»ΠΎΡΡŒ (2.6) Π½ΡƒΠΆΠ½ΠΎ ΠΏΠΎΠ»ΠΎΠΆΠΈΡ‚ΡŒ . Π’ Ρ€Π΅Π·ΡƒΠ»ΡŒΡ‚Π°Ρ‚Π΅ ΠΈΠ· (2.5) ΠΏΠΎΠ»ΡƒΡ‡Π°Π΅ΠΌ ΡƒΡ€Π°Π²Π½Π΅Π½ΠΈΠ΅ (1.1):
.
УсловиС (1.2) Ρ‚Π°ΠΊΠΆΠ΅ выполняСтся, Ссли ΠΏΠΎΠ»ΠΎΠΆΠΈΡ‚ΡŒ . Π’Π°ΠΊΠΈΠΌ ΠΎΠ±Ρ€Π°Π·ΠΎΠΌ ΠΌΡ‹ ΠΏΠΎΠ»ΡƒΡ‡ΠΈΠ»ΠΈ, Ρ‡Ρ‚ΠΎ Ссли Π²Ρ‹ΠΏΠΎΠ»Π½ΡΡŽΡ‚ΡΡ условия (2.1), (2.2) ΠΈ (2.3), Ρ‚ΠΎ Π²Ρ‹ΠΏΠΎΠ»Π½ΡΡŽΡ‚ΡΡ условия (1.1) ΠΈ (1.2):
(1.1) Β  ;
(1.2) Β  .
Как ΠΌΡ‹ ΡƒΠΆΠ΅ Π΄ΠΎΠΊΠ°Π·Π°Π»ΠΈ ΠΏΡ€ΠΈ Π²Ρ‹Π²ΠΎΠ΄Π΅ основной Ρ„ΠΎΡ€ΠΌΡ‹ условий равновСсия ⇑, это ΠΎΠ·Π½Π°Ρ‡Π°Π΅Ρ‚, Ρ‡Ρ‚ΠΎ Ρ‚Π΅Π»ΠΎ находится Π² равновСсии.

Бвойство Π΄ΠΎΠΊΠ°Π·Π°Π½ΠΎ.

ЛинСйная Π·Π°Π²ΠΈΡΠΈΠΌΠΎΡΡ‚ΡŒ ΠΌΠΎΠΌΠ΅Π½Ρ‚ΠΎΠ² ΠΎΡ‚Π½ΠΎΡΠΈΡ‚Π΅Π»ΡŒΠ½ΠΎ Π΄Π²ΡƒΡ… Ρ‚ΠΎΡ‡Π΅ΠΊ

Π”ΠΎΠΊΠ°ΠΆΠ΅ΠΌ, Ρ‡Ρ‚ΠΎ уравнСния (2.1) ΠΈ (2.2) Π»ΠΈΠ½Π΅ΠΉΠ½ΠΎ зависимы. Для этого ΠΈΠ· (2.1) Π²Ρ‹Ρ‡Ρ‚Π΅ΠΌ (2.2) ΠΈ Π²ΠΎΡΠΏΠΎΠ»ΡŒΠ·ΡƒΠ΅ΠΌΡΡ (2.4):


.
Π—Π΄Π΅ΡΡŒ ΠΌΡ‹ Π²Π²Π΅Π»ΠΈ ΠΎΠ±ΠΎΠ·Π½Π°Ρ‡Π΅Π½ΠΈΠ΅ . Π£ΠΌΠ½ΠΎΠΆΠΈΠΌ это ΡƒΡ€Π°Π²Π½Π΅Π½ΠΈΠ΅ скалярно Π½Π° :
.
Π’ ΠΏΡ€Π°Π²ΠΎΠΉ части стоит смСшанноС ΠΏΡ€ΠΎΠΈΠ·Π²Π΅Π΄Π΅Π½ΠΈΠ΅ Π²Π΅ΠΊΡ‚ΠΎΡ€ΠΎΠ², Π² ΠΊΠΎΡ‚ΠΎΡ€ΠΎΠ΅ Π²Ρ…ΠΎΠ΄ΠΈΡ‚ Π΄Π²Π° ΠΎΠ΄ΠΈΠ½Π°ΠΊΠΎΠ²Ρ‹Ρ… Π²Π΅ΠΊΡ‚ΠΎΡ€Π° . ΠŸΠΎΡΡ‚ΠΎΠΌΡƒ ΠΎΠ½ΠΎ Ρ€Π°Π²Π½ΠΎ Π½ΡƒΠ»ΡŽ. Π’ Ρ€Π΅Π·ΡƒΠ»ΡŒΡ‚Π°Ρ‚Π΅ ΠΏΠΎΠ»ΡƒΡ‡Π°Π΅ΠΌ Π»ΠΈΠ½Π΅ΠΉΠ½ΡƒΡŽ Π·Π°Π²ΠΈΡΠΈΠΌΠΎΡΡ‚ΡŒ ΠΌΠ΅ΠΆΠ΄Ρƒ уравнСниями (2.1) ΠΈ (2.2):
.

Π’Ρ€Π΅Ρ‚ΡŒΡ Ρ„ΠΎΡ€ΠΌΠ° условий равновСсия

ВсС Ρ„ΠΎΡ€ΠΌΡ‹ ⇑ Для равновСсия Ρ‚Π²Π΅Ρ€Π΄ΠΎΠ³ΠΎ Ρ‚Π΅Π»Π° Π½Π΅ΠΎΠ±Ρ…ΠΎΠ΄ΠΈΠΌΠΎ ΠΈ достаточно, Ρ‡Ρ‚ΠΎΠ±Ρ‹ вСкторная сумма ΠΌΠΎΠΌΠ΅Π½Ρ‚ΠΎΠ² Π²Π½Π΅ΡˆΠ½ΠΈΡ… сил, Π΄Π΅ΠΉΡΡ‚Π²ΡƒΡŽΡ‰ΠΈΡ… Π½Π° Ρ‚Π΅Π»ΠΎ, ΠΎΡ‚Π½ΠΎΡΠΈΡ‚Π΅Π»ΡŒΠ½ΠΎ ΠΏΡ€ΠΎΠΈΠ·Π²ΠΎΠ»ΡŒΠ½Ρ‹ΠΌ ΠΎΠ±Ρ€Π°Π·ΠΎΠΌ Π²Ρ‹Π±Ρ€Π°Π½Π½Ρ‹Ρ… Ρ‚ΠΎΡ‡Π΅ΠΊ ΠΈ , Π½Π΅ Π»Π΅ΠΆΠ°Ρ‰ΠΈΡ… Π½Π° ΠΎΠ΄Π½ΠΎΠΉ прямой, Ρ€Π°Π²Π½ΡΠ»Π°ΡΡŒ Π½ΡƒΠ»ΡŽ:
(3.1) Β  ;
(3.2) Β  ;
(3.3) Β  ;
(3.4) Β  .

Π”ΠΎΠΊΠ°Π·Π°Ρ‚Π΅Π»ΡŒΡΡ‚Π²ΠΎ

ΠΠ΅ΠΎΠ±Ρ…ΠΎΠ΄ΠΈΠΌΠΎΡΡ‚ΡŒ.

ΠŸΡƒΡΡ‚ΡŒ Ρ‚Π΅Π»ΠΎ находится Π² состоянии равновСсия. Как ΡƒΠΆΠ΅ Π±Ρ‹Π»ΠΎ Π΄ΠΎΠΊΠ°Π·Π°Π½ΠΎ ΠΏΡ€ΠΈ Π²Ρ‹Π²ΠΎΠ΄Π΅ основной Ρ„ΠΎΡ€ΠΌΡ‹ условий равновСсия ⇑, ΠΏΡ€ΠΈ этом выполняСтся условиС (1.2):
(1.2) Β  .
Π’ΠΎΠ·ΡŒΠΌΠ΅ΠΌ ΠΏΡ€ΠΎΠΈΠ·Π²ΠΎΠ»ΡŒΠ½Ρ‹Π΅ Ρ‚ΠΎΡ‡ΠΊΠΈ , ΠΈ , Π½Π΅ Π»Π΅ΠΆΠ°Ρ‰ΠΈΠ΅ Π½Π° ΠΎΠ΄Π½ΠΎΠΉ прямой. ΠŸΠΎΡΠΊΠΎΠ»ΡŒΠΊΡƒ Π² (1.2)Β  C – ΠΏΡ€ΠΎΠΈΠ·Π²ΠΎΠ»ΡŒΠ½Π°Ρ Ρ‚ΠΎΡ‡ΠΊΠ°, Ρ‚ΠΎ Π² качСствС Π½Π΅Π΅ возьмСм ΠΏΠΎΡΠ»Π΅Π΄ΠΎΠ²Π°Ρ‚Π΅Π»ΡŒΠ½ΠΎ Ρ‚ΠΎΡ‡ΠΊΠΈ , ΠΈ . Π’ Ρ€Π΅Π·ΡƒΠ»ΡŒΡ‚Π°Ρ‚Π΅ ΠΏΠΎΠ»ΡƒΡ‡ΠΈΠΌ уравнСния (3.1), (3.2) ΠΈ (3.3):
(3.1) Β  ;
(3.2) Β  ;
(3.3) Β  .
Π­Ρ‚ΠΈ уравнСния Π²Ρ‹ΠΏΠΎΠ»Π½ΡΡŽΡ‚ΡΡ для Π»ΡŽΠ±Ρ‹Ρ… Ρ‚ΠΎΡ‡Π΅ΠΊ , ΠΈ . Π’ Ρ‚ΠΎΠΌ числС ΠΈ для Ρ‚Π΅Ρ…, ΠΊΠΎΡ‚ΠΎΡ€Ρ‹Π΅ Π½Π΅ Π»Π΅ΠΆΠ°Ρ‚ Π½Π° ΠΎΠ΄Π½ΠΎΠΉ прямой:
(3.4) Β  .
ΠΠ΅ΠΎΠ±Ρ…ΠΎΠ΄ΠΈΠΌΠΎΡΡ‚ΡŒ Π΄ΠΎΠΊΠ°Π·Π°Π½Π°.

Π”ΠΎΡΡ‚Π°Ρ‚ΠΎΡ‡Π½ΠΎΡΡ‚ΡŒ.

ΠŸΡƒΡΡ‚ΡŒ Π²Ρ‹ΠΏΠΎΠ»Π½ΡΡŽΡ‚ΡΡ условия (3.1), (3.2), (3.3) ΠΈ (3.4). Π”ΠΎΠΊΠ°ΠΆΠ΅ΠΌ, Ρ‡Ρ‚ΠΎ Ρ‚Π΅Π»ΠΎ Π±ΡƒΠ΄Π΅Ρ‚ Π½Π°Ρ…ΠΎΠ΄ΠΈΡ‚ΡŒΡΡ Π² состоянии равновСсия. Как ΠΈ ΠΏΡ€ΠΈ Π΄ΠΎΠΊΠ°Π·Π°Ρ‚Π΅Π»ΡŒΡΡ‚Π²Π΅ Π²Ρ‚ΠΎΡ€ΠΎΠΉ Ρ„ΠΎΡ€ΠΌΡ‹, Π²ΠΎΡΠΏΠΎΠ»ΡŒΠ·ΡƒΠ΅ΠΌΡΡ Π²Π΅ΠΊΡ‚ΠΎΡ€Π½Ρ‹ΠΌ ΡƒΡ€Π°Π²Π½Π΅Π½ΠΈΠ΅ΠΌ:
(3.5) Β  .
ΠŸΠΎΠ΄ΡΡ‚Π°Π²ΠΈΠΌ Π΅Π³ΠΎ Π² (3.1):

.
ΠŸΠΎΡΠΊΠΎΠ»ΡŒΠΊΡƒ ΠΈΠ· (3.2), Β  , Ρ‚ΠΎ Β  .
ΠžΡ‚ΡΡŽΠ΄Π°
(3.6) Β  ,
Π³Π΄Π΅ – ΠΏΡ€ΠΎΠΈΠ·Π²ΠΎΠ»ΡŒΠ½Π°Ρ постоянная.

Выполняя Ρ‚Π΅ ΠΆΠ΅ дСйствия с Ρ‚ΠΎΡ‡ΠΊΠ°ΠΌΠΈ ΠΈ , Π½Π°ΠΉΠ΄Π΅ΠΌ:
(3.7) Β  ,
Π³Π΄Π΅ – Ρ‚Π°ΠΊΠΆΠ΅ ΠΏΡ€ΠΎΠΈΠ·Π²ΠΎΠ»ΡŒΠ½Π°Ρ постоянная. Бравнивая (3.6) ΠΈ (3.7) ΠΈΠΌΠ΅Π΅ΠΌ:
(3.8) Β  .
ΠŸΠΎΡΠΊΠΎΠ»ΡŒΠΊΡƒ Π²Π΅ΠΊΡ‚ΠΎΡ€Ρ‹ Β  ΠΈ Β  Π½Π΅ ΠΏΠ°Ρ€Π°Π»Π»Π΅Π»ΡŒΠ½Ρ‹, Ρ‚ΠΎ ΡƒΡ€Π°Π²Π½Π΅Π½ΠΈΠ΅ (3.8) ΠΌΠΎΠΆΠ΅Ρ‚ Π²Ρ‹ΠΏΠΎΠ»Π½ΡΡ‚ΡŒΡΡ Ρ‚ΠΎΠ»ΡŒΠΊΠΎ ΠΏΡ€ΠΈ . Π’ΠΎΠ³Π΄Π° ΠΈΠ· (3.6) слСдуСт, Ρ‡Ρ‚ΠΎ .
Для Π΄ΠΎΠΊΠ°Π·Π°Ρ‚Π΅Π»ΡŒΡΡ‚Π²Π° Ρ‚ΠΎΠ³ΠΎ, Ρ‡Ρ‚ΠΎ Β , достаточно ΡƒΠΌΠ½ΠΎΠΆΠΈΡ‚ΡŒ скалярно ΡƒΡ€Π°Π²Π½Π΅Π½ΠΈΠ΅ (3.8) Π½Π° Π²Π΅ΠΊΡ‚ΠΎΡ€, пСрпСндикулярный ΠΈ Π²Π΅ΠΊΡ‚ΠΎΡ€, пСрпСндикулярный .

Если ΠΎΠ±ΠΎΠ·Π½Π°Ρ‡ΠΈΡ‚ΡŒ Ρ‚ΠΎΡ‡ΠΊΡƒ ΠΊΠ°ΠΊ C, Ρ‚ΠΎ (3.1) ΠΏΡ€ΠΈΠΌΠ΅Ρ‚ Π²ΠΈΠ΄:
.

Π˜Ρ‚Π°ΠΊ, ΠΌΡ‹ ΠΏΠΎΠ»ΡƒΡ‡ΠΈΠ»ΠΈ, Ρ‡Ρ‚ΠΎ Ссли Π²Ρ‹ΠΏΠΎΠ»Π½ΡΡŽΡ‚ΡΡ условия (3.1), (3.2) ΠΈ (3.3), Ρ‚ΠΎ Π²Ρ‹ΠΏΠΎΠ»Π½ΡΡŽΡ‚ΡΡ условия (1.1) ΠΈ (1.2):
(1.1) Β  ;
(1.2) Β  .
Как ΠΌΡ‹ ΡƒΠΆΠ΅ Π΄ΠΎΠΊΠ°Π·Π°Π»ΠΈ ΠΏΡ€ΠΈ Π²Ρ‹Π²ΠΎΠ΄Π΅ основной Ρ„ΠΎΡ€ΠΌΡ‹ условий равновСсия ⇑, это ΠΎΠ·Π½Π°Ρ‡Π°Π΅Ρ‚, Ρ‡Ρ‚ΠΎ Ρ‚Π΅Π»ΠΎ находится Π² равновСсии.

Бвойство Π΄ΠΎΠΊΠ°Π·Π°Π½ΠΎ.

Автор: ОлСг ΠžΠ΄ΠΈΠ½Ρ†ΠΎΠ². Β  Β  ΠžΠΏΡƒΠ±Π»ΠΈΠΊΠΎΠ²Π°Π½ΠΎ:

Условия равновСсия ΠΏΡ€ΠΎΠΈΠ·Π²ΠΎΠ»ΡŒΠ½ΠΎΠΉ плоской систСмы сил. Π€ΠΎΡ€ΠΌΡ‹ ΡƒΡ€Π°Π²Π½Π΅Π½ΠΈΠΉ равновСсия

Π‘ΠΎΠ΄Π΅Ρ€ΠΆΠ°Π½ΠΈΠ΅:

Условия равновСсия ΠΏΡ€ΠΎΠΈΠ·Π²ΠΎΠ»ΡŒΠ½ΠΎΠΉ плоской систСмы сил. Π€ΠΎΡ€ΠΌΡ‹ ΡƒΡ€Π°Π²Π½Π΅Π½ΠΈΠΉ равновСсия

  • УсловиС равновСсия ΠΏΡ€ΠΎΠΈΠ·Π²ΠΎΠ»ΡŒΠ½ΠΎΠΉ плоской систСмы сил. Π€ΠΎΡ€ΠΌΠ° Ρ‚Π΅ΠΎΡ€Π΅ΠΌΡ‹ уравнСния равновСсия. Для равновСсия свободного Ρ‚Π²Π΅Ρ€Π΄ΠΎΠ³ΠΎ Ρ‚Π΅Π»Π° ΠΏΠΎΠ΄ дСйствиСм ΠΏΡ€ΠΎΠΈΠ·Π²ΠΎΠ»ΡŒΠ½ΠΎΠΉ плоской силовой систСмы Π³Π»Π°Π²Π½Ρ‹ΠΉ Π²Π΅ΠΊΡ‚ΠΎΡ€ ΠΈ Π³Π»Π°Π²Π½Ρ‹ΠΉ ΠΌΠΎΠΌΠ΅Π½Ρ‚ этой систСмы ΠΎΡ‚Π½ΠΎΡΠΈΡ‚Π΅Π»ΡŒΠ½ΠΎ ΠΏΡ€ΠΎΠΈΠ·Π²ΠΎΠ»ΡŒΠ½ΠΎ Π²Ρ‹Π±Ρ€Π°Π½Π½ΠΎΠ³ΠΎ Ρ†Π΅Π½Ρ‚Ρ€Π° (Ρ‚ΠΎΡ‡ΠΊΠΈ) GL=Β°, MS=0. (4.9)Π½ΡƒΠΆΠ½Π° сСртификация.

ΠŸΡ€ΠΈΠ²Π΅Π΄Π΅ΠΌ ΠΏΠ»ΠΎΡΠΊΡƒΡŽ систСму ΠΏΡ€ΠΎΠΈΠ·Π²ΠΎΠ»ΡŒΠ½Ρ‹Ρ… сил с Π½Π΅ΠΏΠΎΠ΄Π²ΠΈΠΆΠ½Ρ‹ΠΌ Ρ‚Π΅Π»ΠΎΠΌ Π² равновСсии, Π³Π»Π°Π²Π½Ρ‹ΠΉ Π²Π΅ΠΊΡ‚ΠΎΡ€ ll, ΠΏΡ€ΠΈΠ»ΠΎΠΆΠ΅Π½Π½Ρ‹ΠΉ ΠΊ Ρ†Π΅Π½Ρ‚Ρ€Ρƒ C, ΠΈ ΠΏΠ°Ρ€Π½Ρ‹ΠΉ (F, F’) ΠΌΠΎΠΌΠ΅Π½Ρ‚ массы массы, Ρ€Π°Π²Π½Ρ‹ΠΉ основному Π²Ρ€Π΅ΠΌΠ΅Π½ΠΈ систСмы. Для Ρ‚ΠΎΠ³ΠΎ Ρ‡Ρ‚ΠΎΠ±Ρ‹ систСма сил сходимости, ΠΏΡ€ΠΈΠ»ΠΎΠΆΠ΅Π½Π½Ρ‹Ρ… ΠΊ Ρ†Π΅Π½Ρ‚Ρ€Ρƒ Π‘, Π±Ρ‹Π»Π° сбалансирована, Π΄ΠΎΠ»ΠΆΠ½ΠΎ Π±Ρ‹Ρ‚ΡŒ Π²Ρ‹ΠΏΠΎΠ»Π½Π΅Π½ΠΎ условиС RGL=0. Для Ρ‚ΠΎΠ³ΠΎ Ρ‡Ρ‚ΠΎΠ±Ρ‹ сумма ΠΌΠΎΠΌΠ΅Π½Ρ‚ΠΎΠ² присоСдинСнной ΠΏΠ°Ρ€Ρ‹ Π±Ρ‹Π»Π° Ρ€Π°Π²Π½Π° Π½ΡƒΠ»ΡŽ, Π½Π΅ΠΎΠ±Ρ…ΠΎΠ΄ΠΈΠΌΠΎ Π²Ρ‹ΠΏΠΎΠ»Π½ΠΈΡ‚ΡŒ условиС MS=0.

Π’Π°ΠΊΠΈΠΌ ΠΎΠ±Ρ€Π°Π·ΠΎΠΌ, для равновСсия плоской систСмы любой силы Π½Π΅ΠΎΠ±Ρ…ΠΎΠ΄ΠΈΠΌΠΎ ΠΎΠ΄Π½ΠΎΠ²Ρ€Π΅ΠΌΠ΅Π½Π½ΠΎΠ΅ Π›ΡŽΠ΄ΠΌΠΈΠ»Π° Π€ΠΈΡ€ΠΌΠ°Π»ΡŒ

Π²Ρ‹ΠΏΠΎΠ»Π½Π΅Π½ΠΈΠ΅ условий (4.)=0-(4-11) * =1k=l K = L ΡƒΡ€Π°Π²Π½Π΅Π½ΠΈΠ΅(4.11) прСдставляСт собой ΡƒΡ€Π°Π²Π½Π΅Π½ΠΈΠ΅ равновСсия свободного Ρ‚Π²Π΅Ρ€Π΄ΠΎΠ³ΠΎ Ρ‚Π΅Π»Π° ΠΏΠΎΠ΄ дСйствиСм ΠΏΡ€ΠΎΠΈΠ·Π²ΠΎΠ»ΡŒΠ½ΠΎΠΉ силовой плоской систСмы. Π’Π°ΠΊΠΈΠΌ ΠΎΠ±Ρ€Π°Π·ΠΎΠΌ, для равновСсия свободного Ρ‚Π²Π΅Ρ€Π΄ΠΎΠ³ΠΎ Ρ‚Π΅Π»Π° ΠΏΠΎΠ΄ дСйствиСм ΠΏΡ€ΠΎΠΈΠ·Π²ΠΎΠ»ΡŒΠ½ΠΎΠΉ плоской систСмы сил сумма ΠΏΡ€ΠΎΠ΅ΠΊΡ†ΠΈΠΉ всСх сил этой систСмы для ΠΊΠ°ΠΆΠ΄ΠΎΠΉ ΠΈΠ· Π΄Π²ΡƒΡ… осСй

  • ΠΊΠΎΠΎΡ€Π΄ΠΈΠ½Π°Ρ‚ Π΄ΠΎΠ»ΠΆΠ½Π° Π±Ρ‹Ρ‚ΡŒ Ρ€Π°Π²Π½Π°. Вторая Ρ„ΠΎΡ€ΠΌΠ° уравнСния равновСсия. Π§Ρ‚ΠΎ касаСтся равновСсия свободных Ρ‚Π²Π΅Ρ€Π΄Ρ‹Ρ… Ρ‚Π΅Π» ΠΏΠΎΠ΄ дСйствиСм сил любой плоской систСмы, Ρ‚ΠΎ сумма ΠΏΡ€ΠΎΠ΅ΠΊΡ†ΠΈΠΉ всСх сил этой систСмы Π½Π° ΠΏΡ€ΠΎΠΈΠ·Π²ΠΎΠ»ΡŒΠ½ΠΎ Π²Ρ‹Π±Ρ€Π°Π½Π½ΡƒΡŽ ось Ρ€Π°Π²Π½Π° Π½ΡƒΠ»ΡŽ, ΠΈ эти силы Ρ€Π°Π²Π½Ρ‹ Π½ΡƒΠ»ΡŽ. 2W)=Β°-(412) /Π³=1Π»=1 1 Π³=1 Π”ΠΎΠΊΠ°Π·Π°Ρ‚Π΅Π»ΡŒΡΡ‚Π²ΠΎ нСобходимости. ΠŸΠΎΡΠΊΠΎΠ»ΡŒΠΊΡƒ Ρ‚Π΅Π»ΠΎ находится Π² равновСсии, Ρ‚ΠΎ сумма ΠΏΡ€ΠΎΠ΅ΠΊΡ†ΠΈΠΉ всСх сил Π½Π° Π»ΡŽΠ±ΡƒΡŽ ось ΠΈ сумма ΠΌΠΎΠΌΠ΅Π½Ρ‚ΠΎΠ² для любой Ρ‚ΠΎΡ‡ΠΊΠΈ плоскости, Π² ΠΊΠΎΡ‚ΠΎΡ€ΠΎΠΉ сущСствуСт систСма Π΄Π°Π½Π½ΠΎΠΉ силы. Π”ΠΎΠΊΠ°Π·Π°Ρ‚Π΅Π»ΡŒΡΡ‚Π²ΠΎ адСкватности.

Π”Π°Π²Π°ΠΉΡ‚Π΅ Π΄ΠΎΠΊΠ°ΠΆΠ΅ΠΌ это ΡƒΡ‚Π²Π΅Ρ€ΠΆΠ΄Π΅Π½ΠΈΠ΅ ΠΏΡ€ΠΎΡ‚ΠΈΠ²ΠΎΠΏΠΎΠ»ΠΎΠΆΠ½Ρ‹ΠΌ ΠΎΠ±Ρ€Π°Π·ΠΎΠΌ. Если Π²Ρ‹ΠΏΠΎΠ»Π½ΡΡŽΡ‚ΡΡ Ρ‚ΠΎΠ»ΡŒΠΊΠΎ Π΄Π²Π° условия для Π΄Π°Π½Π½ΠΎΠΉ систСмы силы (4.12). Π’ΠΎ Π΅ΡΡ‚ΡŒ n_n_2A I o (F/e)=0, 2m in (Fk)=0, систСма Ρ‚Π°ΠΊΠΈΡ… сил co-k=\k=I гласная ΠΏ.)=0. Но с L=1 k=l Π² нашСм случаС ось x Π½Π΅ пСрпСндикулярна прямой S, поэтому ΠΏΠ΅Ρ€Π²ΠΎΠ΅ Π²Ρ‹Ρ€Π°ΠΆΠ΅Π½ΠΈΠ΅ Π² Π€ΠΎΡ€ΠΌΡƒΠ»Π΅ (4.12) выполняСтся Ρ‚ΠΎΠ»ΡŒΠΊΠΎ Π² случаС Vrav=0.

Π’Ρ€Π΅Ρ‚ΡŒΡ Ρ„ΠΎΡ€ΠΌΠ° уравнСния равновСсия. Для равновСсия свободного Ρ‚Π²Π΅Ρ€Π΄ΠΎΠ³ΠΎ Ρ‚Π΅Π»Π° ΠΏΠΎΠ΄ дСйствиСм силы любой плоской систСмы Ρ‚Π΅ΠΎΡ€Π΅ΠΌΠ° гласит, Ρ‡Ρ‚ΠΎ Π›ΡŽΠ΄ΠΌΠΈΠ»Π° Π€ΠΈΡ€ΠΌΠ°Π»ΡŒ

алгСбраичСская сумма всСх силовых ΠΌΠΎΠΌΠ΅Π½Ρ‚ΠΎΠ² этой систСмы Ρ€Π°Π²Π½Π° Π½ΡƒΠ»ΡŽ, сумма всСх силовых ΠΌΠΎΠΌΠ΅Π½Ρ‚ΠΎΠ² систСмы сил, ΠΈΠ»ΠΈ сумма всСх силовых ΠΌΠΎΠΌΠ΅Π½Ρ‚ΠΎΠ² систСмы сил Π² любой Ρ‚ΠΎΡ‡ΠΊΠ΅ я Π½Π΅ ΡƒΠ²Π΅Ρ€Π΅Π½. Π”ΠΎΠΊΠ°Π·Π°Ρ‚Π΅Π»ΡŒΡΡ‚Π²ΠΎ адСкватности. Для Ρ‚Ρ€Π΅Ρ… условий (4.13)Π²Ρ‹ΠΏΠΎΠ»Π½ΡΡŽΡ‚ΡΡ Ρ‚ΠΎΠ»ΡŒΠΊΠΎ ΠΏΠ΅Ρ€Π²Ρ‹Π΅ Π΄Π²Π°,ΠΈ всС Ρ‚Ρ€ΠΈ Ρ‚ΠΎΡ‡ΠΊΠΈ O, B ΠΈ C находятся Π½Π° ΠΎΠ΄Π½ΠΎΠΉ прямой. Π’ΠΎΠ³Π΄Π°, исходя ΠΈΠ· ΡƒΡ€Π°Π²Π½Π΅Π½ΠΈΠΉ p-P-2Afo (F) fe)=0 ΠΈ 2L1B (/7K)=0, Ρ€Π΅Π·ΡƒΠ»ΡŒΡ‚Π°Ρ‚ плоской систСмы сил Π΄ΠΎΠ»ΠΆΠ΅Π½ Π±Ρ‹Ρ‚ΡŒ Ρ€Π°Π²Π΅Π½ Π½ΡƒΠ»ΡŽ, Ρ‚Π°ΠΊ ΠΊΠ°ΠΊ Π² соотвСтствии с k=i k=iΒ§4.3 систСма Ρ‚Π°ΠΊΠΈΡ… сил Π½Π΅ становится равновСсной, ΠΈ Ρ€Π΅Π·ΡƒΠ»ΡŒΡ‚ΠΈΡ€ΡƒΡŽΡ‰ΠΈΠΌΠΈ линиями ΡΠ²Π»ΡΡŽΡ‚ΡΡ Π»ΠΈΠ½ΠΈΠΈ S, Π° Π½Π΅ Ρ‚ΠΎΡ‡ΠΊΠΈ O, B ΠΈ C находятся Π½Π° ΠΎΠ΄Π½ΠΈΡ… ΠΈ Ρ‚Π΅Ρ… ΠΆΠ΅ линиях. Π’Π°ΠΊΠΈΠΌ ΠΎΠ±Ρ€Π°Π·ΠΎΠΌ,

ΠΏΡ€ΠΈ Π²Ρ‹ΠΏΠΎΠ»Π½Π΅Π½ΠΈΠΈ всСх Ρ‚Ρ€Π΅Ρ… условий (4.)=Β°. 2L4V (L)=Β° β€” <4-15 A β€˜ =1 Π’Π°ΠΊΠΈΠΌ ΠΎΠ±Ρ€Π°Π·ΠΎΠΌ, для равновСсия свободного Ρ‚Π²Π΅Ρ€Π΄ΠΎΠ³ΠΎ Ρ‚Π΅Π»Π° ΠΏΠΎΠ΄ дСйствиСм плоской систСмы ΠΏΠ°Ρ€Π°Π»Π»Π΅Π»ΡŒΠ½Ρ‹Ρ… сил ΡΡƒΡ‰Π΅ΡΡ‚Π²ΡƒΡŽΡ‚ Π΄Π²Π΅ ΠΏΡ€ΠΎΠΈΠ·Π²ΠΎΠ»ΡŒΠ½ΠΎ Π²Ρ‹Π±Ρ€Π°Π½Π½Ρ‹Π΅ Ρ‚ΠΎΡ‡ΠΊΠΈ, ΠΊΠΎΡ‚ΠΎΡ€Ρ‹Π΅ Π½Π΅ ΡΡƒΡ‰Π΅ΡΡ‚Π²ΡƒΡŽΡ‚ Π½Π° ΠΎΠ΄Π½ΠΎΠΉ прямой, ΠΏΠ°Ρ€Π°Π»Π»Π΅Π»ΡŒΠ½ΠΎΠΉ силовой Π»ΠΈΠ½ΠΈΠΈ.

Π‘ΠΌΠΎΡ‚Ρ€ΠΈΡ‚Π΅ Ρ‚Π°ΠΊΠΆΠ΅:

ΠŸΡ€Π΅Π΄ΠΌΠ΅Ρ‚ тСхничСская ΠΌΠ΅Ρ…Π°Π½ΠΈΠΊΠ°Β 

Условия ΠΈ уравнСния равновСсия Ρ‚Π²Π΅Ρ€Π΄ΠΎΠ³ΠΎ Ρ‚Π΅Π»Π°

ΠŸΡ€ΠΈΠ²Π΅Ρ‚! МСня Π·ΠΎΠ²ΡƒΡ‚ ΠšΠΎΠ½ΡΡ‚Π°Π½Ρ‚ΠΈΠ½ Π’Π°Π²ΠΈΠ»ΠΎΠ² ΠΈ Π² этой ΡΡ‚Π°Ρ‚ΡŒΠ΅ я рассказу ΠΎΠ± условиях, ΠΏΡ€ΠΈ ΠΊΠΎΡ‚ΠΎΡ€Ρ‹Ρ… любая систСма сил, Ρ‚Π²Π΅Ρ€Π΄ΠΎΠ΅ Ρ‚Π΅Π»ΠΎ, элСмСнт конструкции ΠΈΠ»ΠΈ конструкция Π² Ρ†Π΅Π»ΠΎΠΌ находится Π² равновСсии. А Ρ‚Π°ΠΊΠΆΠ΅ Π½Π°ΠΏΠΈΡˆΡƒ ΠΏΡ€ΠΎ уравнСния равновСсия, ΠΊΠΎΡ‚ΠΎΡ€Ρ‹Π΅ Π²Ρ‹Ρ‚Π΅ΠΊΠ°ΡŽΡ‚ ΠΈΠ· этих условий. Рассмотрим Ρ‚Ρ€ΠΈ основныС Ρ„ΠΎΡ€ΠΌΡ‹ этих ΡƒΡ€Π°Π²Π½Π΅Π½ΠΈΠΉ.

Условия равновСсия ΠΏΡ€ΠΎΠΈΠ·Π²ΠΎΠ»ΡŒΠ½ΠΎΠΉ систСмы сил

Π•Ρ‰Π΅ ΠΡŒΡŽΡ‚ΠΎΠ½ Π³ΠΎΠ²ΠΎΡ€ΠΈΠ», Ρ‡Ρ‚ΠΎ Ссли гСомСтричСская сумма сил, Π΄Π΅ΠΉΡΡ‚Π²ΡƒΡŽΡ‰Π°Ρ Π½Π° Ρ‚Π΅Π»ΠΎ, Ρ€Π°Π²Π½Π° Π½ΡƒΠ»ΡŽ, Ρ‚ΠΎ Ρ‚Π΅Π»ΠΎ:

  • Π»ΠΈΠ±ΠΎ находится Π² состоянии покоя;
  • Π»ΠΈΠ±ΠΎ двиТСтся Ρ€Π°Π²Π½ΠΎΠΌΠ΅Ρ€Π½ΠΎ прямолинСйно.

Из тСорСтичСской ΠΌΠ΅Ρ…Π°Π½ΠΈΠΊΠΈ извСстно, Ρ‡Ρ‚ΠΎ дСйствиС Π½Π΅ΡΠΊΠΎΠ»ΡŒΠΊΠΈΡ… сил, просуммировав, ΠΌΠΎΠΆΠ½ΠΎ Π·Π°ΠΌΠ΅Π½ΠΈΡ‚ΡŒ Ρ€Π°Π²Π½ΠΎΠ΄Π΅ΠΉΡΡ‚Π²ΡƒΡŽΡ‰Π΅ΠΉ силой:

\[ \bar { { R }_{ 1 } } +\bar { { R }_{ 2 } } +\bar { { R }_{ 3 } } +\bar { { R }_{ n } } =\bar { R } \]

Π’ΠΎΠ³Π΄Π° ΠΎΠ±ΡΠ·Π°Ρ‚Π΅Π»ΡŒΠ½ΠΎΠ΅ условиС равновСсия ΠΌΠΎΠΆΠ½ΠΎ Π·Π°ΠΏΠΈΡΠ°Ρ‚ΡŒ Ρ‚Π°ΠΊ:

\[ \bar { R } =0 \]

Однако для ΠΏΠΎΠ»Π½ΠΎΠ³ΠΎ равновСсия, часто, этого условия нСдостаточно, Ссли Ρ‚Π΅Π»ΠΎ ΠΈΠΌΠ΅Π΅Ρ‚ Π²ΠΎΠ·ΠΌΠΎΠΆΠ½ΠΎΡΡ‚ΡŒ Π²Ρ€Π°Ρ‰Π°Ρ‚ΡŒΡΡ ΠΎΡ‚Π½ΠΎΡΠΈΡ‚Π΅Π»ΡŒΠ½ΠΎ ΠΊΠ°ΠΊΠΎΠΉ-Ρ‚ΠΎ Ρ‚ΠΎΡ‡ΠΊΠΈ ΠΈΠ»ΠΈ оси, Ρ‚ΠΎ для равновСсия Ρ‚Π°ΠΊΠΎΠΉ систСмы, Π½Π΅ΠΎΠ±Ρ…ΠΎΠ΄ΠΈΠΌΠΎ, Ρ‡Ρ‚ΠΎΠ±Ρ‹ Π²Ρ‹ΠΏΠΎΠ»Π½ΡΠ»ΠΎΡΡŒ условиС:

\[ \bar { M } =0 \]

Π³Π΄Π΅ MΒ β€”Β Π³Π»Π°Π²Π½Ρ‹Π΅ ΠΌΠΎΠΌΠ΅Π½Ρ‚ систСмы, ΠΊΠΎΡ‚ΠΎΡ€Ρ‹ΠΉ эквивалСнтСн суммС ΠΌΠΎΠΌΠ΅Π½Ρ‚ΠΎΠ² систСмы ΠΎΡ‚Π½ΠΎΡΠΈΡ‚Π΅Π»ΡŒΠ½ΠΎ Π½Π΅ΠΊΠΎΡ‚ΠΎΡ€ΠΎΠ³ΠΎ Ρ†Π΅Π½Ρ‚Ρ€Π°.

Условия равновСсия плоской систСмы сил

Π’Ρ‹ΡˆΠ΅ описанныС условия ΠΎΠ·Π½Π°Ρ‡Π°ΡŽΡ‚, Ρ‡Ρ‚ΠΎ систСма Π±ΡƒΠ΄Π΅Ρ‚ находится Π² равновСсии, ΠΊΠΎΠ³Π΄Π° всС силы, Π΄Π΅ΠΉΡΡ‚Π²ΡƒΡŽΡ‰ΠΈΠ΅ Π½Π° систСму, Π±ΡƒΠ΄ΡƒΡ‚ Π²Π·Π°ΠΈΠΌΠ½ΠΎ ΡƒΡ€Π°Π²Π½ΠΎΠ²Π΅ΡˆΠΈΠ²Π°Ρ‚ΡŒΡΡ ΠΈ ΠΌΠΎΠΌΠ΅Π½Ρ‚ ΠΎΡ‚Π½ΠΎΡΠΈΡ‚Π΅Π»ΡŒΠ½ΠΎ любой ΠΏΡ€ΠΎΠΈΠ·Π²ΠΎΠ»ΡŒΠ½ΠΎΠΉ Ρ‚ΠΎΡ‡ΠΊΠΈ Π±ΡƒΠ΄Π΅Ρ‚ Ρ€Π°Π²Π΅Π½ Π½ΡƒΠ»ΡŽ, ΠΎΡ‚ΡΡŽΠ΄Π° Π²Ρ‹Ρ‚Π΅ΠΊΠ°Π΅Ρ‚ пСрвая ΠΈ основная Ρ„ΠΎΡ€ΠΌΠ°

условий равновСсия для плоской систСмы сил:

\[ \begin{cases} { Ξ£M }_{ A }=0 \\ { Ξ£F }_{ kx }=0 \\ { Ξ£F }_{ ky }=0 \end{cases} \]

Вторая Ρ„ΠΎΡ€ΠΌΠ° условий равновСсия записываСтся ΡΠ»Π΅Π΄ΡƒΡŽΡ‰ΠΈΠΌ ΠΎΠ±Ρ€Π°Π·ΠΎΠΌ:

\[ \begin{cases} { Ξ£M }_{ A }=0 \\ { Ξ£M }_{ B }=0 \\ { Ξ£F }_{ ky }=0 \end{cases} \]

Из Π΄Π°Π½Π½ΠΎΠ³ΠΎ условия слСдуСт, Ρ‡Ρ‚ΠΎ для равновСсия систСмы достаточно равСнство Π½ΡƒΠ»ΡŽ суммы ΠΌΠΎΠΌΠ΅Π½Ρ‚ΠΎΠ² ΠΎΡ‚Π½ΠΎΡΠΈΡ‚Π΅Π»ΡŒΠ½ΠΎ Π΄Π²ΡƒΡ… Ρ‚ΠΎΡ‡Π΅ΠΊ (A ΠΈ B), Π° Ρ‚Π°ΠΊΠΆΠ΅ суммы ΠΏΡ€ΠΎΠ΅ΠΊΡ†ΠΈΠΉ всСх сил ΠΎΡ‚Π½ΠΎΡΠΈΡ‚Π΅Π»ΡŒΠ½ΠΎ Π½Π΅ΠΊΠΎΡ‚ΠΎΡ€ΠΎΠΉ оси.

Π’Π°ΠΆΠ½ΠΎ! Ось Π½Π΅ Π΄ΠΎΠ»ΠΆΠ½Π° Π±Ρ‹Ρ‚ΡŒ пСрпСндикулярна прямой AB.

И, Π½Π°ΠΊΠΎΠ½Π΅Ρ†, Ρ‚Ρ€Π΅Ρ‚ΡŒΡ Ρ„ΠΎΡ€ΠΌΠ° условий равновСсия выглядит Ρ‚Π°ΠΊ:

\[ \begin{cases} { Ξ£M }_{ A }=0 \\ { Ξ£M }_{ B }=0 \\ { Ξ£M }_{ Π‘ }=0 \end{cases} \]

Из Π΄Π°Π½Π½ΠΎΠΉ систСмы ΡƒΡ€Π°Π²Π½Π΅Π½ΠΈΠΉ слСдуСт, Ρ‡Ρ‚ΠΎ для равновСсия систСмы достаточно равСнства Π½ΡƒΠ»ΡŽ суммы ΠΌΠΎΠΌΠ΅Π½Ρ‚ΠΎΠ² ΠΎΡ‚Π½ΠΎΡΠΈΡ‚Π΅Π»ΡŒΠ½ΠΎ Ρ‚Ρ€Π΅Ρ… Ρ‚ΠΎΡ‡Π΅ΠΊ.

Π’Π°ΠΆΠ½ΠΎ! Π’ΠΎΡ‡ΠΊΠΈ, ΠΎΡ‚Π½ΠΎΡΠΈΡ‚Π΅Π»ΡŒΠ½ΠΎ ΠΊΠΎΡ‚ΠΎΡ€Ρ‹Ρ… Π·Π°ΠΏΠΈΡΡ‹Π²Π°ΡŽΡ‚ΡΡ уравнСния Π½Π΅ Π΄ΠΎΠ»ΠΆΠ½Ρ‹ Π»Π΅ΠΆΠ°Ρ‚ΡŒ Π½Π° ΠΎΠ΄Π½ΠΎΠΉ прямой.

УравнСния равновСсия для плоской систСмы сил

Рассмотрим Π½Π° ΠΏΡ€ΠΈΠΌΠ΅Ρ€Π΅ плоской Π±Π°Π»ΠΊΠΈ, ΠΊΠ°ΠΊ Π·Π°ΠΏΠΈΡΡ‹Π²Π°ΡŽΡ‚ΡΡ уравнСния равновСсия. Π˜ΡΠΏΠΎΠ»ΡŒΠ·ΠΎΠ²Π°Ρ‚ΡŒ Π±ΡƒΠ΄Π΅Ρ‚ ΠΊΠ»Π°ΡΡΠΈΡ‡Π΅ΡΠΊΡƒΡŽ (ΠΏΠ΅Ρ€Π²ΡƒΡŽ) Ρ„ΠΎΡ€ΠΌΡƒ условия равновСсия:

\[ \begin{cases} { Ξ£M }_{ A }=0 \\ { Ξ£F }_{ kx }=0 \\ { Ξ£F }_{ ky }=0 \end{cases} \]

Π‘ΡƒΠΌΠΌΠ° ΠΌΠΎΠΌΠ΅Π½Ρ‚ΠΎΠ² ΠΎΡ‚Π½ΠΎΡΠΈΡ‚Π΅Π»ΡŒΠ½ΠΎ Ρ‚ΠΎΡ‡ΠΊΠΈ A:

\[ { Ξ£M }_{ A }=-q\cdot 4\cdot 4-M+{ R }_{ B }\cdot 8=0 \]

Π‘ΡƒΠΌΠΌΠ° ΠΏΡ€ΠΎΠ΅ΠΊΡ†ΠΈΠΉ всСх сил Π½Π° Π²Π΅Ρ€Ρ‚ΠΈΠΊΠ°Π»ΡŒΠ½ΡƒΡŽ ось (y):

\[ { Ξ£F }_{ ky }=-q\cdot 4+{ R }_{ A }+{ R }_{ B }-F=0 \]

Π‘ΡƒΠΌΠΌΠ° ΠΏΡ€ΠΎΠ΅ΠΊΡ†ΠΈΠΉ всСх сил Π½Π° Π³ΠΎΡ€ΠΈΠ·ΠΎΠ½Ρ‚Π°Π»ΡŒΠ½ΡƒΡŽ ось(x):

\[ { Ξ£F }_{ kx }={ H }_{ A }=0 \]

УсловиС равновСсия пространствСнной систСмы сил

Для пространствСнной систСмы сил условиС равновСсиС выглядит Π²ΠΎΡ‚ Ρ‚Π°ΠΊ:

\[ \begin{cases} \begin{matrix} { Ξ£F }_{ kx }=0 \\ { Ξ£F }_{ ky }=0 \\ { Ξ£F }_{ kz }=0 \end{matrix} \\ \begin{matrix} { Ξ£M }_{ x }=0 \\ { Ξ£M }_{ y }=0 \\ { Ξ£M }_{ z }=0 \end{matrix} \end{cases} \]

Π’Π°ΠΊΠΈΠΌ ΠΎΠ±Ρ€Π°Π·ΠΎΠΌ, пространствСнная систСма Π±ΡƒΠ΄Π΅Ρ‚ Π½Π°Ρ…ΠΎΠ΄ΠΈΡ‚ΡŒΡΡ Π² равновСсии, Ссли суммы ΠΏΡ€ΠΎΠ΅ΠΊΡ†ΠΈΠΉ сил Π½Π° ΠΊΠΎΠΎΡ€Π΄ΠΈΠ½Π°Ρ‚Π½Ρ‹Π΅ оси, Π° Ρ‚Π°ΠΊΠΆΠ΅ суммы ΠΌΠΎΠΌΠ΅Π½Ρ‚ΠΎΠ² ΠΎΡ‚Π½ΠΎΡΠΈΡ‚Π΅Π»ΡŒΠ½ΠΎ осСй Π±ΡƒΠ΄ΡƒΡ‚ Ρ€Π°Π²Π½Ρ‹ Π½ΡƒΠ»ΡŽ.

УравнСния равновСсия для пространствСнной систСмы сил

Π’ качСствС ΠΏΡ€ΠΈΠΌΠ΅Ρ€Π° рассмотрим ΠΏΡ€ΠΎΡΡ‚Ρ€Π°Π½ΡΡ‚Π²Π΅Π½Π½ΡƒΡŽ Ρ€Π°ΠΌΡƒ, Π·Π°ΠΊΡ€ΡƒΠΆΠ΅Π½Π½ΡƒΡŽ сосрСдоточСнными силами. Боставим для Π½Π΅Π΅ ΡˆΠ΅ΡΡ‚ΡŒ ΡƒΡ€Π°Π²Π½Π΅Π½ΠΈΠΉ равновСсия:

\[ { Ξ£F }_{ kx }=F=0 \]

\[ { Ξ£F }_{ ky }=P=0 \]

\[ { Ξ£F }_{ kz }=T-G=0 \]

\[ { Ξ£M }_{ x }=-T\cdot b+G\cdot b=0 \]

\[ { Ξ£M }_{ y }=-T\cdot a+G\cdot (a+c)=0 \]

\[ { Ξ£M }_{ z }=P\cdot a=0 \]

ΠŸΡ€ΠΎΡΡ‚Ρ€Π°Π½ΡΡ‚Π²Π΅Π½Π½Π°Ρ систСма сил. Π’Π΅ΠΎΡ€Π΅ΠΌΠ° Π’Π°Ρ€ΠΈΠ½ΡŒΠΎΠ½Π°.

ΠŸΡ€ΠΎΡΡ‚Ρ€Π°Π½ΡΡ‚Π²Π΅Π½Π½Π°Ρ систСма сил

ο»Ώ

ΠŸΡ€ΠΎΡΡ‚Ρ€Π°Π½ΡΡ‚Π²Π΅Π½Π½Π°Ρ систСма сходящихся сил

БистСма сил, Π»ΠΈΠ½ΠΈΠΈ дСйствия ΠΊΠΎΡ‚ΠΎΡ€Ρ‹Ρ… располоТСны Π² Ρ€Π°Π·Π»ΠΈΡ‡Π½Ρ‹Ρ… плоскостях, называСтся пространствСнной систСмой сил.

ΠŸΡ€ΠΎΡΡ‚Ρ€Π°Π½ΡΡ‚Π²Π΅Π½Π½Π°Ρ систСма сил называСтся сходящСйся, Ссли Π»ΠΈΠ½ΠΈΠΈ дСйствия всСх сил систСмы ΠΏΠ΅Ρ€Π΅ΡΠ΅ΠΊΠ°ΡŽΡ‚ΡΡ Π² ΠΎΠ΄Π½ΠΎΠΉ Ρ‚ΠΎΡ‡ΠΊΠ΅.

Π’Π΅ΠΎΡ€Π΅ΠΌΠ°: пространствСнная систСма сходящихся сил эквивалСнтна Ρ€Π°Π²Π½ΠΎΠ΄Π΅ΠΉΡΡ‚Π²ΡƒΡŽΡ‰Π΅ΠΉ, которая Ρ€Π°Π²Π½Π° Π²Π΅ΠΊΡ‚ΠΎΡ€Π½ΠΎΠΉ суммС этих сил; линия дСйствия Ρ€Π°Π²Π½ΠΎΠ΄Π΅ΠΉΡΡ‚Π²ΡƒΡŽΡ‰Π΅ΠΉ ΠΏΡ€ΠΎΡ…ΠΎΠ΄ΠΈΡ‚ Ρ‡Π΅Ρ€Π΅Π· Ρ‚ΠΎΡ‡ΠΊΡƒ пСрСсСчСния Π»ΠΈΠ½ΠΈΠΉ дСйствия ΡΠΎΡΡ‚Π°Π²Π»ΡΡŽΡ‰ΠΈΡ… сил.

ΠŸΡƒΡΡ‚ΡŒ Π΄Π°Π½Π° пространствСнная систСма n сходящихся сил (F1, F2, F3,….Fn). На основании слСдствия ΠΈΠ· аксиом III ΠΈ IV пСрСнСсСм всС силы систСмы вдоль Π»ΠΈΠ½ΠΈΠΉ дСйствия Π² Ρ‚ΠΎΡ‡ΠΊΡƒ ΠΈΡ… пСрСсСчСния. Π—Π°Ρ‚Π΅ΠΌ Π½Π° основании аксиомы ΠΏΠ°Ρ€Π°Π»Π»Π΅Π»ΠΎΠ³Ρ€Π°ΠΌΠΌΠ° ΠΏΠΎΡΠ»Π΅Π΄ΠΎΠ²Π°Ρ‚Π΅Π»ΡŒΠ½ΠΎ слоТим всС силы ΠΈ ΠΏΠΎΠ»ΡƒΡ‡ΠΈΠΌ ΠΈΡ… Ρ€Π°Π²Π½ΠΎΠ΄Π΅ΠΉΡΡ‚Π²ΡƒΡŽΡ‰ΡƒΡŽ:

FΞ£ = F1 + F2 + F3 + ….+ Fn, Β Β Β ΠΈΠ»ΠΈ Β Β Β FΞ£ = Ξ£Fi.

Π‘ΠΈΠ»ΠΎΠ²ΠΎΠΉ ΠΌΠ½ΠΎΠ³ΠΎΡƒΠ³ΠΎΠ»ΡŒΠ½ΠΈΠΊ пространствСнной систСмы сил Π½Π΅ Π»Π΅ΠΆΠΈΡ‚ Π² ΠΎΠ΄Π½ΠΎΠΉ плоскости, поэтому гСомСтричСский ΠΈ графичСский способы нахоТдСния Ρ€Π°Π²Π½ΠΎΠ΄Π΅ΠΉΡΡ‚Π²ΡƒΡŽΡ‰Π΅ΠΉ пространствСнной систСмы сходящихся сил Π½Π΅ΠΏΡ€ΠΈΠ΅ΠΌΠ»Π΅ΠΌΡ‹, Π° примСняСтся Ρ‚ΠΎΠ»ΡŒΠΊΠΎ аналитичСский способ (ΠΌΠ΅Ρ‚ΠΎΠ΄ ΠΏΡ€ΠΎΠ΅ΠΊΡ†ΠΈΠΉ).

ΠŸΡ€ΠΎΠ΅ΠΊΡ†ΠΈΡ силы Π½Π° ось Π² пространствС находится ΠΏΠΎ ΠΏΡ€ΠΎΠ΅Ρ†ΠΈΡ€ΡƒΡŽΡ‰ΠΈΠΌ пСрпСндикулярам, ΠΈ ΠΌΠΎΠΆΠ΅Ρ‚ Π±Ρ‹Ρ‚ΡŒ ΠΎΠΏΡ€Π΅Π΄Π΅Π»Π΅Π½Π° ΠΏΡ€ΠΈ ΠΏΠΎΠΌΠΎΡ‰ΠΈ тригономСтричСских Ρ„ΡƒΠ½ΠΊΡ†ΠΈΠΉ. ΠŸΡ€ΠΈ ΠΎΠΏΡ€Π΅Π΄Π΅Π»Π΅Π½ΠΈΠΈ ΠΏΡ€ΠΎΠ΅ΠΊΡ†ΠΈΠΉ сил пространствСнной систСмы потрСбуСтся систСма ΠΊΠΎΠΎΡ€Π΄ΠΈΠ½Π°Ρ‚ с осями X, Y, Z, ΠΏΠΎΡΠΊΠΎΠ»ΡŒΠΊΡƒ силы систСмы Π½Π΅ Ρ€Π°ΡΠΏΠΎΠ»Π°Π³Π°ΡŽΡ‚ΡΡ Π² ΠΎΠ΄Π½ΠΎΠΉ плоскости.

ΠŸΡ€Π°Π²ΠΈΠ»ΠΎ Π·Π½Π°ΠΊΠΎΠ² для ΠΏΡ€ΠΎΠ΅ΠΊΡ†ΠΈΠΉ Π±ΡƒΠ΄Π΅Ρ‚ Ρ‚Π°ΠΊΠΈΠΌ ΠΆΠ΅, ΠΊΠ°ΠΊ ΠΈ для плоской систСмы сил – ΡΠΎΠ²ΠΏΠ°Π΄Π°ΡŽΡ‰ΠΈΠ΅ ΠΏΠΎ Π½Π°ΠΏΡ€Π°Π²Π»Π΅Π½ΠΈΡŽ с ΠΊΠΎΠΎΡ€Π΄ΠΈΠ½Π°Ρ‚Π½ΠΎΠΉ осью силы ΡΡ‡ΠΈΡ‚Π°ΡŽΡ‚ΡΡ ΠΏΠΎΠ»ΠΎΠΆΠΈΡ‚Π΅Π»ΡŒΠ½Ρ‹ΠΌΠΈ, Π² ΠΏΡ€ΠΎΡ‚ΠΈΠ²Π½ΠΎΠΌ случаС – ΠΎΡ‚Ρ€ΠΈΡ†Π°Ρ‚Π΅Π»ΡŒΠ½Ρ‹ΠΌΠΈ. Если Π²Π΅ΠΊΡ‚ΠΎΡ€ силы ΠΏΠ°Ρ€Π°Π»Π»Π΅Π»Π΅Π½ ΠΊΠ°ΠΊΠΎΠΉ-Π»ΠΈΠ±ΠΎ оси ΠΊΠΎΠΎΡ€Π΄ΠΈΠ½Π°Ρ‚, Ρ‚ΠΎ ΠΎΠ½ проСцируСтся Π½Π° эту ось Π² Π½Π°Ρ‚ΡƒΡ€Π°Π»ΡŒΠ½ΡƒΡŽ Π²Π΅Π»ΠΈΡ‡ΠΈΠ½Ρƒ, Ссли ΠΆΠ΅ Π²Π΅ΠΊΡ‚ΠΎΡ€ пСрпСндикулярСн оси, Π΅Π³ΠΎ проСкция Π½Π° эту ось Π±ΡƒΠ΄Π΅Ρ‚ Ρ€Π°Π²Π½Π° Π½ΡƒΠ»ΡŽ.

***

Π Π°Π·Π»ΠΎΠΆΠ΅Π½ΠΈΠ΅ силы ΠΏΠΎ Ρ‚Ρ€Π΅ΠΌ осям ΠΊΠΎΠΎΡ€Π΄ΠΈΠ½Π°Ρ‚

ΠŸΡƒΡΡ‚ΡŒ Π΄Π°Π½Π° сила F (см. рисунок 1).
Π’ΠΎΠ·ΡŒΠΌΠ΅ΠΌ систСму ΠΊΠΎΠΎΡ€Π΄ΠΈΠ½Π°Ρ‚ Ρ‚Π°ΠΊ, Ρ‡Ρ‚ΠΎΠ±Ρ‹ Π½Π°Ρ‡Π°Π»ΠΎ ΠΊΠΎΠΎΡ€Π΄ΠΈΠ½Π°Ρ‚ совпадало с Π½Π°Ρ‡Π°Π»ΠΎΠΌ Π²Π΅ΠΊΡ‚ΠΎΡ€Π° силы F (Ρ‚. Π΅. с Ρ‚ΠΎΡ‡ΠΊΠΎΠΉ прилоТСния силы). Из ΠΊΠΎΠ½Ρ†Π° этого Π²Π΅ΠΊΡ‚ΠΎΡ€Π° опустим пСрпСндикуляр Π½Π° ΠΏΠ»ΠΎΡΠΊΠΎΡΡ‚ΡŒ xy ΠΈ Ρ€Π°Π·Π»ΠΎΠΆΠΈΠΌ силу F Π½Π° ΡΠΎΡΡ‚Π°Π²Π»ΡΡŽΡ‰ΠΈΠ΅ Fxy ΠΈ Fz, Π° ΡΠΎΡΡ‚Π°Π²Π»ΡΡŽΡ‰ΡƒΡŽ Fxy – Π½Π° ΡΠΎΡΡ‚Π°Π²Π»ΡΡŽΡ‰ΠΈΠ΅ Fx ΠΈ Fy. Π’ΠΎΠ³Π΄Π°:

F = Fx + Fy + Fz.

Достроим ΠΏΠΎΠ»ΡƒΡ‡Π΅Π½Π½ΠΎΠ΅ ΠΈΠ·ΠΎΠ±Ρ€Π°ΠΆΠ΅Π½ΠΈΠ΅ Π΄ΠΎ ΠΏΠ°Ρ€Π°Π»Π»Π΅Π»Π΅ΠΏΠΈΠΏΠ΅Π΄Π°, Ρƒ ΠΊΠΎΡ‚ΠΎΡ€ΠΎΠ³ΠΎ ΡΠΎΡΡ‚Π°Π²Π»ΡΡŽΡ‰ΠΈΠ΅ Fx, Fy ΠΈ Fz ΡΠ²Π»ΡΡŽΡ‚ΡΡ Ρ€Π΅Π±Ρ€Π°ΠΌΠΈ, Π° сила F – диагональю.

Из ΠΈΠ·Π»ΠΎΠΆΠ΅Π½Π½ΠΎΠ³ΠΎ ΠΌΠΎΠΆΠ½ΠΎ ΡΠ΄Π΅Π»Π°Ρ‚ΡŒ Π²Ρ‹Π²ΠΎΠ΄: Ρ€Π°Π²Π½ΠΎΠ΄Π΅ΠΉΡΡ‚Π²ΡƒΡŽΡ‰Π°Ρ Ρ‚Ρ€Π΅Ρ… Π²Π·Π°ΠΈΠΌΠ½ΠΎ-пСрпСндикулярных сил выраТаСтся ΠΏΠΎ ΠΌΠΎΠ΄ΡƒΠ»ΡŽ ΠΈ Π½Π°ΠΏΡ€Π°Π²Π»Π΅Π½ΠΈΡŽ диагональю ΠΏΠ°Ρ€Π°Π»Π»Π΅Π»Π΅ΠΏΠΈΠΏΠ΅Π΄Π°, построСнного Π½Π° этих силах.

Из рисунка Π²ΠΈΠ΄Π½ΠΎ, Ρ‡Ρ‚ΠΎ Π² случаях разлоТСния силы F ΠΏΠΎ Ρ‚Ρ€Π΅ΠΌ Π²Π·Π°ΠΈΠΌΠ½ΠΎ-пСрпСндикулярным направлСниям x, y, z ΡΠΎΡΡ‚Π°Π²Π»ΡΡŽΡ‰ΠΈΠ΅ Fx, Fy ΠΈ Fz Ρ€Π°Π²Π½Ρ‹ ΠΏΠΎ ΠΌΠΎΠ΄ΡƒΠ»ΡŽ проСкциям силы F Π½Π° эти оси.

Зная ΠΏΡ€ΠΎΠ΅ΠΊΡ†ΠΈΠΈ силы Π½Π° Ρ‚Ρ€ΠΈ Π²Π·Π°ΠΈΠΌΠ½ΠΎ-пСрпСндикулярныС оси ΠΊΠΎΠΎΡ€Π΄ΠΈΠ½Π°Ρ‚, ΠΌΠΎΠΆΠ½ΠΎ ΠΎΠΏΡ€Π΅Π΄Π΅Π»ΠΈΡ‚ΡŒ ΠΌΠΎΠ΄ΡƒΠ»ΡŒ ΠΈ Π½Π°ΠΏΡ€Π°Π²Π»Π΅Π½ΠΈΠ΅ Π²Π΅ΠΊΡ‚ΠΎΡ€Π° силы ΠΏΠΎ Ρ„ΠΎΡ€ΠΌΡƒΠ»Π°ΠΌ:

ΠΌΠΎΠ΄ΡƒΠ»ΡŒ силы: Β Β F = √(Fx2 + Fy2 + Fz2) Β Β Β (здСсь ΠΈ Π΄Π°Π»Π΅Π΅ √ — Π·Π½Π°ΠΊ корня);

Π½Π°ΠΏΡ€Π°Π²Π»ΡΡŽΡ‰ΠΈΠ΅ косинусы: Β Β cos(F,x) = Fx/F;Β Β Β Β cos(F,y) = Fy/F;Β Β Β Β cos(F,z) = Fz/F.

***

АналитичСский способ опрСдСлСния Ρ€Π°Π²Π½ΠΎΠ΄Π΅ΠΉΡΡ‚Π²ΡƒΡŽΡ‰Π΅ΠΉ пространствСнной систСмы сходящихся сил

РассмотрСнный Π²Ρ‹ΡˆΠ΅ способ разлоТСния силы F Π½Π° Ρ‚Ρ€ΠΈ ΡΠΎΡΡ‚Π°Π²Π»ΡΡŽΡ‰ΠΈΠ΅ ΠΏΠΎ Π½Π°ΠΏΡ€Π°Π²Π»Π΅Π½ΠΈΡŽ ΠΊΠΎΠΎΡ€Π΄ΠΈΠ½Π°Ρ‚Π½Ρ‹Ρ… осСй x, y, z ΠΌΠΎΠΆΠ½ΠΎ ΠΏΡ€ΠΈΠΌΠ΅Π½ΠΈΡ‚ΡŒ для ΠΊΠ°ΠΆΠ΄ΠΎΠΉ ΠΈΠ· сходящихся сил пространствСнной систСмы. Π’ΠΎΠ³Π΄Π° вмСсто Π΄Π°Π½Π½ΠΎΠΉ систСмы n сходящихся сил ΠΌΡ‹ ΠΏΠΎΠ»ΡƒΡ‡ΠΈΠΌ ΡΠΊΠ²ΠΈΠ²Π°Π»Π΅Π½Ρ‚Π½ΡƒΡŽ Π΅ΠΉ систСму 3n сил, ΠΈΠ· ΠΊΠΎΡ‚ΠΎΡ€Ρ‹Ρ… n сил Π΄Π΅ΠΉΡΡ‚Π²ΡƒΡŽΡ‚ ΠΏΠΎ оси x, n сил – ΠΏΠΎ оси y, ΠΈ n сил – ΠΏΠΎ оси z.
Π Π°Π²Π½ΠΎΠ΄Π΅ΠΉΡΡ‚Π²ΡƒΡŽΡ‰Π°Ρ ΠΏΡ€ΠΎΠ΅ΠΊΡ†ΠΈΠΉ сил систСмы Π½Π° ось x Ρ€Π°Π²Π½Π° ΠΈΡ… гСомСтричСской суммС, Ρ‚ΠΎ ΠΆΠ΅ самоС ΠΌΠΎΠΆΠ½ΠΎ ΡΠΊΠ°Π·Π°Ρ‚ΡŒ ΠΈ ΠΎ Ρ€Π°Π²Π½ΠΎΠ΄Π΅ΠΉΡΡ‚Π²ΡƒΡŽΡ‰ΠΈΡ… ΠΏΡ€ΠΎΠ΅ΠΊΡ†ΠΈΠΉ сил Π½Π° оси y ΠΈ z.
Π’Π°ΠΊΠΈΠΌ ΠΎΠ±Ρ€Π°Π·ΠΎΠΌ, систСму 3n сил ΠΌΠΎΠΆΠ½ΠΎ Π·Π°ΠΌΠ΅Π½ΠΈΡ‚ΡŒ эквивалСнтной Π΅ΠΉ систСмой Ρ‚Ρ€Π΅Ρ… сил, каТдая ΠΈΠ· ΠΊΠΎΡ‚ΠΎΡ€Ρ‹Ρ… прСдставляСт собой Ρ€Π°Π²Π½ΠΎΠ΄Π΅ΠΉΡΡ‚Π²ΡƒΡŽΡ‰ΡƒΡŽ ΠΏΡ€ΠΎΠ΅ΠΊΡ†ΠΈΠΉ сил Π΄Π°Π½Π½ΠΎΠΉ систСмы Π½Π° Ρ‚Ρƒ ΠΈΠ»ΠΈ ΠΈΠ½ΡƒΡŽ ось ΠΊΠΎΠΎΡ€Π΄ΠΈΠ½Π°Ρ‚.

ΠŸΡ€ΠΎΠ΅ΠΊΡ†ΠΈΠΈ силы Π½Π° Ρ‚Ρ€ΠΈ Π²Π·Π°ΠΈΠΌΠ½ΠΎ-пСрпСндикулярныС оси ΠΈ ΡΠΎΡΡ‚Π°Π²Π»ΡΡŽΡ‰ΠΈΠ΅ силы, Π½Π°ΠΏΡ€Π°Π²Π»Π΅Π½Π½Ρ‹Π΅ ΠΏΠΎ этим осям, Ρ€Π°Π²Π½Ρ‹ ΠΏΠΎ ΠΌΠΎΠ΄ΡƒΠ»ΡŽ, ΡΠ»Π΅Π΄ΠΎΠ²Π°Ρ‚Π΅Π»ΡŒΠ½ΠΎ, ΠΏΡ€ΠΎΠ΅ΠΊΡ†ΠΈΠΈ Ρ€Π°Π²Π½ΠΎΠ΄Π΅ΠΉΡΡ‚Π²ΡƒΡŽΡ‰Π΅ΠΉ Ρ€Π°Π²Π½Ρ‹:

FΞ£x = Ξ£X;Β Β Β Β Β FΞ£y = Ξ£Y;Β Β Β Β Β FΞ£z = Ξ£Z.

ΠžΡ‡Π΅Π²ΠΈΠ΄Π½ΠΎ, Ρ‡Ρ‚ΠΎ Ρ€Π°Π²Π½ΠΎΠ΄Π΅ΠΉΡΡ‚Π²ΡƒΡŽΡ‰Π°Ρ Ρ‚Ρ€Π΅Ρ… Π²Π·Π°ΠΈΠΌΠ½ΠΎ пСрпСндикулярных сил выраТаСтся ΠΏΠΎ ΠΌΠΎΠ΄ΡƒΠ»ΡŽ ΠΈ Π½Π°ΠΏΡ€Π°Π²Π»Π΅Π½ΠΈΡŽ диагональю ΠΏΠ°Ρ€Π°Π»Π»Π΅Π»Π΅ΠΏΠΈΠΏΠ΅Π΄Π°, построСнного Π½Π° этих силах, ΠΈ ΠΏΠΎ извСстным проСкциям Ρ€Π°Π²Π½ΠΎΠ΄Π΅ΠΉΡΡ‚Π²ΡƒΡŽΡ‰Π΅ΠΉ ΠΌΠΎΠΆΠ½ΠΎ ΠΎΠΏΡ€Π΅Π΄Π΅Π»ΠΈΡ‚ΡŒ ΠΌΠΎΠ΄ΡƒΠ»ΡŒ ΠΈ Π½Π°ΠΏΡ€Π°Π²Π»Π΅Π½ΠΈΠ΅ самой Ρ€Π°Π²Π½ΠΎΠ΄Π΅ΠΉΡΡ‚Π²ΡƒΡŽΡ‰Π΅ΠΉ.

***

АналитичСскиС условия равновСсия пространствСнной систСмы сходящихся сил

Π˜Π·Π²Π΅ΡΡ‚Π½ΠΎ, Ρ‡Ρ‚ΠΎ пространствСнная систСма сходящихся сил эквивалСнтна Ρ€Π°Π²Π½ΠΎΠ΄Π΅ΠΉΡΡ‚Π²ΡƒΡŽΡ‰Π΅ΠΉ. Если такая систСма сил находится Π² равновСсии, Ρ‚. Π΅. эквивалСнтна Π½ΡƒΠ»ΡŽ, Ρ‚ΠΎ ΠΌΠΎΠΆΠ½ΠΎ ΡΠ΄Π΅Π»Π°Ρ‚ΡŒ Π²Ρ‹Π²ΠΎΠ΄, Ρ‡Ρ‚ΠΎ Ρ€Π°Π²Π½ΠΎΠ΄Π΅ΠΉΡΡ‚Π²ΡƒΡŽΡ‰Π°Ρ этой систСмы Ρ€Π°Π²Π½Π° Π½ΡƒΠ»ΡŽ, Π° ΡΠ»Π΅Π΄ΠΎΠ²Π°Ρ‚Π΅Π»ΡŒΠ½ΠΎ, ΠΈ ΠΏΡ€ΠΎΠ΅ΠΊΡ†ΠΈΠΈ Ρ€Π°Π²Π½ΠΎΠ΄Π΅ΠΉΡΡ‚Π²ΡƒΡŽΡ‰Π΅ΠΉ Ρ‚ΠΎΠΆΠ΅ Ρ€Π°Π²Π½Ρ‹ Π½ΡƒΠ»ΡŽ, ΠΏΡ€ΠΈΡ‡Π΅ΠΌ эти ΠΏΡ€ΠΎΠ΅ΠΊΡ†ΠΈΠΈ Ρ€Π°Π²Π½Ρ‹ суммС ΠΏΡ€ΠΎΠ΅ΠΊΡ†ΠΈΠΉ ΡΠΎΡΡ‚Π°Π²Π»ΡΡŽΡ‰ΠΈΡ….
ΠžΡ‚ΡΡŽΠ΄Π° Π²Ρ‹Ρ‚Π΅ΠΊΠ°ΡŽΡ‚ условия равновСсия пространствСнной систСмы сходящихся сил:

Ξ£X = 0;Β Β Β Β Ξ£Y = 0;Β Β Β Β Ξ£Z = 0.

Π­Ρ‚ΠΈ условия Ρ„ΠΎΡ€ΠΌΠΈΡ€ΡƒΡŽΡ‚ΡΡ ΡΠ»Π΅Π΄ΡƒΡŽΡ‰ΠΈΠΌ ΠΎΠ±Ρ€Π°Π·ΠΎΠΌ: для равновСсия пространствСнной систСмы сходящихся сил Π½Π΅ΠΎΠ±Ρ…ΠΎΠ΄ΠΈΠΌΠΎ ΠΈ достаточно, Ρ‡Ρ‚ΠΎΠ±Ρ‹ алгСбраичСская сумма ΠΏΡ€ΠΎΠ΅ΠΊΡ†ΠΈΠΉ всСх сил Π½Π° ΠΊΠ°ΠΆΠ΄ΡƒΡŽ ΠΈΡ… Ρ‚Ρ€Π΅Ρ… ΠΊΠΎΠΎΡ€Π΄ΠΈΠ½Π°Ρ‚Π½Ρ‹Ρ… осСй Ρ€Π°Π²Π½ΡΠ»Π°ΡΡŒ Π½ΡƒΠ»ΡŽ.

***

ο»Ώ

ΠœΠΎΠΌΠ΅Π½Ρ‚ силы ΠΎΡ‚Π½ΠΎΡΠΈΡ‚Π΅Π»ΡŒΠ½ΠΎ оси

Рассмотрим колСсо чСрвячной ΠΏΠ΅Ρ€Π΅Π΄Π°Ρ‡ΠΈ, ΡƒΠΊΡ€Π΅ΠΏΠ»Π΅Π½Π½ΠΎΠ΅ Π½Π° Π²Π°Π»Ρƒ, Π²Ρ€Π°Ρ‰Π°ΡŽΡ‰Π΅ΠΌΡΡ Π² ΠΏΠΎΠ΄ΡˆΠΈΠΏΠ½ΠΈΠΊΠ°Ρ… (см. рисунок 2). ЧСрвяк ΠΏΠ΅Ρ€Π΅Π΄Π°Π΅Ρ‚ чСрвячному колСсу силу F, Π½Π΅ Π»Π΅ΠΆΠ°Ρ‰ΡƒΡŽ Π² плоскости, пСрпСндикулярной оси.

Π Π°Π·Π»ΠΎΠΆΠΈΠΌ силу F Π½Π° Ρ‚Ρ€ΠΈ Π²Π·Π°ΠΈΠΌΠ½ΠΎ-пСрпСндикулярныС ΡΠΎΡΡ‚Π°Π²Π»ΡΡŽΡ‰ΠΈΠ΅ F1, F2 ΠΈ F3.
Π‘ΠΎΡΡ‚Π°Π²Π»ΡΡŽΡ‰ΡƒΡŽ F1 Π½Π°Π·ΠΎΠ²Π΅ΠΌ ΠΎΠΊΡ€ΡƒΠΆΠ½ΠΎΠΉ силой, ΡΠΎΡΡ‚Π°Π²Π»ΡΡŽΡ‰ΡƒΡŽ F2 – осСвой силой, Π° ΡΠΎΡΡ‚Π°Π²Π»ΡΡŽΡ‰ΡƒΡŽ F3 – Ρ€Π°Π΄ΠΈΠ°Π»ΡŒΠ½ΠΎΠΉ силой.
Из рисунка Π²ΠΈΠ΄Π½ΠΎ, Ρ‡Ρ‚ΠΎ ΡΠΎΡΡ‚Π°Π²Π»ΡΡŽΡ‰Π°Ρ F1 Π²Ρ‹Π·Ρ‹Π²Π°Π΅Ρ‚ Π²Ρ€Π°Ρ‰Π°Ρ‚Π΅Π»ΡŒΠ½ΠΎΠ΅ дСйствиС, ΠΊΠΎΡ‚ΠΎΡ€ΠΎΠ΅ измСряСтся ΠΏΡ€ΠΎΠΈΠ·Π²Π΅Π΄Π΅Π½ΠΈΠ΅ΠΌ силы F1 Π½Π° радиус колСса r; ΡΠΎΡΡ‚Π°Π²Π»ΡΡŽΡ‰Π°Ρ F2 стрСмится ΡΠ΄Π²ΠΈΠ½ΡƒΡ‚ΡŒ чСрвячноС колСсо вдоль оси, Π° ΡΠΎΡΡ‚Π°Π²Π»ΡΡŽΡ‰Π°Ρ F3 стрСмится ΠΈΠ·ΠΎΠ³Π½ΡƒΡ‚ΡŒ ось колСса.
ΠžΡ‡Π΅Π²ΠΈΠ΄Π½ΠΎ, Ρ‡Ρ‚ΠΎ Π²Ρ€Π°Ρ‰Π°ΡŽΡ‰Π΅Π΅ дСйствиС сил F2 ΠΈ F3 ΠΎΡ‚Π½ΠΎΡΠΈΡ‚Π΅Π»ΡŒΠ½ΠΎ оси колСса Ρ€Π°Π²Π½ΠΎ Π½ΡƒΠ»ΡŽ.
Π’Π°ΠΊΠΈΠΌ ΠΎΠ±Ρ€Π°Π·ΠΎΠΌ, Ссли Π½ΡƒΠΆΠ½ΠΎ Π½Π°ΠΉΡ‚ΠΈ ΠΌΠΎΠΌΠ΅Π½Ρ‚ силы ΠΎΡ‚Π½ΠΎΡΠΈΡ‚Π΅Π»ΡŒΠ½ΠΎ оси, Ρ‚ΠΎ слСдуСт ΠΏΡ€ΠΈΠ½ΠΈΠΌΠ°Ρ‚ΡŒ Π² расчСт Ρ‚ΠΎΠ»ΡŒΠΊΠΎ ΡΠΎΡΡ‚Π°Π²Π»ΡΡŽΡ‰ΡƒΡŽ F1, Π»Π΅ΠΆΠ°Ρ‰ΡƒΡŽ Π² плоскости, пСрпСндикулярной оси, ΠΈ Π½Π΅ ΠΏΠ΅Ρ€Π΅ΡΠ΅ΠΊΠ°ΡŽΡ‰ΡƒΡŽ ось (ΠΈΠ½Π°Ρ‡Π΅ Π΅Π΅ ΠΌΠΎΠΌΠ΅Π½Ρ‚ Π±ΡƒΠ΄Π΅Ρ‚ Ρ€Π°Π²Π΅Π½ Π½ΡƒΠ»ΡŽ).

Π Π°Π½Π΅Π΅ Π±Ρ‹Π»ΠΎ ΠΎΡ‚ΠΌΠ΅Ρ‡Π΅Π½ΠΎ, Ρ‡Ρ‚ΠΎ проСкция Π²Π΅ΠΊΡ‚ΠΎΡ€Π° силы Π½Π° ось Π΅ΡΡ‚ΡŒ скалярная алгСбраичСская Π²Π΅Π»ΠΈΡ‡ΠΈΠ½Π°. Π’ ΠΎΡ‚Π»ΠΈΡ‡ΠΈΠ΅ ΠΎΡ‚ ΠΏΡ€ΠΎΠ΅ΠΊΡ†ΠΈΠΈ Π½Π° ось проСкция силы Π½Π° ΠΏΠ»ΠΎΡΠΊΠΎΡΡ‚ΡŒ Π΅ΡΡ‚ΡŒ Π²Π΅Π»ΠΈΡ‡ΠΈΠ½Π° вСкторная, Ρ‚Π°ΠΊ ΠΊΠ°ΠΊ эта проСкция характСризуСтся Π½Π΅ Ρ‚ΠΎΠ»ΡŒΠΊΠΎ числовым Π·Π½Π°Ρ‡Π΅Π½ΠΈΠ΅ΠΌ, Π½ΠΎ ΠΈ ΠΏΠΎΠ»ΠΎΠΆΠ΅Π½ΠΈΠ΅ΠΌ Π½Π° плоскости, Ρ‚. Π΅. Π½Π°ΠΏΡ€Π°Π²Π»Π΅Π½ΠΈΠ΅ΠΌ.
ΠŸΠΎΡΡ‚ΠΎΠΌΡƒ ΠΌΠΎΠΌΠ΅Π½Ρ‚Ρƒ силы ΠΎΡ‚Π½ΠΎΡΠΈΡ‚Π΅Π»ΡŒΠ½ΠΎ оси ΠΌΠΎΠΆΠ½ΠΎ Π΄Π°Ρ‚ΡŒ Ρ‚Π°ΠΊΠΎΠ΅ ΠΎΠΏΡ€Π΅Π΄Π΅Π»Π΅Π½ΠΈΠ΅: ΠΌΠΎΠΌΠ΅Π½Ρ‚ΠΎΠΌ силы ΠΎΡ‚Π½ΠΎΡΠΈΡ‚Π΅Π»ΡŒΠ½ΠΎ оси называСтся Π²Π΅Π»ΠΈΡ‡ΠΈΠ½Π°, равная ΠΌΠΎΠΌΠ΅Π½Ρ‚Ρƒ ΠΏΡ€ΠΎΠ΅ΠΊΡ†ΠΈΠΈ этой силы Π½Π° ΠΏΠ»ΠΎΡΠΊΠΎΡΡ‚ΡŒ, ΠΏΠ΅Ρ€ΠΏΠ΅Π½Π΄ΠΈΠΊΡƒΠ»ΡΡ€Π½ΡƒΡŽ оси, ΠΎΡ‚Π½ΠΎΡΠΈΡ‚Π΅Π»ΡŒΠ½ΠΎ Ρ‚ΠΎΡ‡ΠΊΠΈ пСрСсСчСния оси с ΠΏΠ»ΠΎΡΠΊΠΎΡΡ‚ΡŒΡŽ.

Π­Ρ‚ΠΎ ΠΎΠΏΡ€Π΅Π΄Π΅Π»Π΅Π½ΠΈΠ΅ поясняСт рисунок 3.
ΠœΠΎΠΌΠ΅Π½Ρ‚ силы ΠΎΡ‚Π½ΠΎΡΠΈΡ‚Π΅Π»ΡŒΠ½ΠΎ оси условимся Π·Π°ΠΏΠΈΡΡ‹Π²Π°Ρ‚ΡŒ ΡΠ»Π΅Π΄ΡƒΡŽΡ‰ΠΈΠΌ ΠΎΠ±Ρ€Π°Π·ΠΎΠΌ:

Mz(F) = FΠ½Π°.

Условимся ΡΡ‡ΠΈΡ‚Π°Ρ‚ΡŒ ΠΌΠΎΠΌΠ΅Π½Ρ‚ силы ΠΏΠΎΠ»ΠΎΠΆΠΈΡ‚Π΅Π»ΡŒΠ½Ρ‹ΠΌ, Ссли ΡΠΌΠΎΡ‚Ρ€Π΅Ρ‚ΡŒ с ΠΏΠΎΠ»ΠΎΠΆΠΈΡ‚Π΅Π»ΡŒΠ½ΠΎΠ³ΠΎ ΠΊΠΎΠ½Ρ†Π° оси ΠΈ сила стрСмится Π²Ρ‹Π·Π²Π°Ρ‚ΡŒ Π²Ρ€Π°Ρ‰Π΅Π½ΠΈΠ΅ ΠΏΡ€ΠΎΡ‚ΠΈΠ² часовой стрСлки, Ссли ΠΆΠ΅ сила стрСмится Π²Ρ‹Π·Π²Π°Ρ‚ΡŒ Π²Ρ€Π°Ρ‰Π΅Π½ΠΈΠ΅ ΠΏΠΎ часовой стрСлкС, Π΅Π΅ ΠΌΠΎΠΌΠ΅Π½Ρ‚ считаСм ΠΎΡ‚Ρ€ΠΈΡ†Π°Ρ‚Π΅Π»ΡŒΠ½Ρ‹ΠΌ.

ΠœΠΎΠΌΠ΅Π½Ρ‚ силы ΠΎΡ‚Π½ΠΎΡΠΈΡ‚Π΅Π»ΡŒΠ½ΠΎ оси Π½Π΅ мСняСтся ΠΏΡ€ΠΈ ΠΏΠ΅Ρ€Π΅ΠΌΠ΅Ρ‰Π΅Π½ΠΈΠΈ силы вдоль оси Π΅Π΅ дСйствия.

ΠœΠΎΠΌΠ΅Π½Ρ‚ силы Π±ΡƒΠ΄Π΅Ρ‚ Ρ€Π°Π²Π΅Π½ Π½ΡƒΠ»ΡŽ Π² Π΄Π²ΡƒΡ… случаях (Π½Π΅ считая случаСв, ΠΊΠΎΠ³Π΄Π° сила Ρ€Π°Π²Π½Π° Π½ΡƒΠ»ΡŽ ΠΈΠ»ΠΈ Π½Π°ΠΏΡ€Π°Π²Π»Π΅Π½Π° вдоль оси):
  • Ссли Π²Π΅ΠΊΡ‚ΠΎΡ€ силы ΠΏΠ°Ρ€Π°Π»Π»Π΅Π»Π΅Π½ оси, Ρ‚Π°ΠΊ ΠΊΠ°ΠΊ ΠΏΡ€ΠΈ этом проСкция силы Π½Π° ΠΏΠ»ΠΎΡΠΊΠΎΡΡ‚ΡŒ, ΠΏΠ΅Ρ€ΠΏΠ΅Π½Π΄ΠΈΠΊΡƒΠ»ΡΡ€Π½ΡƒΡŽ оси, Ρ€Π°Π²Π½Π° Π½ΡƒΠ»ΡŽ (см. рисунок 3, сила FZ);
  • Ссли линия дСйствия силы пСрСсСкаСт ось, Ρ‚Π°ΠΊ ΠΊΠ°ΠΊ ΠΏΡ€ΠΈ этом ΠΏΠ»Π΅Ρ‡ΠΎ Ρ€Π°Π²Π½ΠΎ Π½ΡƒΠ»ΡŽ (сила F3 Π½Π° рисункС 2).

***

АналитичСскиС условия равновСсия пространствСнной систСмы ΠΏΡ€ΠΎΠΈΠ·Π²ΠΎΠ»ΡŒΠ½ΠΎ располоТСнных сил

ΠŸΡ€ΠΎΡΡ‚Ρ€Π°Π½ΡΡ‚Π²Π΅Π½Π½Π°Ρ систСма сил, Π² ΠΊΠΎΡ‚ΠΎΡ€ΠΎΠΉ Π»ΠΈΠ½ΠΈΠΈ дСйствия ΡΠΎΡΡ‚Π°Π²Π»ΡΡŽΡ‰ΠΈΡ… сил располоТСны ΠΏΡ€ΠΎΠΈΠ·Π²ΠΎΠ»ΡŒΠ½ΠΎ, Ρ‚. Π΅. Π»ΠΈΠ½ΠΈΠΈ ΠΈΡ… дСйствия ΠΌΠΎΠ³ΡƒΡ‚ Π½Π΅ ΠΏΠ΅Ρ€Π΅ΡΠ΅ΠΊΠ°Ρ‚ΡŒΡΡ ΠΈ Π½Π°Ρ…ΠΎΠ΄ΠΈΡ‚ΡŒΡΡ Π² Ρ€Π°Π·Π½Ρ‹Ρ… плоскостях, называСтся ΠΏΡ€ΠΎΠΈΠ·Π²ΠΎΠ»ΡŒΠ½ΠΎ располоТСнной систСмой сил.

Для равновСсия пространствСнной систСмы ΠΏΡ€ΠΎΠΈΠ·Π²ΠΎΠ»ΡŒΠ½ΠΎ располоТСнных сил Π½Π΅ΠΎΠ±Ρ…ΠΎΠ΄ΠΈΠΌΠΎ ΠΈ достаточно, Ρ‡Ρ‚ΠΎΠ±Ρ‹ алгСбраичСская сумма ΠΏΡ€ΠΎΠ΅ΠΊΡ†ΠΈΠΉ всСх сил Π½Π° ΠΊΠ°ΠΆΠ΄ΡƒΡŽ ΠΈΠ· Ρ‚Ρ€Π΅Ρ… осСй ΠΊΠΎΠΎΡ€Π΄ΠΈΠ½Π°Ρ‚ Π±Ρ‹Π»Π° Ρ€Π°Π²Π½Π° Π½ΡƒΠ»ΡŽ ΠΈ Ρ‡Ρ‚ΠΎΠ±Ρ‹ алгСбраичСская сумма ΠΌΠΎΠΌΠ΅Π½Ρ‚ΠΎΠ² всСх сил ΠΎΡ‚Π½ΠΎΡΠΈΡ‚Π΅Π»ΡŒΠ½ΠΎ ΠΊΠ°ΠΆΠ΄ΠΎΠΉ ΠΈΠ· этих осСй Π±Ρ‹Π»Π° Ρ€Π°Π²Π½Π° Π½ΡƒΠ»ΡŽ.

Π‘Ρ‚Ρ€ΠΎΠ³ΠΎΠ΅ обоснованиС ΠΏΡ€ΠΈΠ²Π΅Π΄Π΅Π½Π½ΠΎΠ³ΠΎ Π²Ρ‹ΡˆΠ΅ условия равновСсия пространствСнной систСмы ΠΏΡ€ΠΎΠΈΠ·Π²ΠΎΠ»ΡŒΠ½ΠΎ располоТСнных сил Ρ‚Ρ€Π΅Π±ΡƒΠ΅Ρ‚ знания Π½Π΅ΠΊΠΎΡ‚ΠΎΡ€Ρ‹Ρ… вопросов, Π½Π΅ прСдусмотрСнных ΠΏΡ€ΠΎΠ³Ρ€Π°ΠΌΠΌΠ°ΠΌΠΈ ΡƒΡ‡Ρ€Π΅ΠΆΠ΄Π΅Π½ΠΈΠΉ срСднСго ΠΏΡ€ΠΎΡ„Π΅ΡΡΠΈΠΎΠ½Π°Π»ΡŒΠ½ΠΎΠ³ΠΎ образования, поэтому условиС равновСсия Ρ‚Π°ΠΊΠΎΠΉ систСмы здСсь приводится Π±Π΅Π· Π΄ΠΎΠΊΠ°Π·Π°Ρ‚Π΅Π»ΡŒΡΡ‚Π²Π°.

ΠœΠ°Ρ‚Π΅ΠΌΠ°Ρ‚ΠΈΡ‡Π΅ΡΠΊΠΈ условиС равновСсия пространствСнной систСмы ΠΏΡ€ΠΎΠΈΠ·Π²ΠΎΠ»ΡŒΠ½ΠΎ располоТСнных сил ΠΌΠΎΠΆΠ½ΠΎ Π·Π°ΠΏΠΈΡΠ°Ρ‚ΡŒ Π² Π²ΠΈΠ΄Π΅ ΡƒΡ€Π°Π²Π½Π΅Π½ΠΈΠΉ:

  • Ξ£X = 0;Β Β Β Β Β Ξ£Mx(Fi) = 0;
  • Ξ£Y = 0;Β Β Β Β Β Ξ£My(Fi) = 0;
  • Ξ£Z = 0;Β Β Β Β Β Ξ£Mz(Fi) = 0.

Π‘Π²ΠΎΠ±ΠΎΠ΄Π½ΠΎΠ΅ Ρ‚Π΅Π»ΠΎ Π² пространствС ΠΈΠΌΠ΅Π΅Ρ‚ ΡˆΠ΅ΡΡ‚ΡŒ стСпСнСй свободы, Π° ΠΈΠΌΠ΅Π½Π½ΠΎ: Π²ΠΎΠ·ΠΌΠΎΠΆΠ½ΠΎΡΡ‚ΡŒ ΠΏΠ΅Ρ€Π΅ΠΌΠ΅Ρ‰Π°Ρ‚ΡŒΡΡ Π² направлСниях Ρ‚Ρ€Π΅Ρ… Π²Π·Π°ΠΈΠΌΠ½ΠΎ-пСрпСндикулярных осСй ΠΊΠΎΠΎΡ€Π΄ΠΈΠ½Π°Ρ‚ ΠΈ Π²ΠΎΠ·ΠΌΠΎΠΆΠ½ΠΎΡΡ‚ΡŒ Π²Ρ€Π°Ρ‰Π°Ρ‚ΡŒΡΡ Π²ΠΎΠΊΡ€ΡƒΠ³ этих осСй. Π’Π°ΠΊΠΈΠΌ ΠΎΠ±Ρ€Π°Π·ΠΎΠΌ, ΡˆΠ΅ΡΡ‚ΠΈ стСпСням свободы Ρ‚Π΅Π»Π° Π² пространствС ΡΠΎΠΎΡ‚Π²Π΅Ρ‚ΡΡ‚Π²ΡƒΡŽΡ‚ ΡˆΠ΅ΡΡ‚ΡŒ условий равновСсия.
Если систСма сил, ΠΏΡ€ΠΈΠ»ΠΎΠΆΠ΅Π½Π½Ρ‹Ρ… ΠΊ свободному Ρ‚Π΅Π»Ρƒ, удовлСтворяСт всСм ΡˆΠ΅ΡΡ‚ΠΈ условиям равновСсия, Ρ‚ΠΎ Π²ΠΎΠ·ΠΌΠΎΠΆΠ½ΠΎΡΡ‚ΡŒ Ρ‚Ρ€Π΅Ρ… ΠΏΠ΅Ρ€Π΅ΠΌΠ΅Ρ‰Π΅Π½ΠΈΠΉ ΠΈ Ρ‚Ρ€Π΅Ρ… Π²Ρ€Π°Ρ‰Π΅Π½ΠΈΠΉ Ρ‚Π΅Π»Π° ΠΏΠΎΠ΄ дСйствиСм сил систСмы ΠΈΡΠΊΠ»ΡŽΡ‡Π΅Π½Π°, поэтому Ρ‚Π΅Π»ΠΎ Π±ΡƒΠ΄Π΅Ρ‚ находится Π² равновСсии.

ΠžΡ‡Π΅Π²ΠΈΠ΄Π½ΠΎ, Ρ‡Ρ‚ΠΎ всС Π²Ρ‹Π²Π΅Π΄Π΅Π½Π½Ρ‹Π΅ Ρ€Π°Π½Π΅Π΅ условия равновСсия для Ρ€Π°Π·Π»ΠΈΡ‡Π½Ρ‹Ρ… систСм сил ΡΠ²Π»ΡΡŽΡ‚ΡΡ частными случаями условия равновСсия пространствСнной систСмы ΠΏΡ€ΠΎΠΈΠ·Π²ΠΎΠ»ΡŒΠ½ΠΎ располоТСнных сил.

Π’Π°ΠΊ ΠΊΠ°ΠΊ условия равновСсия пространствСнной систСмы сил справСдливы для Π»ΡŽΠ±Ρ‹Ρ… ΠΏΡ€ΡΠΌΠΎΡƒΠ³ΠΎΠ»ΡŒΠ½Ρ‹Ρ… осСй ΠΊΠΎΠΎΡ€Π΄ΠΈΠ½Π°Ρ‚, Ρ‚ΠΎ ΠΏΡ€ΠΈ Ρ€Π΅ΡˆΠ΅Π½ΠΈΠΈ Π΄Π°Π½Π½ΠΎΠΉ Π·Π°Π΄Π°Ρ‡ΠΈ систСму ΠΊΠΎΠΎΡ€Π΄ΠΈΠ½Π°Ρ‚ ΠΌΠΎΠΆΠ½ΠΎ ΠΈΠ·ΠΌΠ΅Π½ΡΡ‚ΡŒ, Ρ‚. Π΅. Ρ‡Π°ΡΡ‚ΡŒ ΡƒΡ€Π°Π²Π½Π΅Π½ΠΈΠΉ равновСсия ΡΠΎΡΡ‚Π°Π²ΠΈΡ‚ΡŒ для ΠΎΠ΄Π½ΠΈΡ… осСй ΠΊΠΎΠΎΡ€Π΄ΠΈΠ½Π°Ρ‚, Π° Ρ‡Π°ΡΡ‚ΡŒ – для ΠΈΠ·ΠΌΠ΅Π½Π΅Π½Π½Ρ‹Ρ…. Π’ Π½Π΅ΠΊΠΎΡ‚ΠΎΡ€Ρ‹Ρ… случаях этот ΠΏΡ€ΠΈΠ΅ΠΌ ΡƒΠΏΡ€ΠΎΡ‰Π°Π΅Ρ‚ Ρ€Π΅ΡˆΠ΅Π½ΠΈΠ΅ Π·Π°Π΄Π°Ρ‡.

***

Π’Π΅ΠΎΡ€Π΅ΠΌΠ° ΠΎ ΠΌΠΎΠΌΠ΅Π½Ρ‚Π΅ Ρ€Π°Π²Π½ΠΎΠ΄Π΅ΠΉΡΡ‚Π²ΡƒΡŽΡ‰Π΅ΠΉ ΠΎΡ‚Π½ΠΎΡΠΈΡ‚Π΅Π»ΡŒΠ½ΠΎ оси


(Ρ‚Π΅ΠΎΡ€Π΅ΠΌΠ° Π’Π°Ρ€ΠΈΠ½ΡŒΠΎΠ½Π°)

Π’Π΅ΠΎΡ€Π΅ΠΌΠ°: ΠΌΠΎΠΌΠ΅Π½Ρ‚ Ρ€Π°Π²Π½ΠΎΠ΄Π΅ΠΉΡΡ‚Π²ΡƒΡŽΡ‰Π΅ΠΉ ΠΎΡ‚Π½ΠΎΡΠΈΡ‚Π΅Π»ΡŒΠ½ΠΎ оси Ρ€Π°Π²Π΅Π½ алгСбраичСской суммС ΠΌΠΎΠΌΠ΅Π½Ρ‚ΠΎΠ², ΡΠΎΡΡ‚Π°Π²Π»ΡΡŽΡ‰ΠΈΡ… сил ΠΎΡ‚Π½ΠΎΡΠΈΡ‚Π΅Π»ΡŒΠ½ΠΎ этой ΠΆΠ΅ оси.

ΠŸΡƒΡΡ‚ΡŒ Π΄Π°Π½Ρ‹ пространствСнная систСма n ΠΏΡ€ΠΎΠΈΠ·Π²ΠΎΠ»ΡŒΠ½ΠΎ располоТСнных сил, ΠΏΡ€ΠΈΠ»ΠΎΠΆΠ΅Π½Π½Ρ‹Ρ… ΠΊ Ρ‚Π΅Π»Ρƒ, ΠΈ Ρ€Π°Π²Π½ΠΎΠ΄Π΅ΠΉΡΡ‚Π²ΡƒΡŽΡ‰Π°Ρ этой систСмы сил FΞ£ (см. рисунок 4):

(F1, F2, F3,….Fn) ≑ FΞ£.

ΠŸΡ€ΠΈΠ»ΠΎΠΆΠΈΠΌ ΠΊ Ρ‚Π΅Π»Ρƒ Π΄Ρ€ΡƒΠ³ΡƒΡŽ систСму сил, Ρ€Π°Π²Π½ΠΎΠ΄Π΅ΠΉΡΡ‚Π²ΡƒΡŽΡ‰Π°Ρ ΠΊΠΎΡ‚ΠΎΡ€ΠΎΠΉ F’Σ ΠΏΠΎ ΠΌΠΎΠ΄ΡƒΠ»ΡŽ Ρ€Π°Π²Π½Π° FΞ£ ΠΈ Π½Π°ΠΏΡ€Π°Π²Π»Π΅Π½Π° ΠΏΠΎ Ρ‚ΠΎΠΉ ΠΆΠ΅ Π»ΠΈΠ½ΠΈΠΈ дСйствия, Π½ΠΎ Π² ΠΏΡ€ΠΎΡ‚ΠΈΠ²ΠΎΠΏΠΎΠ»ΠΎΠΆΠ½ΡƒΡŽ сторону, Ρ‚. Π΅. являСтся ΡƒΡ€Π°Π²Π½ΠΎΠ²Π΅ΡˆΠΈΠ²Π°ΡŽΡ‰Π΅ΠΉ Π΄Π°Π½Π½ΠΎΠΉ систСмы сил.
Π’ΠΎΠ³Π΄Π° ΠΌΠΎΠΆΠ½ΠΎ Π·Π°ΠΏΠΈΡΠ°Ρ‚ΡŒ:

(F1, F2, F3,….Fn, F’Σ) ≑ 0 ,Β Β Β ΠΈΠ»ΠΈΒ Β Β (FΞ£, F’Σ) ≑ 0.

Π’Π°ΠΊ ΠΊΠ°ΠΊ ΠΎΠ±Π΅ записанныС Π²Ρ‹ΡˆΠ΅ систСмы сил эквивалСнтны Π½ΡƒΠ»ΡŽ, Ρ‚. Π΅. ΡƒΡ€Π°Π²Π½ΠΎΠ²Π΅ΡˆΠ΅Π½Ρ‹, Ρ‚ΠΎ ΠΊ Π½ΠΈΠΌ ΠΌΠΎΠΆΠ½ΠΎ ΠΏΡ€ΠΈΠΌΠ΅Π½ΠΈΡ‚ΡŒ любоС условиС равновСсия, Π½Π°ΠΏΡ€ΠΈΠΌΠ΅Ρ€

Ξ£Mx(Fi) = 0.

Π—Π°ΠΏΠΈΡˆΠ΅ΠΌ это условиС для ΠΎΠ±Π΅ΠΈΡ… систСм:

Mx(F1) = Mx(F2) + Mx(F2) + …. + Mx(Fn) + Mx(F’Σ) = 0;
MΞ£(FΞ£) + Mx(F’Σ) = 0.

Π’Π°ΠΊ ΠΊΠ°ΠΊ ΠΏΡ€Π°Π²Ρ‹Π΅ части этих равСнств Ρ€Π°Π²Π½Ρ‹, Ρ‚ΠΎ Π±ΡƒΠ΄ΡƒΡ‚ Ρ€Π°Π²Π½Ρ‹ ΠΈ Π»Π΅Π²Ρ‹Π΅ :

Mx(F1) = Mx(F2) + Mx(F3) + …. +Mx(Fn) + Mx(F’Σ) = Mx(FΞ£) + Mx(F’Σ).

Π‘ΠΎΠΊΡ€Π°Ρ‚ΠΈΠ² ΠΎΠ±Ρ‰Π΅Π΅ слагаСмоС Mx(F’Σ), ΠΏΠΎΠ»ΡƒΡ‡ΠΈΠΌ:

Mx(F1) = Mx(F2) + Mx(F3) + …. +Mx(Fn) = Mx(FΞ£)Β Β Β ΠΈΠ»ΠΈΒ Β Β Ξ£Mx(Fi) + Mx(FΞ£).

Π’Π΅ΠΎΡ€Π΅ΠΌΠ° Π΄ΠΎΠΊΠ°Π·Π°Π½Π°.

***

Π’Ρ€Π΅Π½ΠΈΠ΅ — ΡΡƒΡ‰Π½ΠΎΡΡ‚ΡŒ явлСния, Π·Π°ΠΊΠΎΠ½Ρ‹ ΠΈ зависимости

ο»Ώ
Главная страница


ДистанционноС ΠΎΠ±Ρ€Π°Π·ΠΎΠ²Π°Π½ΠΈΠ΅

Π‘ΠΏΠ΅Ρ†ΠΈΠ°Π»ΡŒΠ½ΠΎΡΡ‚ΠΈ

Π£Ρ‡Π΅Π±Π½Ρ‹Π΅ дисциплины

ΠžΠ»ΠΈΠΌΠΏΠΈΠ°Π΄Ρ‹ ΠΈ тСсты

Условия равновСсия ΠΏΡ€ΠΎΠΈΠ·Π²ΠΎΠ»ΡŒΠ½ΠΎΠΉ систСмы сил. ЧастныС случаи.

Условия равновСсия ΠΏΡ€ΠΎΠΈΠ·Π²ΠΎΠ»ΡŒΠ½ΠΎΠΉ систСмы сил. ЧастныС случаи.

41) Условия равновСсия ΠΏΡ€ΠΎΠΈΠ·Π²ΠΎΠ»ΡŒΠ½ΠΎΠΉ систСмы сил. ЧастныС случаи.

ΠŸΡ€ΠΎΠΈΠ·Π²ΠΎΠ»ΡŒΠ½Π°Ρ систСма сил, ΠΏΡ€ΠΈΠ»ΠΎΠΆΠ΅Π½Π½Ρ‹Ρ… ΠΊ Ρ‚Π²Π΅Ρ€Π΄ΠΎΠΌΡƒ Ρ‚Π΅Π»Ρƒ, эквивалСнтна силС, Ρ€Π°Π²Π½ΠΎΠΉ Π³Π»Π°Π²Π½ΠΎΠΌΡƒ Π²Π΅ΠΊΡ‚ΠΎΡ€Ρƒ R, ΠΈ ΠΏΠ°Ρ€Π΅ сил с ΠΌΠΎΠΌΠ΅Π½Ρ‚ΠΎΠΌ, Ρ€Π°Π²Π½Ρ‹ΠΌ Π³Π»Π°Π²Π½ΠΎΠΌΡƒ ΠΌΠΎΠΌΠ΅Π½Ρ‚Ρƒ L0 ΠΎΡ‚Π½ΠΎΡΠΈΡ‚Π΅Π»ΡŒΠ½ΠΎ ΠΊΠ°ΠΊΠΎΠ³ΠΎ-Π»ΠΈΠ±ΠΎ Ρ†Π΅Π½Ρ‚Ρ€Π° О. Π§Ρ‚ΠΎΠ±Ρ‹ такая систСма Π½Π°Ρ…ΠΎΠ΄ΠΈΠ»Π°ΡΡŒ Π² равновСсии, Π½Π΅ΠΎΠ±Ρ…ΠΎΠ΄ΠΈΠΌΠΎ ΠΈ достаточно равСнство Π½ΡƒΠ»ΡŽ ΠΈ Π³Π»Π°Π²Π½ΠΎΠ³ΠΎ Π²Π΅ΠΊΡ‚ΠΎΡ€Π° R, ΠΈ Π³Π»Π°Π²Π½ΠΎΠ³ΠΎ ΠΌΠΎΠΌΠ΅Π½Ρ‚Π° L0. ΠŸΠΎΡΡ‚ΠΎΠΌΡƒ условия равновСсия пространствСнной систСмы сил ΠΌΠΎΠ³ΡƒΡ‚ Π±Ρ‹Ρ‚ΡŒ прСдставлСны Π² Π²Π΅ΠΊΡ‚ΠΎΡ€Π½ΠΎΠΉ Ρ„ΠΎΡ€ΠΌΠ΅
Π”Π²Π° Π²Π΅ΠΊΡ‚ΠΎΡ€Π½Ρ‹Ρ… условия эквивалСнтны ΡΠ»Π΅Π΄ΡƒΡŽΡ‰ΠΈΠΌ ΡˆΠ΅ΡΡ‚ΠΈ аналитичСским условиям равновСсия:

Условия равновСсия ΠΌΠΎΠΆΠ½ΠΎ ΡΡ„ΠΎΡ€ΠΌΡƒΠ»ΠΈΡ€ΠΎΠ²Π°Ρ‚ΡŒ Ρ‚Π°ΠΊ: для равновСсия ΠΏΡ€ΠΎΠΈΠ·Π²ΠΎΠ»ΡŒΠ½ΠΎΠΉ систСмы сил, ΠΏΡ€ΠΈΠ»ΠΎΠΆΠ΅Π½Π½Ρ‹Ρ… ΠΊ Ρ‚Π²Π΅Ρ€Π΄ΠΎΠΌΡƒ Ρ‚Π΅Π»Ρƒ, Π½Π΅ΠΎΠ±Ρ…ΠΎΠ΄ΠΈΠΌΠΎ ΠΈ достаточно, Ρ‡Ρ‚ΠΎΠ±Ρ‹ суммы ΠΏΡ€ΠΎΠ΅ΠΊΡ†ΠΈΠΉ всСх сил Π½Π° оси Π΄Π΅ΠΊΠ°Ρ€Ρ‚ΠΎΠ²ΠΎΠΉ систСмы ΠΊΠΎΠΎΡ€Π΄ΠΈΠ½Π°Ρ‚ Ρ€Π°Π²Π½ΡΠ»ΠΈΡΡŒ Π½ΡƒΠ»ΡŽ ΠΈ суммы ΠΌΠΎΠΌΠ΅Π½Ρ‚ΠΎΠ² всСх сил ΠΎΡ‚Π½ΠΎΡΠΈΡ‚Π΅Π»ΡŒΠ½ΠΎ этих осСй Ρ‚Π°ΠΊΠΆΠ΅ Ρ€Π°Π²Π½ΡΠ»ΠΈΡΡŒ Π½ΡƒΠ»ΡŽ.

ЧастныС случаи.

Условия равновСсия пространствСнной систСмы ΠΏΠ°Ρ€Π°Π»Π»Π΅Π»ΡŒΠ½Ρ‹Ρ… сил.

Если силы, Π΄Π΅ΠΉΡΡ‚Π²ΡƒΡŽΡ‰ΠΈΠ΅ Π½Π° Ρ‚Π²Π΅Ρ€Π΄ΠΎΠ΅ Ρ‚Π΅Π»ΠΎ, ΠΏΠ°Ρ€Π°Π»Π»Π΅Π»ΡŒΠ½Ρ‹ ΠΌΠ΅ΠΆΠ΄Ρƒ собой, Ρ‚ΠΎ ΠΌΠΎΠΆΠ½ΠΎ Π²Ρ‹Π±Ρ€Π°Ρ‚ΡŒ Ρ‚Π°ΠΊΡƒΡŽ систСму ΠΊΠΎΠΎΡ€Π΄ΠΈΠ½Π°Ρ‚, ΠΊΠΎΠ³Π΄Π° ΠΎΠ΄Π½Π° ΠΈΠ· Π΅Π΅ осСй, Π½Π°ΠΏΡ€ΠΈΠΌΠ΅Ρ€ Oz, ΠΏΠ°Ρ€Π°Π»Π»Π΅Π»ΡŒΠ½Π° Π½Π°ΠΏΡ€Π°Π²Π»Π΅Π½ΠΈΡŽ дСйствия сил (рис.). Π’ΠΎΠ³Π΄Π° ΠΈΠ· ΡˆΠ΅ΡΡ‚ΠΈ аналитичСских условий равновСсия Ρ‚Ρ€ΠΈ Π²Ρ‹ΠΏΠΎΠ»Π½ΡΡŽΡ‚ΡΡ тоТдСствСнно, ΠΈ систСма ΠΏΠ°Ρ€Π°Π»Π»Π΅Π»ΡŒΠ½Ρ‹Ρ… сил Π±ΡƒΠ΄Π΅Ρ‚ ΠΈΠΌΠ΅Ρ‚ΡŒ Ρ‚ΠΎΠ»ΡŒΠΊΠΎ Ρ‚Ρ€ΠΈ условия равновСсия:

Условия равновСсия плоской систСмы сил.

Для плоской систСмы сил условия равновСсия Π±ΡƒΠ΄ΡƒΡ‚ частным
случаСм ΡƒΡ€Π°Π²Π½Π΅Π½ΠΈΠΉ , ΠΎΠΏΡ€Π΅Π΄Π΅Π»ΡΡŽΡ‰ΠΈΡ… условия равновСсия пространствСнной систСмы сил. НапримСр, Ссли силы располоТСны Π² плоскости ΠžΡ…Ρƒ, Ρ‚ΠΎ аналитичСскиС условия равновСсия ΠΌΠΎΠΆΠ½ΠΎ Π·Π°ΠΏΠΈΡΠ°Ρ‚ΡŒ Π² Π²ΠΈΠ΄Π΅:
Для равновСсия ΠΏΡ€ΠΎΠΈΠ·Π²ΠΎΠ»ΡŒΠ½ΠΎΠΉ плоской систСмы сил Π½Π΅ΠΎΠ±Ρ…ΠΎΠ΄ΠΈΠΌΠΎ ΠΈ достаточно, Ρ‡Ρ‚ΠΎΠ±Ρ‹ суммы ΠΏΡ€ΠΎΠ΅ΠΊΡ†ΠΈΠΉ всСх сил Π½Π° ΠΊΠ°ΠΆΠ΄ΡƒΡŽ ΠΈΠ· Π΄Π²ΡƒΡ… ΠΊΠΎΠΎΡ€Π΄ΠΈΠ½Π°Ρ‚Π½Ρ‹Ρ… осСй ΠΈ сумма алгСбраичСских ΠΌΠΎΠΌΠ΅Π½Ρ‚ΠΎΠ² этих сил ΠΎΡ‚Π½ΠΎΡΠΈΡ‚Π΅Π»ΡŒΠ½ΠΎ любого Ρ†Π΅Π½Ρ‚Ρ€Π° О Π±Ρ‹Π»ΠΈ Ρ€Π°Π²Π½Ρ‹ Π½ΡƒΠ»ΡŽ. АлгСбраичСским ΠΌΠΎΠΌΠ΅Π½Ρ‚ΠΎΠΌ силы ΠΎΡ‚Π½ΠΎΡΠΈΡ‚Π΅Π»ΡŒΠ½ΠΎ Ρ‚ΠΎΡ‡ΠΊΠΈ Π½Π°Π·Ρ‹Π²Π°ΡŽΡ‚ ΠΌΠΎΠΌΠ΅Π½Ρ‚ силы ΠΎΡ‚Π½ΠΎΡΠΈΡ‚Π΅Π»ΡŒΠ½ΠΎ оси, проходящСй Ρ‡Π΅Ρ€Π΅Π· Π΄Π°Π½Π½ΡƒΡŽ Ρ‚ΠΎΡ‡ΠΊΡƒ пСрпСндикулярно плоскости, Π² ΠΊΠΎΡ‚ΠΎΡ€ΠΎΠΉ располоТСна сила ΠΈ
Ρ‚ΠΎΡ‡ΠΊΠ°.
ВмСсто ΠΈΠ½ΠΎΠ³Π΄Π° ΡƒΠ΄ΠΎΠ±Π½ΠΎ ΠΏΡ€ΠΈΠΌΠ΅Π½ΠΈΡ‚ΡŒ условия равновСсия Π² Π²ΠΈΠ΄Π΅ ΡƒΡ€Π°Π²Π½Π΅Π½ΠΈΠΉ Ρ‚Ρ€Π΅Ρ… ΠΌΠΎΠΌΠ΅Π½Ρ‚ΠΎΠ²: для равновСсия ΠΏΡ€ΠΎΠΈΠ·Π²ΠΎΠ»ΡŒΠ½ΠΎΠΉ плоской систСмы сил Π½Π΅ΠΎΠ±Ρ…ΠΎΠ΄ΠΈΠΌΠΎ ΠΈ достаточно, Ρ‡Ρ‚ΠΎΠ±Ρ‹ суммы алгСбраичСских ΠΌΠΎΠΌΠ΅Π½Ρ‚ΠΎΠ² всСх этих сил ΠΎΡ‚Π½ΠΎΡΠΈΡ‚Π΅Π»ΡŒΠ½ΠΎ Π»ΡŽΠ±Ρ‹Ρ… Ρ‚Ρ€Π΅Ρ… Ρ†Π΅Π½Ρ‚Ρ€ΠΎΠ² А, Π’, Π‘, Π½Π΅ Π»Π΅ΠΆΠ°Ρ‰ΠΈΡ… Π½Π° ΠΎΠ΄Π½ΠΎΠΉ прямой, Π±Ρ‹Π»ΠΈ Ρ€Π°Π²Π½Ρ‹ Π½ΡƒΠ»ΡŽ: .

ΠΠ΅ΠΎΠ±Ρ…ΠΎΠ΄ΠΈΠΌΠΎΡΡ‚ΡŒ утвСрТдСния слСдуСт ΠΈΠ· Ρ‚ΠΎΠ³ΠΎ, Ρ‡Ρ‚ΠΎ Ρ‚Ρ€Π΅Ρ‚ΡŒΠ΅ условиС справСдливо для любой Ρ‚ΠΎΡ‡ΠΊΠΈ. Π”ΠΎΡΡ‚Π°Ρ‚ΠΎΡ‡Π½ΠΎΡΡ‚ΡŒ Π΄ΠΎΠΊΠ°ΠΆΠ΅ΠΌ ΠΌΠ΅Ρ‚ΠΎΠ΄ΠΎΠΌ ΠΎΡ‚ ΠΏΡ€ΠΎΡ‚ΠΈΠ²Π½ΠΎΠ³ΠΎ, ΠΈΡΠΏΠΎΠ»ΡŒΠ·ΡƒΡ Ρ‚Π΅ΠΎΡ€Π΅ΠΌΡƒ ΠΎ ΠΏΡ€ΠΈΠ²Π΅Π΄Π΅Π½ΠΈΠΈ ΠΏΡ€ΠΎΠΈΠ·Π²ΠΎΠ»ΡŒΠ½ΠΎΠΉ систСмы сил ΠΊ Ρ†Π΅Π½Ρ‚Ρ€Ρƒ. Допустим, Ρ‡Ρ‚ΠΎ плоская систСма сил Π½Π΅ находится Π² равновСсии. Π’ΠΎΠ³Π΄Π°, приводя Π΅Π΅ ΠΏΠΎΠΎΡ‡Π΅Ρ€Π΅Π΄Π½ΠΎ ΠΊ Ρ‚ΠΎΡ‡ΠΊΠ°ΠΌ А, Π’, Π‘, Π±ΡƒΠ΄Π΅ΠΌ ΠΈΠΌΠ΅Ρ‚ΡŒ Π² этих Ρ‚ΠΎΡ‡ΠΊΠ°Ρ… Ρ€Π°Π²Π½ΠΎΠ΄Π΅ΠΉΡΡ‚Π²ΡƒΡŽΡ‰ΡƒΡŽ R . Для выполнСния равСнств Ρ€Π°Π²Π½ΠΎΠ΄Π΅ΠΉΡΡ‚Π²ΡƒΡŽΡ‰Π°Ρ Π΄ΠΎΠ»ΠΆΠ½Π° ΠΏΡ€ΠΎΠΉΡ‚ΠΈ ΠΎΠ΄Π½ΠΎΠ²Ρ€Π΅ΠΌΠ΅Π½Π½ΠΎ Ρ‡Π΅Ρ€Π΅Π· всС Ρ‚Ρ€ΠΈ Ρ‚ΠΎΡ‡ΠΊΠΈ, Π° это Π½Π΅Π²ΠΎΠ·ΠΌΠΎΠΆΠ½ΠΎ, Ρ‚Π°ΠΊ ΠΊΠ°ΠΊ Ρ‚ΠΎΡ‡ΠΊΠΈ Π½Π΅ Π»Π΅ΠΆΠ°Ρ‚ Π½Π° ΠΎΠ΄Π½ΠΎΠΉ прямой. Π‘Π»Π΅Π΄ΠΎΠ²Π°Ρ‚Π΅Π»ΡŒΠ½ΠΎ, Ρ€Π°Π²Π½ΠΎΠ΄Π΅ΠΉΡΡ‚Π²ΡƒΡŽΡ‰Π°Ρ Ρ€Π°Π²Π½Π° Π½ΡƒΠ»ΡŽ ΠΈ систСма сил, ΡƒΠ΄ΠΎΠ²Π»Π΅Ρ‚Π²ΠΎΡ€ΡΡŽΡ‰Π°Ρ
равСнствам , находится Π² равновСсии.

Π˜ΡΠΏΠΎΠ»ΡŒΠ·ΡƒΡŽΡ‚ΡΡ Ρ‚Π΅Ρ…Π½ΠΎΠ»ΠΎΠ³ΠΈΠΈ uCoz

Условия равновСсия ΠΏΡ€ΠΎΠΈΠ·Π²ΠΎΠ»ΡŒΠ½ΠΎΠΉ систСмы сил Π² Π²Π΅ΠΊΡ‚ΠΎΡ€Π½ΠΎΠΉ Ρ„ΠΎΡ€ΠΌΠ΅

Π’Π΅ΠΊΡ‚ΠΎΡ€Π½Ρ‹Π΅ условия равновСсия ΠΏΡ€ΠΎΠΈΠ·Π²ΠΎΠ»ΡŒΠ½ΠΎΠΉ систСмы сил: для равновСсия систСмы сил, ΠΏΡ€ΠΈΠ»ΠΎΠΆΠ΅Π½Π½Ρ‹Ρ… ΠΊ Ρ‚Π²Π΅Ρ€Π΄ΠΎΠΌΡƒ Ρ‚Π΅Π»Ρƒ, Π½Π΅ΠΎΠ±Ρ…ΠΎΠ΄ΠΈΠΌΠΎ ΠΈ достаточно, Ρ‡Ρ‚ΠΎΠ±Ρ‹ Π³Π»Π°Π²Π½Ρ‹ΠΉ Π²Π΅ΠΊΡ‚ΠΎΡ€ систСмы сил Π±Ρ‹Π» Ρ€Π°Π²Π΅Π½ Π½ΡƒΠ»ΡŽ ΠΈ Π³Π»Π°Π²Π½Ρ‹ΠΉ ΠΌΠΎΠΌΠ΅Π½Ρ‚ систСмы сил ΠΎΡ‚Π½ΠΎΡΠΈΡ‚Π΅Π»ΡŒΠ½ΠΎ любого Ρ†Π΅Π½Ρ‚Ρ€Π° привСдСния Ρ‚Π°ΠΊΠΆΠ΅ Π±Ρ‹Π» Ρ€Π°Π²Π΅Π½ Π½ΡƒΠ»ΡŽ. Π˜Π½Π°Ρ‡Π΅: для Ρ‚ΠΎΠ³ΠΎ Ρ‡Ρ‚ΠΎΠ±Ρ‹ ~0, Π½Π΅ΠΎΠ±Ρ…ΠΎΠ΄ΠΈΠΌΡ‹ ΠΈ достаточны условия:

, ΠΈΠ»ΠΈ,. (19)

Условия равновСсия пространствСнной систСмы сил Π² аналитичСской Ρ„ΠΎΡ€ΠΌΠ΅

Для равновСсия пространствСнной систСмы сил, ΠΏΡ€ΠΈΠ»ΠΎΠΆΠ΅Π½Π½Ρ‹Ρ… ΠΊ Ρ‚Π²Π΅Ρ€Π΄ΠΎΠΌΡƒ Ρ‚Π΅Π»Ρƒ, Π½Π΅ΠΎΠ±Ρ…ΠΎΠ΄ΠΈΠΌΠΎ ΠΈ достаточно, Ρ‡Ρ‚ΠΎΠ±Ρ‹ Ρ‚Ρ€ΠΈ суммы ΠΏΡ€ΠΎΠ΅ΠΊΡ†ΠΈΠΉ всСх сил Π½Π° оси Π΄Π΅ΠΊΠ°Ρ€Ρ‚ΠΎΠ²Ρ‹Ρ… ΠΊΠΎΠΎΡ€Π΄ΠΈΠ½Π°Ρ‚ Π±Ρ‹Π»ΠΈ Ρ€Π°Π²Π½Ρ‹ Π½ΡƒΠ»ΡŽ ΠΈ Ρ‚Ρ€ΠΈ суммы ΠΌΠΎΠΌΠ΅Π½Ρ‚ΠΎΠ² всСх сил ΠΎΡ‚Π½ΠΎΡΠΈΡ‚Π΅Π»ΡŒΠ½ΠΎ Ρ‚Ρ€Π΅Ρ… осСй ΠΊΠΎΠΎΡ€Π΄ΠΈΠ½Π°Ρ‚ Ρ‚Π°ΠΊΠΆΠ΅ Π±Ρ‹Π»ΠΈ Ρ€Π°Π²Π½Ρ‹ Π½ΡƒΠ»ΡŽ.

. (20)

Условия равновСсия пространствСнной систСмы сходящихся сил

Для равновСсия пространствСнной систСмы сходящихся сил, ΠΏΡ€ΠΈΠ»ΠΎΠΆΠ΅Π½Π½Ρ‹Ρ… ΠΊ Ρ‚Π²Π΅Ρ€Π΄ΠΎΠΌΡƒ Ρ‚Π΅Π»Ρƒ, Π½Π΅ΠΎΠ±Ρ…ΠΎΠ΄ΠΈΠΌΠΎ ΠΈ достаточно, Ρ‡Ρ‚ΠΎΠ±Ρ‹ суммы ΠΏΡ€ΠΎΠ΅ΠΊΡ†ΠΈΠΉ сил Π½Π° ΠΊΠ°ΠΆΠ΄ΡƒΡŽ ΠΈΠ· Ρ‚Ρ€Π΅Ρ… ΠΏΡ€ΡΠΌΠΎΡƒΠ³ΠΎΠ»ΡŒΠ½Ρ‹Ρ… осСй ΠΊΠΎΠΎΡ€Π΄ΠΈΠ½Π°Ρ‚ Π±Ρ‹Π»ΠΈ Ρ€Π°Π²Π½Ρ‹ Π½ΡƒΠ»ΡŽ:

; ;, (21)

Π’ случаС плоской систСмы сходящихся сил ΠΎΠ΄Π½Ρƒ ΠΈΠ· осСй ΠΊΠΎΠΎΡ€Π΄ΠΈΠ½Π°Ρ‚, ΠΎΠ±Ρ‹Ρ‡Π½ΠΎ , Π²Ρ‹Π±ΠΈΡ€Π°ΡŽΡ‚ пСрпСндикулярной силам, Π° Π΄Π²Π΅ Π΄Ρ€ΡƒΠ³ΠΈΠ΅ оси – соотвСтствСнно Π² плоскости сил. Для равновСсия плоской систСмы сходящихся сил, Π΄Π΅ΠΉΡΡ‚Π²ΡƒΡŽΡ‰ΠΈΡ… Π½Π° Ρ‚Π²Π΅Ρ€Π΄ΠΎΠ΅ Ρ‚Π΅Π»ΠΎ, Π½Π΅ΠΎΠ±Ρ…ΠΎΠ΄ΠΈΠΌΠΎ ΠΈ достаточно, Ρ‡Ρ‚ΠΎΠ±Ρ‹ суммы ΠΏΡ€ΠΎΠ΅ΠΊΡ†ΠΈΠΉ этих сил Π½Π° ΠΊΠ°ΠΆΠ΄ΡƒΡŽ ΠΈΠ· Π΄Π²ΡƒΡ… ΠΏΡ€ΡΠΌΠΎΡƒΠ³ΠΎΠ»ΡŒΠ½Ρ‹Ρ… ΠΊΠΎΠΎΡ€Π΄ΠΈΠ½Π°Ρ‚Π½Ρ‹Ρ… осСй, Π»Π΅ΠΆΠ°Ρ‰ΠΈΡ… Π² плоскости сил, Π±Ρ‹Π»ΠΈ Ρ€Π°Π²Π½Ρ‹ Π½ΡƒΠ»ΡŽ:

; , (22)

Условия равновСсия пространствСнной систСмы ΠΏΠ°Ρ€Π°Π»Π»Π΅Π»ΡŒΠ½Ρ‹Ρ… сил

Направим ось ΠΏΠ°Ρ€Π°Π»Π»Π΅Π»ΡŒΠ½ΠΎ силам:для равновСсия пространствСнной систСмы ΠΏΠ°Ρ€Π°Π»Π»Π΅Π»ΡŒΠ½Ρ‹Ρ… сил, ΠΏΡ€ΠΈΠ»ΠΎΠΆΠ΅Π½Π½Ρ‹Ρ… ΠΊ Ρ‚Π²Π΅Ρ€Π΄ΠΎΠΌΡƒ Ρ‚Π΅Π»Ρƒ, Π½Π΅ΠΎΠ±Ρ…ΠΎΠ΄ΠΈΠΌΠΎ ΠΈ достаточно, Ρ‡Ρ‚ΠΎΠ±Ρ‹ алгСбраичСская сумма этих сил Π±Ρ‹Π»Π° Ρ€Π°Π²Π½Π° Π½ΡƒΠ»ΡŽ ΠΈ суммы ΠΌΠΎΠΌΠ΅Π½Ρ‚ΠΎΠ² сил ΠΎΡ‚Π½ΠΎΡΠΈΡ‚Π΅Π»ΡŒΠ½ΠΎ Π΄Π²ΡƒΡ… ΠΊΠΎΠΎΡ€Π΄ΠΈΠ½Π°Ρ‚Π½Ρ‹Ρ… осСй, пСрпСндикулярных силам, Ρ‚Π°ΠΊΠΆΠ΅ Π±Ρ‹Π»ΠΈ Ρ€Π°Π²Π½Ρ‹ Π½ΡƒΠ»ΡŽ:

(23)

Условия равновСсия плоской систСмы сил

РасполоТим оси ΠΈΠ² плоскости дСйствия сил.

Условия равновСсия плоской систСмы сил Π² ΠΏΠ΅Ρ€Π²ΠΎΠΉ Ρ„ΠΎΡ€ΠΌΠ΅: для равновСсия плоской систСмы сил, Π΄Π΅ΠΉΡΡ‚Π²ΡƒΡŽΡ‰ΠΈΡ… Π½Π° Ρ‚Π²Π΅Ρ€Π΄ΠΎΠ΅ Ρ‚Π΅Π»ΠΎ, Π½Π΅ΠΎΠ±Ρ…ΠΎΠ΄ΠΈΠΌΠΎ ΠΈ достаточно, Ρ‡Ρ‚ΠΎΠ±Ρ‹ суммы ΠΏΡ€ΠΎΠ΅ΠΊΡ†ΠΈΠΉ этих сил Π½Π° ΠΊΠ°ΠΆΠ΄ΡƒΡŽ ΠΈΠ· Π΄Π²ΡƒΡ… ΠΏΡ€ΡΠΌΠΎΡƒΠ³ΠΎΠ»ΡŒΠ½Ρ‹Ρ… осСй ΠΊΠΎΠΎΡ€Π΄ΠΈΠ½Π°Ρ‚, располоТСнных Π² плоскости дСйствия сил, Π±Ρ‹Π»ΠΈ Ρ€Π°Π²Π½Ρ‹ Π½ΡƒΠ»ΡŽ ΠΈ сумма алгСбраичСских ΠΌΠΎΠΌΠ΅Π½Ρ‚ΠΎΠ² сил ΠΎΡ‚Π½ΠΎΡΠΈΡ‚Π΅Π»ΡŒΠ½ΠΎ любой Ρ‚ΠΎΡ‡ΠΊΠΈ, находящСйся Π² плоскости дСйствия сил, Ρ‚Π°ΠΊΠΆΠ΅ Π±Ρ‹Π»Π° Ρ€Π°Π²Π½Π° Π½ΡƒΠ»ΡŽ:

(24)

Для равновСсия плоской систСмы ΠΏΠ°Ρ€Π°Π»Π»Π΅Π»ΡŒΠ½Ρ‹Ρ… сил, ΠΏΡ€ΠΈΠ»ΠΎΠΆΠ΅Π½Π½Ρ‹Ρ… ΠΊ Ρ‚Π²Π΅Ρ€Π΄ΠΎΠΌΡƒ Ρ‚Π΅Π»Ρƒ, Π½Π΅ΠΎΠ±Ρ…ΠΎΠ΄ΠΈΠΌΠΎ ΠΈ достаточно, Ρ‡Ρ‚ΠΎΠ±Ρ‹ алгСбраичСская сумма сил Π±Ρ‹Π»Π° Ρ€Π°Π²Π½Π° Π½ΡƒΠ»ΡŽ ΠΈ сумма алгСбраичСских ΠΌΠΎΠΌΠ΅Π½Ρ‚ΠΎΠ² сил ΠΎΡ‚Π½ΠΎΡΠΈΡ‚Π΅Π»ΡŒΠ½ΠΎ любой Ρ‚ΠΎΡ‡ΠΊΠΈ, находящСйся Π² плоскости сил, Ρ‚Π°ΠΊΠΆΠ΅ Π±Ρ‹Π»Π° Ρ€Π°Π²Π½Π° Π½ΡƒΠ»ΡŽ:

(25)

Π’Π΅ΠΎΡ€Π΅ΠΌΠ° ΠΎ Ρ‚Ρ€Π΅Ρ… ΠΌΠΎΠΌΠ΅Π½Ρ‚Π°Ρ… (вторая Ρ„ΠΎΡ€ΠΌΠ° условий равновСсия): для равновСсия плоской систСмы сил, ΠΏΡ€ΠΈΠ»ΠΎΠΆΠ΅Π½Π½Ρ‹Ρ… ΠΊ Ρ‚Π²Π΅Ρ€Π΄ΠΎΠΌΡƒ Ρ‚Π΅Π»Ρƒ, Π½Π΅ΠΎΠ±Ρ…ΠΎΠ΄ΠΈΠΌΠΎ ΠΈ достаточно, Ρ‡Ρ‚ΠΎΠ±Ρ‹ суммы алгСбраичСских ΠΌΠΎΠΌΠ΅Π½Ρ‚ΠΎΠ² сил систСмы ΠΎΡ‚Π½ΠΎΡΠΈΡ‚Π΅Π»ΡŒΠ½ΠΎ Ρ‚Ρ€Π΅Ρ… Π»ΡŽΠ±Ρ‹Ρ… Ρ‚ΠΎΡ‡Π΅ΠΊ, располоТСнных Π² плоскости дСйствия сил ΠΈ Π½Π΅ Π»Π΅ΠΆΠ°Ρ‰ΠΈΡ… Π½Π° ΠΎΠ΄Π½ΠΎΠΉ прямой, Π±Ρ‹Π»ΠΈ Ρ€Π°Π²Π½Ρ‹ Π½ΡƒΠ»ΡŽ:

. (26)

Π’Ρ€Π΅Ρ‚ΡŒΡ Ρ„ΠΎΡ€ΠΌΠ° условий равновСсия: для равновСсия плоской систСмы сил, ΠΏΡ€ΠΈΠ»ΠΎΠΆΠ΅Π½Π½Ρ‹Ρ… ΠΊ Ρ‚Π²Π΅Ρ€Π΄ΠΎΠΌΡƒ Ρ‚Π΅Π»Ρƒ, Π½Π΅ΠΎΠ±Ρ…ΠΎΠ΄ΠΈΠΌΠΎ ΠΈ достаточно, Ρ‡Ρ‚ΠΎΠ±Ρ‹ суммы алгСбраичСских ΠΌΠΎΠΌΠ΅Π½Ρ‚ΠΎΠ² сил ΠΎΡ‚Π½ΠΎΡΠΈΡ‚Π΅Π»ΡŒΠ½ΠΎ Π΄Π²ΡƒΡ… Π»ΡŽΠ±Ρ‹Ρ… Ρ‚ΠΎΡ‡Π΅ΠΊ, Π»Π΅ΠΆΠ°Ρ‰ΠΈΡ… Π² плоскости дСйствия сил, Π±Ρ‹Π»ΠΈ Ρ€Π°Π²Π½Ρ‹ Π½ΡƒΠ»ΡŽ ΠΈ алгСбраичСская сумма ΠΏΡ€ΠΎΠ΅ΠΊΡ†ΠΈΠΉ этих сил Π½Π° ΠΊΠ°ΠΊΡƒΡŽ-Π»ΠΈΠ±ΠΎ ось плоскости, Π½Π΅ ΠΏΠ΅Ρ€ΠΏΠ΅Π½Π΄ΠΈΠΊΡƒΠ»ΡΡ€Π½ΡƒΡŽ прямой, проходящСй Ρ‡Π΅Ρ€Π΅Π· Π΄Π²Π΅ ΠΌΠΎΠΌΠ΅Π½Ρ‚Π½Ρ‹Π΅ Ρ‚ΠΎΡ‡ΠΊΠΈ, Ρ‚Π°ΠΊΠΆΠ΅ Π±Ρ‹Π»Π° Ρ€Π°Π²Π½Π° Π½ΡƒΠ»ΡŽ, Ρ‚.Π΅.

. (27)

12.1 Условия статичСского равновСсия

Π¦Π΅Π»ΠΈ обучСния

К ΠΊΠΎΠ½Ρ†Ρƒ этого Ρ€Π°Π·Π΄Π΅Π»Π° Π²Ρ‹ смоТСтС:

  • ΠžΠΏΡ€Π΅Π΄Π΅Π»ΠΈΡ‚Π΅ физичСскиС условия статичСского равновСсия.
  • НарисуйтС Π΄ΠΈΠ°Π³Ρ€Π°ΠΌΠΌΡƒ свободного Ρ‚Π΅Π»Π° для Ρ‚Π²Π΅Ρ€Π΄ΠΎΠ³ΠΎ Ρ‚Π΅Π»Π°, Π½Π° ΠΊΠΎΡ‚ΠΎΡ€ΠΎΠ΅ Π΄Π΅ΠΉΡΡ‚Π²ΡƒΡŽΡ‚ силы.
  • ΠžΠ±ΡŠΡΡΠ½ΠΈΡ‚Π΅, ΠΊΠ°ΠΊ условия равновСсия ΠΏΠΎΠ·Π²ΠΎΠ»ΡΡŽΡ‚ Π½Π°ΠΌ Ρ€Π΅ΡˆΠ°Ρ‚ΡŒ Π·Π°Π΄Π°Ρ‡ΠΈ статики.

ΠœΡ‹ Π³ΠΎΠ²ΠΎΡ€ΠΈΠΌ, Ρ‡Ρ‚ΠΎ Ρ‚Π²Π΅Ρ€Π΄ΠΎΠ΅ Ρ‚Π΅Π»ΠΎ находится Π² состоянии равновСсия , ΠΊΠΎΠ³Π΄Π° Π΅Π³ΠΎ Π»ΠΈΠ½Π΅ΠΉΠ½ΠΎΠ΅ ΠΈ ΡƒΠ³Π»ΠΎΠ²ΠΎΠ΅ ускорСниС Ρ€Π°Π²Π½Ρ‹ Π½ΡƒΠ»ΡŽ ΠΎΡ‚Π½ΠΎΡΠΈΡ‚Π΅Π»ΡŒΠ½ΠΎ ΠΈΠ½Π΅Ρ€Ρ†ΠΈΠ°Π»ΡŒΠ½ΠΎΠΉ систСмы отсчСта.Π­Ρ‚ΠΎ ΠΎΠ·Π½Π°Ρ‡Π°Π΅Ρ‚, Ρ‡Ρ‚ΠΎ Ρ‚Π΅Π»ΠΎ Π² состоянии равновСсия ΠΌΠΎΠΆΠ΅Ρ‚ Π΄Π²ΠΈΠ³Π°Ρ‚ΡŒΡΡ, Π½ΠΎ Π² этом случаС Π΅Π³ΠΎ линСйная ΠΈ угловая скорости Π΄ΠΎΠ»ΠΆΠ½Ρ‹ Π±Ρ‹Ρ‚ΡŒ постоянными. ΠœΡ‹ Π³ΠΎΠ²ΠΎΡ€ΠΈΠΌ, Ρ‡Ρ‚ΠΎ Ρ‚Π²Π΅Ρ€Π΄ΠΎΠ΅ Ρ‚Π΅Π»ΠΎ находится Π² состоянии статичСского равновСсия , ΠΊΠΎΠ³Π΄Π° ΠΎΠ½ΠΎ находится Π² состоянии покоя Π² нашСй Π²Ρ‹Π±Ρ€Π°Π½Π½ΠΎΠΉ систСмС ΠΊΠΎΠΎΡ€Π΄ΠΈΠ½Π°Ρ‚ . ΠžΠ±Ρ€Π°Ρ‚ΠΈΡ‚Π΅ Π²Π½ΠΈΠΌΠ°Π½ΠΈΠ΅, Ρ‡Ρ‚ΠΎ Ρ€Π°Π·Π»ΠΈΡ‡ΠΈΠ΅ ΠΌΠ΅ΠΆΠ΄Ρƒ состояниСм покоя ΠΈ состояниСм Ρ€Π°Π²Π½ΠΎΠΌΠ΅Ρ€Π½ΠΎΠ³ΠΎ двиТСния являСтся искусствСнным, Ρ‚ΠΎ Π΅ΡΡ‚ΡŒ ΠΎΠ±ΡŠΠ΅ΠΊΡ‚ ΠΌΠΎΠΆΠ΅Ρ‚ Π½Π°Ρ…ΠΎΠ΄ΠΈΡ‚ΡŒΡΡ Π² состоянии покоя Π² Π²Ρ‹Π±Ρ€Π°Π½Π½ΠΎΠΉ Π½Π°ΠΌΠΈ систСмС отсчСта, Π½ΠΎ для Π½Π°Π±Π»ΡŽΠ΄Π°Ρ‚Π΅Π»Ρ, двиТущСгося с постоянной ΡΠΊΠΎΡ€ΠΎΡΡ‚ΡŒΡŽ ΠΎΡ‚Π½ΠΎΡΠΈΡ‚Π΅Π»ΡŒΠ½ΠΎ нашСй систСмы ΠΊΠΎΠΎΡ€Π΄ΠΈΠ½Π°Ρ‚, Ρ‚ΠΎΡ‚ ΠΆΠ΅ ΠΎΠ±ΡŠΠ΅ΠΊΡ‚ каТСтся, находится Π² Ρ€Π°Π²Π½ΠΎΠΌΠ΅Ρ€Π½ΠΎΠΌ Π΄Π²ΠΈΠΆΠ΅Π½ΠΈΠΈ с постоянной ΡΠΊΠΎΡ€ΠΎΡΡ‚ΡŒΡŽ.ΠŸΠΎΡΠΊΠΎΠ»ΡŒΠΊΡƒ Π΄Π²ΠΈΠΆΠ΅Π½ΠΈΠ΅ составляСт ΠΎΡ‚Π½ΠΎΡΠΈΡ‚Π΅Π»ΡŒΠ½ΠΎ , Ρ‚ΠΎ, Ρ‡Ρ‚ΠΎ для нас находится Π² статичСском равновСсии, находится Π² динамичСском равновСсии для двиТущСгося Π½Π°Π±Π»ΡŽΠ΄Π°Ρ‚Π΅Π»Ρ, ΠΈ Π½Π°ΠΎΠ±ΠΎΡ€ΠΎΡ‚. ΠŸΠΎΡΠΊΠΎΠ»ΡŒΠΊΡƒ Π·Π°ΠΊΠΎΠ½Ρ‹ Ρ„ΠΈΠ·ΠΈΠΊΠΈ ΠΈΠ΄Π΅Π½Ρ‚ΠΈΡ‡Π½Ρ‹ для всСх ΠΈΠ½Π΅Ρ€Ρ†ΠΈΠ°Π»ΡŒΠ½Ρ‹Ρ… систСм отсчСта, Π² ΠΈΠ½Π΅Ρ€Ρ†ΠΈΠ°Π»ΡŒΠ½ΠΎΠΉ систСмС отсчСта Π½Π΅Ρ‚ различия ΠΌΠ΅ΠΆΠ΄Ρƒ статичСским равновСсиСм ΠΈ равновСсиСм.

Богласно Π²Ρ‚ΠΎΡ€ΠΎΠΌΡƒ Π·Π°ΠΊΠΎΠ½Ρƒ двиТСния ΠΡŒΡŽΡ‚ΠΎΠ½Π°, Π»ΠΈΠ½Π΅ΠΉΠ½ΠΎΠ΅ ускорСниС Ρ‚Π²Π΅Ρ€Π΄ΠΎΠ³ΠΎ Ρ‚Π΅Π»Π° вызываСтся Π΄Π΅ΠΉΡΡ‚Π²ΡƒΡŽΡ‰Π΅ΠΉ Π½Π° Π½Π΅Π³ΠΎ чистой силой, ΠΈΠ»ΠΈ

[латСкс] \ sum _ {k} {\ overset {\ to} {F}} _ {k} = m {\ overset {\ to} {a}} _ {\ text {CM}}.[/ латСкс]

Π—Π΄Π΅ΡΡŒ сумма всСх Π²Π½Π΅ΡˆΠ½ΠΈΡ… сил, Π΄Π΅ΠΉΡΡ‚Π²ΡƒΡŽΡ‰ΠΈΡ… Π½Π° Ρ‚Π΅Π»ΠΎ, Π³Π΄Π΅ ΠΌ — это Π΅Π³ΠΎ масса, Π° [latex] {\ overset {\ to} {a}} _ {\ text {CM}} [/ latex] — Π»ΠΈΠ½Π΅ΠΉΠ½ΠΎΠ΅ ускорСниС Π΅Π³ΠΎ Ρ†Π΅Π½Ρ‚Ρ€Π° масс (концСпция, ΠΊΠΎΡ‚ΠΎΡ€ΡƒΡŽ ΠΌΡ‹ обсуТдали Π² ΡΡ‚Π°Ρ‚ΡŒΡΡ… Β«Π›ΠΈΠ½Π΅ΠΉΠ½Ρ‹ΠΉ ΠΈΠΌΠΏΡƒΠ»ΡŒΡ ΠΈ столкновСния ΠΏΠΎ ΠΈΠΌΠΏΡƒΠ»ΡŒΡΡƒ ΠΈ столкновСниям»). Π’ состоянии равновСсия Π»ΠΈΠ½Π΅ΠΉΠ½ΠΎΠ΅ ускорСниС Ρ€Π°Π²Π½ΠΎ Π½ΡƒΠ»ΡŽ. Если ΡƒΡΡ‚Π°Π½ΠΎΠ²ΠΈΡ‚ΡŒ Π½ΡƒΠ»Π΅Π²ΠΎΠ΅ ускорСниС Π½Π° (рисунок), ΠΌΡ‹ ΠΏΠΎΠ»ΡƒΡ‡ΠΈΠΌ ΡΠ»Π΅Π΄ΡƒΡŽΡ‰Π΅Π΅ ΡƒΡ€Π°Π²Π½Π΅Π½ΠΈΠ΅:

ΠŸΠ΅Ρ€Π²ΠΎΠ΅ условиС равновСсия

ΠŸΠ΅Ρ€Π²ΠΎΠ΅ условиС равновСсия для статичСского равновСсия Ρ‚Π²Π΅Ρ€Π΄ΠΎΠ³ΠΎ Ρ‚Π΅Π»Π° Π²Ρ‹Ρ€Π°ΠΆΠ°Π΅Ρ‚ ΠΏΠΎΡΡ‚ΡƒΠΏΠ°Ρ‚Π΅Π»ΡŒΠ½ΠΎΠ΅ равновСсиС:

[латСкс] \ sum _ {k} {\ overset {\ to} {F}} _ {k} = \ overset {\ to} {0}.[/ латСкс]

ΠŸΠ΅Ρ€Π²ΠΎΠ΅ условиС равновСсия (рисунок) — это условиС равновСсия сил, с ΠΊΠΎΡ‚ΠΎΡ€Ρ‹ΠΌ ΠΌΡ‹ ΡΡ‚ΠΎΠ»ΠΊΠ½ΡƒΠ»ΠΈΡΡŒ ΠΏΡ€ΠΈ ΠΈΠ·ΡƒΡ‡Π΅Π½ΠΈΠΈ ΠΏΡ€ΠΈΠ»ΠΎΠΆΠ΅Π½ΠΈΠΉ Π·Π°ΠΊΠΎΠ½ΠΎΠ² ΠΡŒΡŽΡ‚ΠΎΠ½Π°.

Π­Ρ‚ΠΎ Π²Π΅ΠΊΡ‚ΠΎΡ€Π½ΠΎΠ΅ ΡƒΡ€Π°Π²Π½Π΅Π½ΠΈΠ΅ эквивалСнтно ΡΠ»Π΅Π΄ΡƒΡŽΡ‰ΠΈΠΌ Ρ‚Ρ€Π΅ΠΌ скалярным уравнСниям для ΠΊΠΎΠΌΠΏΠΎΠ½Π΅Π½Ρ‚ΠΎΠ² чистой силы:

[латСкс] \ sum _ {k} {F} _ {kx} = 0, \ quad \ sum _ {k} {F} _ {ky} = 0, \ quad \ sum _ {k} {F} _ {kz} = 0. [/ латСкс]

Аналогично (рисунок), ΠΌΡ‹ ΠΌΠΎΠΆΠ΅ΠΌ ΡƒΡ‚Π²Π΅Ρ€ΠΆΠ΄Π°Ρ‚ΡŒ, Ρ‡Ρ‚ΠΎ Π²Ρ€Π°Ρ‰Π°Ρ‚Π΅Π»ΡŒΠ½ΠΎΠ΅ ускорСниС [латСкс] \ overset {\ to} {\ alpha} [/ latex] Ρ‚Π²Π΅Ρ€Π΄ΠΎΠ³ΠΎ Ρ‚Π΅Π»Π° Π²ΠΎΠΊΡ€ΡƒΠ³ фиксированной оси вращСния вызываСтся чистым крутящим ΠΌΠΎΠΌΠ΅Π½Ρ‚ΠΎΠΌ, Π΄Π΅ΠΉΡΡ‚Π²ΡƒΡŽΡ‰ΠΈΠΌ Π½Π° body, ΠΈΠ»ΠΈ

[латСкс] \ sum _ {k} {\ overset {\ to} {\ tau}} _ {k} = I \ overset {\ to} {\ alpha}.[/ латСкс]

Π—Π΄Π΅ΡΡŒ [латСкс] I [/ латСкс] — это инСрция вращСния Ρ‚Π΅Π»Π° ΠΏΡ€ΠΈ Π²Ρ€Π°Ρ‰Π΅Π½ΠΈΠΈ Π²ΠΎΠΊΡ€ΡƒΠ³ этой оси, Π° сумма составляСт Π±ΠΎΠ»Π΅Π΅ всСх крутящих ΠΌΠΎΠΌΠ΅Π½Ρ‚ΠΎΠ² [латСкс] {\ overset {\ to} {\ tau}} _ {k} [/ latex] Π²Π½Π΅ΡˆΠ½ΠΈΡ… сил Π² (Рисунок). Π’ состоянии равновСсия ускорСниС вращСния Ρ€Π°Π²Π½ΠΎ Π½ΡƒΠ»ΡŽ. ΠžΠ±Π½ΡƒΠ»ΡΡ ΠΏΡ€Π°Π²ΡƒΡŽ Ρ‡Π°ΡΡ‚ΡŒ (рисунок), ΠΌΡ‹ ΠΏΠΎΠ»ΡƒΡ‡Π°Π΅ΠΌ Π²Ρ‚ΠΎΡ€ΠΎΠ΅ условиС равновСсия:

Π’Ρ‚ΠΎΡ€ΠΎΠ΅ состояниС равновСсия

Π’Ρ‚ΠΎΡ€ΠΎΠ΅ условиС равновСсия для статичСского равновСсия Ρ‚Π²Π΅Ρ€Π΄ΠΎΠ³ΠΎ Ρ‚Π΅Π»Π° Π²Ρ‹Ρ€Π°ΠΆΠ°Π΅Ρ‚ Π²Ρ€Π°Ρ‰Π°Ρ‚Π΅Π»ΡŒΠ½ΠΎΠ΅ равновСсиС:

[латСкс] \ sum _ {k} {\ overset {\ to} {\ tau}} _ {k} = \ overset {\ to} {0}.[/ латСкс]

Π’Ρ‚ΠΎΡ€ΠΎΠ΅ условиС равновСсия (рисунок) — это условиС равновСсия для крутящих ΠΌΠΎΠΌΠ΅Π½Ρ‚ΠΎΠ², с ΠΊΠΎΡ‚ΠΎΡ€Ρ‹ΠΌ ΠΌΡ‹ ΡΡ‚ΠΎΠ»ΠΊΠ½ΡƒΠ»ΠΈΡΡŒ ΠΏΡ€ΠΈ ΠΈΠ·ΡƒΡ‡Π΅Π½ΠΈΠΈ Π΄ΠΈΠ½Π°ΠΌΠΈΠΊΠΈ вращСния. Π‘Ρ‚ΠΎΠΈΡ‚ ΠΎΡ‚ΠΌΠ΅Ρ‚ΠΈΡ‚ΡŒ, Ρ‡Ρ‚ΠΎ это ΡƒΡ€Π°Π²Π½Π΅Π½ΠΈΠ΅ равновСсия ΠΎΠ±Ρ‹Ρ‡Π½ΠΎ справСдливо для Π²Ρ€Π°Ρ‰Π°Ρ‚Π΅Π»ΡŒΠ½ΠΎΠ³ΠΎ равновСсия Π²ΠΎΠΊΡ€ΡƒΠ³ любой оси вращСния (фиксированной ΠΈΠ»ΠΈ ΠΈΠ½ΠΎΠΉ). ΠžΠΏΡΡ‚ΡŒ ΠΆΠ΅, это Π²Π΅ΠΊΡ‚ΠΎΡ€Π½ΠΎΠ΅ ΡƒΡ€Π°Π²Π½Π΅Π½ΠΈΠ΅ эквивалСнтно Ρ‚Ρ€Π΅ΠΌ скалярным уравнСниям для Π²Π΅ΠΊΡ‚ΠΎΡ€Π½Ρ‹Ρ… ΠΊΠΎΠΌΠΏΠΎΠ½Π΅Π½Ρ‚ΠΎΠ² чистого крутящСго ΠΌΠΎΠΌΠ΅Π½Ρ‚Π°:

[латСкс] \ sum _ {k} {\ tau} _ {kx} = 0, \ quad \ sum _ {k} {\ tau} _ {ky} = 0, \ quad \ sum _ {k} {\ Ρ‚Π°Ρƒ} _ {kz} = 0.[/ латСкс]

Π’Ρ‚ΠΎΡ€ΠΎΠ΅ условиС равновСсия ΠΎΠ·Π½Π°Ρ‡Π°Π΅Ρ‚, Ρ‡Ρ‚ΠΎ Π² равновСсии Π½Π΅Ρ‚ чистого внСшнСго крутящСго ΠΌΠΎΠΌΠ΅Π½Ρ‚Π°, Π²Ρ‹Π·Ρ‹Π²Π°ΡŽΡ‰Π΅Π³ΠΎ Π²Ρ€Π°Ρ‰Π΅Π½ΠΈΠ΅ Π²ΠΎΠΊΡ€ΡƒΠ³ любой оси.

ΠŸΠ΅Ρ€Π²ΠΎΠ΅ ΠΈ Π²Ρ‚ΠΎΡ€ΠΎΠ΅ условия равновСсия ΡƒΠΊΠ°Π·Π°Π½Ρ‹ Π² ΠΊΠΎΠ½ΠΊΡ€Π΅Ρ‚Π½ΠΎΠΉ систСмС отсчСта. ΠŸΠ΅Ρ€Π²ΠΎΠ΅ условиС Π²ΠΊΠ»ΡŽΡ‡Π°Π΅Ρ‚ Ρ‚ΠΎΠ»ΡŒΠΊΠΎ силы ΠΈ поэтому Π½Π΅ зависит ΠΎΡ‚ источника систСмы отсчСта. Однако Π²Ρ‚ΠΎΡ€ΠΎΠ΅ условиС Π²ΠΊΠ»ΡŽΡ‡Π°Π΅Ρ‚ крутящий ΠΌΠΎΠΌΠ΅Π½Ρ‚, ΠΊΠΎΡ‚ΠΎΡ€Ρ‹ΠΉ опрСдСляСтся ΠΊΠ°ΠΊ пСрСкрСстноС ΠΏΡ€ΠΎΠΈΠ·Π²Π΅Π΄Π΅Π½ΠΈΠ΅, [латСкс] {\ overset {\ to} {\ tau}} _ {k} = {\ overset {\ to} {r}} _ {k } \, Γ— \, {\ overset {\ to} {F}} _ {k}, [/ latex], Π³Π΄Π΅ Π²Π΅ΠΊΡ‚ΠΎΡ€ полоТСния [latex] {\ overset {\ to} {r}} _ {k} [/ латСкс] ΠΎΡ‚Π½ΠΎΡΠΈΡ‚Π΅Π»ΡŒΠ½ΠΎ оси вращСния Ρ‚ΠΎΡ‡ΠΊΠΈ прилоТСния силы Π²Ρ…ΠΎΠ΄ΠΈΡ‚ Π² ΡƒΡ€Π°Π²Π½Π΅Π½ΠΈΠ΅.Π‘Π»Π΅Π΄ΠΎΠ²Π°Ρ‚Π΅Π»ΡŒΠ½ΠΎ, крутящий ΠΌΠΎΠΌΠ΅Π½Ρ‚ зависит ΠΎΡ‚ полоТСния оси Π² систСмС отсчСта. Однако, ΠΊΠΎΠ³Π΄Π° условия Π²Ρ€Π°Ρ‰Π°Ρ‚Π΅Π»ΡŒΠ½ΠΎΠ³ΠΎ ΠΈ ΠΏΠΎΡΡ‚ΡƒΠΏΠ°Ρ‚Π΅Π»ΡŒΠ½ΠΎΠ³ΠΎ равновСсия Π²Ρ‹ΠΏΠΎΠ»Π½ΡΡŽΡ‚ΡΡ ΠΎΠ΄Π½ΠΎΠ²Ρ€Π΅ΠΌΠ΅Π½Π½ΠΎ Π² ΠΎΠ΄Π½ΠΎΠΉ систСмС отсчСта, ΠΎΠ½ΠΈ Ρ‚Π°ΠΊΠΆΠ΅ ΡΠΎΡ…Ρ€Π°Π½ΡΡŽΡ‚ΡΡ Π² любой Π΄Ρ€ΡƒΠ³ΠΎΠΉ ΠΈΠ½Π΅Ρ€Ρ†ΠΈΠ°Π»ΡŒΠ½ΠΎΠΉ систСмС отсчСта, Ρ‚Π°ΠΊ Ρ‡Ρ‚ΠΎ чистый крутящий ΠΌΠΎΠΌΠ΅Π½Ρ‚ Π²ΠΎΠΊΡ€ΡƒΠ³ любой оси вращСния ΠΏΠΎ-ΠΏΡ€Π΅ΠΆΠ½Π΅ΠΌΡƒ Ρ€Π°Π²Π΅Π½ Π½ΡƒΠ»ΡŽ. ОбъяснСниС этому довольно простоС.

ΠŸΡ€Π΅Π΄ΠΏΠΎΠ»ΠΎΠΆΠΈΠΌ, Ρ‡Ρ‚ΠΎ Π²Π΅ΠΊΡ‚ΠΎΡ€ [latex] \ overset {\ to} {R} [/ latex] — это позиция Π½Π°Ρ‡Π°Π»Π° ΠΊΠΎΠΎΡ€Π΄ΠΈΠ½Π°Ρ‚ Π½ΠΎΠ²ΠΎΠΉ ΠΈΠ½Π΅Ρ€Ρ†ΠΈΠ°Π»ΡŒΠ½ΠΎΠΉ систСмы отсчСта [latex] S \ prime [/ latex] Π² старой ΠΈΠ½Π΅Ρ€Ρ†ΠΈΠ°Π»ΡŒΠ½ΠΎΠΉ систСмС отсчСта S .{\ prime}} _ {k} \, Γ— \, {\ overset {\ to} {F}} _ {k} = \ sum _ {k} ({\ overset {\ to} {r}} _ { k} — \ overset {\ to} {R}) \, Γ— \, {\ overset {\ to} {F}} _ {k} = \ sum _ {k} {\ overset {\ to} {r} } _ {k} \, Γ— \, {\ overset {\ to} {F}} _ {k} — \ sum _ {k} \ overset {\ to} {R} \, Γ— \, {\ overset { \ to} {F}} _ {k} = \ sum _ {k} {\ overset {\ to} {\ tau}} _ {k} — \ overset {\ to} {R} \, Γ— \, \ сумма _ {k} {\ overset {\ to} {F}} _ {k} = \ overset {\ to} {0}. [/ латСкс]

На послСднСм этапС этой Ρ†Π΅ΠΏΠΎΡ‡ΠΊΠΈ рассуТдСний ΠΌΡ‹ использовали Ρ‚ΠΎΡ‚ Ρ„Π°ΠΊΡ‚, Ρ‡Ρ‚ΠΎ Π² равновСсии Π² старой систСмС отсчСта, S , ΠΏΠ΅Ρ€Π²Ρ‹ΠΉ Ρ‡Π»Π΅Π½ исчСзаСт ΠΈΠ·-Π·Π° (Рисунок), Π° Π²Ρ‚ΠΎΡ€ΠΎΠΉ Ρ‡Π»Π΅Π½ исчСзаСт ΠΈΠ·-Π·Π° (Рисунок).Π‘Π»Π΅Π΄ΠΎΠ²Π°Ρ‚Π΅Π»ΡŒΠ½ΠΎ, ΠΌΡ‹ Π²ΠΈΠ΄ΠΈΠΌ, Ρ‡Ρ‚ΠΎ чистый крутящий ΠΌΠΎΠΌΠ΅Π½Ρ‚ Π² любой ΠΈΠ½Π΅Ρ€Ρ†ΠΈΠ°Π»ΡŒΠ½ΠΎΠΉ систСмС отсчСта [латСкс] S \ prime [/ latex] Ρ€Π°Π²Π΅Π½ Π½ΡƒΠ»ΡŽ, ΠΏΡ€ΠΈ условии, Ρ‡Ρ‚ΠΎ ΠΎΠ±Π° условия равновСсия Π²Ρ‹ΠΏΠΎΠ»Π½ΡΡŽΡ‚ΡΡ Π² ΠΈΠ½Π΅Ρ€Ρ†ΠΈΠ°Π»ΡŒΠ½ΠΎΠΉ систСмС отсчСта S .

ΠŸΡ€Π°ΠΊΡ‚ΠΈΡ‡Π΅ΡΠΊΠΎΠ΅ Π·Π½Π°Ρ‡Π΅Π½ΠΈΠ΅ этого состоит Π² Ρ‚ΠΎΠΌ, Ρ‡Ρ‚ΠΎ ΠΏΡ€ΠΈ ΠΏΡ€ΠΈΠΌΠ΅Π½Π΅Π½ΠΈΠΈ условий равновСсия для Ρ‚Π²Π΅Ρ€Π΄ΠΎΠ³ΠΎ Ρ‚Π΅Π»Π° ΠΌΡ‹ ΠΌΠΎΠΆΠ΅ΠΌ Π²Ρ‹Π±Ρ€Π°Ρ‚ΡŒ Π»ΡŽΠ±ΡƒΡŽ Ρ‚ΠΎΡ‡ΠΊΡƒ Π² качСствС Π½Π°Ρ‡Π°Π»Π° отсчСта систСмы отсчСта. Наш Π²Ρ‹Π±ΠΎΡ€ систСмы отсчСта ΠΏΡ€ΠΎΠ΄ΠΈΠΊΡ‚ΠΎΠ²Π°Π½ физичСскими особСнностями Ρ€Π΅ΡˆΠ°Π΅ΠΌΠΎΠΉ ΠΏΡ€ΠΎΠ±Π»Π΅ΠΌΡ‹. Π’ ΠΎΠ΄Π½ΠΎΠΉ систСмС отсчСта матСматичСская Ρ„ΠΎΡ€ΠΌΠ° условий равновСсия ΠΌΠΎΠΆΠ΅Ρ‚ Π±Ρ‹Ρ‚ΡŒ довольно слоТной, Ρ‚ΠΎΠ³Π΄Π° ΠΊΠ°ΠΊ Π² Π΄Ρ€ΡƒΠ³ΠΎΠΉ систСмС ΠΊΠΎΠΎΡ€Π΄ΠΈΠ½Π°Ρ‚ Ρ‚Π΅ ΠΆΠ΅ условия ΠΌΠΎΠ³ΡƒΡ‚ ΠΈΠΌΠ΅Ρ‚ΡŒ Π±ΠΎΠ»Π΅Π΅ ΠΏΡ€ΠΎΡΡ‚ΡƒΡŽ ΠΌΠ°Ρ‚Π΅ΠΌΠ°Ρ‚ΠΈΡ‡Π΅ΡΠΊΡƒΡŽ Ρ„ΠΎΡ€ΠΌΡƒ, ΠΊΠΎΡ‚ΠΎΡ€ΡƒΡŽ Π»Π΅Π³ΠΊΠΎ Ρ€Π΅ΡˆΠΈΡ‚ΡŒ.Начало Π²Ρ‹Π±Ρ€Π°Π½Π½ΠΎΠΉ систСмы отсчСта называСтся Ρ‚ΠΎΡ‡ΠΊΠΎΠΉ ΠΏΠΎΠ²ΠΎΡ€ΠΎΡ‚Π° .

Π’ самом ΠΎΠ±Ρ‰Π΅ΠΌ случаС условия равновСсия Π²Ρ‹Ρ€Π°ΠΆΠ°ΡŽΡ‚ΡΡ ΡˆΠ΅ΡΡ‚ΡŒΡŽ скалярными уравнСниями ((Рисунок) ΠΈ (Рисунок)). Для плоских Π·Π°Π΄Π°Ρ‡ равновСсия с Π²Ρ€Π°Ρ‰Π΅Π½ΠΈΠ΅ΠΌ Π²ΠΎΠΊΡ€ΡƒΠ³ фиксированной оси, ΠΊΠΎΡ‚ΠΎΡ€Ρ‹Π΅ ΠΌΡ‹ рассматриваСм Π² этой Π³Π»Π°Π²Π΅, ΠΌΡ‹ ΠΌΠΎΠΆΠ΅ΠΌ ΡΠΎΠΊΡ€Π°Ρ‚ΠΈΡ‚ΡŒ количСство ΡƒΡ€Π°Π²Π½Π΅Π½ΠΈΠΉ Π΄ΠΎ Ρ‚Ρ€Π΅Ρ…. Бтандартная ΠΏΡ€ΠΎΡ†Π΅Π΄ΡƒΡ€Π° состоит Π² Ρ‚ΠΎΠΌ, Ρ‡Ρ‚ΠΎΠ±Ρ‹ ΠΏΡ€ΠΈΠ½ΡΡ‚ΡŒ систСму отсчСта, Π² ΠΊΠΎΡ‚ΠΎΡ€ΠΎΠΉ ось z являСтся осью вращСния. ΠŸΡ€ΠΈ Ρ‚Π°ΠΊΠΎΠΌ Π²Ρ‹Π±ΠΎΡ€Π΅ оси чистый крутящий ΠΌΠΎΠΌΠ΅Π½Ρ‚ ΠΈΠΌΠ΅Π΅Ρ‚ Ρ‚ΠΎΠ»ΡŒΠΊΠΎ ΠΊΠΎΠΌΠΏΠΎΠ½Π΅Π½Ρ‚ z , всС силы, ΠΊΠΎΡ‚ΠΎΡ€Ρ‹Π΅ ΠΈΠΌΠ΅ΡŽΡ‚ Π½Π΅Π½ΡƒΠ»Π΅Π²Ρ‹Π΅ крутящиС ΠΌΠΎΠΌΠ΅Π½Ρ‚Ρ‹, Π»Π΅ΠΆΠ°Ρ‚ Π² плоскости xy , ΠΈ, ΡΠ»Π΅Π΄ΠΎΠ²Π°Ρ‚Π΅Π»ΡŒΠ½ΠΎ, Π²ΠΊΠ»Π°Π΄ Π² чистый крутящий ΠΌΠΎΠΌΠ΅Π½Ρ‚ поступаСт Ρ‚ΠΎΠ»ΡŒΠΊΠΎ ΠΎΡ‚ x — ΠΈ y — ΡΠΎΡΡ‚Π°Π²Π»ΡΡŽΡ‰ΠΈΠ΅ Π²Π½Π΅ΡˆΠ½ΠΈΡ… сил.Π’Π°ΠΊΠΈΠΌ ΠΎΠ±Ρ€Π°Π·ΠΎΠΌ, для плоских Π·Π°Π΄Π°Ρ‡ с осью вращСния, пСрпСндикулярной плоскости xy , ΠΌΡ‹ ΠΈΠΌΠ΅Π΅ΠΌ ΡΠ»Π΅Π΄ΡƒΡŽΡ‰ΠΈΠ΅ Ρ‚Ρ€ΠΈ условия равновСсия для сил ΠΈ ΠΌΠΎΠΌΠ΅Π½Ρ‚ΠΎΠ²:

[латСкс] {F} _ {1x} + {F} _ {2x} + \ text {β‹―} + {F} _ {Nx} = 0 [/ латСкс]

[латСкс] {F} _ {1y} + {F} _ {2y} + \ text {β‹―} + {F} _ {Ny} = 0 [/ латСкс]

[латСкс] {\ tau} _ {1} + {\ tau} _ {2} + \ text {β‹―} + {\ tau} _ {N} = 0 [/ латСкс]

, Π³Π΄Π΅ суммированиС вСдСтся ΠΏΠΎ всСм внСшним силам N , Π΄Π΅ΠΉΡΡ‚Π²ΡƒΡŽΡ‰ΠΈΠΌ Π½Π° Ρ‚Π΅Π»ΠΎ, ΠΈ ΠΈΡ… крутящим ΠΌΠΎΠΌΠ΅Π½Ρ‚Π°ΠΌ.На (Рисунок) ΠΌΡ‹ упростили обозначСния, опустив индСкс z , Π½ΠΎ ΠΌΡ‹ ΠΏΠΎΠ½ΠΈΠΌΠ°Π΅ΠΌ, Ρ‡Ρ‚ΠΎ здСсь суммированиС вСдСтся ΠΏΠΎ всСм Π²ΠΊΠ»Π°Π΄Π°ΠΌ вдоль оси z , которая являСтся осью вращСния. На (Рисунок) z -ΠΊΠΎΠΌΠΏΠΎΠ½Π΅Π½Ρ‚ крутящСго ΠΌΠΎΠΌΠ΅Π½Ρ‚Π° [латСкс] {\ overset {\ to} {\ tau}} _ {k} [/ latex] ΠΎΡ‚ силы [латСкс] {\ overset {\ to} { F}} _ {k} [/ latex] —

[латСкс] {\ tau} _ {k} = {r} _ {k} {F} _ {k} \ text {sin} \, \ theta [/ latex]

Π³Π΄Π΅ [латСкс] {r} _ {k} [/ latex] — Π΄Π»ΠΈΠ½Π° ΠΏΠ»Π΅Ρ‡Π° Ρ€Ρ‹Ρ‡Π°Π³Π° силы, Π° [latex] {F} _ {k} [/ latex] — Π²Π΅Π»ΠΈΡ‡ΠΈΠ½Π° силы (ΠΊΠ°ΠΊ Π²Ρ‹ ΠΏΠΈΠ»Π° Π² Ρ€Π΅ΠΆΠΈΠΌΠ΅ вращСния с фиксированной осью).Π£Π³ΠΎΠ» [latex] \ theta [/ latex] — это ΡƒΠ³ΠΎΠ» ΠΌΠ΅ΠΆΠ΄Ρƒ Π²Π΅ΠΊΡ‚ΠΎΡ€Π°ΠΌΠΈ [latex] {\ overset {\ to} {r}} _ {k} [/ latex] ΠΈ [latex] {\ overset {\ to} { F}} _ {k}, [/ latex] ΠΈΠ·ΠΌΠ΅Ρ€Π΅Π½ΠΈΠ΅ ΠΎΡ‚ Π²Π΅ΠΊΡ‚ΠΎΡ€Π° [latex] {\ overset {\ to} {r}} _ {k} [/ latex] Π΄ΠΎ Π²Π΅ΠΊΡ‚ΠΎΡ€Π° [latex] {\ overset { \ to} {F}} _ {k} [/ latex] Π² Π½Π°ΠΏΡ€Π°Π²Π»Π΅Π½ΠΈΠΈ ΠΏΡ€ΠΎΡ‚ΠΈΠ² часовой стрСлки ((рисунок)). ΠŸΡ€ΠΈ использовании (Рисунок) ΠΌΡ‹ часто вычисляСм Π²Π΅Π»ΠΈΡ‡ΠΈΠ½Ρƒ крутящСго ΠΌΠΎΠΌΠ΅Π½Ρ‚Π° ΠΈ Π½Π°Π·Π½Π°Ρ‡Π°Π΅ΠΌ Π΅Π³ΠΎ Π·Π½Π°Ρ‡Π΅Π½ΠΈΠ΅ Π»ΠΈΠ±ΠΎ ΠΏΠΎΠ»ΠΎΠΆΠΈΡ‚Π΅Π»ΡŒΠ½Ρ‹ΠΌ [латСкс] (+) [/ латСкс], Π»ΠΈΠ±ΠΎ ΠΎΡ‚Ρ€ΠΈΡ†Π°Ρ‚Π΅Π»ΡŒΠ½Ρ‹ΠΌ [латСкс] (-), [/ латСкс] Π² зависимости ΠΎΡ‚ направлСния вращСния, Π²Ρ‹Π·Π²Π°Π½Π½ΠΎΠ³ΠΎ Ρ‚ΠΎΠ»ΡŒΠΊΠΎ этим крутящим ΠΌΠΎΠΌΠ΅Π½Ρ‚ΠΎΠΌ.На (Рисунок) чистый крутящий ΠΌΠΎΠΌΠ΅Π½Ρ‚ прСдставляСт собой сумму Ρ‡Π»Π΅Π½ΠΎΠ², ΠΊΠ°ΠΆΠ΄Ρ‹ΠΉ Ρ‡Π»Π΅Π½ рассчитываСтся ΠΈΠ· (Рисунок), ΠΈ ΠΊΠ°ΠΆΠ΄Ρ‹ΠΉ Ρ‡Π»Π΅Π½ Π΄ΠΎΠ»ΠΆΠ΅Π½ ΠΈΠΌΠ΅Ρ‚ΡŒ ΠΏΡ€Π°Π²ΠΈΠ»ΡŒΠ½ΠΎΠ΅ Π·Π½Π°Ρ‡Π΅Π½ΠΈΠ΅ . Π’ΠΎΡ‡Π½ΠΎ Ρ‚Π°ΠΊ ΠΆΠ΅ Π½Π° (Рисунок) ΠΌΡ‹ Π½Π°Π·Π½Π°Ρ‡Π°Π΅ΠΌ Π·Π½Π°ΠΊ [latex] + [/ latex] ΠΊΠΎΠΌΠΏΠΎΠ½Π΅Π½Ρ‚Π°ΠΌ Π² Π½Π°ΠΏΡ€Π°Π²Π»Π΅Π½ΠΈΠΈ [latex] + [/ latex] x ΠΈ Π·Π½Π°ΠΊ [latex] — [/ latex] ΠΊΠΎΠΌΠΏΠΎΠ½Π΅Π½Ρ‚Π°ΠΌ. Π² Π½Π°ΠΏΡ€Π°Π²Π»Π΅Π½ΠΈΠΈ [латСкс] — [/ латСкс] x . Π­Ρ‚ΠΎ ΠΆΠ΅ ΠΏΡ€Π°Π²ΠΈΠ»ΠΎ Π΄ΠΎΠ»ΠΆΠ½ΠΎ ΠΏΠΎΡΠ»Π΅Π΄ΠΎΠ²Π°Ρ‚Π΅Π»ΡŒΠ½ΠΎ ΡΠΎΠ±Π»ΡŽΠ΄Π°Ρ‚ΡŒΡΡ Π½Π° (Рисунок) ΠΏΡ€ΠΈ вычислСнии ΡΠΎΡΡ‚Π°Π²Π»ΡΡŽΡ‰ΠΈΡ… силы ΠΏΠΎ оси y .

Рисунок 12.2 ΠšΡ€ΡƒΡ‚ΡΡ‰ΠΈΠΉ ΠΌΠΎΠΌΠ΅Π½Ρ‚ силы: (a) Когда крутящий ΠΌΠΎΠΌΠ΅Π½Ρ‚ силы Π²Ρ‹Π·Ρ‹Π²Π°Π΅Ρ‚ Π²Ρ€Π°Ρ‰Π΅Π½ΠΈΠ΅ ΠΏΡ€ΠΎΡ‚ΠΈΠ² часовой стрСлки Π²ΠΎΠΊΡ€ΡƒΠ³ оси вращСния, ΠΌΡ‹ Π³ΠΎΠ²ΠΎΡ€ΠΈΠΌ, Ρ‡Ρ‚ΠΎ Π΅Π³ΠΎ Π½Π°ΠΏΡ€Π°Π²Π»Π΅Π½ΠΈΠ΅ ΠΏΠΎΠ»ΠΎΠΆΠΈΡ‚Π΅Π»ΡŒΠ½ΠΎΠ΅, Ρ‡Ρ‚ΠΎ ΠΎΠ·Π½Π°Ρ‡Π°Π΅Ρ‚, Ρ‡Ρ‚ΠΎ Π²Π΅ΠΊΡ‚ΠΎΡ€ крутящСго ΠΌΠΎΠΌΠ΅Π½Ρ‚Π° ΠΏΠ°Ρ€Π°Π»Π»Π΅Π»Π΅Π½ оси вращСния. (b) Когда крутящий ΠΌΠΎΠΌΠ΅Π½Ρ‚ силы Π²Ρ‹Π·Ρ‹Π²Π°Π΅Ρ‚ Π²Ρ€Π°Ρ‰Π΅Π½ΠΈΠ΅ Π²ΠΎΠΊΡ€ΡƒΠ³ оси ΠΏΠΎ часовой стрСлкС, ΠΌΡ‹ Π³ΠΎΠ²ΠΎΡ€ΠΈΠΌ, Ρ‡Ρ‚ΠΎ Π΅Π³ΠΎ Π½Π°ΠΏΡ€Π°Π²Π»Π΅Π½ΠΈΠ΅ ΠΎΡ‚Ρ€ΠΈΡ†Π°Ρ‚Π΅Π»ΡŒΠ½ΠΎΠ΅, Ρ‡Ρ‚ΠΎ ΠΎΠ·Π½Π°Ρ‡Π°Π΅Ρ‚, Ρ‡Ρ‚ΠΎ Π²Π΅ΠΊΡ‚ΠΎΡ€ крутящСго ΠΌΠΎΠΌΠ΅Π½Ρ‚Π° Π°Π½Ρ‚ΠΈΠΏΠ°Ρ€Π°Π»Π»Π΅Π»Π΅Π½ оси вращСния.

Π’ΠΎ ΠΌΠ½ΠΎΠ³ΠΈΡ… ситуациях равновСсия ΠΎΠ΄Π½ΠΎΠΉ ΠΈΠ· сил, Π΄Π΅ΠΉΡΡ‚Π²ΡƒΡŽΡ‰ΠΈΡ… Π½Π° Ρ‚Π΅Π»ΠΎ, являСтся Π΅Π³ΠΎ вСс.На Π΄ΠΈΠ°Π³Ρ€Π°ΠΌΠΌΠ°Ρ… свободного Ρ‚Π΅Π»Π° Π²Π΅ΠΊΡ‚ΠΎΡ€ вСса привязан ΠΊ Ρ†Π΅Π½Ρ‚Ρ€Ρƒ тяТСсти Ρ‚Π΅Π»Π°. Для всСх практичСских Ρ†Π΅Π»Π΅ΠΉ Ρ†Π΅Π½Ρ‚Ρ€ тяТСсти ΠΈΠ΄Π΅Π½Ρ‚ΠΈΡ‡Π΅Π½ Ρ†Π΅Π½Ρ‚Ρ€Ρƒ масс, ΠΊΠ°ΠΊ Π²Ρ‹ ΡƒΠ·Π½Π°Π»ΠΈ ΠΈΠ· статСй Β«Π›ΠΈΠ½Π΅ΠΉΠ½Ρ‹ΠΉ ΠΈΠΌΠΏΡƒΠ»ΡŒΡΒ» ΠΈ «БтолкновСния» ΠΎ Π»ΠΈΠ½Π΅ΠΉΠ½ΠΎΠΌ ΠΈΠΌΠΏΡƒΠ»ΡŒΡΠ΅ ΠΈ столкновСниях. Волько Π² Ρ‚Π΅Ρ… случаях, ΠΊΠΎΠ³Π΄Π° Ρ‚Π΅Π»ΠΎ ΠΈΠΌΠ΅Π΅Ρ‚ Π±ΠΎΠ»ΡŒΡˆΡƒΡŽ ΠΏΡ€ΠΎΡΡ‚Ρ€Π°Π½ΡΡ‚Π²Π΅Π½Π½ΡƒΡŽ ΠΏΡ€ΠΎΡ‚ΡΠΆΠ΅Π½Π½ΠΎΡΡ‚ΡŒ, Ρ‚Π°ΠΊ Ρ‡Ρ‚ΠΎ Π³Ρ€Π°Π²ΠΈΡ‚Π°Ρ†ΠΈΠΎΠ½Π½ΠΎΠ΅ ΠΏΠΎΠ»Π΅ Π½Π΅ΠΎΠ΄Π½ΠΎΡ€ΠΎΠ΄Π½ΠΎ ΠΏΠΎ всСму Π΅Π³ΠΎ ΠΎΠ±ΡŠΠ΅ΠΌΡƒ, Ρ†Π΅Π½Ρ‚Ρ€ тяТСсти ΠΈ Ρ†Π΅Π½Ρ‚Ρ€ масс располоТСны Π² Ρ€Π°Π·Π½Ρ‹Ρ… Ρ‚ΠΎΡ‡ΠΊΠ°Ρ…. Однако Π½Π° ΠΏΡ€Π°ΠΊΡ‚ΠΈΠΊΠ΅ Π΄Π°ΠΆΠ΅ Ρ‚Π°ΠΊΠΈΠ΅ большиС ΠΎΠ±ΡŠΠ΅ΠΊΡ‚Ρ‹, ΠΊΠ°ΠΊ здания ΠΈΠ»ΠΈ ΠΊΡ€ΡƒΠΈΠ·Π½Ρ‹Π΅ Π»Π°ΠΉΠ½Π΅Ρ€Ρ‹, находятся Π² ΠΎΠ΄Π½ΠΎΡ€ΠΎΠ΄Π½ΠΎΠΌ Π³Ρ€Π°Π²ΠΈΡ‚Π°Ρ†ΠΈΠΎΠ½Π½ΠΎΠΌ ΠΏΠΎΠ»Π΅ Π½Π° повСрхности Π—Π΅ΠΌΠ»ΠΈ, Π³Π΄Π΅ ускорСниС свободного падСния ΠΈΠΌΠ΅Π΅Ρ‚ ΠΏΠΎΡΡ‚ΠΎΡΠ½Π½ΡƒΡŽ Π²Π΅Π»ΠΈΡ‡ΠΈΠ½Ρƒ [латСкс] g = 9.{2}. [/ latex] Π’ этих ситуациях Ρ†Π΅Π½Ρ‚Ρ€ тяТСсти ΠΈΠ΄Π΅Π½Ρ‚ΠΈΡ‡Π΅Π½ Ρ†Π΅Π½Ρ‚Ρ€Ρƒ масс. ΠŸΠΎΡΡ‚ΠΎΠΌΡƒ Π² этой Π³Π»Π°Π²Π΅ ΠΌΡ‹ ΠΈΡΠΏΠΎΠ»ΡŒΠ·ΡƒΠ΅ΠΌ Ρ†Π΅Π½Ρ‚Ρ€ масс (CM) ΠΊΠ°ΠΊ Ρ‚ΠΎΡ‡ΠΊΡƒ, ΠΊ ΠΊΠΎΡ‚ΠΎΡ€ΠΎΠΉ ΠΏΡ€ΠΈΠΊΡ€Π΅ΠΏΠ»Π΅Π½ Π²Π΅ΠΊΡ‚ΠΎΡ€ вСса. Напомним, Ρ‡Ρ‚ΠΎ ЦМ ΠΈΠΌΠ΅Π΅Ρ‚ особый физичСский смысл: ΠΊΠΎΠ³Π΄Π° внСшняя сила ΠΏΡ€ΠΈΠ»ΠΎΠΆΠ΅Π½Π° ΠΊ Ρ‚Π΅Π»Ρƒ Ρ‚ΠΎΡ‡Π½ΠΎ Π² Π΅Π³ΠΎ ЦМ, Ρ‚Π΅Π»ΠΎ Π² Ρ†Π΅Π»ΠΎΠΌ ΡΠΎΠ²Π΅Ρ€ΡˆΠ°Π΅Ρ‚ ΠΏΠΎΡΡ‚ΡƒΠΏΠ°Ρ‚Π΅Π»ΡŒΠ½ΠΎΠ΅ Π΄Π²ΠΈΠΆΠ΅Π½ΠΈΠ΅, ΠΈ такая сила Π½Π΅ Π²Ρ‹Π·Ρ‹Π²Π°Π΅Ρ‚ вращСния.

Когда ЦМ располоТСн Π²Π½Π΅ оси вращСния, Π½Π° ΠΎΠ±ΡŠΠ΅ΠΊΡ‚Π΅ Π²ΠΎΠ·Π½ΠΈΠΊΠ°Π΅Ρ‚ чистый Π³Ρ€Π°Π²ΠΈΡ‚Π°Ρ†ΠΈΠΎΠ½Π½Ρ‹ΠΉ ΠΌΠΎΠΌΠ΅Π½Ρ‚ .Π“Ρ€Π°Π²ΠΈΡ‚Π°Ρ†ΠΈΠΎΠ½Π½Ρ‹ΠΉ ΠΌΠΎΠΌΠ΅Π½Ρ‚ — это крутящий ΠΌΠΎΠΌΠ΅Π½Ρ‚, Π²Ρ‹Π·Π²Π°Π½Π½Ρ‹ΠΉ вСсом. Π­Ρ‚ΠΎΡ‚ Π³Ρ€Π°Π²ΠΈΡ‚Π°Ρ†ΠΈΠΎΠ½Π½Ρ‹ΠΉ ΠΌΠΎΠΌΠ΅Π½Ρ‚ ΠΌΠΎΠΆΠ΅Ρ‚ Π²Ρ€Π°Ρ‰Π°Ρ‚ΡŒ ΠΎΠ±ΡŠΠ΅ΠΊΡ‚, Ссли Π½Π΅Ρ‚ ΠΎΠΏΠΎΡ€Ρ‹ для Π΅Π³ΠΎ балансировки. Π’Π΅Π»ΠΈΡ‡ΠΈΠ½Π° Π³Ρ€Π°Π²ΠΈΡ‚Π°Ρ†ΠΈΠΎΠ½Π½ΠΎΠ³ΠΎ ΠΌΠΎΠΌΠ΅Π½Ρ‚Π° зависит ΠΎΡ‚ Ρ‚ΠΎΠ³ΠΎ, ΠΊΠ°ΠΊ Π΄Π°Π»Π΅ΠΊΠΎ ΠΎΡ‚ оси находится ЦМ. НапримСр, Π² случаС самосвала ((Рисунок)) ось ΠΏΠΎΠ²ΠΎΡ€ΠΎΡ‚Π° располоТСна Π½Π° Π»ΠΈΠ½ΠΈΠΈ, Π³Π΄Π΅ ΡˆΠΈΠ½Ρ‹ ΡΠΎΠΏΡ€ΠΈΠΊΠ°ΡΠ°ΡŽΡ‚ΡΡ с ΠΏΠΎΠ²Π΅Ρ€Ρ…Π½ΠΎΡΡ‚ΡŒΡŽ Π΄ΠΎΡ€ΠΎΠ³ΠΈ. Если CM располоТСн высоко Π½Π°Π΄ ΠΏΠΎΠ²Π΅Ρ€Ρ…Π½ΠΎΡΡ‚ΡŒΡŽ Π΄ΠΎΡ€ΠΎΠ³ΠΈ, Π³Ρ€Π°Π²ΠΈΡ‚Π°Ρ†ΠΈΠΎΠ½Π½Ρ‹ΠΉ ΠΌΠΎΠΌΠ΅Π½Ρ‚ ΠΌΠΎΠΆΠ΅Ρ‚ Π±Ρ‹Ρ‚ΡŒ достаточно большим, Ρ‡Ρ‚ΠΎΠ±Ρ‹ ΠΏΠ΅Ρ€Π΅Π²Π΅Ρ€Π½ΡƒΡ‚ΡŒ Π³Ρ€ΡƒΠ·ΠΎΠ²ΠΈΠΊ.Π›Π΅Π³ΠΊΠΎΠ²Ρ‹Π΅ Π°Π²Ρ‚ΠΎΠΌΠΎΠ±ΠΈΠ»ΠΈ с Π½ΠΈΠ·ΠΊΠΎ располоТСнной КМ, Π±Π»ΠΈΠ·ΠΊΠΎΠΉ ΠΊ Ρ‚Ρ€ΠΎΡ‚ΡƒΠ°Ρ€Ρƒ, Π±ΠΎΠ»Π΅Π΅ устойчивы ΠΊ ΠΎΠΏΡ€ΠΎΠΊΠΈΠ΄Ρ‹Π²Π°Π½ΠΈΡŽ, Ρ‡Π΅ΠΌ Π³Ρ€ΡƒΠ·ΠΎΠ²ΠΈΠΊΠΈ.

Рисунок 12.3 РаспрСдСлСниС массы влияСт Π½Π° ΠΏΠΎΠ»ΠΎΠΆΠ΅Π½ΠΈΠ΅ Ρ†Π΅Π½Ρ‚Ρ€Π° масс (CM), ΠΊ ΠΊΠΎΡ‚ΠΎΡ€ΠΎΠΌΡƒ ΠΏΡ€ΠΈΠΊΡ€Π΅ΠΏΠ»Π΅Π½ Π²Π΅ΠΊΡ‚ΠΎΡ€ вСса [latex] \ overset {\ to} {w} [/ latex]. Если Ρ†Π΅Π½Ρ‚Ρ€ тяТСсти находится Π² Π·ΠΎΠ½Π΅ ΠΎΠΏΠΎΡ€Ρ‹, ΠΏΠΎΠ³Ρ€ΡƒΠ·Ρ‡ΠΈΠΊ возвращаСтся Π² исходноС ΠΏΠΎΠ»ΠΎΠΆΠ΅Π½ΠΈΠ΅ послС опрокидывания [см. Π›Π΅Π²ΡƒΡŽ панСль Π² (b)]. Но Ссли Ρ†Π΅Π½Ρ‚Ρ€ тяТСсти находится Π·Π° ΠΏΡ€Π΅Π΄Π΅Π»Π°ΠΌΠΈ Π·ΠΎΠ½Ρ‹ ΠΎΠΏΠΎΡ€Ρ‹, Π³Ρ€ΡƒΠ·ΠΎΠ²ΠΈΠΊ пСрСвСрнСтся [см. ΠŸΡ€Π°Π²ΡƒΡŽ панСль Π² (b)].Оба транспортных срСдства Π² (b) находятся Π²Π½Π΅ равновСсия. ΠžΠ±Ρ€Π°Ρ‚ΠΈΡ‚Π΅ Π²Π½ΠΈΠΌΠ°Π½ΠΈΠ΅, Ρ‡Ρ‚ΠΎ Π°Π²Ρ‚ΠΎΠΌΠΎΠ±ΠΈΠ»ΡŒ Π½Π° (Π°) находится Π² равновСсии: Π½ΠΈΠ·ΠΊΠΎΠ΅ располоТСниС Ρ†Π΅Π½Ρ‚Ρ€Π° тяТСсти затрудняСт ΠΎΠΏΡ€ΠΎΠΊΠΈΠ΄Ρ‹Π²Π°Π½ΠΈΠ΅.

ΠŸΡ€ΠΈΠΌΠ΅Ρ€

Π¦Π΅Π½Ρ‚Ρ€ тяТСсти автомобиля

Π›Π΅Π³ΠΊΠΎΠ²ΠΎΠΉ Π°Π²Ρ‚ΠΎΠΌΠΎΠ±ΠΈΠ»ΡŒ с колСсной Π±Π°Π·ΠΎΠΉ 2,5 ΠΌ ΠΈΠΌΠ΅Π΅Ρ‚ 52% вСса Π½Π° ΠΏΠ΅Ρ€Π΅Π΄Π½ΠΈΠ΅ колСса Π½Π° Ρ€ΠΎΠ²Π½ΠΎΠΉ повСрхности, ΠΊΠ°ΠΊ ΠΏΠΎΠΊΠ°Π·Π°Π½ΠΎ Π½Π° (Рисунок). Π“Π΄Π΅ находится ЦМ этого автомобиля ΠΏΠΎ ΠΎΡ‚Π½ΠΎΡˆΠ΅Π½ΠΈΡŽ ΠΊ Π·Π°Π΄Π½Π΅ΠΉ оси?

Рисунок 12.4 РаспрСдСлСниС вСса ΠΌΠ΅ΠΆΠ΄Ρƒ осями автомобиля.Π“Π΄Π΅ находится Ρ†Π΅Π½Ρ‚Ρ€ тяТСсти?

БтратСгия

ВСс автомобиля w Π½Π°ΠΌ нСизвСстСн. ВсС, Ρ‡Ρ‚ΠΎ ΠΌΡ‹ Π·Π½Π°Π΅ΠΌ, это Ρ‚ΠΎ, Ρ‡Ρ‚ΠΎ ΠΊΠΎΠ³Π΄Π° Π°Π²Ρ‚ΠΎΠΌΠΎΠ±ΠΈΠ»ΡŒ стоит Π½Π° Ρ€ΠΎΠ²Π½ΠΎΠΉ повСрхности, 0,52 w Π΄Π°Π²ΠΈΡ‚ Π½Π° ΠΏΠΎΠ²Π΅Ρ€Ρ…Π½ΠΎΡΡ‚ΡŒ Π² Ρ‚ΠΎΡ‡ΠΊΠ°Ρ… ΠΊΠΎΠ½Ρ‚Π°ΠΊΡ‚Π° ΠΏΠ΅Ρ€Π΅Π΄Π½ΠΈΡ… колСс, Π° 0,48 w Ρ‚ΠΎΠ»ΠΊΠ°Π΅Ρ‚ Π²Π½ΠΈΠ· Π½Π° ΠΏΠΎΠ²Π΅Ρ€Ρ…Π½ΠΎΡΡ‚ΡŒ Π² Ρ‚ΠΎΡ‡ΠΊΠ°Ρ… ΠΊΠΎΠ½Ρ‚Π°ΠΊΡ‚Π° Π·Π°Π΄Π½ΠΈΡ… колСс. Π’Π°ΠΊΠΆΠ΅ Ρ‚ΠΎΡ‡ΠΊΠΈ ΠΊΠΎΠ½Ρ‚Π°ΠΊΡ‚Π° ΠΎΡ‚Π΄Π΅Π»Π΅Π½Ρ‹ Π΄Ρ€ΡƒΠ³ ΠΎΡ‚ Π΄Ρ€ΡƒΠ³Π° расстояниСм [латСкс] d = 2,5 \, \ text {m}. [/ latex] Π’ этих Ρ‚ΠΎΡ‡ΠΊΠ°Ρ… ΠΊΠΎΠ½Ρ‚Π°ΠΊΡ‚Π° Π°Π²Ρ‚ΠΎΠΌΠΎΠ±ΠΈΠ»ΡŒ испытываСт Π½ΠΎΡ€ΠΌΠ°Π»ΡŒΠ½Ρ‹Π΅ силы Ρ€Π΅Π°ΠΊΡ†ΠΈΠΈ с Π²Π΅Π»ΠΈΡ‡ΠΈΠ½ΠΎΠΉ [латСкс] {F} _ {\ text {F}} = 0.52 Π’Ρ‚ [/ латСкс] ΠΈ [латСкс] {F} _ {\ text {R}} = 0,48 Π’Ρ‚ [/ латСкс] Π½Π° ΠΏΠ΅Ρ€Π΅Π΄Π½Π΅ΠΉ ΠΈ Π·Π°Π΄Π½Π΅ΠΉ осях соотвСтствСнно. ΠœΡ‹ Ρ‚Π°ΠΊΠΆΠ΅ Π·Π½Π°Π΅ΠΌ, Ρ‡Ρ‚ΠΎ Π°Π²Ρ‚ΠΎΠΌΠΎΠ±ΠΈΠ»ΡŒ являСтся ΠΏΡ€ΠΈΠΌΠ΅Ρ€ΠΎΠΌ Ρ‚Π²Π΅Ρ€Π΄ΠΎΠ³ΠΎ Ρ‚Π΅Π»Π°, находящСгося Π² равновСсии, вСсь вСс ΠΊΠΎΡ‚ΠΎΡ€ΠΎΠ³ΠΎ w дСйствуСт Π½Π° Π΅Π³ΠΎ ЦМ. CM находится Π³Π΄Π΅-Ρ‚ΠΎ ΠΌΠ΅ΠΆΠ΄Ρƒ Ρ‚ΠΎΡ‡ΠΊΠ°ΠΌΠΈ, Π³Π΄Π΅ Π΄Π΅ΠΉΡΡ‚Π²ΡƒΡŽΡ‚ Π½ΠΎΡ€ΠΌΠ°Π»ΡŒΠ½Ρ‹Π΅ силы Ρ€Π΅Π°ΠΊΡ†ΠΈΠΈ, Π³Π΄Π΅-Ρ‚ΠΎ Π½Π° расстоянии x ΠΎΡ‚ Ρ‚ΠΎΡ‡ΠΊΠΈ, Π³Π΄Π΅ дСйствуСт [латСкс] {F} _ {R} [/ латСкс]. Наша Π·Π°Π΄Π°Ρ‡Π° Π½Π°ΠΉΡ‚ΠΈ Ρ… . Π’Π°ΠΊΠΈΠΌ ΠΎΠ±Ρ€Π°Π·ΠΎΠΌ, ΠΌΡ‹ ΠΈΠ΄Π΅Π½Ρ‚ΠΈΡ„ΠΈΡ†ΠΈΡ€ΡƒΠ΅ΠΌ Ρ‚Ρ€ΠΈ силы, Π΄Π΅ΠΉΡΡ‚Π²ΡƒΡŽΡ‰ΠΈΠ΅ Π½Π° Ρ‚Π΅Π»ΠΎ (Π°Π²Ρ‚ΠΎΠΌΠΎΠ±ΠΈΠ»ΡŒ), ΠΈ ΠΌΠΎΠΆΠ΅ΠΌ Π½Π°Ρ€ΠΈΡΠΎΠ²Π°Ρ‚ΡŒ Π΄ΠΈΠ°Π³Ρ€Π°ΠΌΠΌΡƒ свободного Ρ‚Π΅Π»Π° для Ρ€Π°ΡΡˆΠΈΡ€Π΅Π½Π½ΠΎΠ³ΠΎ Ρ‚Π²Π΅Ρ€Π΄ΠΎΠ³ΠΎ Ρ‚Π΅Π»Π°, ΠΊΠ°ΠΊ ΠΏΠΎΠΊΠ°Π·Π°Π½ΠΎ Π½Π° (Рисунок).

Рисунок 12.5 Π”ΠΈΠ°Π³Ρ€Π°ΠΌΠΌΠ° свободного Ρ‚Π΅Π»Π° для автомобиля Ρ‡Π΅Ρ‚ΠΊΠΎ ΡƒΠΊΠ°Π·Ρ‹Π²Π°Π΅Ρ‚ Π²Π΅ΠΊΡ‚ΠΎΡ€Ρ‹ сил, Π΄Π΅ΠΉΡΡ‚Π²ΡƒΡŽΡ‰ΠΈΡ… Π½Π° Π°Π²Ρ‚ΠΎΠΌΠΎΠ±ΠΈΠ»ΡŒ, ΠΈ расстояния Π΄ΠΎ Ρ†Π΅Π½Ρ‚Ρ€Π° масс (CM). Когда CM Π²Ρ‹Π±Ρ€Π°Π½ Π² качСствС Ρ‚ΠΎΡ‡ΠΊΠΈ ΠΏΠΎΠ²ΠΎΡ€ΠΎΡ‚Π°, эти расстояния ΠΏΡ€Π΅Π΄ΡΡ‚Π°Π²Π»ΡΡŽΡ‚ собой ΠΏΠ»Π΅Ρ‡ΠΈ Ρ€Ρ‹Ρ‡Π°Π³Π° Π½ΠΎΡ€ΠΌΠ°Π»ΡŒΠ½Ρ‹Ρ… сил Ρ€Π΅Π°ΠΊΡ†ΠΈΠΈ. ΠžΠ±Ρ€Π°Ρ‚ΠΈΡ‚Π΅ Π²Π½ΠΈΠΌΠ°Π½ΠΈΠ΅, Ρ‡Ρ‚ΠΎ Π²Π΅Π»ΠΈΡ‡ΠΈΠ½Ρ‹ Π²Π΅ΠΊΡ‚ΠΎΡ€ΠΎΠ² ΠΈ Ρ€Ρ‹Ρ‡Π°Π³ΠΈ Π½Π΅ Π½ΡƒΠΆΠ½ΠΎ Ρ€ΠΈΡΠΎΠ²Π°Ρ‚ΡŒ Π² ΠΌΠ°ΡΡˆΡ‚Π°Π±Π΅, Π½ΠΎ всС Ρ€Π΅Π»Π΅Π²Π°Π½Ρ‚Π½Ρ‹Π΅ Π²Π΅Π»ΠΈΡ‡ΠΈΠ½Ρ‹ Π΄ΠΎΠ»ΠΆΠ½Ρ‹ Π±Ρ‹Ρ‚ΡŒ Ρ‡Π΅Ρ‚ΠΊΠΎ ΠΎΠ±ΠΎΠ·Π½Π°Ρ‡Π΅Π½Ρ‹.

ΠœΡ‹ ΠΏΠΎΡ‡Ρ‚ΠΈ Π³ΠΎΡ‚ΠΎΠ²Ρ‹ Π·Π°ΠΏΠΈΡΠ°Ρ‚ΡŒ условия равновСсия (рисунок) — (рисунок) для автомобиля, Π½ΠΎ сначала ΠΌΡ‹ Π΄ΠΎΠ»ΠΆΠ½Ρ‹ ΠΎΠΏΡ€Π΅Π΄Π΅Π»ΠΈΡ‚ΡŒΡΡ с систСмой отсчСта.ΠŸΡ€Π΅Π΄ΠΏΠΎΠ»ΠΎΠΆΠΈΠΌ, ΠΌΡ‹ Π²Ρ‹Π±Ρ€Π°Π»ΠΈ ось x ΠΏΠΎ Π΄Π»ΠΈΠ½Π΅ ΠΊΠ°Π±ΠΈΠ½Ρ‹, ось y — Π²Π΅Ρ€Ρ‚ΠΈΠΊΠ°Π»ΡŒΠ½ΡƒΡŽ, Π° ось z — пСрпСндикулярно этой плоскости xy . ΠŸΡ€ΠΈ Ρ‚Π°ΠΊΠΎΠΌ Π²Ρ‹Π±ΠΎΡ€Π΅ Π½Π°ΠΌ Π½ΡƒΠΆΠ½ΠΎ Ρ‚ΠΎΠ»ΡŒΠΊΠΎ Π½Π°ΠΏΠΈΡΠ°Ρ‚ΡŒ (рисунок) ΠΈ (рисунок), ΠΏΠΎΡ‚ΠΎΠΌΡƒ Ρ‡Ρ‚ΠΎ всС ΠΊΠΎΠΌΠΏΠΎΠ½Π΅Π½Ρ‚Ρ‹ y тоТдСствСнно Ρ€Π°Π²Π½Ρ‹ Π½ΡƒΠ»ΡŽ. Π’Π΅ΠΏΠ΅Ρ€ΡŒ Π½Π°ΠΌ Π½ΡƒΠΆΠ½ΠΎ ΠΎΠΏΡ€Π΅Π΄Π΅Π»ΠΈΡ‚ΡŒΡΡ с располоТСниСм Ρ‚ΠΎΡ‡ΠΊΠΈ ΠΏΠΎΠ²ΠΎΡ€ΠΎΡ‚Π°. ΠœΡ‹ ΠΌΠΎΠΆΠ΅ΠΌ Π²Ρ‹Π±Ρ€Π°Ρ‚ΡŒ Π»ΡŽΠ±ΡƒΡŽ Ρ‚ΠΎΡ‡ΠΊΡƒ Π² качСствС мСстополоТСния оси вращСния ( z -ось). ΠŸΡ€Π΅Π΄ΠΏΠΎΠ»ΠΎΠΆΠΈΠΌ, ΠΌΡ‹ размСстили ось вращСния Π½Π° CM, ΠΊΠ°ΠΊ ΠΏΠΎΠΊΠ°Π·Π°Π½ΠΎ Π½Π° схСмС свободного Ρ‚Π΅Π»Π° для автомобиля.На этом этапС ΠΌΡ‹ Π³ΠΎΡ‚ΠΎΠ²Ρ‹ Π½Π°ΠΏΠΈΡΠ°Ρ‚ΡŒ условия равновСсия для автомобиля.

РСшСниС

КаТдоС условиС равновСсия содСрТит Ρ‚ΠΎΠ»ΡŒΠΊΠΎ Ρ‚Ρ€ΠΈ Ρ‡Π»Π΅Π½Π°, ΠΏΠΎΡ‚ΠΎΠΌΡƒ Ρ‡Ρ‚ΠΎ Π½Π° Π°Π²Ρ‚ΠΎΠΌΠΎΠ±ΠΈΠ»ΡŒ Π΄Π΅ΠΉΡΡ‚Π²ΡƒΡŽΡ‚ силы [latex] N = 3 [/ latex]. ΠŸΠ΅Ρ€Π²ΠΎΠ΅ условиС равновСсия (рисунок) читаСтся ΠΊΠ°ΠΊ

.

[латСкс] + {F} _ {\ text {F}} — w + {F} _ {\ text {R}} = 0. [/ латСкс]

Π­Ρ‚ΠΎ условиС Ρ‚Ρ€ΠΈΠ²ΠΈΠ°Π»ΡŒΠ½ΠΎ выполняСтся, ΠΏΠΎΡ‚ΠΎΠΌΡƒ Ρ‡Ρ‚ΠΎ, ΠΊΠΎΠ³Π΄Π° ΠΌΡ‹ подставляСм Π΄Π°Π½Π½Ρ‹Π΅, (рисунок) становится [латСкс] + 0,52w-w + 0,48w = 0. [/ latex] Π’Ρ‚ΠΎΡ€ΠΎΠ΅ условиС равновСсия (рисунок) читаСтся ΠΊΠ°ΠΊ

.

[латСкс] {\ tau} _ {\ text {F}} + {\ tau} _ {w} + {\ tau} _ {\ text {R}} = 0 [/ латСкс]

Π³Π΄Π΅ [латСкс] {\ tau} _ {\ text {F}} [/ latex] — это крутящий ΠΌΠΎΠΌΠ΅Π½Ρ‚ силы [латСкс] {F} _ {\ text {F}}, \, {\ tau} _ {w } [/ latex] — это Π³Ρ€Π°Π²ΠΈΡ‚Π°Ρ†ΠΈΠΎΠ½Π½Ρ‹ΠΉ ΠΌΠΎΠΌΠ΅Π½Ρ‚ силы w , Π° [latex] {\ tau} _ {\ text {R}} [/ latex] — это крутящий ΠΌΠΎΠΌΠ΅Π½Ρ‚ силы [latex] {F} _ {\ text {Π }}.[/ latex] Когда ось располоТСна Π² CM, Π³Ρ€Π°Π²ΠΈΡ‚Π°Ρ†ΠΈΠΎΠ½Π½Ρ‹ΠΉ ΠΌΠΎΠΌΠ΅Π½Ρ‚ ΠΈΠ΄Π΅Π½Ρ‚ΠΈΡ‡Π½ΠΎ Π½ΡƒΠ»ΡŽ, ΠΏΠΎΡ‚ΠΎΠΌΡƒ Ρ‡Ρ‚ΠΎ ΠΏΠ»Π΅Ρ‡ΠΎ Ρ€Ρ‹Ρ‡Π°Π³Π° вСса ΠΎΡ‚Π½ΠΎΡΠΈΡ‚Π΅Π»ΡŒΠ½ΠΎ оси, которая ΠΏΡ€ΠΎΡ…ΠΎΠ΄ΠΈΡ‚ Ρ‡Π΅Ρ€Π΅Π· CM, Ρ€Π°Π²Π½ΠΎ Π½ΡƒΠ»ΡŽ. Π›ΠΈΠ½ΠΈΠΈ дСйствия ΠΎΠ±Π΅ΠΈΡ… Π½ΠΎΡ€ΠΌΠ°Π»ΡŒΠ½Ρ‹Ρ… сил Ρ€Π΅Π°ΠΊΡ†ΠΈΠΈ пСрпСндикулярны ΠΏΠ»Π΅Ρ‡Π°ΠΌ ΠΈΡ… Ρ€Ρ‹Ρ‡Π°Π³ΠΎΠ², поэтому Π½Π° (Рисунок) ΠΌΡ‹ ΠΈΠΌΠ΅Π΅ΠΌ [latex] | \, \ text {sin} \, \ theta | = 1 [/ latex] для ΠΎΠ±Π΅ΠΈΡ… сил. Из Π΄ΠΈΠ°Π³Ρ€Π°ΠΌΠΌΡ‹ свободного Ρ‚Π΅Π»Π° ΠΌΡ‹ Ρ‡ΠΈΡ‚Π°Π΅ΠΌ, Ρ‡Ρ‚ΠΎ крутящий ΠΌΠΎΠΌΠ΅Π½Ρ‚ [латСкс] {\ tau} _ {\ text {F}} [/ latex] Π²Ρ‹Π·Ρ‹Π²Π°Π΅Ρ‚ Π²Ρ€Π°Ρ‰Π΅Π½ΠΈΠ΅ ΠΏΠΎ часовой стрСлкС Π²ΠΎΠΊΡ€ΡƒΠ³ оси Π² CM, поэтому Π΅Π³ΠΎ смысл ΠΎΡ‚Ρ€ΠΈΡ†Π°Ρ‚Π΅Π»ΡŒΠ½Ρ‹ΠΉ; ΠΈ крутящий ΠΌΠΎΠΌΠ΅Π½Ρ‚ [latex] {\ tau} _ {\ text {R}} [/ latex] Π²Ρ‹Π·Ρ‹Π²Π°Π΅Ρ‚ Π²Ρ€Π°Ρ‰Π΅Π½ΠΈΠ΅ ΠΏΡ€ΠΎΡ‚ΠΈΠ² часовой стрСлки Π²ΠΎΠΊΡ€ΡƒΠ³ оси Π² CM, поэтому Π΅Π³ΠΎ смысл ΠΏΠΎΠ»ΠΎΠΆΠΈΡ‚Π΅Π»ΡŒΠ½Ρ‹ΠΉ.ИмСя эту ΠΈΠ½Ρ„ΠΎΡ€ΠΌΠ°Ρ†ΠΈΡŽ, запишСм Π²Ρ‚ΠΎΡ€ΠΎΠ΅ условиС равновСсия ΠΊΠ°ΠΊ

[латСкс] \ text {-} {r} _ {\ text {F}} {F} _ {\ text {F}} + {r} _ {\ text {R}} {F} _ {\ text {R}} = 0. [/ латСкс]

Π‘ ΠΏΠΎΠΌΠΎΡ‰ΡŒΡŽ Π΄ΠΈΠ°Π³Ρ€Π°ΠΌΠΌΡ‹ свободного Ρ‚Π΅Π»Π° ΠΌΡ‹ опрСдСляСм Π²Π΅Π»ΠΈΡ‡ΠΈΠ½Ρ‹ силы [латСкс] {F} _ {\ text {R}} = 0,48w [/ латСкс] ΠΈ [латСкс] {F} _ {\ text {F }} = 0,52w, [/ latex] ΠΈ ΡΠΎΠΎΡ‚Π²Π΅Ρ‚ΡΡ‚Π²ΡƒΡŽΡ‰ΠΈΠ΅ ΠΈΠΌ ΠΏΠ»Π΅Ρ‡ΠΈ [латСкс] {r} _ {\ text {R}} = x [/ latex] ΠΈ [latex] {r} _ {\ text {F}} = dx. [/ latex] Π’Π΅ΠΏΠ΅Ρ€ΡŒ ΠΌΡ‹ ΠΌΠΎΠΆΠ΅ΠΌ Π·Π°ΠΏΠΈΡΠ°Ρ‚ΡŒ Π²Ρ‚ΠΎΡ€ΠΎΠ΅ условиС равновСсия (рисунок) Π² явном Π²ΠΈΠ΄Π΅ Π² Ρ‚Π΅Ρ€ΠΌΠΈΠ½Π°Ρ… нСизвСстного расстояния x :

[латСкс] -0.52 (Π³-Ρ…) ш + 0,48 Ρ… ш = 0. [/ латСкс]

Π—Π΄Π΅ΡΡŒ вСс w отмСняСтся, ΠΈ ΠΌΡ‹ ΠΌΠΎΠΆΠ΅ΠΌ Ρ€Π΅ΡˆΠΈΡ‚ΡŒ ΡƒΡ€Π°Π²Π½Π΅Π½ΠΈΠ΅ для нСизвСстного полоТСния x CM. ΠžΡ‚Π²Π΅Ρ‚: [латСкс] x = 0,52d = 0,52 (2,5 \, \ text {m}) = 1,3 \, \ text {m} \ text {.} [/ Latex]

РСшСниС

Π’Ρ‹Π±ΠΎΡ€ ΡˆΠ°Ρ€Π½ΠΈΡ€Π° Π² ΠΏΠΎΠ»ΠΎΠΆΠ΅Π½ΠΈΠΈ ΠΏΠ΅Ρ€Π΅Π΄Π½Π΅ΠΉ оси Π½Π΅ мСняСт Ρ€Π΅Π·ΡƒΠ»ΡŒΡ‚Π°Ρ‚Π°. Π‘Ρ…Π΅ΠΌΠ° свободного Ρ‚Π΅Π»Π° для этого мСста ΠΏΠΎΠ²ΠΎΡ€ΠΎΡ‚Π° прСдставлСна ​​на (Рисунок). Для этого Π²Ρ‹Π±ΠΎΡ€Π° Ρ‚ΠΎΡ‡ΠΊΠΈ ΠΏΠΎΠ²ΠΎΡ€ΠΎΡ‚Π° Π²Ρ‚ΠΎΡ€ΠΎΠ΅ условиС равновСсия Ρ€Π°Π²Π½ΠΎ

.

[латСкс] \ text {-} {r} _ {w} w + {r} _ {\ text {R}} {F} _ {\ text {R}} = 0.[/ латСкс]

Когда ΠΌΡ‹ подставляСм числа, ΡƒΠΊΠ°Π·Π°Π½Π½Ρ‹Π΅ Π½Π° Π΄ΠΈΠ°Π³Ρ€Π°ΠΌΠΌΠ΅, ΠΏΠΎΠ»ΡƒΡ‡Π°Π΅ΠΌ

[латСкс] \ text {-} (d-x) w + 0,48dw = 0. [/ латСкс]

ΠžΡ‚Π²Π΅Ρ‚, ΠΏΠΎΠ»ΡƒΡ‡Π΅Π½Π½Ρ‹ΠΉ ΠΏΡƒΡ‚Π΅ΠΌ Ρ€Π΅ΡˆΠ΅Π½ΠΈΡ (рисунок), ΠΎΠΏΡΡ‚ΡŒ ΠΆΠ΅, [латСкс] x = 0,52d = 1,3 \, \ text {m}. [/ латСкс]

Рисунок 12.6 ЭквивалСнтная Π΄ΠΈΠ°Π³Ρ€Π°ΠΌΠΌΠ° свободного Ρ‚Π΅Π»Π° для автомобиля; Ρ‚ΠΎΡ‡ΠΊΠ° ΠΏΠΎΠ²ΠΎΡ€ΠΎΡ‚Π° Ρ‡Π΅Ρ‚ΠΊΠΎ ΠΎΠ±ΠΎΠ·Π½Π°Ρ‡Π΅Π½Π°.

Π—Π½Π°Ρ‡Π΅Π½ΠΈΠ΅

Π­Ρ‚ΠΎΡ‚ ΠΏΡ€ΠΈΠΌΠ΅Ρ€ ΠΏΠΎΠΊΠ°Π·Ρ‹Π²Π°Π΅Ρ‚, Ρ‡Ρ‚ΠΎ ΠΏΡ€ΠΈ Ρ€Π΅ΡˆΠ΅Π½ΠΈΠΈ Π·Π°Π΄Π°Ρ‡ статичСского равновСсия ΠΌΡ‹ ΠΌΠΎΠΆΠ΅ΠΌ Π²Ρ‹Π±Ρ€Π°Ρ‚ΡŒ Ρ‚ΠΎΡ‡ΠΊΡƒ ΠΏΠΎΠ²ΠΎΡ€ΠΎΡ‚Π°.Для Ρ€Π°Π·Π»ΠΈΡ‡Π½Ρ‹Ρ… Π²Π°Ρ€ΠΈΠ°Π½Ρ‚ΠΎΠ² Π²Ρ‹Π±ΠΎΡ€Π° Ρ‚ΠΎΡ‡ΠΊΠΈ ΠΏΠΎΠ²ΠΎΡ€ΠΎΡ‚Π° Ρƒ нас Π΅ΡΡ‚ΡŒ Ρ€Π°Π·Π½Ρ‹Π΅ Π½Π°Π±ΠΎΡ€Ρ‹ условий равновСсия, ΠΊΠΎΡ‚ΠΎΡ€Ρ‹Π΅ Π½Π΅ΠΎΠ±Ρ…ΠΎΠ΄ΠΈΠΌΠΎ Ρ€Π΅ΡˆΠΈΡ‚ΡŒ. Однако любой Π²Ρ‹Π±ΠΎΡ€ ΠΏΡ€ΠΈΠ²ΠΎΠ΄ΠΈΡ‚ ΠΊ ΠΎΠ΄Π½ΠΎΠΌΡƒ ΠΈ Ρ‚ΠΎΠΌΡƒ ΠΆΠ΅ Ρ€Π΅ΡˆΠ΅Π½ΠΈΡŽ ΠΏΡ€ΠΎΠ±Π»Π΅ΠΌΡ‹.

ΠŸΡ€ΠΎΠ²Π΅Ρ€ΡŒΡ‚Π΅ своС ΠΏΠΎΠ½ΠΈΠΌΠ°Π½ΠΈΠ΅

Π Π΅ΡˆΠΈΡ‚Π΅ (рисунок), Π²Ρ‹Π±Ρ€Π°Π² ΡˆΠ°Ρ€Π½ΠΈΡ€ Π² мСстС располоТСния Π·Π°Π΄Π½Π΅ΠΉ оси.

ΠŸΠΎΠΊΠ°Π·Π°Ρ‚ΡŒ Ρ€Π΅ΡˆΠ΅Π½ΠΈΠ΅

[латСкс] x = 1,3 \, \ text {m} [/ латСкс]

ΠŸΡ€ΠΎΠ²Π΅Ρ€ΡŒΡ‚Π΅ своС ΠΏΠΎΠ½ΠΈΠΌΠ°Π½ΠΈΠ΅

ΠžΠ±ΡŠΡΡΠ½ΠΈΡ‚Π΅, какая ΠΈΠ· ΡΠ»Π΅Π΄ΡƒΡŽΡ‰ΠΈΡ… ситуаций удовлСтворяСт ΠΎΠ±ΠΎΠΈΠΌ условиям равновСсия: (Π°) тСннисный мяч, ΠΊΠΎΡ‚ΠΎΡ€Ρ‹ΠΉ Π½Π΅ вращаСтся ΠΏΡ€ΠΈ Π΄Π²ΠΈΠΆΠ΅Π½ΠΈΠΈ Π² Π²ΠΎΠ·Π΄ΡƒΡ…Π΅; (Π±) ΠΏΠ΅Π»ΠΈΠΊΠ°Π½, ΠΊΠΎΡ‚ΠΎΡ€Ρ‹ΠΉ ΠΏΠ°Ρ€ΠΈΡ‚ Π² Π²ΠΎΠ·Π΄ΡƒΡ…Π΅ с постоянной ΡΠΊΠΎΡ€ΠΎΡΡ‚ΡŒΡŽ Π½Π° ΠΎΠ΄Π½ΠΎΠΉ высотС; ΠΈΠ»ΠΈ (c) ΠΊΠΎΠ»Π΅Π½Ρ‡Π°Ρ‚Ρ‹ΠΉ Π²Π°Π» двигатСля ΠΏΡ€ΠΈΠΏΠ°Ρ€ΠΊΠΎΠ²Π°Π½Π½ΠΎΠ³ΠΎ автомобиля.

ΠžΡΠΎΠ±Ρ‹ΠΉ случай статичСского равновСсия Π²ΠΎΠ·Π½ΠΈΠΊΠ°Π΅Ρ‚, ΠΊΠΎΠ³Π΄Π° всС внСшниС силы Π½Π° ΠΎΠ±ΡŠΠ΅ΠΊΡ‚ Π΄Π΅ΠΉΡΡ‚Π²ΡƒΡŽΡ‚ Π½Π° оси вращСния ΠΈΠ»ΠΈ вдоль Π½Π΅Π΅, ΠΈΠ»ΠΈ ΠΊΠΎΠ³Π΄Π° пространствСнноС протяТСниС ΠΎΠ±ΡŠΠ΅ΠΊΡ‚Π° ΠΌΠΎΠΆΠ½ΠΎ Π½Π΅ ΠΏΡ€ΠΈΠ½ΠΈΠΌΠ°Ρ‚ΡŒ Π²ΠΎ Π²Π½ΠΈΠΌΠ°Π½ΠΈΠ΅. Π’ Ρ‚Π°ΠΊΠΎΠΌ случаС ΠΎΠ±ΡŠΠ΅ΠΊΡ‚ ΠΌΠΎΠΆΠ½ΠΎ эффСктивно Ρ€Π°ΡΡΠΌΠ°Ρ‚Ρ€ΠΈΠ²Π°Ρ‚ΡŒ ΠΊΠ°ΠΊ Ρ‚ΠΎΡ‡Π΅Ρ‡Π½ΡƒΡŽ массу. Π’ этом частном случаС Π½Π°ΠΌ Π½Π΅ Π½ΡƒΠΆΠ½ΠΎ Π±Π΅ΡΠΏΠΎΠΊΠΎΠΈΡ‚ΡŒΡΡ ΠΎ Π²Ρ‚ΠΎΡ€ΠΎΠΌ условии равновСсия (рисунок), ΠΏΠΎΡ‚ΠΎΠΌΡƒ Ρ‡Ρ‚ΠΎ всС крутящиС ΠΌΠΎΠΌΠ΅Π½Ρ‚Ρ‹ тоТдСствСнно Ρ€Π°Π²Π½Ρ‹ Π½ΡƒΠ»ΡŽ, Π° ΠΏΠ΅Ρ€Π²ΠΎΠ΅ условиС равновСсия (для сил) являСтся СдинствСнным условиСм, ΠΊΠΎΡ‚ΠΎΡ€ΠΎΠ΅ Π΄ΠΎΠ»ΠΆΠ½ΠΎ Π²Ρ‹ΠΏΠΎΠ»Π½ΡΡ‚ΡŒΡΡ. Π”ΠΈΠ°Π³Ρ€Π°ΠΌΠΌΠ° свободного Ρ‚Π΅Π»Π° ΠΈ стратСгия Ρ€Π΅ΡˆΠ΅Π½ΠΈΡ Π·Π°Π΄Π°Ρ‡ для этого особого случая Π±Ρ‹Π»ΠΈ ΠΈΠ·Π»ΠΎΠΆΠ΅Π½Ρ‹ Π² Β«Π—Π°ΠΊΠΎΠ½Π°Ρ… ΠΡŒΡŽΡ‚ΠΎΠ½Π°Β» ΠΈ Β«ΠŸΡ€ΠΈΠ»ΠΎΠΆΠ΅Π½ΠΈΡΡ… Π·Π°ΠΊΠΎΠ½ΠΎΠ² ΠΡŒΡŽΡ‚ΠΎΠ½Π°Β».Π’ ΡΠ»Π΅Π΄ΡƒΡŽΡ‰Π΅ΠΌ ΠΏΡ€ΠΈΠΌΠ΅Ρ€Π΅ Π²Ρ‹ ΡƒΠ²ΠΈΠ΄ΠΈΡ‚Π΅ Ρ‚ΠΈΠΏΠΈΡ‡Π½ΡƒΡŽ ΡΠΈΡ‚ΡƒΠ°Ρ†ΠΈΡŽ равновСсия, Π²ΠΊΠ»ΡŽΡ‡Π°ΡŽΡ‰ΡƒΡŽ Ρ‚ΠΎΠ»ΡŒΠΊΠΎ ΠΏΠ΅Ρ€Π²ΠΎΠ΅ условиС равновСсия.

ΠŸΡ€ΠΈΠΌΠ΅Ρ€

Π Π°Π·Ρ€Ρ‹Π²Π½ΠΎΠ΅ напряТСниС

МалСнькая ΠΊΠ°ΡΡ‚Ρ€ΡŽΠ»Ρ массой 42,0 Π³ поддСрТиваСтся двумя струнами, ΠΊΠ°ΠΊ ΠΏΠΎΠΊΠ°Π·Π°Π½ΠΎ Π½Π° (Рисунок). МаксимальноС натяТСниС, ΠΊΠΎΡ‚ΠΎΡ€ΠΎΠ΅ ΠΌΠΎΠΆΠ΅Ρ‚ Π²Ρ‹Π΄Π΅Ρ€ΠΆΠ°Ρ‚ΡŒ струна, составляСт 2,80 Н. Масса постСпСнно добавляСтся ΠΊ Ρ‡Π°ΡˆΠ΅, ΠΏΠΎΠΊΠ° ΠΎΠ΄Π½Π° ΠΈΠ· струн Π½Π΅ сломаСтся. Какая это струна? ΠšΠ°ΠΊΡƒΡŽ массу Π½ΡƒΠΆΠ½ΠΎ Π΄ΠΎΠ±Π°Π²ΠΈΡ‚ΡŒ, Ρ‡Ρ‚ΠΎΠ±Ρ‹ это ΠΏΡ€ΠΎΠΈΠ·ΠΎΡˆΠ»ΠΎ?

Рисунок 12.7 Масса постСпСнно добавляСтся ΠΊ ΠΊΠ°ΡΡ‚Ρ€ΡŽΠ»Π΅, ΠΏΠΎΠΊΠ° ΠΎΠ΄Π½Π° ΠΈΠ· струн Π½Π΅ Π»ΠΎΠΏΠ½Π΅Ρ‚.

БтратСгия

Π­Ρ‚Π° мСханичСская систСма, состоящая ΠΈΠ· струн, масс ΠΈ сковороды, находится Π² статичСском равновСсии. Π’ частности, ΡƒΠ·Π΅Π», ΠΊΠΎΡ‚ΠΎΡ€Ρ‹ΠΉ привязываСт струны ΠΊ ΠΊΠ°ΡΡ‚Ρ€ΡŽΠ»Π΅, находится Π² статичСском равновСсии. Π£Π·Π΅Π» ΠΌΠΎΠΆΠ½ΠΎ Ρ€Π°ΡΡΠΌΠ°Ρ‚Ρ€ΠΈΠ²Π°Ρ‚ΡŒ ΠΊΠ°ΠΊ Ρ‚ΠΎΡ‡ΠΊΡƒ; ΡΠ»Π΅Π΄ΠΎΠ²Π°Ρ‚Π΅Π»ΡŒΠ½ΠΎ, Π½Π°ΠΌ Π½ΡƒΠΆΠ½ΠΎ Ρ‚ΠΎΠ»ΡŒΠΊΠΎ ΠΏΠ΅Ρ€Π²ΠΎΠ΅ условиС равновСсия. Π’Ρ€ΠΈ силы, тянущиС ΠΊ ΡƒΠ·Π»Ρƒ: натяТСниС [латСкс] {\ overset {\ to} {T}} _ {1} [/ latex] Π² 5,0-сантимСтровой струнС, натяТСниС [латСкс] {\ overset {\ to } {T}} _ {2} [/ latex] Π² Π²Π΅Ρ€Π΅Π²ΠΊΠ΅ Π΄Π»ΠΈΠ½ΠΎΠΉ 10,0 см ΠΈ вСс [латСкс] \ overset {\ to} {w} [/ latex] сковороды, ΡƒΠ΄Π΅Ρ€ΠΆΠΈΠ²Π°ΡŽΡ‰Π΅ΠΉ Π³ΠΈΡ€ΠΈ.ΠœΡ‹ ΠΏΡ€ΠΈΠ½ΠΈΠΌΠ°Π΅ΠΌ ΠΏΡ€ΡΠΌΠΎΡƒΠ³ΠΎΠ»ΡŒΠ½ΡƒΡŽ систСму ΠΊΠΎΠΎΡ€Π΄ΠΈΠ½Π°Ρ‚ с осью y , Π½Π°ΠΏΡ€Π°Π²Π»Π΅Π½Π½ΠΎΠΉ ΠΏΡ€ΠΎΡ‚ΠΈΠ²ΠΎΠΏΠΎΠ»ΠΎΠΆΠ½ΠΎ Π½Π°ΠΏΡ€Π°Π²Π»Π΅Π½ΠΈΡŽ силы тяТСсти, ΠΈ рисуСм Π΄ΠΈΠ°Π³Ρ€Π°ΠΌΠΌΡƒ свободного Ρ‚Π΅Π»Π° для ΡƒΠ·Π»Π° (см. (Рисунок)). Π§Ρ‚ΠΎΠ±Ρ‹ Π½Π°ΠΉΡ‚ΠΈ ΠΊΠΎΠΌΠΏΠΎΠ½Π΅Π½Ρ‚Ρ‹ натяТСния, ΠΌΡ‹ Π΄ΠΎΠ»ΠΆΠ½Ρ‹ ΠΎΠΏΡ€Π΅Π΄Π΅Π»ΠΈΡ‚ΡŒ ΡƒΠ³Π»Ρ‹ направлСния [латСкс] {\ alpha} _ {1} [/ latex] ΠΈ [latex] {\ alpha} _ {2} [/ latex], ΠΊΠΎΡ‚ΠΎΡ€Ρ‹Π΅ ΠΎΠ±Ρ€Π°Π·ΡƒΡŽΡ‚ струны с Π³ΠΎΡ€ΠΈΠ·ΠΎΠ½Ρ‚Π°Π»ΡŒΠ½Ρ‹ΠΌ Π½Π°ΠΏΡ€Π°Π²Π»Π΅Π½ΠΈΠ΅ оси x . Как Π²Ρ‹ ΠΌΠΎΠΆΠ΅Ρ‚Π΅ Π²ΠΈΠ΄Π΅Ρ‚ΡŒ Π½Π° (Рисунок), струны ΡΠΎΡΡ‚Π°Π²Π»ΡΡŽΡ‚ Π΄Π²Π΅ стороны ΠΏΡ€ΡΠΌΠΎΡƒΠ³ΠΎΠ»ΡŒΠ½ΠΎΠ³ΠΎ Ρ‚Ρ€Π΅ΡƒΠ³ΠΎΠ»ΡŒΠ½ΠΈΠΊΠ°. ΠœΡ‹ ΠΌΠΎΠΆΠ΅ΠΌ ΠΈΡΠΏΠΎΠ»ΡŒΠ·ΠΎΠ²Π°Ρ‚ΡŒ Ρ‚Π΅ΠΎΡ€Π΅ΠΌΡƒ ΠŸΠΈΡ„Π°Π³ΠΎΡ€Π°, Ρ‡Ρ‚ΠΎΠ±Ρ‹ Ρ€Π΅ΡˆΠΈΡ‚ΡŒ этот Ρ‚Ρ€Π΅ΡƒΠ³ΠΎΠ»ΡŒΠ½ΠΈΠΊ, ΠΏΠΎΠΊΠ°Π·Π°Π½Π½Ρ‹ΠΉ Π½Π° (Рисунок), ΠΈ Π½Π°ΠΉΡ‚ΠΈ синус ΠΈ косинус ΡƒΠ³Π»ΠΎΠ² [латСкс] {\ alpha} _ {1} [/ latex] ΠΈ [latex] {\ alpha} _ {2}.[/ latex] Π—Π°Ρ‚Π΅ΠΌ ΠΌΡ‹ ΠΌΠΎΠΆΠ΅ΠΌ Ρ€Π°Π·Π»ΠΎΠΆΠΈΡ‚ΡŒ натяТСния Π½Π° ΠΈΡ… ΠΏΡ€ΡΠΌΠΎΡƒΠ³ΠΎΠ»ΡŒΠ½Ρ‹Π΅ ΡΠΎΡΡ‚Π°Π²Π»ΡΡŽΡ‰ΠΈΠ΅, ΠΏΠΎΠ΄ΡΡ‚Π°Π²ΠΈΡ‚ΡŒ Π² ΠΏΠ΅Ρ€Π²ΠΎΠ΅ условиС равновСсия ((Рисунок) ΠΈ (Рисунок)) ΠΈ Π½Π°ΠΉΡ‚ΠΈ натяТСния Π² струнах. ΠŸΠ΅Ρ€Π²ΠΎΠΉ порвСтся струна с большим натяТСниСм.

Рисунок 12.8 Π‘Ρ…Π΅ΠΌΠ° свободного Ρ‚Π΅Π»Π° для ΡƒΠ·Π»Π° Π½Π° (Рисунок).

РСшСниС

ВСс w , Π½Π°Ρ‚ΡΠ³ΠΈΠ²Π°ΡŽΡ‰ΠΈΠΉ ΡƒΠ·Π΅Π», обусловлСн массой M ΠΊΠ°ΡΡ‚Ρ€ΡŽΠ»ΠΈ ΠΈ массой m , Π΄ΠΎΠ±Π°Π²Π»Π΅Π½Π½ΠΎΠΉ ΠΊ ΠΊΠ°ΡΡ‚Ρ€ΡŽΠ»Π΅, ΠΈΠ»ΠΈ [латСкс] w = (M + m) g.[/ latex] Π‘ ΠΏΠΎΠΌΠΎΡ‰ΡŒΡŽ Π΄ΠΈΠ°Π³Ρ€Π°ΠΌΠΌΡ‹ свободного Ρ‚Π΅Π»Π° Π½Π° (Рисунок) ΠΌΡ‹ ΠΌΠΎΠΆΠ΅ΠΌ ΡƒΡΡ‚Π°Π½ΠΎΠ²ΠΈΡ‚ΡŒ условия равновСсия для ΡƒΠ·Π»Π°:

[латСкс] \ begin {array} {ccccc} \ text {Π² Π½Π°ΠΏΡ€Π°Π²Π»Π΅Π½ΠΈΠΈ} \, x \ text {-direction,} \ hfill & & \ hfill \ text {-} {T} _ {1x} + {T} _ {2x} & = \ hfill & 0 \ hfill \\ \ text {Π² Π½Π°ΠΏΡ€Π°Π²Π»Π΅Π½ΠΈΠΈ} \, y \ text {,} \ hfill & & \ hfill \ text {+} {T} _ {1y} + { T} _ {2y} -w & = \ hfill & 0. \ hfill \ end {array} [/ latex]

Из Π΄ΠΈΠ°Π³Ρ€Π°ΠΌΠΌΡ‹ свободного Ρ‚Π΅Π»Π°, Π²Π΅Π»ΠΈΡ‡ΠΈΠ½Ρ‹ ΠΊΠΎΠΌΠΏΠΎΠ½Π΅Π½Ρ‚ΠΎΠ² Π² этих уравнСниях Ρ€Π°Π²Π½Ρ‹

.

[латСкс] \ begin {array} {ccc} {T} _ {1x} = {T} _ {1} \ text {cos} \, {\ alpha} _ {1} = {T} _ {1} \ text {/} \ sqrt {5}, \ hfill & & {T} _ {1y} = {T} _ {1} \ text {sin} \, {\ alpha} _ {1} = 2 {T} _ {1} \ text {/} \ sqrt {5} \ hfill \\ {T} _ {2x} = {T} _ {2} \ text {cos} \, {\ alpha} _ {2} = 2 {T} _ {2} \ text {/} \ sqrt {5}, \ hfill & & {T} _ {2y} = {T} _ {2} \ text {sin} \, {\ alpha} _ { 2} = {T} _ {2} \ text {/} \ sqrt {5}.\ hfill \ end {array} [/ latex]

ΠŸΠΎΠ΄ΡΡ‚Π°Π²Π»ΡΠ΅ΠΌ эти ΠΊΠΎΠΌΠΏΠΎΠ½Π΅Π½Ρ‚Ρ‹ Π² условия равновСсия ΠΈ ΡƒΠΏΡ€ΠΎΡ‰Π°Π΅ΠΌ. Π—Π°Ρ‚Π΅ΠΌ ΠΌΡ‹ ΠΏΠΎΠ»ΡƒΡ‡Π°Π΅ΠΌ Π΄Π²Π° уравнСния равновСсия для натяТСний:

[латСкс] \ begin {array} {ccccc} \ text {in} \, x \ text {-direction,} \ hfill & & \ hfill {T} _ {1} & = \ hfill & 2 {T} _ {2} \ hfill \\ \ text {in} \, y \ text {-direction,} \ hfill & & \ hfill \ frac {2 {T} _ {1}} {\ sqrt {5}} + \ frac {{T} _ {2}} {\ sqrt {5}} & = \ hfill & (M + m) g. \ Hfill \ end {array} [/ latex]

Π£Ρ€Π°Π²Π½Π΅Π½ΠΈΠ΅ равновСсия для направлСния x Π³ΠΎΠ²ΠΎΡ€ΠΈΡ‚ Π½Π°ΠΌ, Ρ‡Ρ‚ΠΎ натяТСниС [латСкс] {T} _ {1} [/ латСкс] Π² 5.0-сантимСтровая струна Π²Π΄Π²ΠΎΠ΅ ΠΏΡ€Π΅Π²Ρ‹ΡˆΠ°Π΅Ρ‚ натяТСниС [латСкс] {T} _ {2} [/ latex] Π² 10-сантимСтровой струнС. Π’Π°ΠΊΠΈΠΌ ΠΎΠ±Ρ€Π°Π·ΠΎΠΌ, Π±ΠΎΠ»Π΅Π΅ короткая струна порвСтся. Когда ΠΌΡ‹ ΠΈΡΠΏΠΎΠ»ΡŒΠ·ΡƒΠ΅ΠΌ ΠΏΠ΅Ρ€Π²ΠΎΠ΅ ΡƒΡ€Π°Π²Π½Π΅Π½ΠΈΠ΅, Ρ‡Ρ‚ΠΎΠ±Ρ‹ ΠΈΡΠΊΠ»ΡŽΡ‡ΠΈΡ‚ΡŒ [латСкс] {T} _ {2} [/ latex] ΠΈΠ· Π²Ρ‚ΠΎΡ€ΠΎΠ³ΠΎ уравнСния, ΠΌΡ‹ ΠΏΠΎΠ»ΡƒΡ‡Π°Π΅ΠΌ ΡΠΎΠΎΡ‚Π½ΠΎΡˆΠ΅Π½ΠΈΠ΅ ΠΌΠ΅ΠΆΠ΄Ρƒ массой [латСкс] ΠΌ [/ латСкс] Π½Π° сковородС ΠΈ натяТСниСм [латСкс ] {T} _ {1} [/ latex] Π² Π±ΠΎΠ»Π΅Π΅ ΠΊΠΎΡ€ΠΎΡ‚ΠΊΠΎΠΉ строкС:

[латСкс] 2,5 {T} _ {1} \ text {/} \ sqrt {5} = (M + m) Π³. [/ латСкс]

Π‘Ρ‚Ρ€ΡƒΠ½Π° разрываСтся, ΠΊΠΎΠ³Π΄Π° натяТСниС достигаСт критичСского значСния [латСкс] {T} _ {1} = 2.{2}} — 0,042 \, \ text {kg} = 0,277 \, \ text {kg} = 277,0 \, \ text {g.} [/ Latex]

Π—Π½Π°Ρ‡Π΅Π½ΠΈΠ΅

ΠŸΡ€Π΅Π΄ΠΏΠΎΠ»ΠΎΠΆΠΈΠΌ, Ρ‡Ρ‚ΠΎ мСханичСская систСма, рассматриваСмая Π² этом ΠΏΡ€ΠΈΠΌΠ΅Ρ€Π΅, ΠΏΡ€ΠΈΠΊΡ€Π΅ΠΏΠ»Π΅Π½Π° ΠΊ ΠΏΠΎΡ‚ΠΎΠ»ΠΊΡƒ Π²Π½ΡƒΡ‚Ρ€ΠΈ Π»ΠΈΡ„Ρ‚Π°, ΠΏΠΎΠ΄Π½ΠΈΠΌΠ°ΡŽΡ‰Π΅Π³ΠΎΡΡ Π²Π²Π΅Ρ€Ρ…. Пока Π»ΠΈΡ„Ρ‚ двиТСтся Π²Π²Π΅Ρ€Ρ… с постоянной ΡΠΊΠΎΡ€ΠΎΡΡ‚ΡŒΡŽ, Ρ€Π΅Π·ΡƒΠ»ΡŒΡ‚Π°Ρ‚ остаСтся Π½Π΅ΠΈΠ·ΠΌΠ΅Π½Π½Ρ‹ΠΌ, ΠΏΠΎΡ‚ΠΎΠΌΡƒ Ρ‡Ρ‚ΠΎ вСс [латСкс] w [/ латСкс] Π½Π΅ мСняСтся. Если Π»ΠΈΡ„Ρ‚ двиТСтся Π²Π²Π΅Ρ€Ρ… с ускорСниСм, критичСская масса мСньшС, ΠΏΠΎΡ‚ΠΎΠΌΡƒ Ρ‡Ρ‚ΠΎ вСс [латСкса] M + m [/ латСкса] становится большС Π½Π° каТущийся вСс ΠΈΠ·-Π·Π° ускорСния Π»ΠΈΡ„Ρ‚Π°.Π’Π΅ΠΌ Π½Π΅ ΠΌΠ΅Π½Π΅Π΅, Π²ΠΎ всСх случаях Π±ΠΎΠ»Π΅Π΅ короткая струна рвСтся ΠΏΠ΅Ρ€Π²ΠΎΠΉ.

Π‘Π²ΠΎΠ΄ΠΊΠ°

  • Π’Π΅Π»ΠΎ находится Π² равновСсии, ΠΊΠΎΠ³Π΄Π° ΠΎΠ½ΠΎ остаСтся Π»ΠΈΠ±ΠΎ Π² Ρ€Π°Π²Π½ΠΎΠΌΠ΅Ρ€Π½ΠΎΠΌ Π΄Π²ΠΈΠΆΠ΅Π½ΠΈΠΈ (ΠΏΠΎΡΡ‚ΡƒΠΏΠ°Ρ‚Π΅Π»ΡŒΠ½ΠΎΠΌ ΠΈ Π²Ρ€Π°Ρ‰Π°Ρ‚Π΅Π»ΡŒΠ½ΠΎΠΌ), Π»ΠΈΠ±ΠΎ Π² состоянии покоя. Когда Ρ‚Π΅Π»ΠΎ Π² Π²Ρ‹Π±Ρ€Π°Π½Π½ΠΎΠΉ ΠΈΠ½Π΅Ρ€Ρ†ΠΈΠ°Π»ΡŒΠ½ΠΎΠΉ систСмС отсчСта Π½Π΅ вращаСтся ΠΈ Π½Π΅ двиТСтся Π² ΠΏΠΎΡΡ‚ΡƒΠΏΠ°Ρ‚Π΅Π»ΡŒΠ½ΠΎΠΌ Π΄Π²ΠΈΠΆΠ΅Π½ΠΈΠΈ, ΠΌΡ‹ Π³ΠΎΠ²ΠΎΡ€ΠΈΠΌ, Ρ‡Ρ‚ΠΎ Ρ‚Π΅Π»ΠΎ находится Π² статичСском равновСсии Π² этой систСмС отсчСта.
  • Условия равновСсия Ρ‚Ρ€Π΅Π±ΡƒΡŽΡ‚, Ρ‡Ρ‚ΠΎΠ±Ρ‹ сумма всСх Π²Π½Π΅ΡˆΠ½ΠΈΡ… сил, Π΄Π΅ΠΉΡΡ‚Π²ΡƒΡŽΡ‰ΠΈΡ… Π½Π° Ρ‚Π΅Π»ΠΎ, Π±Ρ‹Π»Π° Ρ€Π°Π²Π½Π° Π½ΡƒΠ»ΡŽ (ΠΏΠ΅Ρ€Π²ΠΎΠ΅ условиС равновСсия), Π° сумма всСх Π²Π½Π΅ΡˆΠ½ΠΈΡ… ΠΌΠΎΠΌΠ΅Π½Ρ‚ΠΎΠ² ΠΎΡ‚ Π²Π½Π΅ΡˆΠ½ΠΈΡ… сил Π±Ρ‹Π»Π° Ρ€Π°Π²Π½Π° Π½ΡƒΠ»ΡŽ (Π²Ρ‚ΠΎΡ€ΠΎΠ΅ условиС равновСсия).Π­Ρ‚ΠΈ Π΄Π²Π° условия Π΄ΠΎΠ»ΠΆΠ½Ρ‹ ΠΎΠ΄Π½ΠΎΠ²Ρ€Π΅ΠΌΠ΅Π½Π½ΠΎ Π²Ρ‹ΠΏΠΎΠ»Π½ΡΡ‚ΡŒΡΡ Π² состоянии равновСсия. Если ΠΎΠ΄ΠΈΠ½ ΠΈΠ· Π½ΠΈΡ… Π½Π΅ ΡƒΠ΄ΠΎΠ²Π»Π΅Ρ‚Π²ΠΎΡ€Π΅Π½, Ρ‚Π΅Π»ΠΎ Π½Π΅ находится Π² равновСсии.
  • Π”ΠΈΠ°Π³Ρ€Π°ΠΌΠΌΠ° свободного Ρ‚Π΅Π»Π° для Ρ‚Π΅Π»Π° — ΠΏΠΎΠ»Π΅Π·Π½Ρ‹ΠΉ инструмСнт, ΠΊΠΎΡ‚ΠΎΡ€Ρ‹ΠΉ позволяСт Π½Π°ΠΌ ΠΏΡ€Π°Π²ΠΈΠ»ΡŒΠ½ΠΎ ΠΏΠΎΠ΄ΡΡ‡ΠΈΡ‚Π°Ρ‚ΡŒ всС Π²ΠΊΠ»Π°Π΄Ρ‹ ΠΎΡ‚ всСх Π²Π½Π΅ΡˆΠ½ΠΈΡ… сил ΠΈ ΠΌΠΎΠΌΠ΅Π½Ρ‚ΠΎΠ², Π΄Π΅ΠΉΡΡ‚Π²ΡƒΡŽΡ‰ΠΈΡ… Π½Π° Ρ‚Π΅Π»ΠΎ. Π”ΠΈΠ°Π³Ρ€Π°ΠΌΠΌΡ‹ свободного Ρ‚Π΅Π»Π° для равновСсия вытянутого Ρ‚Π²Π΅Ρ€Π΄ΠΎΠ³ΠΎ Ρ‚Π΅Π»Π° Π΄ΠΎΠ»ΠΆΠ½Ρ‹ ΡƒΠΊΠ°Π·Ρ‹Π²Π°Ρ‚ΡŒ Ρ‚ΠΎΡ‡ΠΊΡƒ ΠΏΠΎΠ²ΠΎΡ€ΠΎΡ‚Π° ΠΈ ΠΏΠ»Π΅Ρ‡ΠΈ Ρ€Ρ‹Ρ‡Π°Π³Π° Π΄Π΅ΠΉΡΡ‚Π²ΡƒΡŽΡ‰ΠΈΡ… сил ΠΏΠΎ ΠΎΡ‚Π½ΠΎΡˆΠ΅Π½ΠΈΡŽ ΠΊ оси.

ΠšΠΎΠ½Ρ†Π΅ΠΏΡ‚ΡƒΠ°Π»ΡŒΠ½Ρ‹Π΅ вопросы

Π§Ρ‚ΠΎ Π²Ρ‹ ΠΌΠΎΠΆΠ΅Ρ‚Π΅ ΡΠΊΠ°Π·Π°Ρ‚ΡŒ ΠΎ скорости двиТущСгося Ρ‚Π΅Π»Π°, находящСгося Π² динамичСском равновСсии?

ΠŸΡ€ΠΈ ΠΊΠ°ΠΊΠΈΡ… условиях Π²Ρ€Π°Ρ‰Π°ΡŽΡ‰Π΅Π΅ΡΡ Ρ‚Π΅Π»ΠΎ ΠΌΠΎΠΆΠ΅Ρ‚ Π½Π°Ρ…ΠΎΠ΄ΠΈΡ‚ΡŒΡΡ Π² равновСсии? ΠŸΡ€ΠΈΠ²Π΅Π΄ΠΈΡ‚Π΅ ΠΏΡ€ΠΈΠΌΠ΅Ρ€.

КакиС Ρ‚Ρ€ΠΈ Ρ„Π°ΠΊΡ‚ΠΎΡ€Π° Π²Π»ΠΈΡΡŽΡ‚ Π½Π° крутящий ΠΌΠΎΠΌΠ΅Π½Ρ‚, создаваСмый силой ΠΎΡ‚Π½ΠΎΡΠΈΡ‚Π΅Π»ΡŒΠ½ΠΎ ΠΎΠΏΡ€Π΅Π΄Π΅Π»Π΅Π½Π½ΠΎΠΉ Ρ‚ΠΎΡ‡ΠΊΠΈ ΠΏΠΎΠ²ΠΎΡ€ΠΎΡ‚Π°?

ΠŸΠΎΠΊΠ°Π·Π°Ρ‚ΡŒ Ρ€Π΅ΡˆΠ΅Π½ΠΈΠ΅

Π’Π΅Π»ΠΈΡ‡ΠΈΠ½Π° ΠΈ Π½Π°ΠΏΡ€Π°Π²Π»Π΅Π½ΠΈΠ΅ силы, Π° Π΅Π΅ ΠΏΠ»Π΅Ρ‡ΠΎ Ρ€Ρ‹Ρ‡Π°Π³Π°

ΠœΠ΅Ρ…Π°Π½ΠΈΠΊΠΈ ΠΈΠ½ΠΎΠ³Π΄Π° ΠΊΠ»Π°Π΄ΡƒΡ‚ кусок Ρ‚Ρ€ΡƒΠ±Ρ‹ Π½Π° рукоятку Π³Π°Π΅Ρ‡Π½ΠΎΠ³ΠΎ ΠΊΠ»ΡŽΡ‡Π°, ΠΏΡ‹Ρ‚Π°ΡΡΡŒ ΠΎΡ‚ΠΊΡ€ΡƒΡ‚ΠΈΡ‚ΡŒ ΠΎΡ‡Π΅Π½ΡŒ Ρ‚ΡƒΠ³ΠΎΠΉ Π±ΠΎΠ»Ρ‚. Как это ΠΏΠΎΠΌΠΎΠ³Π°Π΅Ρ‚?

Для ΡΠ»Π΅Π΄ΡƒΡŽΡ‰ΠΈΡ… Ρ‡Π΅Ρ‚Ρ‹Ρ€Π΅Ρ… Π·Π°Π΄Π°Ρ‡ ΠΎΡ†Π΅Π½ΠΈΡ‚Π΅ ΡƒΡ‚Π²Π΅Ρ€ΠΆΠ΄Π΅Π½ΠΈΠ΅ ΠΊΠ°ΠΊ истинноС ΠΈΠ»ΠΈ Π»ΠΎΠΆΠ½ΠΎΠ΅ ΠΈ ΠΎΠ±ΡŠΡΡΠ½ΠΈΡ‚Π΅ свой ΠΎΡ‚Π²Π΅Ρ‚.

Если Π½Π° ΠΎΠ±ΡŠΠ΅ΠΊΡ‚ дСйствуСт Ρ‚ΠΎΠ»ΡŒΠΊΠΎ ΠΎΠ΄Π½Π° внСшняя сила (ΠΈΠ»ΠΈ крутящий ΠΌΠΎΠΌΠ΅Π½Ρ‚), ΠΎΠ½ Π½Π΅ ΠΌΠΎΠΆΠ΅Ρ‚ Π½Π°Ρ…ΠΎΠ΄ΠΈΡ‚ΡŒΡΡ Π² равновСсии.

ΠŸΠΎΠΊΠ°Π·Π°Ρ‚ΡŒ Ρ€Π΅ΡˆΠ΅Π½ΠΈΠ΅

Π’Π΅Ρ€Π½ΠΎ, ΠΏΠΎΡΠΊΠΎΠ»ΡŒΠΊΡƒ Π² этом случаС сумма сил Π½Π΅ ΠΌΠΎΠΆΠ΅Ρ‚ Π±Ρ‹Ρ‚ΡŒ Ρ€Π°Π²Π½Π° Π½ΡƒΠ»ΡŽ, Ссли сама сила Π½Π΅ Ρ€Π°Π²Π½Π° Π½ΡƒΠ»ΡŽ.

Если ΠΎΠ±ΡŠΠ΅ΠΊΡ‚ находится Π² равновСсии, Π½Π° Π½Π΅Π³ΠΎ Π΄ΠΎΠ»ΠΆΠ½ΠΎ Π΄Π΅ΠΉΡΡ‚Π²ΠΎΠ²Π°Ρ‚ΡŒ Ρ‡Π΅Ρ‚Π½ΠΎΠ΅ число сил.

Если Π½Π° ΠΎΠ±ΡŠΠ΅ΠΊΡ‚ дСйствуСт Π½Π΅Ρ‡Π΅Ρ‚Π½ΠΎΠ΅ количСство сил, ΠΎΠ±ΡŠΠ΅ΠΊΡ‚ Π½Π΅ ΠΌΠΎΠΆΠ΅Ρ‚ Π½Π°Ρ…ΠΎΠ΄ΠΈΡ‚ΡŒΡΡ Π² равновСсии.

ΠŸΠΎΠΊΠ°Π·Π°Ρ‚ΡŒ Ρ€Π΅ΡˆΠ΅Π½ΠΈΠ΅

Π›ΠΎΠΆΡŒ, Ссли силы ΡΠΊΠ»Π°Π΄Ρ‹Π²Π°ΡŽΡ‚ΡΡ Π² ноль ΠΊΠ°ΠΊ Π²Π΅ΠΊΡ‚ΠΎΡ€Ρ‹, Ρ‚ΠΎΠ³Π΄Π° ΠΌΠΎΠΆΠ΅Ρ‚ Π±Ρ‹Ρ‚ΡŒ достигнуто равновСсиС.

Π’Π΅Π»ΠΎ, двиТущССся ΠΏΠΎ ΠΊΡ€ΡƒΠ³Ρƒ с постоянной ΡΠΊΠΎΡ€ΠΎΡΡ‚ΡŒΡŽ, находится Π² равновСсии вращСния.

Для Ρ‡Π΅Π³ΠΎ Π½ΡƒΠΆΠ΅Π½ Π΄Π»ΠΈΠ½Π½Ρ‹ΠΉ ΠΈ Π³ΠΈΠ±ΠΊΠΈΠΉ ΡˆΠ΅ΡΡ‚, ΠΊΠΎΡ‚ΠΎΡ€Ρ‹ΠΉ пСрСносят ΠΊΠ°Π½Π°Ρ‚ΠΎΡ…ΠΎΠ΄Ρ†Ρ‹?

ΠŸΠΎΠΊΠ°Π·Π°Ρ‚ΡŒ Ρ€Π΅ΡˆΠ΅Π½ΠΈΠ΅

ΠŸΠΎΠΌΠΎΠ³Π°Π΅Ρ‚ ΠΊΠ°Π½Π°Ρ‚ΠΎΡ…ΠΎΠ΄Ρ†Ρƒ ΡΠΎΡ…Ρ€Π°Π½ΡΡ‚ΡŒ равновСсиС.

ΠŸΡ€ΠΎΠ±Π»Π΅ΠΌΡ‹

ΠŸΡ€ΠΈ затяТкС Π±ΠΎΠ»Ρ‚Π° Π²Ρ‹ Π½Π°ΠΆΠΈΠΌΠ°Π΅Ρ‚Π΅ Π³Π°Π΅Ρ‡Π½Ρ‹ΠΉ ΠΊΠ»ΡŽΡ‡ пСрпСндикулярно с усилиСм 165 Н Π½Π° расстоянии 0,140 ΠΌ ΠΎΡ‚ Ρ†Π΅Π½Ρ‚Ρ€Π° Π±ΠΎΠ»Ρ‚Π°. Какой крутящий ΠΌΠΎΠΌΠ΅Π½Ρ‚ Π²Ρ‹ ΠΏΡ€ΠΈΠΊΠ»Π°Π΄Ρ‹Π²Π°Π΅Ρ‚Π΅ ΠΎΡ‚Π½ΠΎΡΠΈΡ‚Π΅Π»ΡŒΠ½ΠΎ Ρ†Π΅Π½Ρ‚Ρ€Π° Π±ΠΎΠ»Ρ‚Π°?

ΠŸΡ€ΠΈ ΠΎΡ‚ΠΊΡ€Ρ‹Ρ‚ΠΈΠΈ Π΄Π²Π΅Ρ€ΠΈ Π²Ρ‹ Π½Π°ΠΆΠΈΠΌΠ°Π΅Ρ‚Π΅ Π½Π° Π½Π΅Π΅ пСрпСндикулярно с силой 55.0 Н Π½Π° расстоянии 0,850 ΠΌ ΠΎΡ‚ ΠΏΠ΅Ρ‚Π΅Π»ΡŒ. Какой крутящий ΠΌΠΎΠΌΠ΅Π½Ρ‚ Π²Ρ‹ ΠΏΡ€ΠΈΠΊΠ»Π°Π΄Ρ‹Π²Π°Π΅Ρ‚Π΅ ΠΊ пСтлям?

ΠŸΠΎΠΊΠ°Π·Π°Ρ‚ΡŒ Ρ€Π΅ΡˆΠ΅Π½ΠΈΠ΅

[латСкс] 46,8 \, \ text {N} Β· \ text {m} [/ латСкс]

НайдитС Π²Π΅Π»ΠΈΡ‡ΠΈΠ½Ρƒ натяТСния ΠΊΠ°ΠΆΠ΄ΠΎΠ³ΠΎ ΠΏΠΎΠ΄Π΄Π΅Ρ€ΠΆΠΈΠ²Π°ΡŽΡ‰Π΅Π³ΠΎ троса, ΠΏΠΎΠΊΠ°Π·Π°Π½Π½ΠΎΠ³ΠΎ Π½ΠΈΠΆΠ΅. Π’ ΠΊΠ°ΠΆΠ΄ΠΎΠΌ случаС вСс подвСшСнного Ρ‚Π΅Π»Π° составляСт 100,0 Н, Π° массой ΠΊΠ°Π±Π΅Π»Π΅ΠΉ ΠΌΠΎΠΆΠ½ΠΎ ΠΏΡ€Π΅Π½Π΅Π±Ρ€Π΅Ρ‡ΡŒ.

Какая сила Π΄ΠΎΠ»ΠΆΠ½Π° Π±Ρ‹Ρ‚ΡŒ ΠΏΡ€ΠΈΠ»ΠΎΠΆΠ΅Π½Π° Π² Ρ‚ΠΎΡ‡ΠΊΠ΅ P , Ρ‡Ρ‚ΠΎΠ±Ρ‹ ΡƒΠ΄Π΅Ρ€ΠΆΠΈΠ²Π°Ρ‚ΡŒ ΠΏΠΎΠΊΠ°Π·Π°Π½Π½ΡƒΡŽ ΠΊΠΎΠ½ΡΡ‚Ρ€ΡƒΠΊΡ†ΠΈΡŽ Π² равновСсии? ВСс конструкции Π½Π΅Π·Π½Π°Ρ‡ΠΈΡ‚Π΅Π»ΡŒΠ½Ρ‹ΠΉ.

МоТно Π»ΠΈ ΠΏΡ€ΠΈΠ»ΠΎΠΆΠΈΡ‚ΡŒ силу ΠΊ P , Ρ‡Ρ‚ΠΎΠ±Ρ‹ ΡƒΠ΄Π΅Ρ€ΠΆΠΈΠ²Π°Ρ‚ΡŒ ΠΏΠΎΠΊΠ°Π·Π°Π½Π½ΡƒΡŽ ΠΊΠΎΠ½ΡΡ‚Ρ€ΡƒΠΊΡ†ΠΈΡŽ Π² равновСсии? ВСс конструкции Π½Π΅Π·Π½Π°Ρ‡ΠΈΡ‚Π΅Π»ΡŒΠ½Ρ‹ΠΉ.

Π”Π²ΠΎΠ΅ Π΄Π΅Ρ‚Π΅ΠΉ Ρ‚ΠΎΠ»ΠΊΠ°ΡŽΡ‚ ΠΏΡ€ΠΎΡ‚ΠΈΠ²ΠΎΠΏΠΎΠ»ΠΎΠΆΠ½Ρ‹Π΅ стороны Π΄Π²Π΅Ρ€ΠΈ Π²ΠΎ врСмя ΠΈΠ³Ρ€Ρ‹. Оба Ρ‚ΠΎΠ»ΠΊΠ°ΡŽΡ‚ΡΡ Π³ΠΎΡ€ΠΈΠ·ΠΎΠ½Ρ‚Π°Π»ΡŒΠ½ΠΎ ΠΈ пСрпСндикулярно Π΄Π²Π΅Ρ€ΠΈ. Один Ρ€Π΅Π±Π΅Π½ΠΎΠΊ Ρ‚ΠΎΠ»ΠΊΠ°Π΅Ρ‚ с силой 17,5 Н Π½Π° расстоянии 0,600 ΠΌ ΠΎΡ‚ ΠΏΠ΅Ρ‚Π΅Π»ΡŒ, Π° Π²Ρ‚ΠΎΡ€ΠΎΠΉ Ρ€Π΅Π±Π΅Π½ΠΎΠΊ Ρ‚ΠΎΠ»ΠΊΠ°Π΅Ρ‚ Π½Π° расстоянии 0,450 ΠΌ. ΠšΠ°ΠΊΡƒΡŽ силу Π΄ΠΎΠ»ΠΆΠ΅Π½ ΠΏΡ€ΠΈΠ»ΠΎΠΆΠΈΡ‚ΡŒ Π²Ρ‚ΠΎΡ€ΠΎΠΉ Ρ€Π΅Π±Π΅Π½ΠΎΠΊ, Ρ‡Ρ‚ΠΎΠ±Ρ‹ Π΄Π²Π΅Ρ€ΡŒ Π½Π΅ двигалась? ΠŸΡ€Π΅Π΄ΠΏΠΎΠ»ΠΎΠΆΠΈΠΌ, Ρ‚Ρ€Π΅Π½ΠΈΠ΅ Π½Π΅Π·Π½Π°Ρ‡ΠΈΡ‚Π΅Π»ΡŒΠ½ΠΎ.

НСбольшой Π²Π½Π΅Π΄ΠΎΡ€ΠΎΠΆΠ½ΠΈΠΊ массой 1000 ΠΊΠ³ ΠΈΠΌΠ΅Π΅Ρ‚ ΠΊΠΎΠ»Π΅ΡΠ½ΡƒΡŽ Π±Π°Π·Ρƒ 3,0 ΠΌ. Если 60%, Ссли Π΅Π³ΠΎ вСс приходится Π½Π° ΠΏΠ΅Ρ€Π΅Π΄Π½ΠΈΠ΅ колСса, насколько Π΄Π°Π»Π΅ΠΊΠΎ ΠΏΠΎΠ·Π°Π΄ΠΈ ΠΏΠ΅Ρ€Π΅Π΄Π½ΠΈΡ… колСс находится Ρ†Π΅Π½Ρ‚Ρ€ масс Ρ„ΡƒΡ€Π³ΠΎΠ½Π°?

Π£Π½ΠΈΡ„ΠΈΡ†ΠΈΡ€ΠΎΠ²Π°Π½Π½Ρ‹Π΅ ΠΊΠ°Ρ‡Π΅Π»ΠΈ сбалансированы Π² Ρ†Π΅Π½Ρ‚Ρ€Π΅ масс, ΠΊΠ°ΠΊ ΠΏΠΎΠΊΠ°Π·Π°Π½ΠΎ Π½ΠΈΠΆΠ΅. МалСнький ΠΌΠ°Π»ΡŒΡ‡ΠΈΠΊ справа ΠΈΠΌΠ΅Π΅Ρ‚ массу 40,0 ΠΊΠ³. Какая масса Ρƒ Π΅Π³ΠΎ Π΄Ρ€ΡƒΠ³Π°?

Глоссарий

Ρ†Π΅Π½Ρ‚Ρ€ тяТСсти
Ρ‚ΠΎΡ‡ΠΊΠ°, ΠΊ ΠΊΠΎΡ‚ΠΎΡ€ΠΎΠΉ ΠΏΡ€ΠΈΠΊΡ€Π΅ΠΏΠ»Π΅Π½ Π²Π΅ΠΊΡ‚ΠΎΡ€ вСсов
равновСсиС
Π’Π΅Π»ΠΎ
находится Π² равновСсии, ΠΊΠΎΠ³Π΄Π° Π΅Π³ΠΎ Π»ΠΈΠ½Π΅ΠΉΠ½ΠΎΠ΅ ΠΈ ΡƒΠ³Π»ΠΎΠ²ΠΎΠ΅ ускорСния Ρ€Π°Π²Π½Ρ‹ Π½ΡƒΠ»ΡŽ ΠΎΡ‚Π½ΠΎΡΠΈΡ‚Π΅Π»ΡŒΠ½ΠΎ ΠΈΠ½Π΅Ρ€Ρ†ΠΈΠ°Π»ΡŒΠ½ΠΎΠΉ систСмы отсчСта
ΠΏΠ΅Ρ€Π²ΠΎΠ΅ условиС равновСсия
Π²Ρ‹Ρ€Π°ΠΆΠ°Π΅Ρ‚ ΠΏΠΎΡΡ‚ΡƒΠΏΠ°Ρ‚Π΅Π»ΡŒΠ½ΠΎΠ΅ равновСсиС; всС внСшниС силы, Π΄Π΅ΠΉΡΡ‚Π²ΡƒΡŽΡ‰ΠΈΠ΅ Π½Π° Ρ‚Π΅Π»ΠΎ, ΡƒΡ€Π°Π²Π½ΠΎΠ²Π΅ΡˆΠΈΠ²Π°ΡŽΡ‚ΡΡ ΠΈ ΠΈΡ… вСкторная сумма Ρ€Π°Π²Π½Π° Π½ΡƒΠ»ΡŽ
Π³Ρ€Π°Π²ΠΈΡ‚Π°Ρ†ΠΈΠΎΠ½Π½Ρ‹ΠΉ ΠΌΠΎΠΌΠ΅Π½Ρ‚
крутящий ΠΌΠΎΠΌΠ΅Π½Ρ‚ Π½Π° корпусС, Π²Ρ‹Π·Π²Π°Π½Π½Ρ‹ΠΉ Π΅Π³ΠΎ вСсом; Π²ΠΎΠ·Π½ΠΈΠΊΠ°Π΅Ρ‚, ΠΊΠΎΠ³Π΄Π° Ρ†Π΅Π½Ρ‚Ρ€ тяТСсти Ρ‚Π΅Π»Π° Π½Π΅ располоТСн Π½Π° оси вращСния
Π²Ρ‚ΠΎΡ€ΠΎΠ΅ состояниС равновСсия
Π²Ρ‹Ρ€Π°ΠΆΠ°Π΅Ρ‚ Π²Ρ€Π°Ρ‰Π°Ρ‚Π΅Π»ΡŒΠ½ΠΎΠ΅ равновСсиС; всС крутящиС ΠΌΠΎΠΌΠ΅Π½Ρ‚Ρ‹ ΠΎΡ‚ Π²Π½Π΅ΡˆΠ½ΠΈΡ… сил, Π΄Π΅ΠΉΡΡ‚Π²ΡƒΡŽΡ‰ΠΈΡ… Π½Π° Ρ‚Π΅Π»ΠΎ, ΡƒΡ€Π°Π²Π½ΠΎΠ²Π΅ΡˆΠΈΠ²Π°ΡŽΡ‚ΡΡ ΠΈ ΠΈΡ… вСкторная сумма Ρ€Π°Π²Π½Π° Π½ΡƒΠ»ΡŽ
статичСскоС равновСсиС
Π’Π΅Π»ΠΎ находится Π² статичСском равновСсии, ΠΊΠΎΠ³Π΄Π° ΠΎΠ½ΠΎ покоится Π² Π²Ρ‹Π±Ρ€Π°Π½Π½ΠΎΠΉ Π½Π°ΠΌΠΈ ΠΈΠ½Π΅Ρ€Ρ†ΠΈΠ°Π»ΡŒΠ½ΠΎΠΉ систСмС отсчСта

12.1 Условия статичСского равновСсия — University Physics Volume 1

Π¦Π΅Π»ΠΈ обучСния

К ΠΊΠΎΠ½Ρ†Ρƒ этого Ρ€Π°Π·Π΄Π΅Π»Π° Π²Ρ‹ смоТСтС:

  • ΠžΠΏΡ€Π΅Π΄Π΅Π»ΠΈΡ‚Π΅ физичСскиС условия статичСского равновСсия.
  • НарисуйтС Π΄ΠΈΠ°Π³Ρ€Π°ΠΌΠΌΡƒ свободного Ρ‚Π΅Π»Π° для Ρ‚Π²Π΅Ρ€Π΄ΠΎΠ³ΠΎ Ρ‚Π΅Π»Π°, Π½Π° ΠΊΠΎΡ‚ΠΎΡ€ΠΎΠ΅ Π΄Π΅ΠΉΡΡ‚Π²ΡƒΡŽΡ‚ силы.
  • ΠžΠ±ΡŠΡΡΠ½ΠΈΡ‚Π΅, ΠΊΠ°ΠΊ условия равновСсия ΠΏΠΎΠ·Π²ΠΎΠ»ΡΡŽΡ‚ Π½Π°ΠΌ Ρ€Π΅ΡˆΠ°Ρ‚ΡŒ Π·Π°Π΄Π°Ρ‡ΠΈ статики.

ΠœΡ‹ Π³ΠΎΠ²ΠΎΡ€ΠΈΠΌ, Ρ‡Ρ‚ΠΎ Ρ‚Π²Π΅Ρ€Π΄ΠΎΠ΅ Ρ‚Π΅Π»ΠΎ находится Π² состоянии равновСсия , ΠΊΠΎΠ³Π΄Π° Π΅Π³ΠΎ Π»ΠΈΠ½Π΅ΠΉΠ½ΠΎΠ΅ ΠΈ ΡƒΠ³Π»ΠΎΠ²ΠΎΠ΅ ускорСниС Ρ€Π°Π²Π½Ρ‹ Π½ΡƒΠ»ΡŽ ΠΎΡ‚Π½ΠΎΡΠΈΡ‚Π΅Π»ΡŒΠ½ΠΎ ΠΈΠ½Π΅Ρ€Ρ†ΠΈΠ°Π»ΡŒΠ½ΠΎΠΉ систСмы отсчСта.Π­Ρ‚ΠΎ ΠΎΠ·Π½Π°Ρ‡Π°Π΅Ρ‚, Ρ‡Ρ‚ΠΎ Ρ‚Π΅Π»ΠΎ Π² состоянии равновСсия ΠΌΠΎΠΆΠ΅Ρ‚ Π΄Π²ΠΈΠ³Π°Ρ‚ΡŒΡΡ, Π½ΠΎ Π² этом случаС Π΅Π³ΠΎ линСйная ΠΈ угловая скорости Π΄ΠΎΠ»ΠΆΠ½Ρ‹ Π±Ρ‹Ρ‚ΡŒ постоянными. ΠœΡ‹ Π³ΠΎΠ²ΠΎΡ€ΠΈΠΌ, Ρ‡Ρ‚ΠΎ Ρ‚Π²Π΅Ρ€Π΄ΠΎΠ΅ Ρ‚Π΅Π»ΠΎ находится Π² состоянии статичСского равновСсия , ΠΊΠΎΠ³Π΄Π° ΠΎΠ½ΠΎ находится Π² состоянии покоя Π² нашСй Π²Ρ‹Π±Ρ€Π°Π½Π½ΠΎΠΉ систСмС ΠΊΠΎΠΎΡ€Π΄ΠΈΠ½Π°Ρ‚ . ΠžΠ±Ρ€Π°Ρ‚ΠΈΡ‚Π΅ Π²Π½ΠΈΠΌΠ°Π½ΠΈΠ΅, Ρ‡Ρ‚ΠΎ Ρ€Π°Π·Π»ΠΈΡ‡ΠΈΠ΅ ΠΌΠ΅ΠΆΠ΄Ρƒ состояниСм покоя ΠΈ состояниСм Ρ€Π°Π²Π½ΠΎΠΌΠ΅Ρ€Π½ΠΎΠ³ΠΎ двиТСния являСтся искусствСнным, Ρ‚ΠΎ Π΅ΡΡ‚ΡŒ ΠΎΠ±ΡŠΠ΅ΠΊΡ‚ ΠΌΠΎΠΆΠ΅Ρ‚ Π½Π°Ρ…ΠΎΠ΄ΠΈΡ‚ΡŒΡΡ Π² состоянии покоя Π² Π²Ρ‹Π±Ρ€Π°Π½Π½ΠΎΠΉ Π½Π°ΠΌΠΈ систСмС отсчСта, Π½ΠΎ для Π½Π°Π±Π»ΡŽΠ΄Π°Ρ‚Π΅Π»Ρ, двиТущСгося с постоянной ΡΠΊΠΎΡ€ΠΎΡΡ‚ΡŒΡŽ ΠΎΡ‚Π½ΠΎΡΠΈΡ‚Π΅Π»ΡŒΠ½ΠΎ нашСй систСмы ΠΊΠΎΠΎΡ€Π΄ΠΈΠ½Π°Ρ‚, Ρ‚ΠΎΡ‚ ΠΆΠ΅ ΠΎΠ±ΡŠΠ΅ΠΊΡ‚ каТСтся, находится Π² Ρ€Π°Π²Π½ΠΎΠΌΠ΅Ρ€Π½ΠΎΠΌ Π΄Π²ΠΈΠΆΠ΅Π½ΠΈΠΈ с постоянной ΡΠΊΠΎΡ€ΠΎΡΡ‚ΡŒΡŽ.ΠŸΠΎΡΠΊΠΎΠ»ΡŒΠΊΡƒ Π΄Π²ΠΈΠΆΠ΅Π½ΠΈΠ΅ составляСт ΠΎΡ‚Π½ΠΎΡΠΈΡ‚Π΅Π»ΡŒΠ½ΠΎ , Ρ‚ΠΎ, Ρ‡Ρ‚ΠΎ для нас находится Π² статичСском равновСсии, находится Π² динамичСском равновСсии для двиТущСгося Π½Π°Π±Π»ΡŽΠ΄Π°Ρ‚Π΅Π»Ρ, ΠΈ Π½Π°ΠΎΠ±ΠΎΡ€ΠΎΡ‚. ΠŸΠΎΡΠΊΠΎΠ»ΡŒΠΊΡƒ Π·Π°ΠΊΠΎΠ½Ρ‹ Ρ„ΠΈΠ·ΠΈΠΊΠΈ ΠΈΠ΄Π΅Π½Ρ‚ΠΈΡ‡Π½Ρ‹ для всСх ΠΈΠ½Π΅Ρ€Ρ†ΠΈΠ°Π»ΡŒΠ½Ρ‹Ρ… систСм отсчСта, Π² ΠΈΠ½Π΅Ρ€Ρ†ΠΈΠ°Π»ΡŒΠ½ΠΎΠΉ систСмС отсчСта Π½Π΅Ρ‚ различия ΠΌΠ΅ΠΆΠ΄Ρƒ статичСским равновСсиСм ΠΈ равновСсиСм.

Богласно Π²Ρ‚ΠΎΡ€ΠΎΠΌΡƒ Π·Π°ΠΊΠΎΠ½Ρƒ двиТСния ΠΡŒΡŽΡ‚ΠΎΠ½Π°, Π»ΠΈΠ½Π΅ΠΉΠ½ΠΎΠ΅ ускорСниС Ρ‚Π²Π΅Ρ€Π΄ΠΎΠ³ΠΎ Ρ‚Π΅Π»Π° вызываСтся Π΄Π΅ΠΉΡΡ‚Π²ΡƒΡŽΡ‰Π΅ΠΉ Π½Π° Π½Π΅Π³ΠΎ чистой силой, ΠΈΠ»ΠΈ

Π—Π΄Π΅ΡΡŒ сумма всСх Π²Π½Π΅ΡˆΠ½ΠΈΡ… сил, Π΄Π΅ΠΉΡΡ‚Π²ΡƒΡŽΡ‰ΠΈΡ… Π½Π° Ρ‚Π΅Π»ΠΎ, Π³Π΄Π΅ ΠΌ, — Π΅Π³ΠΎ масса, Π°

— это Π»ΠΈΠ½Π΅ΠΉΠ½ΠΎΠ΅ ускорСниС Π΅Π³ΠΎ Ρ†Π΅Π½Ρ‚Ρ€Π° масс (концСпция, ΠΊΠΎΡ‚ΠΎΡ€ΡƒΡŽ ΠΌΡ‹ обсуТдали Π² ΡΡ‚Π°Ρ‚ΡŒΡΡ… Β«Π›ΠΈΠ½Π΅ΠΉΠ½Ρ‹ΠΉ ΠΈΠΌΠΏΡƒΠ»ΡŒΡ ΠΈ столкновСния ΠΏΠΎ ΠΈΠΌΠΏΡƒΠ»ΡŒΡΡƒ ΠΈ столкновСниям»).Π’ состоянии равновСсия Π»ΠΈΠ½Π΅ΠΉΠ½ΠΎΠ΅ ускорСниС Ρ€Π°Π²Π½ΠΎ Π½ΡƒΠ»ΡŽ. Если ΡƒΡΡ‚Π°Π½ΠΎΠ²ΠΈΡ‚ΡŒ Π½ΡƒΠ»Π΅Π²ΠΎΠ΅ ускорСниС Π½Π° (рисунок), ΠΌΡ‹ ΠΏΠΎΠ»ΡƒΡ‡ΠΈΠΌ ΡΠ»Π΅Π΄ΡƒΡŽΡ‰Π΅Π΅ ΡƒΡ€Π°Π²Π½Π΅Π½ΠΈΠ΅:

ΠŸΠ΅Ρ€Π²ΠΎΠ΅ условиС равновСсия

ΠŸΠ΅Ρ€Π²ΠΎΠ΅ условиС равновСсия для статичСского равновСсия Ρ‚Π²Π΅Ρ€Π΄ΠΎΠ³ΠΎ Ρ‚Π΅Π»Π° Π²Ρ‹Ρ€Π°ΠΆΠ°Π΅Ρ‚ ΠΏΠΎΡΡ‚ΡƒΠΏΠ°Ρ‚Π΅Π»ΡŒΠ½ΠΎΠ΅ равновСсиС:

ΠŸΠ΅Ρ€Π²ΠΎΠ΅ условиС равновСсия (рисунок) — это условиС равновСсия сил, с ΠΊΠΎΡ‚ΠΎΡ€Ρ‹ΠΌ ΠΌΡ‹ ΡΡ‚ΠΎΠ»ΠΊΠ½ΡƒΠ»ΠΈΡΡŒ ΠΏΡ€ΠΈ ΠΈΠ·ΡƒΡ‡Π΅Π½ΠΈΠΈ ΠΏΡ€ΠΈΠ»ΠΎΠΆΠ΅Π½ΠΈΠΉ Π·Π°ΠΊΠΎΠ½ΠΎΠ² ΠΡŒΡŽΡ‚ΠΎΠ½Π°.

Π­Ρ‚ΠΎ Π²Π΅ΠΊΡ‚ΠΎΡ€Π½ΠΎΠ΅ ΡƒΡ€Π°Π²Π½Π΅Π½ΠΈΠ΅ эквивалСнтно ΡΠ»Π΅Π΄ΡƒΡŽΡ‰ΠΈΠΌ Ρ‚Ρ€Π΅ΠΌ скалярным уравнСниям для ΠΊΠΎΠΌΠΏΠΎΠ½Π΅Π½Ρ‚ΠΎΠ² чистой силы:

Аналогично (рисунок) ΠΌΠΎΠΆΠ½ΠΎ ΠΊΠΎΠ½ΡΡ‚Π°Ρ‚ΠΈΡ€ΠΎΠ²Π°Ρ‚ΡŒ, Ρ‡Ρ‚ΠΎ ускорСниС вращСния

Ρ‚Π²Π΅Ρ€Π΄ΠΎΠ³ΠΎ Ρ‚Π΅Π»Π° Π²ΠΎΠΊΡ€ΡƒΠ³ фиксированной оси вращСния вызываСтся чистым крутящим ΠΌΠΎΠΌΠ΅Π½Ρ‚ΠΎΠΌ, Π΄Π΅ΠΉΡΡ‚Π²ΡƒΡŽΡ‰ΠΈΠΌ Π½Π° Ρ‚Π΅Π»ΠΎ, ΠΈΠ»ΠΈ

Π—Π΄Π΅ΡΡŒ

— это инСрция вращСния Ρ‚Π΅Π»Π° ΠΏΡ€ΠΈ Π²Ρ€Π°Ρ‰Π΅Π½ΠΈΠΈ Π²ΠΎΠΊΡ€ΡƒΠ³ этой оси, ΠΈ сумма составляСт Π±ΠΎΠ»Π΅Π΅ всСх ΠΌΠΎΠΌΠ΅Π½Ρ‚ΠΎΠ²

Π²Π½Π΅ΡˆΠ½ΠΈΡ… сил Π² (рисунок).Π’ состоянии равновСсия ускорСниС вращСния Ρ€Π°Π²Π½ΠΎ Π½ΡƒΠ»ΡŽ. ΠžΠ±Π½ΡƒΠ»ΡΡ ΠΏΡ€Π°Π²ΡƒΡŽ Ρ‡Π°ΡΡ‚ΡŒ (рисунок), ΠΌΡ‹ ΠΏΠΎΠ»ΡƒΡ‡Π°Π΅ΠΌ Π²Ρ‚ΠΎΡ€ΠΎΠ΅ условиС равновСсия:

Π’Ρ‚ΠΎΡ€ΠΎΠ΅ состояниС равновСсия

Π’Ρ‚ΠΎΡ€ΠΎΠ΅ условиС равновСсия для статичСского равновСсия Ρ‚Π²Π΅Ρ€Π΄ΠΎΠ³ΠΎ Ρ‚Π΅Π»Π° Π²Ρ‹Ρ€Π°ΠΆΠ°Π΅Ρ‚ Π²Ρ€Π°Ρ‰Π°Ρ‚Π΅Π»ΡŒΠ½ΠΎΠ΅ равновСсиС:

Π’Ρ‚ΠΎΡ€ΠΎΠ΅ условиС равновСсия (рисунок) — это условиС равновСсия для крутящих ΠΌΠΎΠΌΠ΅Π½Ρ‚ΠΎΠ², с ΠΊΠΎΡ‚ΠΎΡ€Ρ‹ΠΌ ΠΌΡ‹ ΡΡ‚ΠΎΠ»ΠΊΠ½ΡƒΠ»ΠΈΡΡŒ ΠΏΡ€ΠΈ ΠΈΠ·ΡƒΡ‡Π΅Π½ΠΈΠΈ Π΄ΠΈΠ½Π°ΠΌΠΈΠΊΠΈ вращСния.Π‘Ρ‚ΠΎΠΈΡ‚ ΠΎΡ‚ΠΌΠ΅Ρ‚ΠΈΡ‚ΡŒ, Ρ‡Ρ‚ΠΎ это ΡƒΡ€Π°Π²Π½Π΅Π½ΠΈΠ΅ равновСсия ΠΎΠ±Ρ‹Ρ‡Π½ΠΎ справСдливо для Π²Ρ€Π°Ρ‰Π°Ρ‚Π΅Π»ΡŒΠ½ΠΎΠ³ΠΎ равновСсия Π²ΠΎΠΊΡ€ΡƒΠ³ любой оси вращСния (фиксированной ΠΈΠ»ΠΈ ΠΈΠ½ΠΎΠΉ). ΠžΠΏΡΡ‚ΡŒ ΠΆΠ΅, это Π²Π΅ΠΊΡ‚ΠΎΡ€Π½ΠΎΠ΅ ΡƒΡ€Π°Π²Π½Π΅Π½ΠΈΠ΅ эквивалСнтно Ρ‚Ρ€Π΅ΠΌ скалярным уравнСниям для Π²Π΅ΠΊΡ‚ΠΎΡ€Π½Ρ‹Ρ… ΠΊΠΎΠΌΠΏΠΎΠ½Π΅Π½Ρ‚ΠΎΠ² чистого крутящСго ΠΌΠΎΠΌΠ΅Π½Ρ‚Π°:

Π’Ρ‚ΠΎΡ€ΠΎΠ΅ условиС равновСсия ΠΎΠ·Π½Π°Ρ‡Π°Π΅Ρ‚, Ρ‡Ρ‚ΠΎ Π² равновСсии Π½Π΅Ρ‚ чистого внСшнСго крутящСго ΠΌΠΎΠΌΠ΅Π½Ρ‚Π°, Π²Ρ‹Π·Ρ‹Π²Π°ΡŽΡ‰Π΅Π³ΠΎ Π²Ρ€Π°Ρ‰Π΅Π½ΠΈΠ΅ Π²ΠΎΠΊΡ€ΡƒΠ³ любой оси.

ΠŸΠ΅Ρ€Π²ΠΎΠ΅ ΠΈ Π²Ρ‚ΠΎΡ€ΠΎΠ΅ условия равновСсия ΡƒΠΊΠ°Π·Π°Π½Ρ‹ Π² ΠΊΠΎΠ½ΠΊΡ€Π΅Ρ‚Π½ΠΎΠΉ систСмС отсчСта.ΠŸΠ΅Ρ€Π²ΠΎΠ΅ условиС Π²ΠΊΠ»ΡŽΡ‡Π°Π΅Ρ‚ Ρ‚ΠΎΠ»ΡŒΠΊΠΎ силы ΠΈ поэтому Π½Π΅ зависит ΠΎΡ‚ источника систСмы отсчСта. Однако Π²Ρ‚ΠΎΡ€ΠΎΠ΅ условиС связано с крутящим ΠΌΠΎΠΌΠ΅Π½Ρ‚ΠΎΠΌ, ΠΊΠΎΡ‚ΠΎΡ€Ρ‹ΠΉ опрСдСляСтся ΠΊΠ°ΠΊ пСрСкрСстноС ΠΏΡ€ΠΎΠΈΠ·Π²Π΅Π΄Π΅Π½ΠΈΠ΅,

, Π³Π΄Π΅ Π²Π΅ΠΊΡ‚ΠΎΡ€ полоТСния

ΠΎΡ‚Π½ΠΎΡΠΈΡ‚Π΅Π»ΡŒΠ½ΠΎ оси вращСния Ρ‚ΠΎΡ‡ΠΊΠΈ прилоТСния силы Π²Ρ…ΠΎΠ΄ΠΈΡ‚ Π² ΡƒΡ€Π°Π²Π½Π΅Π½ΠΈΠ΅. Π‘Π»Π΅Π΄ΠΎΠ²Π°Ρ‚Π΅Π»ΡŒΠ½ΠΎ, крутящий ΠΌΠΎΠΌΠ΅Π½Ρ‚ зависит ΠΎΡ‚ полоТСния оси Π² систСмС отсчСта. Однако, ΠΊΠΎΠ³Π΄Π° условия Π²Ρ€Π°Ρ‰Π°Ρ‚Π΅Π»ΡŒΠ½ΠΎΠ³ΠΎ ΠΈ ΠΏΠΎΡΡ‚ΡƒΠΏΠ°Ρ‚Π΅Π»ΡŒΠ½ΠΎΠ³ΠΎ равновСсия Π²Ρ‹ΠΏΠΎΠ»Π½ΡΡŽΡ‚ΡΡ ΠΎΠ΄Π½ΠΎΠ²Ρ€Π΅ΠΌΠ΅Π½Π½ΠΎ Π² ΠΎΠ΄Π½ΠΎΠΉ систСмС отсчСта, ΠΎΠ½ΠΈ Ρ‚Π°ΠΊΠΆΠ΅ ΡΠΎΡ…Ρ€Π°Π½ΡΡŽΡ‚ΡΡ Π² любой Π΄Ρ€ΡƒΠ³ΠΎΠΉ ΠΈΠ½Π΅Ρ€Ρ†ΠΈΠ°Π»ΡŒΠ½ΠΎΠΉ систСмС отсчСта, Ρ‚Π°ΠΊ Ρ‡Ρ‚ΠΎ чистый крутящий ΠΌΠΎΠΌΠ΅Π½Ρ‚ Π²ΠΎΠΊΡ€ΡƒΠ³ любой оси вращСния ΠΏΠΎ-ΠΏΡ€Π΅ΠΆΠ½Π΅ΠΌΡƒ Ρ€Π°Π²Π΅Π½ Π½ΡƒΠ»ΡŽ.ОбъяснСниС этому довольно простоС.

ΠŸΡ€Π΅Π΄ΠΏΠΎΠ»ΠΎΠΆΠΈΠΌ Π²Π΅ΠΊΡ‚ΠΎΡ€

— позиция Π½Π°Ρ‡Π°Π»Π° Π½ΠΎΠ²ΠΎΠΉ ΠΈΠ½Π΅Ρ€Ρ†ΠΈΠ°Π»ΡŒΠ½ΠΎΠΉ систСмы отсчСта

Π² старой ΠΈΠ½Π΅Ρ€Ρ†ΠΈΠ°Π»ΡŒΠ½ΠΎΠΉ систСмС отсчСта S . Из нашСго исслСдования ΠΎΡ‚Π½ΠΎΡΠΈΡ‚Π΅Π»ΡŒΠ½ΠΎΠ³ΠΎ двиТСния ΠΌΡ‹ Π·Π½Π°Π΅ΠΌ, Ρ‡Ρ‚ΠΎ Π² Π½ΠΎΠ²ΠΎΠΉ систСмС отсчСта

Π²Π΅ΠΊΡ‚ΠΎΡ€ полоТСния

Ρ‚ΠΎΡ‡ΠΊΠΈ, Π³Π΄Π΅ дСйствуСт сила

относится ΠΊ

Ρ‡Π΅Ρ€Π΅Π· ΡƒΡ€Π°Π²Π½Π΅Π½ΠΈΠ΅

Π’Π΅ΠΏΠ΅Ρ€ΡŒ ΠΌΡ‹ ΠΌΠΎΠΆΠ΅ΠΌ ΠΏΡ€ΠΎΡΡƒΠΌΠΌΠΈΡ€ΠΎΠ²Π°Ρ‚ΡŒ всС крутящиС ΠΌΠΎΠΌΠ΅Π½Ρ‚Ρ‹

всСх Π²Π½Π΅ΡˆΠ½ΠΈΡ… сил Π² Π½ΠΎΠ²ΠΎΠΉ систСмС отсчСта,

На послСднСм этапС этой Ρ†Π΅ΠΏΠΎΡ‡ΠΊΠΈ рассуТдСний ΠΌΡ‹ использовали Ρ‚ΠΎΡ‚ Ρ„Π°ΠΊΡ‚, Ρ‡Ρ‚ΠΎ Π² равновСсии Π² старой систСмС отсчСта, S , ΠΏΠ΅Ρ€Π²Ρ‹ΠΉ Ρ‡Π»Π΅Π½ исчСзаСт ΠΈΠ·-Π·Π° (Рисунок), Π° Π²Ρ‚ΠΎΡ€ΠΎΠΉ Ρ‡Π»Π΅Π½ исчСзаСт ΠΈΠ·-Π·Π° (Рисунок).Π‘Π»Π΅Π΄ΠΎΠ²Π°Ρ‚Π΅Π»ΡŒΠ½ΠΎ, ΠΌΡ‹ Π²ΠΈΠ΄ΠΈΠΌ, Ρ‡Ρ‚ΠΎ чистый крутящий ΠΌΠΎΠΌΠ΅Π½Ρ‚ Π² любой ΠΈΠ½Π΅Ρ€Ρ†ΠΈΠ°Π»ΡŒΠ½ΠΎΠΉ систСмС отсчСта

Ρ€Π°Π²Π½ΠΎ Π½ΡƒΠ»ΡŽ ΠΏΡ€ΠΈ условии, Ρ‡Ρ‚ΠΎ ΠΎΠ±Π° условия равновСсия Π²Ρ‹ΠΏΠΎΠ»Π½ΡΡŽΡ‚ΡΡ Π² ΠΈΠ½Π΅Ρ€Ρ†ΠΈΠ°Π»ΡŒΠ½ΠΎΠΉ систСмС отсчСта S .

ΠŸΡ€Π°ΠΊΡ‚ΠΈΡ‡Π΅ΡΠΊΠΎΠ΅ Π·Π½Π°Ρ‡Π΅Π½ΠΈΠ΅ этого состоит Π² Ρ‚ΠΎΠΌ, Ρ‡Ρ‚ΠΎ ΠΏΡ€ΠΈ ΠΏΡ€ΠΈΠΌΠ΅Π½Π΅Π½ΠΈΠΈ условий равновСсия для Ρ‚Π²Π΅Ρ€Π΄ΠΎΠ³ΠΎ Ρ‚Π΅Π»Π° ΠΌΡ‹ ΠΌΠΎΠΆΠ΅ΠΌ Π²Ρ‹Π±Ρ€Π°Ρ‚ΡŒ Π»ΡŽΠ±ΡƒΡŽ Ρ‚ΠΎΡ‡ΠΊΡƒ Π² качСствС Π½Π°Ρ‡Π°Π»Π° отсчСта систСмы отсчСта. Наш Π²Ρ‹Π±ΠΎΡ€ систСмы отсчСта ΠΏΡ€ΠΎΠ΄ΠΈΠΊΡ‚ΠΎΠ²Π°Π½ физичСскими особСнностями Ρ€Π΅ΡˆΠ°Π΅ΠΌΠΎΠΉ ΠΏΡ€ΠΎΠ±Π»Π΅ΠΌΡ‹. Π’ ΠΎΠ΄Π½ΠΎΠΉ систСмС отсчСта матСматичСская Ρ„ΠΎΡ€ΠΌΠ° условий равновСсия ΠΌΠΎΠΆΠ΅Ρ‚ Π±Ρ‹Ρ‚ΡŒ довольно слоТной, Ρ‚ΠΎΠ³Π΄Π° ΠΊΠ°ΠΊ Π² Π΄Ρ€ΡƒΠ³ΠΎΠΉ систСмС ΠΊΠΎΠΎΡ€Π΄ΠΈΠ½Π°Ρ‚ Ρ‚Π΅ ΠΆΠ΅ условия ΠΌΠΎΠ³ΡƒΡ‚ ΠΈΠΌΠ΅Ρ‚ΡŒ Π±ΠΎΠ»Π΅Π΅ ΠΏΡ€ΠΎΡΡ‚ΡƒΡŽ ΠΌΠ°Ρ‚Π΅ΠΌΠ°Ρ‚ΠΈΡ‡Π΅ΡΠΊΡƒΡŽ Ρ„ΠΎΡ€ΠΌΡƒ, ΠΊΠΎΡ‚ΠΎΡ€ΡƒΡŽ Π»Π΅Π³ΠΊΠΎ Ρ€Π΅ΡˆΠΈΡ‚ΡŒ.Начало Π²Ρ‹Π±Ρ€Π°Π½Π½ΠΎΠΉ систСмы отсчСта называСтся Ρ‚ΠΎΡ‡ΠΊΠΎΠΉ ΠΏΠΎΠ²ΠΎΡ€ΠΎΡ‚Π° .

Π’ самом ΠΎΠ±Ρ‰Π΅ΠΌ случаС условия равновСсия Π²Ρ‹Ρ€Π°ΠΆΠ°ΡŽΡ‚ΡΡ ΡˆΠ΅ΡΡ‚ΡŒΡŽ скалярными уравнСниями ((Рисунок) ΠΈ (Рисунок)). Для плоских Π·Π°Π΄Π°Ρ‡ равновСсия с Π²Ρ€Π°Ρ‰Π΅Π½ΠΈΠ΅ΠΌ Π²ΠΎΠΊΡ€ΡƒΠ³ фиксированной оси, ΠΊΠΎΡ‚ΠΎΡ€Ρ‹Π΅ ΠΌΡ‹ рассматриваСм Π² этой Π³Π»Π°Π²Π΅, ΠΌΡ‹ ΠΌΠΎΠΆΠ΅ΠΌ ΡΠΎΠΊΡ€Π°Ρ‚ΠΈΡ‚ΡŒ количСство ΡƒΡ€Π°Π²Π½Π΅Π½ΠΈΠΉ Π΄ΠΎ Ρ‚Ρ€Π΅Ρ…. Бтандартная ΠΏΡ€ΠΎΡ†Π΅Π΄ΡƒΡ€Π° состоит Π² Ρ‚ΠΎΠΌ, Ρ‡Ρ‚ΠΎΠ±Ρ‹ ΠΏΡ€ΠΈΠ½ΡΡ‚ΡŒ систСму отсчСта, Π² ΠΊΠΎΡ‚ΠΎΡ€ΠΎΠΉ ось z являСтся осью вращСния. ΠŸΡ€ΠΈ Ρ‚Π°ΠΊΠΎΠΌ Π²Ρ‹Π±ΠΎΡ€Π΅ оси чистый крутящий ΠΌΠΎΠΌΠ΅Π½Ρ‚ ΠΈΠΌΠ΅Π΅Ρ‚ Ρ‚ΠΎΠ»ΡŒΠΊΠΎ ΠΊΠΎΠΌΠΏΠΎΠ½Π΅Π½Ρ‚ z , всС силы, ΠΊΠΎΡ‚ΠΎΡ€Ρ‹Π΅ ΠΈΠΌΠ΅ΡŽΡ‚ Π½Π΅Π½ΡƒΠ»Π΅Π²Ρ‹Π΅ крутящиС ΠΌΠΎΠΌΠ΅Π½Ρ‚Ρ‹, Π»Π΅ΠΆΠ°Ρ‚ Π² плоскости xy , ΠΈ, ΡΠ»Π΅Π΄ΠΎΠ²Π°Ρ‚Π΅Π»ΡŒΠ½ΠΎ, Π²ΠΊΠ»Π°Π΄ Π² чистый крутящий ΠΌΠΎΠΌΠ΅Π½Ρ‚ поступаСт Ρ‚ΠΎΠ»ΡŒΠΊΠΎ ΠΎΡ‚ x — ΠΈ y — ΡΠΎΡΡ‚Π°Π²Π»ΡΡŽΡ‰ΠΈΠ΅ Π²Π½Π΅ΡˆΠ½ΠΈΡ… сил.Π’Π°ΠΊΠΈΠΌ ΠΎΠ±Ρ€Π°Π·ΠΎΠΌ, для плоских Π·Π°Π΄Π°Ρ‡ с осью вращСния, пСрпСндикулярной плоскости xy , ΠΌΡ‹ ΠΈΠΌΠ΅Π΅ΠΌ ΡΠ»Π΅Π΄ΡƒΡŽΡ‰ΠΈΠ΅ Ρ‚Ρ€ΠΈ условия равновСсия для сил ΠΈ ΠΌΠΎΠΌΠ΅Π½Ρ‚ΠΎΠ²:

, Π³Π΄Π΅ суммированиС вСдСтся ΠΏΠΎ всСм внСшним силам N , Π΄Π΅ΠΉΡΡ‚Π²ΡƒΡŽΡ‰ΠΈΠΌ Π½Π° Ρ‚Π΅Π»ΠΎ, ΠΈ ΠΈΡ… крутящим ΠΌΠΎΠΌΠ΅Π½Ρ‚Π°ΠΌ. На (Рисунок) ΠΌΡ‹ упростили обозначСния, опустив индСкс z , Π½ΠΎ ΠΌΡ‹ ΠΏΠΎΠ½ΠΈΠΌΠ°Π΅ΠΌ, Ρ‡Ρ‚ΠΎ здСсь суммированиС вСдСтся ΠΏΠΎ всСм Π²ΠΊΠ»Π°Π΄Π°ΠΌ вдоль оси z , которая являСтся осью вращСния.На (Рисунок) z — ΡΠΎΡΡ‚Π°Π²Π»ΡΡŽΡ‰Π°Ρ крутящСго ΠΌΠΎΠΌΠ΅Π½Ρ‚Π°

ΠΎΡ‚ силы

это

Π³Π΄Π΅

— Π΄Π»ΠΈΠ½Π° ΠΏΠ»Π΅Ρ‡Π° Ρ€Ρ‹Ρ‡Π°Π³Π° силы ΠΈ

— это Π²Π΅Π»ΠΈΡ‡ΠΈΠ½Π° силы (ΠΊΠ°ΠΊ Π²Ρ‹ Π²ΠΈΠ΄Π΅Π»ΠΈ Π² Ρ€Π°Π·Π΄Π΅Π»Π΅ Β«Π’Ρ€Π°Ρ‰Π΅Π½ΠΈΠ΅ с фиксированной осью»). Π£Π³ΠΎΠ»

— ΡƒΠ³ΠΎΠ» ΠΌΠ΅ΠΆΠ΄Ρƒ Π²Π΅ΠΊΡ‚ΠΎΡ€Π°ΠΌΠΈ

ΠΈ

ΠΈΠ·ΠΌΠ΅Ρ€Π΅Π½ΠΈΠ΅ ΠΎΡ‚ Π²Π΅ΠΊΡ‚ΠΎΡ€Π°

Π² Π²Π΅ΠΊΡ‚ΠΎΡ€

Π² Π½Π°ΠΏΡ€Π°Π²Π»Π΅Π½ΠΈΠΈ ΠΏΡ€ΠΎΡ‚ΠΈΠ² часовой стрСлки ((Рисунок)).ΠŸΡ€ΠΈ использовании (рисунок) ΠΌΡ‹ часто вычисляСм Π²Π΅Π»ΠΈΡ‡ΠΈΠ½Ρƒ крутящСго ΠΌΠΎΠΌΠ΅Π½Ρ‚Π° ΠΈ присваиваСм Π΅Π΅ Π·Π½Π°Ρ‡Π΅Π½ΠΈΠ΅ ΠΏΠΎΠ»ΠΎΠΆΠΈΡ‚Π΅Π»ΡŒΠ½ΠΎΠΌΡƒ Π·Π½Π°Ρ‡Π΅Π½ΠΈΡŽ

.

ΠΈΠ»ΠΈ ΠΎΡ‚Ρ€ΠΈΡ†Π°Ρ‚Π΅Π»ΡŒΠ½ΠΎΠ΅

Π² зависимости ΠΎΡ‚ направлСния вращСния, Π²Ρ‹Π·Π²Π°Π½Π½ΠΎΠ³ΠΎ Ρ‚ΠΎΠ»ΡŒΠΊΠΎ этим крутящим ΠΌΠΎΠΌΠ΅Π½Ρ‚ΠΎΠΌ. На (Рисунок) чистый крутящий ΠΌΠΎΠΌΠ΅Π½Ρ‚ прСдставляСт собой сумму Ρ‡Π»Π΅Π½ΠΎΠ², ΠΊΠ°ΠΆΠ΄Ρ‹ΠΉ Ρ‡Π»Π΅Π½ рассчитываСтся ΠΈΠ· (Рисунок), ΠΈ ΠΊΠ°ΠΆΠ΄Ρ‹ΠΉ Ρ‡Π»Π΅Π½ Π΄ΠΎΠ»ΠΆΠ΅Π½ ΠΈΠΌΠ΅Ρ‚ΡŒ ΠΏΡ€Π°Π²ΠΈΠ»ΡŒΠ½ΠΎΠ΅ Π·Π½Π°Ρ‡Π΅Π½ΠΈΠ΅ . Аналогично Π½Π° (Рисунок) ΠΌΡ‹ Π½Π°Π·Π½Π°Ρ‡Π°Π΅ΠΌ

Π—Π½Π°ΠΊ

для ΠΏΡ€ΠΈΠ½ΡƒΠ΄ΠΈΡ‚Π΅Π»ΡŒΠ½ΠΎΠ³ΠΎ Π²ΠΊΠ»ΡŽΡ‡Π΅Π½ΠΈΡ ΠΊΠΎΠΌΠΏΠΎΠ½Π΅Π½Ρ‚ΠΎΠ² Π²

x — Π½Π°ΠΏΡ€Π°Π²Π»Π΅Π½ΠΈΠ΅ ΠΈ

Π·Π½Π°ΠΊ ΠΊΠΎΠΌΠΏΠΎΠ½Π΅Π½Ρ‚ΠΎΠ² Π²

x — Π½Π°ΠΏΡ€Π°Π²Π»Π΅Π½ΠΈΠ΅.Π­Ρ‚ΠΎ ΠΆΠ΅ ΠΏΡ€Π°Π²ΠΈΠ»ΠΎ Π΄ΠΎΠ»ΠΆΠ½ΠΎ ΠΏΠΎΡΠ»Π΅Π΄ΠΎΠ²Π°Ρ‚Π΅Π»ΡŒΠ½ΠΎ ΡΠΎΠ±Π»ΡŽΠ΄Π°Ρ‚ΡŒΡΡ Π½Π° (Рисунок) ΠΏΡ€ΠΈ вычислСнии ΡΠΎΡΡ‚Π°Π²Π»ΡΡŽΡ‰ΠΈΡ… силы ΠΏΠΎ оси y .

Рис. 12.2 ΠšΡ€ΡƒΡ‚ΡΡ‰ΠΈΠΉ ΠΌΠΎΠΌΠ΅Π½Ρ‚ силы: (a) Когда крутящий ΠΌΠΎΠΌΠ΅Π½Ρ‚ силы Π²Ρ‹Π·Ρ‹Π²Π°Π΅Ρ‚ Π²Ρ€Π°Ρ‰Π΅Π½ΠΈΠ΅ ΠΏΡ€ΠΎΡ‚ΠΈΠ² часовой стрСлки Π²ΠΎΠΊΡ€ΡƒΠ³ оси вращСния, ΠΌΡ‹ Π³ΠΎΠ²ΠΎΡ€ΠΈΠΌ, Ρ‡Ρ‚ΠΎ Π΅Π³ΠΎ Π½Π°ΠΏΡ€Π°Π²Π»Π΅Π½ΠΈΠ΅ ΠΏΠΎΠ»ΠΎΠΆΠΈΡ‚Π΅Π»ΡŒΠ½ΠΎΠ΅, Ρ‡Ρ‚ΠΎ ΠΎΠ·Π½Π°Ρ‡Π°Π΅Ρ‚, Ρ‡Ρ‚ΠΎ Π²Π΅ΠΊΡ‚ΠΎΡ€ крутящСго ΠΌΠΎΠΌΠ΅Π½Ρ‚Π° ΠΏΠ°Ρ€Π°Π»Π»Π΅Π»Π΅Π½ оси вращСния. (b) Когда крутящий ΠΌΠΎΠΌΠ΅Π½Ρ‚ силы Π²Ρ‹Π·Ρ‹Π²Π°Π΅Ρ‚ Π²Ρ€Π°Ρ‰Π΅Π½ΠΈΠ΅ Π²ΠΎΠΊΡ€ΡƒΠ³ оси ΠΏΠΎ часовой стрСлкС, ΠΌΡ‹ Π³ΠΎΠ²ΠΎΡ€ΠΈΠΌ, Ρ‡Ρ‚ΠΎ Π΅Π³ΠΎ Π½Π°ΠΏΡ€Π°Π²Π»Π΅Π½ΠΈΠ΅ ΠΎΡ‚Ρ€ΠΈΡ†Π°Ρ‚Π΅Π»ΡŒΠ½ΠΎΠ΅, Ρ‡Ρ‚ΠΎ ΠΎΠ·Π½Π°Ρ‡Π°Π΅Ρ‚, Ρ‡Ρ‚ΠΎ Π²Π΅ΠΊΡ‚ΠΎΡ€ крутящСго ΠΌΠΎΠΌΠ΅Π½Ρ‚Π° Π°Π½Ρ‚ΠΈΠΏΠ°Ρ€Π°Π»Π»Π΅Π»Π΅Π½ оси вращСния.

Π’ΠΎ ΠΌΠ½ΠΎΠ³ΠΈΡ… ситуациях равновСсия ΠΎΠ΄Π½ΠΎΠΉ ΠΈΠ· сил, Π΄Π΅ΠΉΡΡ‚Π²ΡƒΡŽΡ‰ΠΈΡ… Π½Π° Ρ‚Π΅Π»ΠΎ, являСтся Π΅Π³ΠΎ вСс. На Π΄ΠΈΠ°Π³Ρ€Π°ΠΌΠΌΠ°Ρ… свободного Ρ‚Π΅Π»Π° Π²Π΅ΠΊΡ‚ΠΎΡ€ вСса привязан ΠΊ Ρ†Π΅Π½Ρ‚Ρ€Ρƒ тяТСсти Ρ‚Π΅Π»Π°. Для всСх практичСских Ρ†Π΅Π»Π΅ΠΉ Ρ†Π΅Π½Ρ‚Ρ€ тяТСсти ΠΈΠ΄Π΅Π½Ρ‚ΠΈΡ‡Π΅Π½ Ρ†Π΅Π½Ρ‚Ρ€Ρƒ масс, ΠΊΠ°ΠΊ Π²Ρ‹ ΡƒΠ·Π½Π°Π»ΠΈ ΠΈΠ· статСй Β«Π›ΠΈΠ½Π΅ΠΉΠ½Ρ‹ΠΉ ΠΈΠΌΠΏΡƒΠ»ΡŒΡΒ» ΠΈ «БтолкновСния» ΠΎ Π»ΠΈΠ½Π΅ΠΉΠ½ΠΎΠΌ ΠΈΠΌΠΏΡƒΠ»ΡŒΡΠ΅ ΠΈ столкновСниях. Волько Π² Ρ‚Π΅Ρ… случаях, ΠΊΠΎΠ³Π΄Π° Ρ‚Π΅Π»ΠΎ ΠΈΠΌΠ΅Π΅Ρ‚ Π±ΠΎΠ»ΡŒΡˆΡƒΡŽ ΠΏΡ€ΠΎΡΡ‚Ρ€Π°Π½ΡΡ‚Π²Π΅Π½Π½ΡƒΡŽ ΠΏΡ€ΠΎΡ‚ΡΠΆΠ΅Π½Π½ΠΎΡΡ‚ΡŒ, Ρ‚Π°ΠΊ Ρ‡Ρ‚ΠΎ Π³Ρ€Π°Π²ΠΈΡ‚Π°Ρ†ΠΈΠΎΠ½Π½ΠΎΠ΅ ΠΏΠΎΠ»Π΅ Π½Π΅ΠΎΠ΄Π½ΠΎΡ€ΠΎΠ΄Π½ΠΎ ΠΏΠΎ всСму Π΅Π³ΠΎ ΠΎΠ±ΡŠΠ΅ΠΌΡƒ, Ρ†Π΅Π½Ρ‚Ρ€ тяТСсти ΠΈ Ρ†Π΅Π½Ρ‚Ρ€ масс располоТСны Π² Ρ€Π°Π·Π½Ρ‹Ρ… Ρ‚ΠΎΡ‡ΠΊΠ°Ρ….Однако Π½Π° ΠΏΡ€Π°ΠΊΡ‚ΠΈΠΊΠ΅ Π΄Π°ΠΆΠ΅ Ρ‚Π°ΠΊΠΈΠ΅ большиС ΠΎΠ±ΡŠΠ΅ΠΊΡ‚Ρ‹, ΠΊΠ°ΠΊ здания ΠΈΠ»ΠΈ ΠΊΡ€ΡƒΠΈΠ·Π½Ρ‹Π΅ Π»Π°ΠΉΠ½Π΅Ρ€Ρ‹, находятся Π² ΠΎΠ΄Π½ΠΎΡ€ΠΎΠ΄Π½ΠΎΠΌ Π³Ρ€Π°Π²ΠΈΡ‚Π°Ρ†ΠΈΠΎΠ½Π½ΠΎΠΌ ΠΏΠΎΠ»Π΅ Π½Π° повСрхности Π—Π΅ΠΌΠ»ΠΈ, Π³Π΄Π΅ ускорСниС свободного падСния ΠΈΠΌΠ΅Π΅Ρ‚ ΠΏΠΎΡΡ‚ΠΎΡΠ½Π½ΡƒΡŽ Π²Π΅Π»ΠΈΡ‡ΠΈΠ½Ρƒ

.

Π’ этих ситуациях Ρ†Π΅Π½Ρ‚Ρ€ тяТСсти ΠΈΠ΄Π΅Π½Ρ‚ΠΈΡ‡Π΅Π½ Ρ†Π΅Π½Ρ‚Ρ€Ρƒ масс. ΠŸΠΎΡΡ‚ΠΎΠΌΡƒ Π² этой Π³Π»Π°Π²Π΅ ΠΌΡ‹ ΠΈΡΠΏΠΎΠ»ΡŒΠ·ΡƒΠ΅ΠΌ Ρ†Π΅Π½Ρ‚Ρ€ масс (CM) ΠΊΠ°ΠΊ Ρ‚ΠΎΡ‡ΠΊΡƒ, ΠΊ ΠΊΠΎΡ‚ΠΎΡ€ΠΎΠΉ ΠΏΡ€ΠΈΠΊΡ€Π΅ΠΏΠ»Π΅Π½ Π²Π΅ΠΊΡ‚ΠΎΡ€ вСса. Напомним, Ρ‡Ρ‚ΠΎ ЦМ ΠΈΠΌΠ΅Π΅Ρ‚ особый физичСский смысл: ΠΊΠΎΠ³Π΄Π° внСшняя сила ΠΏΡ€ΠΈΠ»ΠΎΠΆΠ΅Π½Π° ΠΊ Ρ‚Π΅Π»Ρƒ Ρ‚ΠΎΡ‡Π½ΠΎ Π² Π΅Π³ΠΎ ЦМ, Ρ‚Π΅Π»ΠΎ Π² Ρ†Π΅Π»ΠΎΠΌ ΡΠΎΠ²Π΅Ρ€ΡˆΠ°Π΅Ρ‚ ΠΏΠΎΡΡ‚ΡƒΠΏΠ°Ρ‚Π΅Π»ΡŒΠ½ΠΎΠ΅ Π΄Π²ΠΈΠΆΠ΅Π½ΠΈΠ΅, ΠΈ такая сила Π½Π΅ Π²Ρ‹Π·Ρ‹Π²Π°Π΅Ρ‚ вращСния.

Когда ЦМ располоТСн Π²Π½Π΅ оси вращСния, Π½Π° ΠΎΠ±ΡŠΠ΅ΠΊΡ‚Π΅ Π²ΠΎΠ·Π½ΠΈΠΊΠ°Π΅Ρ‚ чистый Π³Ρ€Π°Π²ΠΈΡ‚Π°Ρ†ΠΈΠΎΠ½Π½Ρ‹ΠΉ ΠΌΠΎΠΌΠ΅Π½Ρ‚ . Π“Ρ€Π°Π²ΠΈΡ‚Π°Ρ†ΠΈΠΎΠ½Π½Ρ‹ΠΉ ΠΌΠΎΠΌΠ΅Π½Ρ‚ — это крутящий ΠΌΠΎΠΌΠ΅Π½Ρ‚, Π²Ρ‹Π·Π²Π°Π½Π½Ρ‹ΠΉ вСсом. Π­Ρ‚ΠΎΡ‚ Π³Ρ€Π°Π²ΠΈΡ‚Π°Ρ†ΠΈΠΎΠ½Π½Ρ‹ΠΉ ΠΌΠΎΠΌΠ΅Π½Ρ‚ ΠΌΠΎΠΆΠ΅Ρ‚ Π²Ρ€Π°Ρ‰Π°Ρ‚ΡŒ ΠΎΠ±ΡŠΠ΅ΠΊΡ‚, Ссли Π½Π΅Ρ‚ ΠΎΠΏΠΎΡ€Ρ‹ для Π΅Π³ΠΎ балансировки. Π’Π΅Π»ΠΈΡ‡ΠΈΠ½Π° Π³Ρ€Π°Π²ΠΈΡ‚Π°Ρ†ΠΈΠΎΠ½Π½ΠΎΠ³ΠΎ ΠΌΠΎΠΌΠ΅Π½Ρ‚Π° зависит ΠΎΡ‚ Ρ‚ΠΎΠ³ΠΎ, ΠΊΠ°ΠΊ Π΄Π°Π»Π΅ΠΊΠΎ ΠΎΡ‚ оси находится ЦМ. НапримСр, Π² случаС самосвала ((Рисунок)) ось ΠΏΠΎΠ²ΠΎΡ€ΠΎΡ‚Π° располоТСна Π½Π° Π»ΠΈΠ½ΠΈΠΈ, Π³Π΄Π΅ ΡˆΠΈΠ½Ρ‹ ΡΠΎΠΏΡ€ΠΈΠΊΠ°ΡΠ°ΡŽΡ‚ΡΡ с ΠΏΠΎΠ²Π΅Ρ€Ρ…Π½ΠΎΡΡ‚ΡŒΡŽ Π΄ΠΎΡ€ΠΎΠ³ΠΈ.Если CM располоТСн высоко Π½Π°Π΄ ΠΏΠΎΠ²Π΅Ρ€Ρ…Π½ΠΎΡΡ‚ΡŒΡŽ Π΄ΠΎΡ€ΠΎΠ³ΠΈ, Π³Ρ€Π°Π²ΠΈΡ‚Π°Ρ†ΠΈΠΎΠ½Π½Ρ‹ΠΉ ΠΌΠΎΠΌΠ΅Π½Ρ‚ ΠΌΠΎΠΆΠ΅Ρ‚ Π±Ρ‹Ρ‚ΡŒ достаточно большим, Ρ‡Ρ‚ΠΎΠ±Ρ‹ ΠΏΠ΅Ρ€Π΅Π²Π΅Ρ€Π½ΡƒΡ‚ΡŒ Π³Ρ€ΡƒΠ·ΠΎΠ²ΠΈΠΊ. Π›Π΅Π³ΠΊΠΎΠ²Ρ‹Π΅ Π°Π²Ρ‚ΠΎΠΌΠΎΠ±ΠΈΠ»ΠΈ с Π½ΠΈΠ·ΠΊΠΎ располоТСнной КМ, Π±Π»ΠΈΠ·ΠΊΠΎΠΉ ΠΊ Ρ‚Ρ€ΠΎΡ‚ΡƒΠ°Ρ€Ρƒ, Π±ΠΎΠ»Π΅Π΅ устойчивы ΠΊ ΠΎΠΏΡ€ΠΎΠΊΠΈΠ΄Ρ‹Π²Π°Π½ΠΈΡŽ, Ρ‡Π΅ΠΌ Π³Ρ€ΡƒΠ·ΠΎΠ²ΠΈΠΊΠΈ.

Рисунок 12.3 РаспрСдСлСниС массы влияСт Π½Π° ΠΏΠΎΠ»ΠΎΠΆΠ΅Π½ΠΈΠ΅ Ρ†Π΅Π½Ρ‚Ρ€Π° масс (CM), Π³Π΄Π΅ Π²Π΅ΠΊΡ‚ΠΎΡ€ вСса

прилагаСтся. Если Ρ†Π΅Π½Ρ‚Ρ€ тяТСсти находится Π² Π·ΠΎΠ½Π΅ ΠΎΠΏΠΎΡ€Ρ‹, ΠΏΠΎΠ³Ρ€ΡƒΠ·Ρ‡ΠΈΠΊ возвращаСтся Π² исходноС ΠΏΠΎΠ»ΠΎΠΆΠ΅Π½ΠΈΠ΅ послС опрокидывания [см. Π›Π΅Π²ΡƒΡŽ панСль Π² (b)].Но Ссли Ρ†Π΅Π½Ρ‚Ρ€ тяТСсти находится Π·Π° ΠΏΡ€Π΅Π΄Π΅Π»Π°ΠΌΠΈ Π·ΠΎΠ½Ρ‹ ΠΎΠΏΠΎΡ€Ρ‹, Π³Ρ€ΡƒΠ·ΠΎΠ²ΠΈΠΊ пСрСвСрнСтся [см. ΠŸΡ€Π°Π²ΡƒΡŽ панСль Π² (b)]. Оба транспортных срСдства Π² (b) находятся Π²Π½Π΅ равновСсия. ΠžΠ±Ρ€Π°Ρ‚ΠΈΡ‚Π΅ Π²Π½ΠΈΠΌΠ°Π½ΠΈΠ΅, Ρ‡Ρ‚ΠΎ Π°Π²Ρ‚ΠΎΠΌΠΎΠ±ΠΈΠ»ΡŒ Π½Π° (Π°) находится Π² равновСсии: Π½ΠΈΠ·ΠΊΠΎΠ΅ располоТСниС Ρ†Π΅Π½Ρ‚Ρ€Π° тяТСсти затрудняСт ΠΎΠΏΡ€ΠΎΠΊΠΈΠ΄Ρ‹Π²Π°Π½ΠΈΠ΅.

ΠŸΡ€ΠΈΠΌΠ΅Ρ€

Π¦Π΅Π½Ρ‚Ρ€ тяТСсти автомобиля

Π›Π΅Π³ΠΊΠΎΠ²ΠΎΠΉ Π°Π²Ρ‚ΠΎΠΌΠΎΠ±ΠΈΠ»ΡŒ с колСсной Π±Π°Π·ΠΎΠΉ 2,5 ΠΌ ΠΈΠΌΠ΅Π΅Ρ‚ 52% вСса Π½Π° ΠΏΠ΅Ρ€Π΅Π΄Π½ΠΈΠ΅ колСса Π½Π° Ρ€ΠΎΠ²Π½ΠΎΠΉ повСрхности, ΠΊΠ°ΠΊ ΠΏΠΎΠΊΠ°Π·Π°Π½ΠΎ Π½Π° (Рисунок). Π“Π΄Π΅ находится ЦМ этого автомобиля ΠΏΠΎ ΠΎΡ‚Π½ΠΎΡˆΠ΅Π½ΠΈΡŽ ΠΊ Π·Π°Π΄Π½Π΅ΠΉ оси?

Рисунок 12.4 РаспрСдСлСниС вСса ΠΌΠ΅ΠΆΠ΄Ρƒ осями автомобиля. Π“Π΄Π΅ находится Ρ†Π΅Π½Ρ‚Ρ€ тяТСсти?
БтратСгия

ВСс автомобиля w Π½Π°ΠΌ нСизвСстСн. ВсС, Ρ‡Ρ‚ΠΎ ΠΌΡ‹ Π·Π½Π°Π΅ΠΌ, это Ρ‚ΠΎ, Ρ‡Ρ‚ΠΎ ΠΊΠΎΠ³Π΄Π° Π°Π²Ρ‚ΠΎΠΌΠΎΠ±ΠΈΠ»ΡŒ стоит Π½Π° Ρ€ΠΎΠ²Π½ΠΎΠΉ повСрхности, 0,52 w Π΄Π°Π²ΠΈΡ‚ Π½Π° ΠΏΠΎΠ²Π΅Ρ€Ρ…Π½ΠΎΡΡ‚ΡŒ Π² Ρ‚ΠΎΡ‡ΠΊΠ°Ρ… ΠΊΠΎΠ½Ρ‚Π°ΠΊΡ‚Π° ΠΏΠ΅Ρ€Π΅Π΄Π½ΠΈΡ… колСс, Π° 0,48 w Ρ‚ΠΎΠ»ΠΊΠ°Π΅Ρ‚ Π²Π½ΠΈΠ· Π½Π° ΠΏΠΎΠ²Π΅Ρ€Ρ…Π½ΠΎΡΡ‚ΡŒ Π² Ρ‚ΠΎΡ‡ΠΊΠ°Ρ… ΠΊΠΎΠ½Ρ‚Π°ΠΊΡ‚Π° Π·Π°Π΄Π½ΠΈΡ… колСс. Π’Π°ΠΊΠΆΠ΅ Ρ‚ΠΎΡ‡ΠΊΠΈ соприкосновСния ΡƒΠ΄Π°Π»Π΅Π½Ρ‹ Π΄Ρ€ΡƒΠ³ ΠΎΡ‚ Π΄Ρ€ΡƒΠ³Π° Π½Π° расстояниС

Π’ этих Ρ‚ΠΎΡ‡ΠΊΠ°Ρ… ΠΊΠΎΠ½Ρ‚Π°ΠΊΡ‚Π° Π°Π²Ρ‚ΠΎΠΌΠΎΠ±ΠΈΠ»ΡŒ испытываСт Π½ΠΎΡ€ΠΌΠ°Π»ΡŒΠ½Ρ‹Π΅ силы Ρ€Π΅Π°ΠΊΡ†ΠΈΠΈ с Π²Π΅Π»ΠΈΡ‡ΠΈΠ½ΠΎΠΉ

.

ΠΈ

Π½Π° ΠΏΠ΅Ρ€Π΅Π΄Π½Π΅ΠΉ ΠΈ Π·Π°Π΄Π½Π΅ΠΉ оси соотвСтствСнно.ΠœΡ‹ Ρ‚Π°ΠΊΠΆΠ΅ Π·Π½Π°Π΅ΠΌ, Ρ‡Ρ‚ΠΎ Π°Π²Ρ‚ΠΎΠΌΠΎΠ±ΠΈΠ»ΡŒ являСтся ΠΏΡ€ΠΈΠΌΠ΅Ρ€ΠΎΠΌ Ρ‚Π²Π΅Ρ€Π΄ΠΎΠ³ΠΎ Ρ‚Π΅Π»Π°, находящСгося Π² равновСсии, вСсь вСс ΠΊΠΎΡ‚ΠΎΡ€ΠΎΠ³ΠΎ w дСйствуСт Π½Π° Π΅Π³ΠΎ ЦМ. КМ находится Π³Π΄Π΅-Ρ‚ΠΎ ΠΌΠ΅ΠΆΠ΄Ρƒ Ρ‚ΠΎΡ‡ΠΊΠ°ΠΌΠΈ дСйствия Π½ΠΎΡ€ΠΌΠ°Π»ΡŒΠ½Ρ‹Ρ… сил Ρ€Π΅Π°ΠΊΡ†ΠΈΠΈ, Π³Π΄Π΅-Ρ‚ΠΎ Π½Π° расстоянии x ΠΎΡ‚ Ρ‚ΠΎΡ‡ΠΊΠΈ, Π³Π΄Π΅

Π°ΠΊΡ‚Π°. Наша Π·Π°Π΄Π°Ρ‡Π° Π½Π°ΠΉΡ‚ΠΈ Ρ… . Π’Π°ΠΊΠΈΠΌ ΠΎΠ±Ρ€Π°Π·ΠΎΠΌ, ΠΌΡ‹ ΠΈΠ΄Π΅Π½Ρ‚ΠΈΡ„ΠΈΡ†ΠΈΡ€ΡƒΠ΅ΠΌ Ρ‚Ρ€ΠΈ силы, Π΄Π΅ΠΉΡΡ‚Π²ΡƒΡŽΡ‰ΠΈΠ΅ Π½Π° Ρ‚Π΅Π»ΠΎ (Π°Π²Ρ‚ΠΎΠΌΠΎΠ±ΠΈΠ»ΡŒ), ΠΈ ΠΌΠΎΠΆΠ΅ΠΌ Π½Π°Ρ€ΠΈΡΠΎΠ²Π°Ρ‚ΡŒ Π΄ΠΈΠ°Π³Ρ€Π°ΠΌΠΌΡƒ свободного Ρ‚Π΅Π»Π° для Ρ€Π°ΡΡˆΠΈΡ€Π΅Π½Π½ΠΎΠ³ΠΎ Ρ‚Π²Π΅Ρ€Π΄ΠΎΠ³ΠΎ Ρ‚Π΅Π»Π°, ΠΊΠ°ΠΊ ΠΏΠΎΠΊΠ°Π·Π°Π½ΠΎ Π½Π° (Рисунок).

Рисунок 12.5 Π”ΠΈΠ°Π³Ρ€Π°ΠΌΠΌΠ° свободного Ρ‚Π΅Π»Π° для автомобиля Ρ‡Π΅Ρ‚ΠΊΠΎ ΡƒΠΊΠ°Π·Ρ‹Π²Π°Π΅Ρ‚ Π²Π΅ΠΊΡ‚ΠΎΡ€Ρ‹ сил, Π΄Π΅ΠΉΡΡ‚Π²ΡƒΡŽΡ‰ΠΈΡ… Π½Π° Π°Π²Ρ‚ΠΎΠΌΠΎΠ±ΠΈΠ»ΡŒ, ΠΈ расстояния Π΄ΠΎ Ρ†Π΅Π½Ρ‚Ρ€Π° масс (CM). Когда CM Π²Ρ‹Π±Ρ€Π°Π½ Π² качСствС Ρ‚ΠΎΡ‡ΠΊΠΈ ΠΏΠΎΠ²ΠΎΡ€ΠΎΡ‚Π°, эти расстояния ΠΏΡ€Π΅Π΄ΡΡ‚Π°Π²Π»ΡΡŽΡ‚ собой ΠΏΠ»Π΅Ρ‡ΠΈ Ρ€Ρ‹Ρ‡Π°Π³Π° Π½ΠΎΡ€ΠΌΠ°Π»ΡŒΠ½Ρ‹Ρ… сил Ρ€Π΅Π°ΠΊΡ†ΠΈΠΈ. ΠžΠ±Ρ€Π°Ρ‚ΠΈΡ‚Π΅ Π²Π½ΠΈΠΌΠ°Π½ΠΈΠ΅, Ρ‡Ρ‚ΠΎ Π²Π΅Π»ΠΈΡ‡ΠΈΠ½Ρ‹ Π²Π΅ΠΊΡ‚ΠΎΡ€ΠΎΠ² ΠΈ Ρ€Ρ‹Ρ‡Π°Π³ΠΈ Π½Π΅ Π½ΡƒΠΆΠ½ΠΎ Ρ€ΠΈΡΠΎΠ²Π°Ρ‚ΡŒ Π² ΠΌΠ°ΡΡˆΡ‚Π°Π±Π΅, Π½ΠΎ всС Ρ€Π΅Π»Π΅Π²Π°Π½Ρ‚Π½Ρ‹Π΅ Π²Π΅Π»ΠΈΡ‡ΠΈΠ½Ρ‹ Π΄ΠΎΠ»ΠΆΠ½Ρ‹ Π±Ρ‹Ρ‚ΡŒ Ρ‡Π΅Ρ‚ΠΊΠΎ ΠΎΠ±ΠΎΠ·Π½Π°Ρ‡Π΅Π½Ρ‹.

ΠœΡ‹ ΠΏΠΎΡ‡Ρ‚ΠΈ Π³ΠΎΡ‚ΠΎΠ²Ρ‹ Π·Π°ΠΏΠΈΡΠ°Ρ‚ΡŒ условия равновСсия (рисунок) — (рисунок) для автомобиля, Π½ΠΎ сначала ΠΌΡ‹ Π΄ΠΎΠ»ΠΆΠ½Ρ‹ ΠΎΠΏΡ€Π΅Π΄Π΅Π»ΠΈΡ‚ΡŒΡΡ с систСмой отсчСта.ΠŸΡ€Π΅Π΄ΠΏΠΎΠ»ΠΎΠΆΠΈΠΌ, ΠΌΡ‹ Π²Ρ‹Π±Ρ€Π°Π»ΠΈ ось x ΠΏΠΎ Π΄Π»ΠΈΠ½Π΅ ΠΊΠ°Π±ΠΈΠ½Ρ‹, ось y — Π²Π΅Ρ€Ρ‚ΠΈΠΊΠ°Π»ΡŒΠ½ΡƒΡŽ, Π° ось z — пСрпСндикулярно этой плоскости xy . ΠŸΡ€ΠΈ Ρ‚Π°ΠΊΠΎΠΌ Π²Ρ‹Π±ΠΎΡ€Π΅ Π½Π°ΠΌ Π½ΡƒΠΆΠ½ΠΎ Ρ‚ΠΎΠ»ΡŒΠΊΠΎ Π½Π°ΠΏΠΈΡΠ°Ρ‚ΡŒ (рисунок) ΠΈ (рисунок), ΠΏΠΎΡ‚ΠΎΠΌΡƒ Ρ‡Ρ‚ΠΎ всС ΠΊΠΎΠΌΠΏΠΎΠ½Π΅Π½Ρ‚Ρ‹ y тоТдСствСнно Ρ€Π°Π²Π½Ρ‹ Π½ΡƒΠ»ΡŽ. Π’Π΅ΠΏΠ΅Ρ€ΡŒ Π½Π°ΠΌ Π½ΡƒΠΆΠ½ΠΎ ΠΎΠΏΡ€Π΅Π΄Π΅Π»ΠΈΡ‚ΡŒΡΡ с располоТСниСм Ρ‚ΠΎΡ‡ΠΊΠΈ ΠΏΠΎΠ²ΠΎΡ€ΠΎΡ‚Π°. ΠœΡ‹ ΠΌΠΎΠΆΠ΅ΠΌ Π²Ρ‹Π±Ρ€Π°Ρ‚ΡŒ Π»ΡŽΠ±ΡƒΡŽ Ρ‚ΠΎΡ‡ΠΊΡƒ Π² качСствС мСстополоТСния оси вращСния ( z -ось). ΠŸΡ€Π΅Π΄ΠΏΠΎΠ»ΠΎΠΆΠΈΠΌ, ΠΌΡ‹ размСстили ось вращСния Π½Π° CM, ΠΊΠ°ΠΊ ΠΏΠΎΠΊΠ°Π·Π°Π½ΠΎ Π½Π° схСмС свободного Ρ‚Π΅Π»Π° для автомобиля.На этом этапС ΠΌΡ‹ Π³ΠΎΡ‚ΠΎΠ²Ρ‹ Π½Π°ΠΏΠΈΡΠ°Ρ‚ΡŒ условия равновСсия для автомобиля.

РСшСниС

КаТдоС условиС равновСсия содСрТит Ρ‚ΠΎΠ»ΡŒΠΊΠΎ Ρ‚Ρ€ΠΈ Ρ‡Π»Π΅Π½Π°, ΠΏΠΎΡ‚ΠΎΠΌΡƒ Ρ‡Ρ‚ΠΎ ΠΈΡ…

силы, Π΄Π΅ΠΉΡΡ‚Π²ΡƒΡŽΡ‰ΠΈΠ΅ Π½Π° Π°Π²Ρ‚ΠΎΠΌΠΎΠ±ΠΈΠ»ΡŒ. ΠŸΠ΅Ρ€Π²ΠΎΠ΅ условиС равновСсия (рисунок) читаСтся ΠΊΠ°ΠΊ

.

Π­Ρ‚ΠΎ условиС Ρ‚Ρ€ΠΈΠ²ΠΈΠ°Π»ΡŒΠ½ΠΎ выполняСтся, ΠΏΠΎΡ‚ΠΎΠΌΡƒ Ρ‡Ρ‚ΠΎ, ΠΊΠΎΠ³Π΄Π° ΠΌΡ‹ подставляСм Π΄Π°Π½Π½Ρ‹Π΅, (рисунок) становится

Π’Ρ‚ΠΎΡ€ΠΎΠ΅ условиС равновСсия (рисунок) читаСтся ΠΊΠ°ΠΊ

.

Π³Π΄Π΅

— ΠΌΠΎΠΌΠ΅Π½Ρ‚ силы

— это Π³Ρ€Π°Π²ΠΈΡ‚Π°Ρ†ΠΈΠΎΠ½Π½Ρ‹ΠΉ ΠΌΠΎΠΌΠ΅Π½Ρ‚ силы w , Π°

— ΠΌΠΎΠΌΠ΅Π½Ρ‚ силы

Когда ΡˆΠ°Ρ€Π½ΠΈΡ€ располоТСн Π² CM, Π³Ρ€Π°Π²ΠΈΡ‚Π°Ρ†ΠΈΠΎΠ½Π½Ρ‹ΠΉ ΠΌΠΎΠΌΠ΅Π½Ρ‚ ΠΈΠ΄Π΅Π½Ρ‚ΠΈΡ‡Π½ΠΎ Π½ΡƒΠ»ΡŽ, ΠΏΠΎΡ‚ΠΎΠΌΡƒ Ρ‡Ρ‚ΠΎ ΠΏΠ»Π΅Ρ‡ΠΎ Ρ€Ρ‹Ρ‡Π°Π³Π° Π³Ρ€ΡƒΠ·Π° ΠΎΡ‚Π½ΠΎΡΠΈΡ‚Π΅Π»ΡŒΠ½ΠΎ оси, проходящСй Ρ‡Π΅Ρ€Π΅Π· CM, Ρ€Π°Π²Π½ΠΎ Π½ΡƒΠ»ΡŽ.Π›ΠΈΠ½ΠΈΠΈ дСйствия ΠΎΠ±Π΅ΠΈΡ… Π½ΠΎΡ€ΠΌΠ°Π»ΡŒΠ½Ρ‹Ρ… сил Ρ€Π΅Π°ΠΊΡ†ΠΈΠΈ пСрпСндикулярны ΠΏΠ»Π΅Ρ‡Π°ΠΌ ΠΈΡ… Ρ€Ρ‹Ρ‡Π°Π³ΠΎΠ², поэтому Π½Π° (Рисунок) ΠΌΡ‹ ΠΈΠΌΠ΅Π΅ΠΌ

для ΠΎΠ±Π΅ΠΈΡ… сил. Из Π΄ΠΈΠ°Π³Ρ€Π°ΠΌΠΌΡ‹ свободного Ρ‚Π΅Π»Π° ΠΌΡ‹ Ρ‡ΠΈΡ‚Π°Π΅ΠΌ, Ρ‡Ρ‚ΠΎ крутящий ΠΌΠΎΠΌΠ΅Π½Ρ‚

Π²Ρ‹Π·Ρ‹Π²Π°Π΅Ρ‚ Π²Ρ€Π°Ρ‰Π΅Π½ΠΈΠ΅ ΠΏΠΎ часовой стрСлкС Π²ΠΎΠΊΡ€ΡƒΠ³ оси CM, поэтому Π΅Π³ΠΎ Π½Π°ΠΏΡ€Π°Π²Π»Π΅Π½ΠΈΠ΅ ΠΎΡ‚Ρ€ΠΈΡ†Π°Ρ‚Π΅Π»ΡŒΠ½ΠΎΠ΅; ΠΈ крутящий ΠΌΠΎΠΌΠ΅Π½Ρ‚

Π²Ρ‹Π·Ρ‹Π²Π°Π΅Ρ‚ Π²Ρ€Π°Ρ‰Π΅Π½ΠΈΠ΅ ΠΏΡ€ΠΎΡ‚ΠΈΠ² часовой стрСлки Π²ΠΎΠΊΡ€ΡƒΠ³ оси CM, поэтому Π΅Π³ΠΎ Π½Π°ΠΏΡ€Π°Π²Π»Π΅Π½ΠΈΠ΅ ΠΏΠΎΠ»ΠΎΠΆΠΈΡ‚Π΅Π»ΡŒΠ½ΠΎΠ΅. ИмСя эту ΠΈΠ½Ρ„ΠΎΡ€ΠΌΠ°Ρ†ΠΈΡŽ, запишСм Π²Ρ‚ΠΎΡ€ΠΎΠ΅ условиС равновСсия ΠΊΠ°ΠΊ

Π‘ ΠΏΠΎΠΌΠΎΡ‰ΡŒΡŽ Π΄ΠΈΠ°Π³Ρ€Π°ΠΌΠΌΡ‹ свободного Ρ‚Π΅Π»Π° опрСдСляСм Π²Π΅Π»ΠΈΡ‡ΠΈΠ½Ρ‹ силы

ΠΈ

ΠΈ ΡΠΎΠΎΡ‚Π²Π΅Ρ‚ΡΡ‚Π²ΡƒΡŽΡ‰ΠΈΠ΅ ΠΈΠΌ Ρ€Ρ‹Ρ‡Π°Π³ΠΈ

ΠΈ

Π’Π΅ΠΏΠ΅Ρ€ΡŒ ΠΌΡ‹ ΠΌΠΎΠΆΠ΅ΠΌ Π·Π°ΠΏΠΈΡΠ°Ρ‚ΡŒ Π²Ρ‚ΠΎΡ€ΠΎΠ΅ условиС равновСсия (рисунок) Π² явном Π²ΠΈΠ΄Π΅ Π² Ρ‚Π΅Ρ€ΠΌΠΈΠ½Π°Ρ… нСизвСстного расстояния x :

Π—Π΄Π΅ΡΡŒ вСс w отмСняСтся, ΠΈ ΠΌΡ‹ ΠΌΠΎΠΆΠ΅ΠΌ Ρ€Π΅ΡˆΠΈΡ‚ΡŒ ΡƒΡ€Π°Π²Π½Π΅Π½ΠΈΠ΅ для нСизвСстного полоТСния x CM.ΠžΡ‚Π²Π΅Ρ‚

РСшСниС

Π’Ρ‹Π±ΠΎΡ€ ΡˆΠ°Ρ€Π½ΠΈΡ€Π° Π² ΠΏΠΎΠ»ΠΎΠΆΠ΅Π½ΠΈΠΈ ΠΏΠ΅Ρ€Π΅Π΄Π½Π΅ΠΉ оси Π½Π΅ мСняСт Ρ€Π΅Π·ΡƒΠ»ΡŒΡ‚Π°Ρ‚Π°. Π‘Ρ…Π΅ΠΌΠ° свободного Ρ‚Π΅Π»Π° для этого мСста ΠΏΠΎΠ²ΠΎΡ€ΠΎΡ‚Π° прСдставлСна ​​на (Рисунок). Для этого Π²Ρ‹Π±ΠΎΡ€Π° Ρ‚ΠΎΡ‡ΠΊΠΈ ΠΏΠΎΠ²ΠΎΡ€ΠΎΡ‚Π° Π²Ρ‚ΠΎΡ€ΠΎΠ΅ условиС равновСсия Ρ€Π°Π²Π½ΠΎ

.

Когда ΠΌΡ‹ подставляСм числа, ΡƒΠΊΠ°Π·Π°Π½Π½Ρ‹Π΅ Π½Π° Π΄ΠΈΠ°Π³Ρ€Π°ΠΌΠΌΠ΅, ΠΏΠΎΠ»ΡƒΡ‡Π°Π΅ΠΌ

ΠžΡ‚Π²Π΅Ρ‚, ΠΏΠΎΠ»ΡƒΡ‡Π΅Π½Π½Ρ‹ΠΉ ΠΏΡƒΡ‚Π΅ΠΌ Ρ€Π΅ΡˆΠ΅Π½ΠΈΡ (рисунок), ΠΎΠΏΡΡ‚ΡŒ ΠΆΠ΅:

.

Рисунок 12.6 ЭквивалСнтная Π΄ΠΈΠ°Π³Ρ€Π°ΠΌΠΌΠ° свободного Ρ‚Π΅Π»Π° для автомобиля; Ρ‚ΠΎΡ‡ΠΊΠ° ΠΏΠΎΠ²ΠΎΡ€ΠΎΡ‚Π° Ρ‡Π΅Ρ‚ΠΊΠΎ ΠΎΠ±ΠΎΠ·Π½Π°Ρ‡Π΅Π½Π°.
Π—Π½Π°Ρ‡Π΅Π½ΠΈΠ΅

Π­Ρ‚ΠΎΡ‚ ΠΏΡ€ΠΈΠΌΠ΅Ρ€ ΠΏΠΎΠΊΠ°Π·Ρ‹Π²Π°Π΅Ρ‚, Ρ‡Ρ‚ΠΎ ΠΏΡ€ΠΈ Ρ€Π΅ΡˆΠ΅Π½ΠΈΠΈ Π·Π°Π΄Π°Ρ‡ статичСского равновСсия ΠΌΡ‹ ΠΌΠΎΠΆΠ΅ΠΌ Π²Ρ‹Π±Ρ€Π°Ρ‚ΡŒ Ρ‚ΠΎΡ‡ΠΊΡƒ ΠΏΠΎΠ²ΠΎΡ€ΠΎΡ‚Π°. Для Ρ€Π°Π·Π»ΠΈΡ‡Π½Ρ‹Ρ… Π²Π°Ρ€ΠΈΠ°Π½Ρ‚ΠΎΠ² Π²Ρ‹Π±ΠΎΡ€Π° Ρ‚ΠΎΡ‡ΠΊΠΈ ΠΏΠΎΠ²ΠΎΡ€ΠΎΡ‚Π° Ρƒ нас Π΅ΡΡ‚ΡŒ Ρ€Π°Π·Π½Ρ‹Π΅ Π½Π°Π±ΠΎΡ€Ρ‹ условий равновСсия, ΠΊΠΎΡ‚ΠΎΡ€Ρ‹Π΅ Π½Π΅ΠΎΠ±Ρ…ΠΎΠ΄ΠΈΠΌΠΎ Ρ€Π΅ΡˆΠΈΡ‚ΡŒ. Однако любой Π²Ρ‹Π±ΠΎΡ€ ΠΏΡ€ΠΈΠ²ΠΎΠ΄ΠΈΡ‚ ΠΊ ΠΎΠ΄Π½ΠΎΠΌΡƒ ΠΈ Ρ‚ΠΎΠΌΡƒ ΠΆΠ΅ Ρ€Π΅ΡˆΠ΅Π½ΠΈΡŽ ΠΏΡ€ΠΎΠ±Π»Π΅ΠΌΡ‹.

ΠŸΡ€ΠΎΠ²Π΅Ρ€ΡŒΡ‚Π΅ своС ΠΏΠΎΠ½ΠΈΠΌΠ°Π½ΠΈΠ΅

Π Π΅ΡˆΠΈΡ‚Π΅ (рисунок), Π²Ρ‹Π±Ρ€Π°Π² ΡˆΠ°Ρ€Π½ΠΈΡ€ Π² мСстС располоТСния Π·Π°Π΄Π½Π΅ΠΉ оси.

[ΠΏΠΎΠΊΠ°Π·Ρ‹Π²Π°Ρ‚ΡŒ-ΠΎΡ‚Π²Π΅Ρ‚ q = ”fs-id1163713143559 β€³] ΠŸΠΎΠΊΠ°Π·Π°Ρ‚ΡŒ Ρ€Π΅ΡˆΠ΅Π½ΠΈΠ΅ [/ ΠΏΠΎΠΊΠ°Π·Ρ‹Π²Π°Ρ‚ΡŒ-ΠΎΡ‚Π²Π΅Ρ‚]

[скрытый-ΠΎΡ‚Π²Π΅Ρ‚ a = ”fs-id1163713143559 β€³]

[/ hidden-answer]

ΠŸΡ€ΠΎΠ²Π΅Ρ€ΡŒΡ‚Π΅ своС ΠΏΠΎΠ½ΠΈΠΌΠ°Π½ΠΈΠ΅

ΠžΠ±ΡŠΡΡΠ½ΠΈΡ‚Π΅, какая ΠΈΠ· ΡΠ»Π΅Π΄ΡƒΡŽΡ‰ΠΈΡ… ситуаций удовлСтворяСт ΠΎΠ±ΠΎΠΈΠΌ условиям равновСсия: (Π°) тСннисный мяч, ΠΊΠΎΡ‚ΠΎΡ€Ρ‹ΠΉ Π½Π΅ вращаСтся ΠΏΡ€ΠΈ Π΄Π²ΠΈΠΆΠ΅Π½ΠΈΠΈ Π² Π²ΠΎΠ·Π΄ΡƒΡ…Π΅; (Π±) ΠΏΠ΅Π»ΠΈΠΊΠ°Π½, ΠΊΠΎΡ‚ΠΎΡ€Ρ‹ΠΉ ΠΏΠ°Ρ€ΠΈΡ‚ Π² Π²ΠΎΠ·Π΄ΡƒΡ…Π΅ с постоянной ΡΠΊΠΎΡ€ΠΎΡΡ‚ΡŒΡŽ Π½Π° ΠΎΠ΄Π½ΠΎΠΉ высотС; ΠΈΠ»ΠΈ (c) ΠΊΠΎΠ»Π΅Π½Ρ‡Π°Ρ‚Ρ‹ΠΉ Π²Π°Π» двигатСля ΠΏΡ€ΠΈΠΏΠ°Ρ€ΠΊΠΎΠ²Π°Π½Π½ΠΎΠ³ΠΎ автомобиля.

[show-answer q = ”fs-id1163709667984 β€³] ΠŸΠΎΠΊΠ°Π·Π°Ρ‚ΡŒ Ρ€Π΅ΡˆΠ΅Π½ΠΈΠ΅ [/ show-answer]

[скрытый-ΠΎΡ‚Π²Π΅Ρ‚ a = ”fs-id1163709667984 β€³]

(Π±), (Π²)

[/ hidden-answer]

ΠžΡΠΎΠ±Ρ‹ΠΉ случай статичСского равновСсия Π²ΠΎΠ·Π½ΠΈΠΊΠ°Π΅Ρ‚, ΠΊΠΎΠ³Π΄Π° всС внСшниС силы Π½Π° ΠΎΠ±ΡŠΠ΅ΠΊΡ‚ Π΄Π΅ΠΉΡΡ‚Π²ΡƒΡŽΡ‚ Π½Π° оси вращСния ΠΈΠ»ΠΈ вдоль Π½Π΅Π΅, ΠΈΠ»ΠΈ ΠΊΠΎΠ³Π΄Π° пространствСнноС протяТСниС ΠΎΠ±ΡŠΠ΅ΠΊΡ‚Π° ΠΌΠΎΠΆΠ½ΠΎ Π½Π΅ ΠΏΡ€ΠΈΠ½ΠΈΠΌΠ°Ρ‚ΡŒ Π²ΠΎ Π²Π½ΠΈΠΌΠ°Π½ΠΈΠ΅. Π’ Ρ‚Π°ΠΊΠΎΠΌ случаС ΠΎΠ±ΡŠΠ΅ΠΊΡ‚ ΠΌΠΎΠΆΠ½ΠΎ эффСктивно Ρ€Π°ΡΡΠΌΠ°Ρ‚Ρ€ΠΈΠ²Π°Ρ‚ΡŒ ΠΊΠ°ΠΊ Ρ‚ΠΎΡ‡Π΅Ρ‡Π½ΡƒΡŽ массу. Π’ этом частном случаС Π½Π°ΠΌ Π½Π΅ Π½ΡƒΠΆΠ½ΠΎ Π±Π΅ΡΠΏΠΎΠΊΠΎΠΈΡ‚ΡŒΡΡ ΠΎ Π²Ρ‚ΠΎΡ€ΠΎΠΌ условии равновСсия (рисунок), ΠΏΠΎΡ‚ΠΎΠΌΡƒ Ρ‡Ρ‚ΠΎ всС крутящиС ΠΌΠΎΠΌΠ΅Π½Ρ‚Ρ‹ тоТдСствСнно Ρ€Π°Π²Π½Ρ‹ Π½ΡƒΠ»ΡŽ, Π° ΠΏΠ΅Ρ€Π²ΠΎΠ΅ условиС равновСсия (для сил) являСтся СдинствСнным условиСм, ΠΊΠΎΡ‚ΠΎΡ€ΠΎΠ΅ Π΄ΠΎΠ»ΠΆΠ½ΠΎ Π²Ρ‹ΠΏΠΎΠ»Π½ΡΡ‚ΡŒΡΡ.Π”ΠΈΠ°Π³Ρ€Π°ΠΌΠΌΠ° свободного Ρ‚Π΅Π»Π° ΠΈ стратСгия Ρ€Π΅ΡˆΠ΅Π½ΠΈΡ Π·Π°Π΄Π°Ρ‡ для этого особого случая Π±Ρ‹Π»ΠΈ ΠΈΠ·Π»ΠΎΠΆΠ΅Π½Ρ‹ Π² Β«Π—Π°ΠΊΠΎΠ½Π°Ρ… ΠΡŒΡŽΡ‚ΠΎΠ½Π°Β» ΠΈ Β«ΠŸΡ€ΠΈΠ»ΠΎΠΆΠ΅Π½ΠΈΡΡ… Π·Π°ΠΊΠΎΠ½ΠΎΠ² ΠΡŒΡŽΡ‚ΠΎΠ½Π°Β». Π’ ΡΠ»Π΅Π΄ΡƒΡŽΡ‰Π΅ΠΌ ΠΏΡ€ΠΈΠΌΠ΅Ρ€Π΅ Π²Ρ‹ ΡƒΠ²ΠΈΠ΄ΠΈΡ‚Π΅ Ρ‚ΠΈΠΏΠΈΡ‡Π½ΡƒΡŽ ΡΠΈΡ‚ΡƒΠ°Ρ†ΠΈΡŽ равновСсия, Π²ΠΊΠ»ΡŽΡ‡Π°ΡŽΡ‰ΡƒΡŽ Ρ‚ΠΎΠ»ΡŒΠΊΠΎ ΠΏΠ΅Ρ€Π²ΠΎΠ΅ условиС равновСсия.

ΠŸΡ€ΠΈΠΌΠ΅Ρ€

Π Π°Π·Ρ€Ρ‹Π²Π½ΠΎΠ΅ напряТСниС

МалСнькая ΠΊΠ°ΡΡ‚Ρ€ΡŽΠ»Ρ массой 42,0 Π³ поддСрТиваСтся двумя струнами, ΠΊΠ°ΠΊ ΠΏΠΎΠΊΠ°Π·Π°Π½ΠΎ Π½Π° (Рисунок). МаксимальноС натяТСниС, ΠΊΠΎΡ‚ΠΎΡ€ΠΎΠ΅ ΠΌΠΎΠΆΠ΅Ρ‚ Π²Ρ‹Π΄Π΅Ρ€ΠΆΠ°Ρ‚ΡŒ струна, составляСт 2,80 Н. Масса постСпСнно добавляСтся ΠΊ Ρ‡Π°ΡˆΠ΅, ΠΏΠΎΠΊΠ° ΠΎΠ΄Π½Π° ΠΈΠ· струн Π½Π΅ сломаСтся.Какая это струна? ΠšΠ°ΠΊΡƒΡŽ массу Π½ΡƒΠΆΠ½ΠΎ Π΄ΠΎΠ±Π°Π²ΠΈΡ‚ΡŒ, Ρ‡Ρ‚ΠΎΠ±Ρ‹ это ΠΏΡ€ΠΎΠΈΠ·ΠΎΡˆΠ»ΠΎ?

Рисунок 12.7 Масса добавляСтся Π² ΠΊΠ°ΡΡ‚Ρ€ΡŽΠ»ΡŽ постСпСнно, ΠΏΠΎΠΊΠ° ΠΎΠ΄Π½Π° ΠΈΠ· струн Π½Π΅ Π»ΠΎΠΏΠ½Π΅Ρ‚.
БтратСгия

Π­Ρ‚Π° мСханичСская систСма, состоящая ΠΈΠ· струн, масс ΠΈ сковороды, находится Π² статичСском равновСсии. Π’ частности, ΡƒΠ·Π΅Π», ΠΊΠΎΡ‚ΠΎΡ€Ρ‹ΠΉ привязываСт струны ΠΊ ΠΊΠ°ΡΡ‚Ρ€ΡŽΠ»Π΅, находится Π² статичСском равновСсии. Π£Π·Π΅Π» ΠΌΠΎΠΆΠ½ΠΎ Ρ€Π°ΡΡΠΌΠ°Ρ‚Ρ€ΠΈΠ²Π°Ρ‚ΡŒ ΠΊΠ°ΠΊ Ρ‚ΠΎΡ‡ΠΊΡƒ; ΡΠ»Π΅Π΄ΠΎΠ²Π°Ρ‚Π΅Π»ΡŒΠ½ΠΎ, Π½Π°ΠΌ Π½ΡƒΠΆΠ½ΠΎ Ρ‚ΠΎΠ»ΡŒΠΊΠΎ ΠΏΠ΅Ρ€Π²ΠΎΠ΅ условиС равновСсия. Π’Ρ€ΠΈ силы, тянущиС ΠΊ ΡƒΠ·Π»Ρƒ, — это натяТСниС

.

Π² Ρ€Π°ΠΉΠΎΠ½Π΅ 5.Π‘Ρ‚Ρ€ΡƒΠ½Π° 0 см, натяТСниС

Π² Ρ‚Π΅Ρ‚ΠΈΠ²Π΅ 10,0 см, Π° гиря

Ρ‡Π°ΡˆΠΈ, ΡƒΠ΄Π΅Ρ€ΠΆΠΈΠ²Π°ΡŽΡ‰Π΅ΠΉ массы. ΠœΡ‹ ΠΏΡ€ΠΈΠ½ΠΈΠΌΠ°Π΅ΠΌ ΠΏΡ€ΡΠΌΠΎΡƒΠ³ΠΎΠ»ΡŒΠ½ΡƒΡŽ систСму ΠΊΠΎΠΎΡ€Π΄ΠΈΠ½Π°Ρ‚ с осью y , Π½Π°ΠΏΡ€Π°Π²Π»Π΅Π½Π½ΠΎΠΉ ΠΏΡ€ΠΎΡ‚ΠΈΠ²ΠΎΠΏΠΎΠ»ΠΎΠΆΠ½ΠΎ Π½Π°ΠΏΡ€Π°Π²Π»Π΅Π½ΠΈΡŽ силы тяТСсти, ΠΈ рисуСм Π΄ΠΈΠ°Π³Ρ€Π°ΠΌΠΌΡƒ свободного Ρ‚Π΅Π»Π° для ΡƒΠ·Π»Π° (см. (Рисунок)). Π§Ρ‚ΠΎΠ±Ρ‹ Π½Π°ΠΉΡ‚ΠΈ ΠΊΠΎΠΌΠΏΠΎΠ½Π΅Π½Ρ‚Ρ‹ натяТСния, Π½Π΅ΠΎΠ±Ρ…ΠΎΠ΄ΠΈΠΌΠΎ ΠΎΠΏΡ€Π΅Π΄Π΅Π»ΠΈΡ‚ΡŒ направлСния ΡƒΠ³Π»ΠΎΠ²

ΠΈ

, Ρ‡Ρ‚ΠΎ струны ΠΎΠ±Ρ€Π°Π·ΡƒΡŽΡ‚ Π³ΠΎΡ€ΠΈΠ·ΠΎΠ½Ρ‚Π°Π»ΡŒΠ½ΠΎΠ΅ Π½Π°ΠΏΡ€Π°Π²Π»Π΅Π½ΠΈΠ΅ оси x .Как Π²Ρ‹ ΠΌΠΎΠΆΠ΅Ρ‚Π΅ Π²ΠΈΠ΄Π΅Ρ‚ΡŒ Π½Π° (Рисунок), струны ΡΠΎΡΡ‚Π°Π²Π»ΡΡŽΡ‚ Π΄Π²Π΅ стороны ΠΏΡ€ΡΠΌΠΎΡƒΠ³ΠΎΠ»ΡŒΠ½ΠΎΠ³ΠΎ Ρ‚Ρ€Π΅ΡƒΠ³ΠΎΠ»ΡŒΠ½ΠΈΠΊΠ°. ΠœΡ‹ ΠΌΠΎΠΆΠ΅ΠΌ ΠΈΡΠΏΠΎΠ»ΡŒΠ·ΠΎΠ²Π°Ρ‚ΡŒ Ρ‚Π΅ΠΎΡ€Π΅ΠΌΡƒ ΠŸΠΈΡ„Π°Π³ΠΎΡ€Π°, Ρ‡Ρ‚ΠΎΠ±Ρ‹ Ρ€Π΅ΡˆΠΈΡ‚ΡŒ этот Ρ‚Ρ€Π΅ΡƒΠ³ΠΎΠ»ΡŒΠ½ΠΈΠΊ, ΠΏΠΎΠΊΠ°Π·Π°Π½Π½Ρ‹ΠΉ Π½Π° (Рисунок), ΠΈ Π½Π°ΠΉΡ‚ΠΈ синус ΠΈ косинус ΡƒΠ³Π»ΠΎΠ²

.

ΠΈ

Π—Π°Ρ‚Π΅ΠΌ ΠΌΡ‹ ΠΌΠΎΠΆΠ΅ΠΌ Ρ€Π°Π·Π»ΠΎΠΆΠΈΡ‚ΡŒ напряТСния Π½Π° ΠΈΡ… ΠΏΡ€ΡΠΌΠΎΡƒΠ³ΠΎΠ»ΡŒΠ½Ρ‹Π΅ ΡΠΎΡΡ‚Π°Π²Π»ΡΡŽΡ‰ΠΈΠ΅, ΠΏΠΎΠ΄ΡΡ‚Π°Π²ΠΈΡ‚ΡŒ Π² ΠΏΠ΅Ρ€Π²ΠΎΠ΅ условиС равновСсия ((Рисунок) ΠΈ (Рисунок)) ΠΈ Ρ€Π΅ΡˆΠΈΡ‚ΡŒ для напряТСний Π² струнах. ΠŸΠ΅Ρ€Π²ΠΎΠΉ порвСтся струна с большим натяТСниСм.

Рисунок 12.8 Π‘Ρ…Π΅ΠΌΠ° свободного Ρ‚Π΅Π»Π° для ΡƒΠ·Π»Π° (рисунок).

РСшСниС

ВСс w , тянущий Π·Π° ΡƒΠ·Π΅Π», связан с массой M Ρ‡Π°ΡˆΠΈ ΠΈ массой m , Π΄ΠΎΠ±Π°Π²Π»Π΅Π½Π½ΠΎΠΉ ΠΊ Ρ‡Π°ΡˆΠ΅, ΠΈΠ»ΠΈ

Π‘ ΠΏΠΎΠΌΠΎΡ‰ΡŒΡŽ Π΄ΠΈΠ°Π³Ρ€Π°ΠΌΠΌΡ‹ свободного Ρ‚Π΅Π»Π° Π½Π° (Рисунок) ΠΌΡ‹ ΠΌΠΎΠΆΠ΅ΠΌ ΡƒΡΡ‚Π°Π½ΠΎΠ²ΠΈΡ‚ΡŒ условия равновСсия для ΡƒΠ·Π»Π°:

Из Π΄ΠΈΠ°Π³Ρ€Π°ΠΌΠΌΡ‹ свободного Ρ‚Π΅Π»Π°, Π²Π΅Π»ΠΈΡ‡ΠΈΠ½Ρ‹ ΠΊΠΎΠΌΠΏΠΎΠ½Π΅Π½Ρ‚ΠΎΠ² Π² этих уравнСниях Ρ€Π°Π²Π½Ρ‹

.

ΠŸΠΎΠ΄ΡΡ‚Π°Π²Π»ΡΠ΅ΠΌ эти ΠΊΠΎΠΌΠΏΠΎΠ½Π΅Π½Ρ‚Ρ‹ Π² условия равновСсия ΠΈ ΡƒΠΏΡ€ΠΎΡ‰Π°Π΅ΠΌ.Π—Π°Ρ‚Π΅ΠΌ ΠΌΡ‹ ΠΏΠΎΠ»ΡƒΡ‡Π°Π΅ΠΌ Π΄Π²Π° уравнСния равновСсия для натяТСний:

Π£Ρ€Π°Π²Π½Π΅Π½ΠΈΠ΅ равновСсия для направлСния x Π³ΠΎΠ²ΠΎΡ€ΠΈΡ‚ Π½Π°ΠΌ, Ρ‡Ρ‚ΠΎ натяТСниС

Π² 5,0-сантимСтровой Ρ‚Π΅Ρ‚ΠΈΠ²Π΅ Π²Π΄Π²ΠΎΠ΅ ΠΏΡ€Π΅Π²Ρ‹ΡˆΠ°Π΅Ρ‚ натяТСниС

Π² 10,0-ΡΠ°Π½Ρ‚ΠΈΠΌΠ΅Ρ‚Ρ€ΠΎΠ²ΡƒΡŽ Ρ‚Π΅Ρ‚ΠΈΠ²Ρƒ. Π’Π°ΠΊΠΈΠΌ ΠΎΠ±Ρ€Π°Π·ΠΎΠΌ, Π±ΠΎΠ»Π΅Π΅ короткая струна порвСтся. Когда ΠΌΡ‹ ΠΈΡΠΏΠΎΠ»ΡŒΠ·ΡƒΠ΅ΠΌ ΠΏΠ΅Ρ€Π²ΠΎΠ΅ ΡƒΡ€Π°Π²Π½Π΅Π½ΠΈΠ΅ для ΠΈΡΠΊΠ»ΡŽΡ‡Π΅Π½ΠΈΡ

ΠΈΠ· Π²Ρ‚ΠΎΡ€ΠΎΠ³ΠΎ уравнСния ΠΏΠΎΠ»ΡƒΡ‡Π°Π΅ΠΌ ΡΠΎΠΎΡ‚Π½ΠΎΡˆΠ΅Π½ΠΈΠ΅ ΠΌΠ΅ΠΆΠ΄Ρƒ массой

Π½Π° сковородС ΠΈ натяТСниС

Π² Π±ΠΎΠ»Π΅Π΅ ΠΊΠΎΡ€ΠΎΡ‚ΠΊΠΎΠΉ строкС:

Π‘Ρ‚Ρ€ΡƒΠ½Π° рвСтся, ΠΊΠΎΠ³Π΄Π° натяТСниС достигаСт критичСского значСния

.

ΠŸΡ€Π΅Π΄Ρ‹Π΄ΡƒΡ‰Π΅Π΅ ΡƒΡ€Π°Π²Π½Π΅Π½ΠΈΠ΅ ΠΌΠΎΠΆΠ΅Ρ‚ Π±Ρ‹Ρ‚ΡŒ Ρ€Π΅ΡˆΠ΅Π½ΠΎ для критичСской массы ΠΌ , которая Ρ€Π°Π·Ρ€Ρ‹Π²Π°Π΅Ρ‚ струну:

Π—Π½Π°Ρ‡Π΅Π½ΠΈΠ΅

ΠŸΡ€Π΅Π΄ΠΏΠΎΠ»ΠΎΠΆΠΈΠΌ, Ρ‡Ρ‚ΠΎ мСханичСская систСма, рассматриваСмая Π² этом ΠΏΡ€ΠΈΠΌΠ΅Ρ€Π΅, ΠΏΡ€ΠΈΠΊΡ€Π΅ΠΏΠ»Π΅Π½Π° ΠΊ ΠΏΠΎΡ‚ΠΎΠ»ΠΊΡƒ Π²Π½ΡƒΡ‚Ρ€ΠΈ Π»ΠΈΡ„Ρ‚Π°, ΠΏΠΎΠ΄Π½ΠΈΠΌΠ°ΡŽΡ‰Π΅Π³ΠΎΡΡ Π²Π²Π΅Ρ€Ρ….Пока Π»ΠΈΡ„Ρ‚ двиТСтся Π²Π²Π΅Ρ€Ρ… с постоянной ΡΠΊΠΎΡ€ΠΎΡΡ‚ΡŒΡŽ, Ρ€Π΅Π·ΡƒΠ»ΡŒΡ‚Π°Ρ‚ остаСтся Π½Π΅ΠΈΠ·ΠΌΠ΅Π½Π½Ρ‹ΠΌ, ΠΏΠΎΡ‚ΠΎΠΌΡƒ Ρ‡Ρ‚ΠΎ вСс

Π½Π΅ мСняСтся. Если Π»ΠΈΡ„Ρ‚ двиТСтся Π²Π²Π΅Ρ€Ρ… с ускорСниСм, критичСская масса мСньшС, ΠΏΠΎΡ‚ΠΎΠΌΡƒ Ρ‡Ρ‚ΠΎ вСс

становится большС Π½Π° ΠΊΠ°ΠΆΡƒΡ‰ΡƒΡŽΡΡ массу ΠΈΠ·-Π·Π° ускорСния Π»ΠΈΡ„Ρ‚Π°. Π’Π΅ΠΌ Π½Π΅ ΠΌΠ΅Π½Π΅Π΅, Π²ΠΎ всСх случаях Π±ΠΎΠ»Π΅Π΅ короткая струна рвСтся ΠΏΠ΅Ρ€Π²ΠΎΠΉ.

Π‘Π²ΠΎΠ΄ΠΊΠ°

  • Π’Π΅Π»ΠΎ находится Π² равновСсии, ΠΊΠΎΠ³Π΄Π° ΠΎΠ½ΠΎ остаСтся Π»ΠΈΠ±ΠΎ Π² Ρ€Π°Π²Π½ΠΎΠΌΠ΅Ρ€Π½ΠΎΠΌ Π΄Π²ΠΈΠΆΠ΅Π½ΠΈΠΈ (ΠΏΠΎΡΡ‚ΡƒΠΏΠ°Ρ‚Π΅Π»ΡŒΠ½ΠΎΠΌ ΠΈ Π²Ρ€Π°Ρ‰Π°Ρ‚Π΅Π»ΡŒΠ½ΠΎΠΌ), Π»ΠΈΠ±ΠΎ Π² состоянии покоя.Когда Ρ‚Π΅Π»ΠΎ Π² Π²Ρ‹Π±Ρ€Π°Π½Π½ΠΎΠΉ ΠΈΠ½Π΅Ρ€Ρ†ΠΈΠ°Π»ΡŒΠ½ΠΎΠΉ систСмС отсчСта Π½Π΅ вращаСтся ΠΈ Π½Π΅ двиТСтся Π² ΠΏΠΎΡΡ‚ΡƒΠΏΠ°Ρ‚Π΅Π»ΡŒΠ½ΠΎΠΌ Π΄Π²ΠΈΠΆΠ΅Π½ΠΈΠΈ, ΠΌΡ‹ Π³ΠΎΠ²ΠΎΡ€ΠΈΠΌ, Ρ‡Ρ‚ΠΎ Ρ‚Π΅Π»ΠΎ находится Π² статичСском равновСсии Π² этой систСмС отсчСта.
  • Условия равновСсия Ρ‚Ρ€Π΅Π±ΡƒΡŽΡ‚, Ρ‡Ρ‚ΠΎΠ±Ρ‹ сумма всСх Π²Π½Π΅ΡˆΠ½ΠΈΡ… сил, Π΄Π΅ΠΉΡΡ‚Π²ΡƒΡŽΡ‰ΠΈΡ… Π½Π° Ρ‚Π΅Π»ΠΎ, Π±Ρ‹Π»Π° Ρ€Π°Π²Π½Π° Π½ΡƒΠ»ΡŽ (ΠΏΠ΅Ρ€Π²ΠΎΠ΅ условиС равновСсия), Π° сумма всСх Π²Π½Π΅ΡˆΠ½ΠΈΡ… ΠΌΠΎΠΌΠ΅Π½Ρ‚ΠΎΠ² ΠΎΡ‚ Π²Π½Π΅ΡˆΠ½ΠΈΡ… сил Π±Ρ‹Π»Π° Ρ€Π°Π²Π½Π° Π½ΡƒΠ»ΡŽ (Π²Ρ‚ΠΎΡ€ΠΎΠ΅ условиС равновСсия). Π­Ρ‚ΠΈ Π΄Π²Π° условия Π΄ΠΎΠ»ΠΆΠ½Ρ‹ ΠΎΠ΄Π½ΠΎΠ²Ρ€Π΅ΠΌΠ΅Π½Π½ΠΎ Π²Ρ‹ΠΏΠΎΠ»Π½ΡΡ‚ΡŒΡΡ Π² состоянии равновСсия. Если ΠΎΠ΄ΠΈΠ½ ΠΈΠ· Π½ΠΈΡ… Π½Π΅ ΡƒΠ΄ΠΎΠ²Π»Π΅Ρ‚Π²ΠΎΡ€Π΅Π½, Ρ‚Π΅Π»ΠΎ Π½Π΅ находится Π² равновСсии.
  • Π”ΠΈΠ°Π³Ρ€Π°ΠΌΠΌΠ° свободного Ρ‚Π΅Π»Π° для Ρ‚Π΅Π»Π° — ΠΏΠΎΠ»Π΅Π·Π½Ρ‹ΠΉ инструмСнт, ΠΊΠΎΡ‚ΠΎΡ€Ρ‹ΠΉ позволяСт Π½Π°ΠΌ ΠΏΡ€Π°Π²ΠΈΠ»ΡŒΠ½ΠΎ ΠΏΠΎΠ΄ΡΡ‡ΠΈΡ‚Π°Ρ‚ΡŒ всС Π²ΠΊΠ»Π°Π΄Ρ‹ ΠΎΡ‚ всСх Π²Π½Π΅ΡˆΠ½ΠΈΡ… сил ΠΈ ΠΌΠΎΠΌΠ΅Π½Ρ‚ΠΎΠ², Π΄Π΅ΠΉΡΡ‚Π²ΡƒΡŽΡ‰ΠΈΡ… Π½Π° Ρ‚Π΅Π»ΠΎ. Π”ΠΈΠ°Π³Ρ€Π°ΠΌΠΌΡ‹ свободного Ρ‚Π΅Π»Π° для равновСсия вытянутого Ρ‚Π²Π΅Ρ€Π΄ΠΎΠ³ΠΎ Ρ‚Π΅Π»Π° Π΄ΠΎΠ»ΠΆΠ½Ρ‹ ΡƒΠΊΠ°Π·Ρ‹Π²Π°Ρ‚ΡŒ Ρ‚ΠΎΡ‡ΠΊΡƒ ΠΏΠΎΠ²ΠΎΡ€ΠΎΡ‚Π° ΠΈ ΠΏΠ»Π΅Ρ‡ΠΈ Ρ€Ρ‹Ρ‡Π°Π³Π° Π΄Π΅ΠΉΡΡ‚Π²ΡƒΡŽΡ‰ΠΈΡ… сил ΠΏΠΎ ΠΎΡ‚Π½ΠΎΡˆΠ΅Π½ΠΈΡŽ ΠΊ оси.

ΠšΠΎΠ½Ρ†Π΅ΠΏΡ‚ΡƒΠ°Π»ΡŒΠ½Ρ‹Π΅ вопросы

Π§Ρ‚ΠΎ Π²Ρ‹ ΠΌΠΎΠΆΠ΅Ρ‚Π΅ ΡΠΊΠ°Π·Π°Ρ‚ΡŒ ΠΎ скорости двиТущСгося Ρ‚Π΅Π»Π°, находящСгося Π² динамичСском равновСсии?

[ΠΏΠΎΠΊΠ°Π·Ρ‹Π²Π°Ρ‚ΡŒ-ΠΎΡ‚Π²Π΅Ρ‚ q = ”fs-id1163713268541 β€³] ΠŸΠΎΠΊΠ°Π·Π°Ρ‚ΡŒ Ρ€Π΅ΡˆΠ΅Π½ΠΈΠ΅ [/ ΠΏΠΎΠΊΠ°Π·Ρ‹Π²Π°Ρ‚ΡŒ-ΠΎΡ‚Π²Π΅Ρ‚]

[скрытый-ΠΎΡ‚Π²Π΅Ρ‚ a = ”fs-id1163713268541 β€³]

постоянная

[/ hidden-answer]

ΠŸΡ€ΠΈ ΠΊΠ°ΠΊΠΈΡ… условиях Π²Ρ€Π°Ρ‰Π°ΡŽΡ‰Π΅Π΅ΡΡ Ρ‚Π΅Π»ΠΎ ΠΌΠΎΠΆΠ΅Ρ‚ Π½Π°Ρ…ΠΎΠ΄ΠΈΡ‚ΡŒΡΡ Π² равновСсии? ΠŸΡ€ΠΈΠ²Π΅Π΄ΠΈΡ‚Π΅ ΠΏΡ€ΠΈΠΌΠ΅Ρ€.

КакиС Ρ‚Ρ€ΠΈ Ρ„Π°ΠΊΡ‚ΠΎΡ€Π° Π²Π»ΠΈΡΡŽΡ‚ Π½Π° крутящий ΠΌΠΎΠΌΠ΅Π½Ρ‚, создаваСмый силой ΠΎΡ‚Π½ΠΎΡΠΈΡ‚Π΅Π»ΡŒΠ½ΠΎ ΠΎΠΏΡ€Π΅Π΄Π΅Π»Π΅Π½Π½ΠΎΠΉ Ρ‚ΠΎΡ‡ΠΊΠΈ ΠΏΠΎΠ²ΠΎΡ€ΠΎΡ‚Π°?

[ΠΏΠΎΠΊΠ°Π·Ρ‹Π²Π°Ρ‚ΡŒ-ΠΎΡ‚Π²Π΅Ρ‚ q = ”fs-id1163713358733 β€³] ΠŸΠΎΠΊΠ°Π·Π°Ρ‚ΡŒ Ρ€Π΅ΡˆΠ΅Π½ΠΈΠ΅ [/ ΠΏΠΎΠΊΠ°Π·Ρ‹Π²Π°Ρ‚ΡŒ-ΠΎΡ‚Π²Π΅Ρ‚]

[скрытый-ΠΎΡ‚Π²Π΅Ρ‚ a = ”fs-id1163713358733 β€³]

Π’Π΅Π»ΠΈΡ‡ΠΈΠ½Π° ΠΈ Π½Π°ΠΏΡ€Π°Π²Π»Π΅Π½ΠΈΠ΅ силы, Π° Π΅Π΅ ΠΏΠ»Π΅Ρ‡ΠΎ Ρ€Ρ‹Ρ‡Π°Π³Π°

[/ hidden-answer]

ΠœΠ΅Ρ…Π°Π½ΠΈΠΊΠΈ ΠΈΠ½ΠΎΠ³Π΄Π° ΠΊΠ»Π°Π΄ΡƒΡ‚ кусок Ρ‚Ρ€ΡƒΠ±Ρ‹ Π½Π° рукоятку Π³Π°Π΅Ρ‡Π½ΠΎΠ³ΠΎ ΠΊΠ»ΡŽΡ‡Π°, ΠΏΡ‹Ρ‚Π°ΡΡΡŒ ΠΎΡ‚ΠΊΡ€ΡƒΡ‚ΠΈΡ‚ΡŒ ΠΎΡ‡Π΅Π½ΡŒ Ρ‚ΡƒΠ³ΠΎΠΉ Π±ΠΎΠ»Ρ‚. Как это ΠΏΠΎΠΌΠΎΠ³Π°Π΅Ρ‚?

Для ΡΠ»Π΅Π΄ΡƒΡŽΡ‰ΠΈΡ… Ρ‡Π΅Ρ‚Ρ‹Ρ€Π΅Ρ… Π·Π°Π΄Π°Ρ‡ ΠΎΡ†Π΅Π½ΠΈΡ‚Π΅ ΡƒΡ‚Π²Π΅Ρ€ΠΆΠ΄Π΅Π½ΠΈΠ΅ ΠΊΠ°ΠΊ истинноС ΠΈΠ»ΠΈ Π»ΠΎΠΆΠ½ΠΎΠ΅ ΠΈ ΠΎΠ±ΡŠΡΡΠ½ΠΈΡ‚Π΅ свой ΠΎΡ‚Π²Π΅Ρ‚.

Если Π½Π° ΠΎΠ±ΡŠΠ΅ΠΊΡ‚ дСйствуСт Ρ‚ΠΎΠ»ΡŒΠΊΠΎ ΠΎΠ΄Π½Π° внСшняя сила (ΠΈΠ»ΠΈ крутящий ΠΌΠΎΠΌΠ΅Π½Ρ‚), ΠΎΠ½ Π½Π΅ ΠΌΠΎΠΆΠ΅Ρ‚ Π½Π°Ρ…ΠΎΠ΄ΠΈΡ‚ΡŒΡΡ Π² равновСсии.

[ΠΏΠΎΠΊΠ°Π·Ρ‹Π²Π°Ρ‚ΡŒ-ΠΎΡ‚Π²Π΅Ρ‚ q = ”fs-id1163713282665 β€³] ΠŸΠΎΠΊΠ°Π·Π°Ρ‚ΡŒ Ρ€Π΅ΡˆΠ΅Π½ΠΈΠ΅ [/ ΠΏΠΎΠΊΠ°Π·Ρ‹Π²Π°Ρ‚ΡŒ-ΠΎΡ‚Π²Π΅Ρ‚]

[скрытый-ΠΎΡ‚Π²Π΅Ρ‚ a = ”fs-id1163713282665 β€³]

Π’Π΅Ρ€Π½ΠΎ, ΠΏΠΎΡΠΊΠΎΠ»ΡŒΠΊΡƒ Π² этом случаС сумма сил Π½Π΅ ΠΌΠΎΠΆΠ΅Ρ‚ Π±Ρ‹Ρ‚ΡŒ Ρ€Π°Π²Π½Π° Π½ΡƒΠ»ΡŽ, Ссли сама сила Π½Π΅ Ρ€Π°Π²Π½Π° Π½ΡƒΠ»ΡŽ.

[/ hidden-answer]

Если ΠΎΠ±ΡŠΠ΅ΠΊΡ‚ находится Π² равновСсии, Π½Π° Π½Π΅Π³ΠΎ Π΄ΠΎΠ»ΠΆΠ½ΠΎ Π΄Π΅ΠΉΡΡ‚Π²ΠΎΠ²Π°Ρ‚ΡŒ Ρ‡Π΅Ρ‚Π½ΠΎΠ΅ число сил.

Если Π½Π° ΠΎΠ±ΡŠΠ΅ΠΊΡ‚ дСйствуСт Π½Π΅Ρ‡Π΅Ρ‚Π½ΠΎΠ΅ количСство сил, ΠΎΠ±ΡŠΠ΅ΠΊΡ‚ Π½Π΅ ΠΌΠΎΠΆΠ΅Ρ‚ Π½Π°Ρ…ΠΎΠ΄ΠΈΡ‚ΡŒΡΡ Π² равновСсии.

[ΠΏΠΎΠΊΠ°Π·Ρ‹Π²Π°Ρ‚ΡŒ-ΠΎΡ‚Π²Π΅Ρ‚ q = ”fs-id11637096

β€³] ΠŸΠΎΠΊΠ°Π·Π°Ρ‚ΡŒ Ρ€Π΅ΡˆΠ΅Π½ΠΈΠ΅ [/ ΠΏΠΎΠΊΠ°Π·Ρ‹Π²Π°Ρ‚ΡŒ-ΠΎΡ‚Π²Π΅Ρ‚]

[скрытый-ΠΎΡ‚Π²Π΅Ρ‚ a = ”fs-id11637096

β€³]

Π›ΠΎΠΆΡŒ, Ссли силы ΡΠΊΠ»Π°Π΄Ρ‹Π²Π°ΡŽΡ‚ΡΡ Π² ноль ΠΊΠ°ΠΊ Π²Π΅ΠΊΡ‚ΠΎΡ€Ρ‹, Ρ‚ΠΎΠ³Π΄Π° ΠΌΠΎΠΆΠ΅Ρ‚ Π±Ρ‹Ρ‚ΡŒ достигнуто равновСсиС.

[/ hidden-answer]

Π’Π΅Π»ΠΎ, двиТущССся ΠΏΠΎ ΠΊΡ€ΡƒΠ³Ρƒ с постоянной ΡΠΊΠΎΡ€ΠΎΡΡ‚ΡŒΡŽ, находится Π² равновСсии вращСния.

Для Ρ‡Π΅Π³ΠΎ Π½ΡƒΠΆΠ΅Π½ Π΄Π»ΠΈΠ½Π½Ρ‹ΠΉ ΠΈ Π³ΠΈΠ±ΠΊΠΈΠΉ ΡˆΠ΅ΡΡ‚, ΠΊΠΎΡ‚ΠΎΡ€Ρ‹ΠΉ пСрСносят ΠΊΠ°Π½Π°Ρ‚ΠΎΡ…ΠΎΠ΄Ρ†Ρ‹?

[ΠΏΠΎΠΊΠ°Π·Ρ‹Π²Π°Ρ‚ΡŒ-ΠΎΡ‚Π²Π΅Ρ‚ q = ”fs-id1163713272740 β€³] ΠŸΠΎΠΊΠ°Π·Π°Ρ‚ΡŒ Ρ€Π΅ΡˆΠ΅Π½ΠΈΠ΅ [/ ΠΏΠΎΠΊΠ°Π·Ρ‹Π²Π°Ρ‚ΡŒ-ΠΎΡ‚Π²Π΅Ρ‚]

[скрытый-ΠΎΡ‚Π²Π΅Ρ‚ a = ”fs-id1163713272740 β€³]

ΠŸΠΎΠΌΠΎΠ³Π°Π΅Ρ‚ ΠΊΠ°Π½Π°Ρ‚ΠΎΡ…ΠΎΠ΄Ρ†Ρƒ ΡΠΎΡ…Ρ€Π°Π½ΡΡ‚ΡŒ равновСсиС.

[/ hidden-answer]

ΠŸΡ€ΠΎΠ±Π»Π΅ΠΌΡ‹

ΠŸΡ€ΠΈ затяТкС Π±ΠΎΠ»Ρ‚Π° Π²Ρ‹ Π½Π°ΠΆΠΈΠΌΠ°Π΅Ρ‚Π΅ Π³Π°Π΅Ρ‡Π½Ρ‹ΠΉ ΠΊΠ»ΡŽΡ‡ пСрпСндикулярно с усилиСм 165 Н Π½Π° расстоянии 0,140 ΠΌ ΠΎΡ‚ Ρ†Π΅Π½Ρ‚Ρ€Π° Π±ΠΎΠ»Ρ‚Π°. Какой крутящий ΠΌΠΎΠΌΠ΅Π½Ρ‚ Π²Ρ‹ ΠΏΡ€ΠΈΠΊΠ»Π°Π΄Ρ‹Π²Π°Π΅Ρ‚Π΅ ΠΎΡ‚Π½ΠΎΡΠΈΡ‚Π΅Π»ΡŒΠ½ΠΎ Ρ†Π΅Π½Ρ‚Ρ€Π° Π±ΠΎΠ»Ρ‚Π°?

ΠŸΡ€ΠΈ ΠΎΡ‚ΠΊΡ€Ρ‹Ρ‚ΠΈΠΈ Π΄Π²Π΅Ρ€ΠΈ Π²Ρ‹ Π½Π°ΠΆΠΈΠΌΠ°Π΅Ρ‚Π΅ Π½Π° Π½Π΅Π΅ пСрпСндикулярно с усилиСм 55,0 Н Π½Π° расстоянии 0,850 ΠΌ ΠΎΡ‚ ΠΏΠ΅Ρ‚Π΅Π»ΡŒ. Какой крутящий ΠΌΠΎΠΌΠ΅Π½Ρ‚ Π²Ρ‹ ΠΏΡ€ΠΈΠΊΠ»Π°Π΄Ρ‹Π²Π°Π΅Ρ‚Π΅ ΠΊ пСтлям?

[ΠΏΠΎΠΊΠ°Π·Ρ‹Π²Π°Ρ‚ΡŒ-ΠΎΡ‚Π²Π΅Ρ‚ q = ”fs-id1163713470139 β€³] ΠŸΠΎΠΊΠ°Π·Π°Ρ‚ΡŒ Ρ€Π΅ΡˆΠ΅Π½ΠΈΠ΅ [/ ΠΏΠΎΠΊΠ°Π·Ρ‹Π²Π°Ρ‚ΡŒ-ΠΎΡ‚Π²Π΅Ρ‚]

[скрытый-ΠΎΡ‚Π²Π΅Ρ‚ a = ”fs-id1163713470139 β€³]

[/ hidden-answer]

НайдитС Π²Π΅Π»ΠΈΡ‡ΠΈΠ½Ρƒ натяТСния ΠΊΠ°ΠΆΠ΄ΠΎΠ³ΠΎ ΠΏΠΎΠ΄Π΄Π΅Ρ€ΠΆΠΈΠ²Π°ΡŽΡ‰Π΅Π³ΠΎ троса, ΠΏΠΎΠΊΠ°Π·Π°Π½Π½ΠΎΠ³ΠΎ Π½ΠΈΠΆΠ΅.Π’ ΠΊΠ°ΠΆΠ΄ΠΎΠΌ случаС вСс подвСшСнного Ρ‚Π΅Π»Π° составляСт 100,0 Н, Π° массой ΠΊΠ°Π±Π΅Π»Π΅ΠΉ ΠΌΠΎΠΆΠ½ΠΎ ΠΏΡ€Π΅Π½Π΅Π±Ρ€Π΅Ρ‡ΡŒ.

Какая сила Π΄ΠΎΠ»ΠΆΠ½Π° Π±Ρ‹Ρ‚ΡŒ ΠΏΡ€ΠΈΠ»ΠΎΠΆΠ΅Π½Π° Π² Ρ‚ΠΎΡ‡ΠΊΠ΅ P , Ρ‡Ρ‚ΠΎΠ±Ρ‹ ΡƒΠ΄Π΅Ρ€ΠΆΠΈΠ²Π°Ρ‚ΡŒ ΠΏΠΎΠΊΠ°Π·Π°Π½Π½ΡƒΡŽ ΠΊΠΎΠ½ΡΡ‚Ρ€ΡƒΠΊΡ†ΠΈΡŽ Π² равновСсии? ВСс конструкции Π½Π΅Π·Π½Π°Ρ‡ΠΈΡ‚Π΅Π»ΡŒΠ½Ρ‹ΠΉ.

[show-answer q = ”264812 β€³] ΠŸΠΎΠΊΠ°Π·Π°Ρ‚ΡŒ ΠΎΡ‚Π²Π΅Ρ‚ [/ show-answer]
[hidden-answer a =” 264812 β€³] 153,4 Β° [/ hidden-answer]

МоТно Π»ΠΈ ΠΏΡ€ΠΈΠ»ΠΎΠΆΠΈΡ‚ΡŒ силу ΠΊ P , Ρ‡Ρ‚ΠΎΠ±Ρ‹ ΡƒΠ΄Π΅Ρ€ΠΆΠΈΠ²Π°Ρ‚ΡŒ ΠΏΠΎΠΊΠ°Π·Π°Π½Π½ΡƒΡŽ ΠΊΠΎΠ½ΡΡ‚Ρ€ΡƒΠΊΡ†ΠΈΡŽ Π² равновСсии? ВСс конструкции Π½Π΅Π·Π½Π°Ρ‡ΠΈΡ‚Π΅Π»ΡŒΠ½Ρ‹ΠΉ.

Π”Π²ΠΎΠ΅ Π΄Π΅Ρ‚Π΅ΠΉ Ρ‚ΠΎΠ»ΠΊΠ°ΡŽΡ‚ ΠΏΡ€ΠΎΡ‚ΠΈΠ²ΠΎΠΏΠΎΠ»ΠΎΠΆΠ½Ρ‹Π΅ стороны Π΄Π²Π΅Ρ€ΠΈ Π²ΠΎ врСмя ΠΈΠ³Ρ€Ρ‹. Оба Ρ‚ΠΎΠ»ΠΊΠ°ΡŽΡ‚ΡΡ Π³ΠΎΡ€ΠΈΠ·ΠΎΠ½Ρ‚Π°Π»ΡŒΠ½ΠΎ ΠΈ пСрпСндикулярно Π΄Π²Π΅Ρ€ΠΈ. Один Ρ€Π΅Π±Π΅Π½ΠΎΠΊ Ρ‚ΠΎΠ»ΠΊΠ°Π΅Ρ‚ с силой 17,5 Н Π½Π° расстоянии 0,600 ΠΌ ΠΎΡ‚ ΠΏΠ΅Ρ‚Π΅Π»ΡŒ, Π° Π²Ρ‚ΠΎΡ€ΠΎΠΉ Ρ€Π΅Π±Π΅Π½ΠΎΠΊ Ρ‚ΠΎΠ»ΠΊΠ°Π΅Ρ‚ Π½Π° расстоянии 0,450 ΠΌ. ΠšΠ°ΠΊΡƒΡŽ силу Π΄ΠΎΠ»ΠΆΠ΅Π½ ΠΏΡ€ΠΈΠ»ΠΎΠΆΠΈΡ‚ΡŒ Π²Ρ‚ΠΎΡ€ΠΎΠΉ Ρ€Π΅Π±Π΅Π½ΠΎΠΊ, Ρ‡Ρ‚ΠΎΠ±Ρ‹ Π΄Π²Π΅Ρ€ΡŒ Π½Π΅ двигалась? ΠŸΡ€Π΅Π΄ΠΏΠΎΠ»ΠΎΠΆΠΈΠΌ, Ρ‚Ρ€Π΅Π½ΠΈΠ΅ Π½Π΅Π·Π½Π°Ρ‡ΠΈΡ‚Π΅Π»ΡŒΠ½ΠΎ.

[ΠΏΠΎΠΊΠ°Π·Ρ‹Π²Π°Ρ‚ΡŒ-ΠΎΡ‚Π²Π΅Ρ‚ q = ”fs-id1163713183876 β€³] ΠŸΠΎΠΊΠ°Π·Π°Ρ‚ΡŒ Ρ€Π΅ΡˆΠ΅Π½ΠΈΠ΅ [/ ΠΏΠΎΠΊΠ°Π·Ρ‹Π²Π°Ρ‚ΡŒ-ΠΎΡ‚Π²Π΅Ρ‚]

[скрытый-ΠΎΡ‚Π²Π΅Ρ‚ a = ”fs-id1163713183876 β€³]

23.3 N

[/ hidden-answer]

НСбольшой Π²Π½Π΅Π΄ΠΎΡ€ΠΎΠΆΠ½ΠΈΠΊ массой 1000 ΠΊΠ³ ΠΈΠΌΠ΅Π΅Ρ‚ ΠΊΠΎΠ»Π΅ΡΠ½ΡƒΡŽ Π±Π°Π·Ρƒ 3,0 ΠΌ. Если 60%, Ссли Π΅Π³ΠΎ вСс приходится Π½Π° ΠΏΠ΅Ρ€Π΅Π΄Π½ΠΈΠ΅ колСса, насколько Π΄Π°Π»Π΅ΠΊΠΎ ΠΏΠΎΠ·Π°Π΄ΠΈ ΠΏΠ΅Ρ€Π΅Π΄Π½ΠΈΡ… колСс находится Ρ†Π΅Π½Ρ‚Ρ€ масс Ρ„ΡƒΡ€Π³ΠΎΠ½Π°?

Π£Π½ΠΈΡ„ΠΈΡ†ΠΈΡ€ΠΎΠ²Π°Π½Π½Ρ‹Π΅ ΠΊΠ°Ρ‡Π΅Π»ΠΈ сбалансированы Π² Ρ†Π΅Π½Ρ‚Ρ€Π΅ масс, ΠΊΠ°ΠΊ ΠΏΠΎΠΊΠ°Π·Π°Π½ΠΎ Π½ΠΈΠΆΠ΅. МалСнький ΠΌΠ°Π»ΡŒΡ‡ΠΈΠΊ справа ΠΈΠΌΠ΅Π΅Ρ‚ массу 40,0 ΠΊΠ³. Какая масса Ρƒ Π΅Π³ΠΎ Π΄Ρ€ΡƒΠ³Π°?

[ΠΏΠΎΠΊΠ°Π·Π°Ρ‚ΡŒ-ΠΎΡ‚Π²Π΅Ρ‚ q = ”5 β€³] ΠŸΠΎΠΊΠ°Π·Π°Ρ‚ΡŒ ΠΎΡ‚Π²Π΅Ρ‚ [/ Ρ€Π°ΡΠΊΡ€Ρ‹Ρ‚ΡŒ-ΠΎΡ‚Π²Π΅Ρ‚]
[скрытый-ΠΎΡ‚Π²Π΅Ρ‚ a =”5 β€³] 80.0 ΠΊΠ³ [/ hidden-answer]

Глоссарий

Ρ†Π΅Π½Ρ‚Ρ€ тяТСсти
Ρ‚ΠΎΡ‡ΠΊΠ°, ΠΊ ΠΊΠΎΡ‚ΠΎΡ€ΠΎΠΉ ΠΏΡ€ΠΈΠΊΡ€Π΅ΠΏΠ»Π΅Π½ Π²Π΅ΠΊΡ‚ΠΎΡ€ вСсов
равновСсиС
Π’Π΅Π»ΠΎ
находится Π² равновСсии, ΠΊΠΎΠ³Π΄Π° Π΅Π³ΠΎ Π»ΠΈΠ½Π΅ΠΉΠ½ΠΎΠ΅ ΠΈ ΡƒΠ³Π»ΠΎΠ²ΠΎΠ΅ ускорСния Ρ€Π°Π²Π½Ρ‹ Π½ΡƒΠ»ΡŽ ΠΎΡ‚Π½ΠΎΡΠΈΡ‚Π΅Π»ΡŒΠ½ΠΎ ΠΈΠ½Π΅Ρ€Ρ†ΠΈΠ°Π»ΡŒΠ½ΠΎΠΉ систСмы отсчСта
ΠΏΠ΅Ρ€Π²ΠΎΠ΅ условиС равновСсия
Π²Ρ‹Ρ€Π°ΠΆΠ°Π΅Ρ‚ ΠΏΠΎΡΡ‚ΡƒΠΏΠ°Ρ‚Π΅Π»ΡŒΠ½ΠΎΠ΅ равновСсиС; всС внСшниС силы, Π΄Π΅ΠΉΡΡ‚Π²ΡƒΡŽΡ‰ΠΈΠ΅ Π½Π° Ρ‚Π΅Π»ΠΎ, ΡƒΡ€Π°Π²Π½ΠΎΠ²Π΅ΡˆΠΈΠ²Π°ΡŽΡ‚ΡΡ ΠΈ ΠΈΡ… вСкторная сумма Ρ€Π°Π²Π½Π° Π½ΡƒΠ»ΡŽ
Π³Ρ€Π°Π²ΠΈΡ‚Π°Ρ†ΠΈΠΎΠ½Π½Ρ‹ΠΉ ΠΌΠΎΠΌΠ΅Π½Ρ‚
крутящий ΠΌΠΎΠΌΠ΅Π½Ρ‚ Π½Π° корпусС, Π²Ρ‹Π·Π²Π°Π½Π½Ρ‹ΠΉ Π΅Π³ΠΎ вСсом; Π²ΠΎΠ·Π½ΠΈΠΊΠ°Π΅Ρ‚, ΠΊΠΎΠ³Π΄Π° Ρ†Π΅Π½Ρ‚Ρ€ тяТСсти Ρ‚Π΅Π»Π° Π½Π΅ располоТСн Π½Π° оси вращСния
Π²Ρ‚ΠΎΡ€ΠΎΠ΅ состояниС равновСсия
Π²Ρ‹Ρ€Π°ΠΆΠ°Π΅Ρ‚ Π²Ρ€Π°Ρ‰Π°Ρ‚Π΅Π»ΡŒΠ½ΠΎΠ΅ равновСсиС; всС крутящиС ΠΌΠΎΠΌΠ΅Π½Ρ‚Ρ‹ ΠΎΡ‚ Π²Π½Π΅ΡˆΠ½ΠΈΡ… сил, Π΄Π΅ΠΉΡΡ‚Π²ΡƒΡŽΡ‰ΠΈΡ… Π½Π° Ρ‚Π΅Π»ΠΎ, ΡƒΡ€Π°Π²Π½ΠΎΠ²Π΅ΡˆΠΈΠ²Π°ΡŽΡ‚ΡΡ ΠΈ ΠΈΡ… вСкторная сумма Ρ€Π°Π²Π½Π° Π½ΡƒΠ»ΡŽ
статичСскоС равновСсиС
Π’Π΅Π»ΠΎ находится Π² статичСском равновСсии, ΠΊΠΎΠ³Π΄Π° ΠΎΠ½ΠΎ покоится Π² Π²Ρ‹Π±Ρ€Π°Π½Π½ΠΎΠΉ Π½Π°ΠΌΠΈ ΠΈΠ½Π΅Ρ€Ρ†ΠΈΠ°Π»ΡŒΠ½ΠΎΠΉ систСмС отсчСта

БостояниС равновСсия — ΠΎΠ±Π·ΠΎΡ€

ΠšΠΎΠ½Ρ‚Ρ€ΠΎΠ»ΡŒ кристалличСской Ρ„Π°Π·Ρ‹

ΠšΡ€ΠΈΡΡ‚Π°Π»Π»ΠΈΡ‡Π΅ΡΠΊΠ°Ρ Ρ„Π°Π·Π° Ρ‚ΠΎΠ½ΠΊΠΎΠΏΠ»Π΅Π½ΠΎΡ‡Π½Ρ‹Ρ… ΠΌΠ°Ρ‚Π΅Ρ€ΠΈΠ°Π»ΠΎΠ² Π² основном контролируСтся Ρ‚Π΅ΠΌΠΏΠ΅Ρ€Π°Ρ‚ΡƒΡ€ΠΎΠΉ роста ΠΈ химичСским составом Ρ‚ΠΎΠ½ΠΊΠΈΡ… ΠΏΠ»Π΅Π½ΠΎΠΊ.ΠšΡ€ΠΈΡΡ‚Π°Π»Π»ΠΈΡ‡Π΅ΡΠΊΠ°Ρ Ρ„Π°Π·Π° Ρ‚ΠΎΠ½ΠΊΠΎΠΏΠ»Π΅Π½ΠΎΡ‡Π½Ρ‹Ρ… ΠΌΠ°Ρ‚Π΅Ρ€ΠΈΠ°Π»ΠΎΠ² Π² основном основана Π½Π° Ρ„Π°Π·ΠΎΠ²ΠΎΠΉ Π΄ΠΈΠ°Π³Ρ€Π°ΠΌΠΌΠ΅ ΠΎΠ±ΡŠΠ΅ΠΌΠ½Ρ‹Ρ… ΠΌΠ°Ρ‚Π΅Ρ€ΠΈΠ°Π»ΠΎΠ². Однако кристалличСская фазовая Π΄ΠΈΠ°Π³Ρ€Π°ΠΌΠΌΠ° Ρ‚ΠΎΠ½ΠΊΠΈΡ… ΠΏΠ»Π΅Π½ΠΎΠΊ сущСствСнно отличаСтся ΠΎΡ‚ ΠΎΠ±ΡŠΠ΅ΠΌΠ½Ρ‹Ρ…, ΠΏΠΎΡΠΊΠΎΠ»ΡŒΠΊΡƒ Ρ‚ΠΎΠ½ΠΊΠΈΠ΅ ΠΏΠ»Π΅Π½ΠΊΠΈ Π²Ρ‹Ρ€Π°Ρ‰ΠΈΠ²Π°ΡŽΡ‚ΡΡ Π² условиях Π½Π΅Ρ‚Π΅ΠΏΠ»ΠΎΠ²ΠΎΠ³ΠΎ равновСсия.

УсловиС Π½Π΅Ρ‚Π΅ΠΏΠ»ΠΎΠ²ΠΎΠ³ΠΎ равновСсия Π²Ρ‹Π·Ρ‹Π²Π°Π΅Ρ‚ ΡƒΠ½ΠΈΠΊΠ°Π»ΡŒΠ½Ρ‹Π΅ структурныС свойства Ρ‚ΠΎΠ½ΠΊΠΈΡ… ΠΏΠ»Π΅Π½ΠΎΠΊ, Π²ΠΊΠ»ΡŽΡ‡Π°Ρ Ρ€Π΅Π»Π°ΠΊΡΠ°Ρ†ΠΈΡŽ растворимости ΠΈ ΠΎΠ±Ρ€Π°Π·ΠΎΠ²Π°Π½ΠΈΠ΅ ΠΌΠ΅Ρ‚Π°ΡΡ‚Π°Π±ΠΈΠ»ΡŒΠ½ΠΎΠΉ кристалличСской Ρ„Π°Π·Ρ‹. Π‘ΠΊΠΎΡ€ΠΎΡΡ‚ΡŒ роста Ρ‚ΠΎΠ½ΠΊΠΈΡ… ΠΏΠ»Π΅Π½ΠΎΠΊ Ρ‚Π°ΠΊΠΆΠ΅ являСтся Π΄Ρ€ΡƒΠ³ΠΈΠΌ Π²Π°ΠΆΠ½Ρ‹ΠΌ ΠΏΠ°Ρ€Π°ΠΌΠ΅Ρ‚Ρ€ΠΎΠΌ, Π²Π»ΠΈΡΡŽΡ‰ΠΈΠΌ Π½Π° ΠΊΡ€ΠΈΡΡ‚Π°Π»Π»ΠΈΡ‡Π΅ΡΠΊΡƒΡŽ Ρ„Π°Π·Ρƒ ΠΈ / ΠΈΠ»ΠΈ ΠΎΡ€ΠΈΠ΅Π½Ρ‚Π°Ρ†ΠΈΡŽ кристаллов Ρ‚ΠΎΠ½ΠΊΠΈΡ… ΠΏΠ»Π΅Π½ΠΎΠΊ.ОбъСмная фазовая Π΄ΠΈΠ°Π³Ρ€Π°ΠΌΠΌΠ° Ρ‚ΠΎΠ½ΠΊΠΈΡ… ΠΏΠ»Π΅Π½ΠΎΠΊ Π΄ΠΎΠ»ΠΆΠ½Π° Π±Ρ‹Ρ‚ΡŒ составлСна β€‹β€‹Ρ‚Ρ‰Π°Ρ‚Π΅Π»ΡŒΠ½ΠΎ, Ρ‡Ρ‚ΠΎΠ±Ρ‹ Π»ΡƒΡ‡ΡˆΠ΅ ΠΏΠΎΠ½ΡΡ‚ΡŒ ΠΊΡ€ΠΈΡΡ‚Π°Π»Π»ΠΈΡ‡Π΅ΡΠΊΡƒΡŽ Ρ„Π°Π·Ρƒ составных Ρ‚ΠΎΠ½ΠΊΠΈΡ… ΠΏΠ»Π΅Π½ΠΎΠΊ. Випичная фазовая Π΄ΠΈΠ°Π³Ρ€Π°ΠΌΠΌΠ° PbTiO 3 для Ρ‚ΠΎΠ½ΠΊΠΈΡ… ΠΏΠ»Π΅Π½ΠΎΠΊ ΠΏΠΎΠΊΠ°Π·Π°Π½Π° Π½Π° Рис. 5.17 (a) 13 , Π° фазовая Π΄ΠΈΠ°Π³Ρ€Π°ΠΌΠΌΠ° объСмного PbTiO 3 ΠΏΠΎΠΊΠ°Π·Π°Π½Π° Π½Π° Рис. 5.17 (b). b

Рисунок 5.17. Ѐазовая Π΄ΠΈΠ°Π³Ρ€Π°ΠΌΠΌΠ° Π½Π°ΠΏΡ‹Π»Π΅Π½Π½Ρ‹Ρ… Ρ‚ΠΎΠ½ΠΊΠΈΡ… ΠΏΠ»Π΅Π½ΠΎΠΊ PbTiO 3 (Π°) ΠΈ объСмной ΠΊΠ΅Ρ€Π°ΠΌΠΈΠΊΠΈ (Π±).

Из-Π·Π° Π½Π΅Ρ‚Π΅ΠΏΠ»ΠΎΠ²ΠΎΠ³ΠΎ равновСсия процСсса осаТдСния Ρ‚ΠΎΠ½ΠΊΠΈΡ… ΠΏΠ»Π΅Π½ΠΎΠΊ, ΡΡƒΡ‰Π΅ΡΡ‚Π²ΡƒΡŽΡ‚ ΡƒΠ½ΠΈΠΊΠ°Π»ΡŒΠ½Ρ‹Π΅ процСссы контроля Ρ„Π°Π·Ρ‹ для слоТных Ρ‚ΠΎΠ½ΠΊΠΈΡ… ΠΏΠ»Π΅Π½ΠΎΠΊ.Один ΠΈΠ· Π½ΠΈΡ… — Π·Π°ΠΊΠ°Π»ΠΊΠ° послС напылСния, Π΄Ρ€ΡƒΠ³ΠΎΠΉ — пСриодичСскоС Π½Π°ΠΏΡ‹Π»Π΅Π½ΠΈΠ΅.

Π˜Π·Π²Π΅ΡΡ‚Π½ΠΎ, Ρ‡Ρ‚ΠΎ ΡΠΊΠΎΡ€ΠΎΡΡ‚ΡŒ охлаТдСния послС ΡΠΏΠΈΡ‚Π°ΠΊΡΠΈΠ°Π»ΡŒΠ½ΠΎΠ³ΠΎ роста влияСт Π½Π° ΠΎΡ€ΠΈΠ΅Π½Ρ‚Π°Ρ†ΠΈΡŽ кристаллов ΡΠΏΠΈΡ‚Π°ΠΊΡΠΈΠ°Π»ΡŒΠ½Ρ‹Ρ… Ρ‚ΠΎΠ½ΠΊΠΈΡ… ΠΏΠ»Π΅Π½ΠΎΠΊ. НапримСр, Ρ‚Π΅Ρ‚Ρ€Π°Π³ΠΎΠ½Π°Π»ΡŒΠ½Ρ‹Π΅ Ρ‚ΠΎΠ½ΠΊΠΈΠ΅ ΠΏΠ»Π΅Π½ΠΊΠΈ PbTiO 3 , ΡΠΏΠΈΡ‚Π°ΠΊΡΠΈΠ°Π»ΡŒΠ½ΠΎ Π²Ρ‹Ρ€Π°Ρ‰Π΅Π½Π½Ρ‹Π΅ Π½Π° ΠΏΠΎΠ΄Π»ΠΎΠΆΠΊΠ°Ρ… (001) MgO ΠΏΡ€ΠΈ ΡΠΏΠΈΡ‚Π°ΠΊΡΠΈΠ°Π»ΡŒΠ½ΠΎΠΉ Ρ‚Π΅ΠΌΠΏΠ΅Ρ€Π°Ρ‚ΡƒΡ€Π΅ 600 Β° C с ΠΏΠΎΡΠ»Π΅Π΄ΡƒΡŽΡ‰ΠΈΠΌ СстСствСнным ΠΎΡ…Π»Π°ΠΆΠ΄Π΅Π½ΠΈΠ΅ΠΌ, Π΄Π΅ΠΌΠΎΠ½ΡΡ‚Ρ€ΠΈΡ€ΡƒΡŽΡ‚ ΠΎΡ€ΠΈΠ΅Π½Ρ‚Π°Ρ†ΠΈΡŽ (100) PbTiO 3 , ΠΏΠΎΡΠΊΠΎΠ»ΡŒΠΊΡƒ ΠΏΠ°Ρ€Π°ΠΌΠ΅Ρ‚Ρ€ Ρ€Π΅ΡˆΠ΅Ρ‚ΠΊΠΈ c Π±Π»ΠΈΠ·ΠΎΠΊ ΠΊ ΠΏΠ°Ρ€Π°ΠΌΠ΅Ρ‚Ρ€ Ρ€Π΅ΡˆΠ΅Ρ‚ΠΊΠΈ MgO.Однако, Ссли ΡΠΏΠΈΡ‚Π°ΠΊΡΠΈΠ°Π»ΡŒΠ½Ρ‹Π΅ Ρ‚ΠΎΠ½ΠΊΠΈΠ΅ ΠΏΠ»Π΅Π½ΠΊΠΈ PbTiO 3 Π·Π°ΠΊΠ°Π»ΠΈΠ²Π°ΡŽΡ‚ послС ΡΠΏΠΈΡ‚Π°ΠΊΡΠΈΠ°Π»ΡŒΠ½ΠΎΠ³ΠΎ осаТдСния, распылСнныС Ρ‚ΠΎΠ½ΠΊΠΈΠ΅ ΠΏΠ»Π΅Π½ΠΊΠΈ ΠΏΠΎΠΊΠ°Π·Ρ‹Π²Π°ΡŽΡ‚ ΠΎΡ€ΠΈΠ΅Π½Ρ‚Π°Ρ†ΠΈΡŽ (001) PbTiO 3 , ΠΊΠ°ΠΊ ΠΏΠΎΠΊΠ°Π·Π°Π½ΠΎ Π½Π° рис. 5.18 14 , ΠΊΠΎΡ‚ΠΎΡ€Ρ‹ΠΉ Π±ΡƒΠ΄Π΅Ρ‚ ΠΏΠΎΠ΄Ρ€ΠΎΠ±Π½ΠΎ описан Π² ΠΏΡ€Π΅Π΄Ρ‹Π΄ΡƒΡ‰Π΅ΠΉ Π³Π»Π°Π²Π΅. . ΠžΡ€ΠΈΠ΅Π½Ρ‚Π°Ρ†ΠΈΡ кристалла контролируСтся ΡΠΊΠΎΡ€ΠΎΡΡ‚ΡŒΡŽ охлаТдСния.

Рисунок 5.18. ΠšΠ°Ρ€Ρ‚ΠΈΠ½Ρ‹ рСнтгСновской Π΄ΠΈΡ„Ρ€Π°ΠΊΡ†ΠΈΠΈ распылСнных Ρ‚ΠΎΠ½ΠΊΠΈΡ… ΠΏΠ»Π΅Π½ΠΎΠΊ PbTiO 3 , Ρ‚ΠΎΠ»Ρ‰ΠΈΠ½ΠΎΠΉ 120 Π½ΠΌ, Π½Π° (001) MgO: (Π°) быстроС ΠΎΡ…Π»Π°ΠΆΠ΄Π΅Π½ΠΈΠ΅; (Π±) ΠΌΠ΅Π΄Π»Π΅Π½Π½ΠΎΠ΅ ΠΎΡ…Π»Π°ΠΆΠ΄Π΅Π½ΠΈΠ΅.

Π₯имичСский состав Π½Π°ΠΏΡ‹Π»Π΅Π½Π½Ρ‹Ρ… Ρ‚ΠΎΠ½ΠΊΠΈΡ… ΠΏΠ»Π΅Π½ΠΎΠΊ Π² основном Ρ‚Π°ΠΊΠΎΠΉ ΠΆΠ΅, ΠΊΠ°ΠΊ ΠΈ состав мишСни.Однако химичСский состав Π½Π°ΠΏΡ‹Π»Π΅Π½Π½Ρ‹Ρ… Ρ‚ΠΎΠ½ΠΊΠΈΡ… ΠΏΠ»Π΅Π½ΠΎΠΊ отличаСтся ΠΎΡ‚ Ρ†Π΅Π»Π΅Π²ΠΎΠ³ΠΎ состава слоистых комплСксных соСдинСний. Π’ΠΈΠΏΠΈΡ‡Π½Ρ‹ΠΌΠΈ ΠΏΡ€ΠΈΠΌΠ΅Ρ€Π°ΠΌΠΈ ΡΠ²Π»ΡΡŽΡ‚ΡΡ двухслойныС пСровскитныС соСдинСния, ΠΏΠΎΠΊΠ°Π·Π°Π½Π½Ρ‹Π΅ Π½Π° рис. 5.19. 15 БущСствуСт нСсколько Ρ‚ΠΈΠΏΠΎΠ² двухслойных пСровскитных соСдинСний, Π²ΠΊΠ»ΡŽΡ‡Π°Ρ BSCCO (BiSrCaCuO): Для осаТдСния распылСниСм BSCCO с нСбольшими количСствами Cu Ρ†Π΅Π»Π΅Π²Ρ‹Π΅ составы пСрСносятся Π½Π° Ρ‚ΠΎΠ½ΠΊΠΈΠ΅ ΠΏΠ»Π΅Π½ΠΊΠΈ, ΠΊΠ°ΠΊ ΠΏΠΎΠΊΠ°Π·Π°Π½ΠΎ Π² ΡƒΡ€Π°Π²Π½Π΅Π½ΠΈΠΈ. (5.10). Когда количСство Cu увСличиваСтся, состав Ρ‚ΠΎΠ½ΠΊΠΈΡ… ΠΏΠ»Π΅Π½ΠΎΠΊ отличаСтся ΠΎΡ‚ Ρ†Π΅Π»Π΅Π²ΠΎΠ³ΠΎ, ΠΊΠ°ΠΊ ΠΏΠΎΠΊΠ°Π·Π°Π½ΠΎ Π² Ρ„ΠΎΡ€ΠΌΡƒΠ»Π΅.(5.11) ΠΈΠ·-Π·Π° разлоТСния ΠΏΡ€ΠΈ ростС ΠΏΠ»Π΅Π½ΠΊΠΈ.

Рисунок 5.19. ПослойноС нанСсСниС Bi 2 Sr 2 Ca n βˆ’1 Cu n O x .

Однако, Ссли Π½Π°ΠΏΡ‹Π»Π΅Π½ΠΈΠ΅ проводится ΠΏΡƒΡ‚Π΅ΠΌ послойного осаТдСния слоСв BiO ΠΈ SrCaCuO с Π²Ρ‹Π΄Π΅Ρ€ΠΆΠΊΠΎΠΉ Π²Ρ€Π΅ΠΌΠ΅Π½ΠΈ ΠΌΠ΅ΠΆΠ΄Ρƒ нанСсСниСм слоя BiO ΠΈ слоя SrCaCuO, состав ΠΏΠΎΠ»ΡƒΡ‡Π°Π΅ΠΌΠΎΠΉ ΠΏΠ»Π΅Π½ΠΊΠΈ Ρ‚Π°ΠΊΠΎΠΉ ΠΆΠ΅, ΠΊΠ°ΠΊ ΠΈ Ρƒ мишСни. ВрСмя оТидания ΠΎΠΊΠΎΠ»ΠΎ 4 ΠΌΠΈΠ½.

Π¦Π΅Π»Π΅Π²Ρ‹Π΅ Ρ‚ΠΎΠ½ΠΊΠΈΠ΅ ΠΏΠ»Π΅Π½ΠΊΠΈ

(5.10) Bi2Sr2Ca2Cu3O10 β†’ Bi2Sr2Ca2Cu3O10

(5.11) Bi2Sr2Ca3Cu4O12 β†’ Bi2Sr2Ca2Cu3O10 + CaCuO2

(5,12) Bi2Sr2Ca3Cu4O12 β†’ Bi2Sr2Ca3Cu4O12intermitted

Разновидности процСссов распылСния доступны для управлСния Ρ„Π°Π·ΠΎΠΉ распылСнных соСдинСния Ρ‚ΠΎΠ½ΠΊΠΈΡ… ΠΏΠ»Π΅Π½ΠΎΠΊ. c

РавновСсиС сил

ΠžΡ‡Π΅Π½ΡŒ простая концСпция ΠΏΡ€ΠΈ Ρ€Π°Π±ΠΎΡ‚Π΅ с силы это идСя равновСсия ΠΈΠ»ΠΈ баланса .Π’ ΠΎΠ±Ρ‰Π΅ΠΌ, Π½Π° ΠΎΠ±ΡŠΠ΅ΠΊΡ‚ ΠΌΠΎΠ³ΡƒΡ‚ Π΄Π΅ΠΉΡΡ‚Π²ΠΎΠ²Π°Ρ‚ΡŒ нСсколько сил. Π² Ρ‚ΠΎ ΠΆΠ΅ врСмя. Π‘ΠΈΠ»Π° — это вСкторная Π²Π΅Π»ΠΈΡ‡ΠΈΠ½Π° Ρ‡Ρ‚ΠΎ Π·Π½Π°Ρ‡ΠΈΡ‚ Ρ‡Ρ‚ΠΎ ΠΎΠ½ ΠΈΠΌΠ΅Π΅Ρ‚ ΠΊΠ°ΠΊ Π²Π΅Π»ΠΈΡ‡ΠΈΠ½Ρƒ (Ρ€Π°Π·ΠΌΠ΅Ρ€), Ρ‚Π°ΠΊ ΠΈ Π½Π°ΠΏΡ€Π°Π²Π»Π΅Π½ΠΈΠ΅, связанноС с Π½ΠΈΠΌ. Если Ρ€Π°Π·ΠΌΠ΅Ρ€ ΠΈ Π½Π°ΠΏΡ€Π°Π²Π»Π΅Π½ΠΈΠ΅ сил, Π΄Π΅ΠΉΡΡ‚Π²ΡƒΡŽΡ‰ΠΈΡ… Π½Π° ΠΎΠ±ΡŠΠ΅ΠΊΡ‚, Ρ€Π°Π²Π½Ρ‹ Ρ‚ΠΎΡ‡Π½ΠΎ сбалансировано, Ρ‚ΠΎΠ³Π΄Π° Π½Π° ΠΎΠ±ΡŠΠ΅ΠΊΡ‚ Π½Π΅ дСйствуСт чистая сила ΠΈ ΠΎΠ±ΡŠΠ΅ΠΊΡ‚ находится Π² состоянии равновСсия . ΠŸΠΎΡΠΊΠΎΠ»ΡŒΠΊΡƒ Π½Π° ΠΎΠ±ΡŠΠ΅ΠΊΡ‚ Π² состоянии равновСсия Π½Π΅ дСйствуСт Ρ€Π΅Π·ΡƒΠ»ΡŒΡ‚ΠΈΡ€ΡƒΡŽΡ‰Π°Ρ сила, Π·Π°Ρ‚Π΅ΠΌ ΠΈΠ· ΠΡŒΡŽΡ‚ΠΎΠ½Π° ΠΏΠ΅Ρ€Π²Ρ‹ΠΉ Π·Π°ΠΊΠΎΠ½ двиТСния, Π½Π΅ΠΏΠΎΠ΄Π²ΠΈΠΆΠ½Ρ‹ΠΉ ΠΎΠ±ΡŠΠ΅ΠΊΡ‚ останСтся Π² ΠΏΠΎΠΊΠΎΠ΅, Π° ΠΎΠ±ΡŠΠ΅ΠΊΡ‚ Π² Π΄Π²ΠΈΠΆΠ΅Π½ΠΈΠΈ останСтся Π² Π΄Π²ΠΈΠΆΠ΅Π½ΠΈΠΈ.

НачнСм с ΠΏΡ€ΠΎΡΡ‚Π΅ΠΉΡˆΠ΅Π³ΠΎ ΠΏΡ€ΠΈΠΌΠ΅Ρ€Π° Π΄Π²ΡƒΡ… сил, Π΄Π΅ΠΉΡΡ‚Π²ΡƒΡŽΡ‰ΠΈΡ… Π½Π° ΠΎΠ±ΡŠΠ΅ΠΊΡ‚. Π—Π°Ρ‚Π΅ΠΌ ΠΌΡ‹ ΠΏΠΎΠΊΠ°ΠΆΠ΅ΠΌ ΠΏΡ€ΠΈΠΌΠ΅Ρ€Ρ‹ Ρ‚Ρ€ΠΈ силы, Π΄Π΅ΠΉΡΡ‚Π²ΡƒΡŽΡ‰ΠΈΠ΅ Π½Π° ΠΏΠ»Π°Π½Π΅Ρ€, ΠΈ Ρ‡Π΅Ρ‚Ρ‹Ρ€Π΅ силы, Π΄Π΅ΠΉΡΡ‚Π²ΡƒΡŽΡ‰ΠΈΠ΅ Π½Π° Π»Π΅Ρ‚Π°Ρ‚Π΅Π»ΡŒΠ½Ρ‹ΠΉ Π°ΠΏΠΏΠ°Ρ€Π°Ρ‚ с Π΄Π²ΠΈΠ³Π°Ρ‚Π΅Π»Π΅ΠΌ.

Π’ ΠΏΡ€ΠΈΠΌΠ΅Ρ€Π΅ 1 Π½Π° слайдС ΠΌΡ‹ ΠΏΠΎΠΊΠ°Π·Ρ‹Π²Π°Π΅ΠΌ синий ΡˆΠ°Ρ€, ΠΊΠΎΡ‚ΠΎΡ€Ρ‹ΠΉ Ρ‚ΠΎΠ»ΠΊΠ°Π΅Ρ‚ Π΄Π²Π΅ силы, ΠΎΠ±ΠΎΠ·Π½Π°Ρ‡Π΅Π½Π½Ρ‹Π΅ Force # 1 F1 ΠΈ Force # 2 F2 . Помни это силы — это Π²Π΅ΠΊΡ‚ΠΎΡ€Π½Ρ‹Π΅ Π²Π΅Π»ΠΈΡ‡ΠΈΠ½Ρ‹, ΠΈ Π½Π°ΠΏΡ€Π°Π²Π»Π΅Π½ΠΈΠ΅ Π²Π°ΠΆΠ½ΠΎ. Π”Π²Π΅ силы ΠΎΠ΄ΠΈΠ½Π°ΠΊΠΎΠ²ΠΎΠΉ Π²Π΅Π»ΠΈΡ‡ΠΈΠ½Ρ‹, Π½ΠΎ Ρ€Π°Π·Π½Ρ‹Π΅ направлСния Π½Π΅ Ρ€Π°Π²Π½Ρ‹Π΅ силы.По Ρ„Π°ΠΊΡ‚Ρƒ,

F1 = — F2

для ΠΏΠΎΠΊΠ°Π·Π°Π½Π½ΠΎΠΉ систСмы ΠΊΠΎΠΎΡ€Π΄ΠΈΠ½Π°Ρ‚ с Π±ΡƒΠΊΠ²ΠΎΠΉ X ΠΏΠΎΠ΄ мячом. Если ΡΠ»ΠΎΠΆΠΈΡ‚ΡŒ силы, Π΄Π΅ΠΉΡΡ‚Π²ΡƒΡŽΡ‰ΠΈΠ΅ Π½Π° мяч, ΠΏΠΎΠ»ΡƒΡ‡Π°Π΅ΠΌ силовоС ΡƒΡ€Π°Π²Π½Π΅Π½ΠΈΠ΅ слСва:

F1 + F2 = F ΡΠ΅Ρ‚ΡŒ = 0

Π³Π΄Π΅ F net — чистая сила, Π΄Π΅ΠΉΡΡ‚Π²ΡƒΡŽΡ‰Π°Ρ Π½Π° мяч. ΠŸΠΎΡΠΊΠΎΠ»ΡŒΠΊΡƒ Ρ€Π΅Π·ΡƒΠ»ΡŒΡ‚ΠΈΡ€ΡƒΡŽΡ‰Π°Ρ сила Ρ€Π°Π²Π½Π° Π½ΡƒΠ»ΡŽ, силы Π² ΠΏΡ€ΠΈΠΌΠ΅Ρ€Π΅ 1 Ρ€Π°Π²Π½Ρ‹ дСйствуСт Π² равновСсии.

Π’ ΠŸΡ€ΠΈΠΌΠ΅Ρ€Π΅ 1 Π½Π° мяч Π½Π΅ дСйствуСт чистая сила.ΠŸΠΎΡΠΊΠΎΠ»ΡŒΠΊΡƒ мяч ΠΈΠ·Π½Π°Ρ‡Π°Π»ΡŒΠ½ΠΎ находится Π² состоянии покоя (ΡΠΊΠΎΡ€ΠΎΡΡ‚ΡŒ Ρ€Π°Π²Π½Π° Π½ΡƒΠ»ΡŽ), мяч останСтся Π² ΠΏΠΎΠΊΠΎΠ΅ согласно ΠΡŒΡŽΡ‚ΠΎΠ½Ρƒ. ΠΏΠ΅Ρ€Π²Ρ‹ΠΉ Π·Π°ΠΊΠΎΠ½ двиТСния. Если Π±Ρ‹ мяч двигался с постоянной ΡΠΊΠΎΡ€ΠΎΡΡ‚ΡŒΡŽ, ΠΎΠ½ ΠΏΡ€ΠΎΠ΄ΠΎΠ»ΠΆΠ°Π» Π±Ρ‹ Π΄Π²ΠΈΠ³Π°Ρ‚ΡŒΡΡ. с Ρ‚ΠΎΠΉ ΠΆΠ΅ ΡΠΊΠΎΡ€ΠΎΡΡ‚ΡŒΡŽ.

Π’ ΠŸΡ€ΠΈΠΌΠ΅Ρ€Π΅ 2 ΠΌΡ‹ ΡƒΠ²Π΅Π»ΠΈΡ‡ΠΈΠ»ΠΈ Π²Π΅Π»ΠΈΡ‡ΠΈΠ½Ρƒ Π‘ΠΈΠ»Ρ‹ β„–1, Ρ‚Π°ΠΊ Ρ‡Ρ‚ΠΎ ΠΎΠ½Π° Π½Π°ΠΌΠ½ΠΎΠ³ΠΎ большС. Ρ‡Π΅ΠΌ Force # 2. Π‘ΠΈΠ»Ρ‹ большС Π½Π΅ находятся Π² равновСсии. Π£Ρ€Π°Π²Π½Π΅Π½ΠΈΠ΅ силы остаСтся ΠΏΡ€Π΅ΠΆΠ½ΠΈΠΌ, Π½ΠΎ Ρ€Π΅Π·ΡƒΠ»ΡŒΡ‚ΠΈΡ€ΡƒΡŽΡ‰Π°Ρ сила Π½Π΅ Ρ€Π°Π²Π½Π° Π½ΡƒΠ»ΡŽ.Π’Π΅Π»ΠΈΡ‡ΠΈΠ½Π° чистой силы опрСдСляСтся ΠΏΠΎ Ρ„ΠΎΡ€ΠΌΡƒΠ»Π΅:

F1> — F2

F1 + F2 = F Π½Π΅Ρ‚Ρ‚ΠΎ

| F net | = | F1 | — | F2 |

Π³Π΄Π΅ «| |» символы ΡƒΠΊΠ°Π·Ρ‹Π²Π°ΡŽΡ‚ Π²Π΅Π»ΠΈΡ‡ΠΈΠ½Ρƒ количСства, Π·Π°ΠΊΠ»ΡŽΡ‡Π΅Π½Π½ΠΎΠ³ΠΎ ΠΌΠ΅ΠΆΠ΄Ρƒ ΠΊΠΎΠ½Ρ†Π°ΠΌΠΈ. НаправлСниС чистой силы Π±ΡƒΠ΄Π΅Ρ‚ Π² ΠΏΠΎΠ»ΠΎΠΆΠΈΡ‚Π΅Π»ΡŒΠ½ΠΎΠΌ Π½Π°ΠΏΡ€Π°Π²Π»Π΅Π½ΠΈΠΈ X , ΠΏΠΎΡ‚ΠΎΠΌΡƒ Ρ‡Ρ‚ΠΎ F1 большС F2 . По мнСнию ΠΡŒΡŽΡ‚ΠΎΠ½Π° Π²Ρ‚ΠΎΡ€ΠΎΠΉ Π·Π°ΠΊΠΎΠ½ двиТСния мяч Π½Π°Ρ‡ΠΈΠ½Π°Π» ΡƒΡΠΊΠΎΡ€ΡΡ‚ΡŒΡΡ Π²ΠΏΡ€Π°Π²ΠΎ.ΠŸΠΎΡ‚ΠΎΠΌΡƒ Ρ‡Ρ‚ΠΎ Π² ΠŸΡ€ΠΈΠΌΠ΅Ρ€ 2, силы Π½Π΅ находятся Π² равновСсии.

Навигация ..


Руководство для Π½Π°Ρ‡ΠΈΠ½Π°ΡŽΡ‰ΠΈΡ… Π”ΠΎΠΌΠ°ΡˆΠ½ΡΡ страница

РавновСсиС ΠΈ статика

Когда всС силы, Π΄Π΅ΠΉΡΡ‚Π²ΡƒΡŽΡ‰ΠΈΠ΅ Π½Π° ΠΎΠ±ΡŠΠ΅ΠΊΡ‚, ΡƒΡ€Π°Π²Π½ΠΎΠ²Π΅ΡˆΠ΅Π½Ρ‹, Ρ‚ΠΎ говорят, Ρ‡Ρ‚ΠΎ ΠΎΠ±ΡŠΠ΅ΠΊΡ‚ находится Π² состоянии равновСсия . Π‘ΠΈΠ»Ρ‹ ΡΡ‡ΠΈΡ‚Π°ΡŽΡ‚ΡΡ ΡƒΡ€Π°Π²Π½ΠΎΠ²Π΅ΡˆΠ΅Π½Π½Ρ‹ΠΌΠΈ , Ссли ΠΏΡ€Π°Π²Ρ‹Π΅ силы ΡƒΡ€Π°Π²Π½ΠΎΠ²Π΅ΡˆΠΈΠ²Π°ΡŽΡ‚ΡΡ Π»Π΅Π²Ρ‹ΠΌΠΈ, Π° восходящиС силы ΡƒΡ€Π°Π²Π½ΠΎΠ²Π΅ΡˆΠΈΠ²Π°ΡŽΡ‚ΡΡ нисходящими.Однако это Π½Π΅ ΠΎΠ±ΡΠ·Π°Ρ‚Π΅Π»ΡŒΠ½ΠΎ ΠΎΠ·Π½Π°Ρ‡Π°Π΅Ρ‚, Ρ‡Ρ‚ΠΎ всС силы Ρ€Π°Π²Π½Ρ‹ Π΄Ρ€ΡƒΠ³ Π΄Ρ€ΡƒΠ³Ρƒ. Рассмотрим Π΄Π²Π° ΠΎΠ±ΡŠΠ΅ΠΊΡ‚Π°, ΠΈΠ·ΠΎΠ±Ρ€Π°ΠΆΠ΅Π½Π½Ρ‹Ρ… Π½Π° силовой Π΄ΠΈΠ°Π³Ρ€Π°ΠΌΠΌΠ΅, ΠΏΠΎΠΊΠ°Π·Π°Π½Π½ΠΎΠΉ Π½ΠΈΠΆΠ΅. ΠžΠ±Ρ€Π°Ρ‚ΠΈΡ‚Π΅ Π²Π½ΠΈΠΌΠ°Π½ΠΈΠ΅, Ρ‡Ρ‚ΠΎ Π΄Π²Π° ΠΎΠ±ΡŠΠ΅ΠΊΡ‚Π° находятся Π² равновСсии, ΠΏΠΎΡ‚ΠΎΠΌΡƒ Ρ‡Ρ‚ΠΎ силы, Π΄Π΅ΠΉΡΡ‚Π²ΡƒΡŽΡ‰ΠΈΠ΅ Π½Π° Π½ΠΈΡ…, ΡƒΡ€Π°Π²Π½ΠΎΠ²Π΅ΡˆΠ΅Π½Ρ‹; ΠΎΠ΄Π½Π°ΠΊΠΎ ΠΎΡ‚Π΄Π΅Π»ΡŒΠ½Ρ‹Π΅ силы Π½Π΅ Ρ€Π°Π²Π½Ρ‹ Π΄Ρ€ΡƒΠ³ Π΄Ρ€ΡƒΠ³Ρƒ. Π‘ΠΈΠ»Π° 50 Н Π½Π΅ Ρ€Π°Π²Π½Π° силС 30 Н.


Если ΠΎΠ±ΡŠΠ΅ΠΊΡ‚ находится Π² равновСсии, силы ΡƒΡ€Π°Π²Π½ΠΎΠ²Π΅ΡˆΠ΅Π½Ρ‹. Ббалансированный — ΠΊΠ»ΡŽΡ‡Π΅Π²ΠΎΠ΅ слово, ΠΈΡΠΏΠΎΠ»ΡŒΠ·ΡƒΠ΅ΠΌΠΎΠ΅ для описания ситуаций равновСсия.Π’Π°ΠΊΠΈΠΌ ΠΎΠ±Ρ€Π°Π·ΠΎΠΌ, Ρ€Π΅Π·ΡƒΠ»ΡŒΡ‚ΠΈΡ€ΡƒΡŽΡ‰Π°Ρ сила Ρ€Π°Π²Π½Π° Π½ΡƒΠ»ΡŽ, Π° ускорСниС Ρ€Π°Π²Π½ΠΎ 0 ΠΌ / с / с. ΠžΠ±ΡŠΠ΅ΠΊΡ‚Ρ‹ Π² состоянии равновСсия Π΄ΠΎΠ»ΠΆΠ½Ρ‹ ΠΈΠΌΠ΅Ρ‚ΡŒ ускорСниС 0 ΠΌ / с / с. Π­Ρ‚ΠΎ происходит ΠΈΠ· ΠΏΠ΅Ρ€Π²ΠΎΠ³ΠΎ Π·Π°ΠΊΠΎΠ½Π° двиТСния ΠΡŒΡŽΡ‚ΠΎΠ½Π°. Но Π½Π°Π»ΠΈΡ‡ΠΈΠ΅ ускорСния 0 ΠΌ / с / с Π½Π΅ ΠΎΠ·Π½Π°Ρ‡Π°Π΅Ρ‚, Ρ‡Ρ‚ΠΎ ΠΎΠ±ΡŠΠ΅ΠΊΡ‚ находится Π² состоянии покоя. ΠžΠ±ΡŠΠ΅ΠΊΡ‚ Π² состоянии равновСсия Π»ΠΈΠ±ΠΎ …

  • Π² состоянии покоя ΠΈ Π² состоянии покоя, ΠΈΠ»ΠΈ
  • Π² Π΄Π²ΠΈΠΆΠ΅Π½ΠΈΠΈ ΠΈ ΠΏΡ€ΠΎΠ΄ΠΎΠ»ΠΆΠ°Π΅Ρ‚ Π΄Π²ΠΈΠΆΠ΅Π½ΠΈΠ΅ с Ρ‚ΠΎΠΉ ΠΆΠ΅ ΡΠΊΠΎΡ€ΠΎΡΡ‚ΡŒΡŽ ΠΈ Π½Π°ΠΏΡ€Π°Π²Π»Π΅Π½ΠΈΠ΅ΠΌ.

Π­Ρ‚ΠΎ Ρ‚ΠΎΠΆΠ΅ происходит ΠΎΡ‚ ΠΏΠ΅Ρ€Π²ΠΎΠ³ΠΎ Π·Π°ΠΊΠΎΠ½Π° двиТСния ΠΡŒΡŽΡ‚ΠΎΠ½Π°.


Анализ ситуации статичСского равновСсия

Если ΠΎΠ±ΡŠΠ΅ΠΊΡ‚ находится Π² состоянии покоя ΠΈ Π² состоянии равновСсия, Ρ‚ΠΎ ΠΌΡ‹ Π±Ρ‹ сказали, Ρ‡Ρ‚ΠΎ ΠΎΠ±ΡŠΠ΅ΠΊΡ‚ находится Π² «статичСском равновСсии». «БтатичСский» ΠΎΠ·Π½Π°Ρ‡Π°Π΅Ρ‚ Π² Π½Π΅ΠΏΠΎΠ΄Π²ΠΈΠΆΠ½ΠΎΠΌ состоянии ΠΈΠ»ΠΈ Π² состоянии покоя . ΠžΠ±Ρ‹Ρ‡Π½Π°Ρ физичСская лаборатория Π·Π°ΠΊΠ»ΡŽΡ‡Π°Π΅Ρ‚ΡΡ Π² Ρ‚ΠΎΠΌ, Ρ‡Ρ‚ΠΎΠ±Ρ‹ ΠΏΠΎΠ΄Π²Π΅ΡΠΈΡ‚ΡŒ ΠΎΠ±ΡŠΠ΅ΠΊΡ‚ Π½Π° Π΄Π²ΡƒΡ… ΠΈΠ»ΠΈ Π±ΠΎΠ»Π΅Π΅ нитях ΠΈ ΠΈΠ·ΠΌΠ΅Ρ€ΠΈΡ‚ΡŒ силы, Π΄Π΅ΠΉΡΡ‚Π²ΡƒΡŽΡ‰ΠΈΠ΅ ΠΏΠΎΠ΄ ΡƒΠ³Π»ΠΎΠΌ Π½Π° ΠΎΠ±ΡŠΠ΅ΠΊΡ‚, Ρ‡Ρ‚ΠΎΠ±Ρ‹ Π²Ρ‹Π΄Π΅Ρ€ΠΆΠ°Ρ‚ΡŒ Π΅Π³ΠΎ вСс. БостояниС ΠΎΠ±ΡŠΠ΅ΠΊΡ‚Π° анализируСтся с Ρ‚ΠΎΡ‡ΠΊΠΈ зрСния сил, Π΄Π΅ΠΉΡΡ‚Π²ΡƒΡŽΡ‰ΠΈΡ… Π½Π° ΠΎΠ±ΡŠΠ΅ΠΊΡ‚. ΠžΠ±ΡŠΠ΅ΠΊΡ‚ прСдставляСт собой Ρ‚ΠΎΡ‡ΠΊΡƒ Π½Π° струнС, Π½Π° ΠΊΠΎΡ‚ΠΎΡ€ΡƒΡŽ Π΄Π΅ΠΉΡΡ‚Π²ΡƒΡŽΡ‚ Ρ‚Ρ€ΠΈ силы. Π‘ΠΌ. Π”ΠΈΠ°Π³Ρ€Π°ΠΌΠΌΡƒ справа. Если ΠΎΠ±ΡŠΠ΅ΠΊΡ‚ находится Π² состоянии равновСсия, Ρ‚ΠΎ Ρ€Π΅Π·ΡƒΠ»ΡŒΡ‚ΠΈΡ€ΡƒΡŽΡ‰Π°Ρ сила, Π΄Π΅ΠΉΡΡ‚Π²ΡƒΡŽΡ‰Π°Ρ Π½Π° ΠΎΠ±ΡŠΠ΅ΠΊΡ‚, Π΄ΠΎΠ»ΠΆΠ½Π° Π±Ρ‹Ρ‚ΡŒ 0 ΠΡŒΡŽΡ‚ΠΎΠ½ΠΎΠ². Π’Π°ΠΊΠΈΠΌ ΠΎΠ±Ρ€Π°Π·ΠΎΠΌ, Ссли всС силы ΡΠΊΠ»Π°Π΄Ρ‹Π²Π°ΡŽΡ‚ΡΡ вмСстС ΠΊΠ°ΠΊ Π²Π΅ΠΊΡ‚ΠΎΡ€Ρ‹, Ρ‚ΠΎ Ρ€Π΅Π·ΡƒΠ»ΡŒΡ‚ΠΈΡ€ΡƒΡŽΡ‰Π°Ρ сила (вСкторная сумма) Π΄ΠΎΠ»ΠΆΠ½Π° Π±Ρ‹Ρ‚ΡŒ 0 ΠΡŒΡŽΡ‚ΠΎΠ½ΠΎΠ². (Напомним, Ρ‡Ρ‚ΠΎ Ρ€Π΅Π·ΡƒΠ»ΡŒΡ‚ΠΈΡ€ΡƒΡŽΡ‰Π°Ρ сила — это «вСкторная сумма всСх сил» ΠΈΠ»ΠΈ Ρ€Π΅Π·ΡƒΠ»ΡŒΡ‚Π°Ρ‚ слоТСния всСх ΠΎΡ‚Π΄Π΅Π»ΡŒΠ½Ρ‹Ρ… сил ΠΏΠΎ Π½Π°ΠΏΡ€Π°Π²Π»Π΅Π½ΠΈΡŽ Β«Π³ΠΎΠ»ΠΎΠ²Π° ΠΊ хвосту».) Π’Π°ΠΊΠΈΠΌ ΠΎΠ±Ρ€Π°Π·ΠΎΠΌ, ΠΌΠΎΠΆΠ½ΠΎ ΠΏΠΎΡΡ‚Ρ€ΠΎΠΈΡ‚ΡŒ Ρ‚ΠΎΡ‡Π½ΠΎ Π½Π°Ρ€ΠΈΡΠΎΠ²Π°Π½Π½ΡƒΡŽ Π΄ΠΈΠ°Π³Ρ€Π°ΠΌΠΌΡƒ слоТСния Π²Π΅ΠΊΡ‚ΠΎΡ€ΠΎΠ² для опрСдСлСния Ρ€Π°Π²Π½ΠΎΠ΄Π΅ΠΉΡΡ‚Π²ΡƒΡŽΡ‰Π΅ΠΉ.НиТС ΠΏΡ€ΠΈΠ²Π΅Π΄Π΅Π½Ρ‹ ΠΏΡ€ΠΈΠΌΠ΅Ρ€Ρ‹ Π΄Π°Π½Π½Ρ‹Ρ… для Ρ‚Π°ΠΊΠΎΠΉ Π»Π°Π±ΠΎΡ€Π°Ρ‚ΠΎΡ€ΠΈΠΈ.

Била А Force B Била C

Π’Π΅Π»ΠΈΡ‡ΠΈΠ½Π°

3.4 с.ш. 9.2 с.ш. 9.8 с.ш.

НаправлСниС

161 Π³Ρ€Π°Π΄. 70 Π³Ρ€Π°Π΄. 270 Π³Ρ€Π°Π΄.


Для Π±ΠΎΠ»ΡŒΡˆΠΈΠ½ΡΡ‚Π²Π° студСнтов Ρ€Π΅Π·ΡƒΠ»ΡŒΡ‚Π°Ρ‚ Π±Ρ‹Π» 0 ΠΡŒΡŽΡ‚ΠΎΠ½ΠΎΠ² (ΠΈΠ»ΠΈ, ΠΏΠΎ ΠΊΡ€Π°ΠΉΠ½Π΅ΠΉ ΠΌΠ΅Ρ€Π΅, ΠΎΡ‡Π΅Π½ΡŒ Π±Π»ΠΈΠ·ΠΎΠΊ ΠΊ 0 Н).Π­Ρ‚ΠΎ Ρ‚ΠΎ, Ρ‡Ρ‚ΠΎ ΠΌΡ‹ ΠΎΠΆΠΈΠ΄Π°Π»ΠΈ — ΠΏΠΎΡΠΊΠΎΠ»ΡŒΠΊΡƒ ΠΎΠ±ΡŠΠ΅ΠΊΡ‚ находился Π² состоянии равновСсия, Ρ€Π΅Π·ΡƒΠ»ΡŒΡ‚ΠΈΡ€ΡƒΡŽΡ‰Π°Ρ сила (вСкторная сумма всСх сил) Π΄ΠΎΠ»ΠΆΠ½Π° Π±Ρ‹Ρ‚ΡŒ 0 Н.


Π”Ρ€ΡƒΠ³ΠΎΠΉ способ опрСдСлСния чистой силы (Π²Π΅ΠΊΡ‚ΠΎΡ€Π½ΠΎΠΉ суммы всСх сил) Π²ΠΊΠ»ΡŽΡ‡Π°Π΅Ρ‚ использованиС тригономСтричСских Ρ„ΡƒΠ½ΠΊΡ†ΠΈΠΉ для раздСлСния ΠΊΠ°ΠΆΠ΄ΠΎΠΉ силы Π½Π° Π΅Π΅ Π³ΠΎΡ€ΠΈΠ·ΠΎΠ½Ρ‚Π°Π»ΡŒΠ½ΡƒΡŽ ΠΈ Π²Π΅Ρ€Ρ‚ΠΈΠΊΠ°Π»ΡŒΠ½ΡƒΡŽ ΡΠΎΡΡ‚Π°Π²Π»ΡΡŽΡ‰ΠΈΠ΅. Как Ρ‚ΠΎΠ»ΡŒΠΊΠΎ ΠΊΠΎΠΌΠΏΠΎΠ½Π΅Π½Ρ‚Ρ‹ извСстны, ΠΈΡ… ΠΌΠΎΠΆΠ½ΠΎ ΡΡ€Π°Π²Π½ΠΈΡ‚ΡŒ, Ρ‡Ρ‚ΠΎΠ±Ρ‹ ΡƒΠ²ΠΈΠ΄Π΅Ρ‚ΡŒ, сбалансированы Π»ΠΈ Π²Π΅Ρ€Ρ‚ΠΈΠΊΠ°Π»ΡŒΠ½Ρ‹Π΅ силы ΠΈ Π³ΠΎΡ€ΠΈΠ·ΠΎΠ½Ρ‚Π°Π»ΡŒΠ½Ρ‹Π΅ силы.На схСмС Π½ΠΈΠΆΠ΅ ΠΏΠΎΠΊΠ°Π·Π°Π½Ρ‹ Π²Π΅ΠΊΡ‚ΠΎΡ€Ρ‹ A, B ΠΈ C ΠΈ ΠΈΡ… ΡΠΎΠΎΡ‚Π²Π΅Ρ‚ΡΡ‚Π²ΡƒΡŽΡ‰ΠΈΠ΅ ΠΊΠΎΠΌΠΏΠΎΠ½Π΅Π½Ρ‚Ρ‹. Для Π²Π΅ΠΊΡ‚ΠΎΡ€ΠΎΠ² A ΠΈ B Π²Π΅Ρ€Ρ‚ΠΈΠΊΠ°Π»ΡŒΠ½Ρ‹Π΅ ΠΊΠΎΠΌΠΏΠΎΠ½Π΅Π½Ρ‚Ρ‹ ΠΌΠΎΠ³ΡƒΡ‚ Π±Ρ‹Ρ‚ΡŒ ΠΎΠΏΡ€Π΅Π΄Π΅Π»Π΅Π½Ρ‹ с использованиСм синуса ΡƒΠ³Π»Π°, Π° Π³ΠΎΡ€ΠΈΠ·ΠΎΠ½Ρ‚Π°Π»ΡŒΠ½Ρ‹Π΅ ΠΊΠΎΠΌΠΏΠΎΠ½Π΅Π½Ρ‚Ρ‹ ΠΌΠΎΠ³ΡƒΡ‚ Π±Ρ‹Ρ‚ΡŒ ΠΏΡ€ΠΎΠ°Π½Π°Π»ΠΈΠ·ΠΈΡ€ΠΎΠ²Π°Π½Ρ‹ с ΠΏΠΎΠΌΠΎΡ‰ΡŒΡŽ косинуса ΡƒΠ³Π»Π°. Π’Π΅Π»ΠΈΡ‡ΠΈΠ½Π° ΠΈ Π½Π°ΠΏΡ€Π°Π²Π»Π΅Π½ΠΈΠ΅ ΠΊΠ°ΠΆΠ΄ΠΎΠ³ΠΎ ΠΊΠΎΠΌΠΏΠΎΠ½Π΅Π½Ρ‚Π° для Π²Ρ‹Π±ΠΎΡ€ΠΎΡ‡Π½Ρ‹Ρ… Π΄Π°Π½Π½Ρ‹Ρ… ΠΏΠΎΠΊΠ°Π·Π°Π½Ρ‹ Π² Ρ‚Π°Π±Π»ΠΈΡ†Π΅ ΠΏΠΎΠ΄ Π΄ΠΈΠ°Π³Ρ€Π°ΠΌΠΌΠΎΠΉ.


Π”Π°Π½Π½Ρ‹Π΅ Π² Ρ‚Π°Π±Π»ΠΈΡ†Π΅ Π²Ρ‹ΡˆΠ΅ ΠΏΠΎΠΊΠ°Π·Ρ‹Π²Π°ΡŽΡ‚, Ρ‡Ρ‚ΠΎ силы ΠΏΠΎΡ‡Ρ‚ΠΈ ΡƒΡ€Π°Π²Π½ΠΎΠ²Π΅ΡˆΠΈΠ²Π°ΡŽΡ‚ .Анализ Π³ΠΎΡ€ΠΈΠ·ΠΎΠ½Ρ‚Π°Π»ΡŒΠ½Ρ‹Ρ… ΠΊΠΎΠΌΠΏΠΎΠ½Π΅Π½Ρ‚ΠΎΠ² ΠΏΠΎΠΊΠ°Π·Ρ‹Π²Π°Π΅Ρ‚, Ρ‡Ρ‚ΠΎ Π»Π΅Π²Ρ‹ΠΉ ΠΊΠΎΠΌΠΏΠΎΠ½Π΅Π½Ρ‚ A ΠΏΠΎΡ‡Ρ‚ΠΈ ΡƒΡ€Π°Π²Π½ΠΎΠ²Π΅ΡˆΠΈΠ²Π°Π΅Ρ‚ ΠΏΡ€Π°Π²Ρ‹ΠΉ ΠΊΠΎΠΌΠΏΠΎΠ½Π΅Π½Ρ‚ B. Анализ Π²Π΅Ρ€Ρ‚ΠΈΠΊΠ°Π»ΡŒΠ½Ρ‹Ρ… ΠΊΠΎΠΌΠΏΠΎΠ½Π΅Π½Ρ‚ΠΎΠ² ΠΏΠΎΠΊΠ°Π·Ρ‹Π²Π°Π΅Ρ‚, Ρ‡Ρ‚ΠΎ сумма восходящих ΠΊΠΎΠΌΠΏΠΎΠ½Π΅Π½Ρ‚ΠΎΠ² A + B ΠΏΠΎΡ‡Ρ‚ΠΈ ΡƒΡ€Π°Π²Π½ΠΎΠ²Π΅ΡˆΠΈΠ²Π°Π΅Ρ‚ нисходящий ΠΊΠΎΠΌΠΏΠΎΠ½Π΅Π½Ρ‚ C. ВСкторная сумма всСх сил ( ΠΏΠΎΡ‡Ρ‚ΠΈ ) Ρ€Π°Π²Π½Π° 0 ΠΡŒΡŽΡ‚ΠΎΠ½Ρƒ. Но ΠΊΠ°ΠΊ насчСт Ρ€Π°Π·Π½ΠΈΡ†Ρ‹ Π² 0,1 Н ΠΌΠ΅ΠΆΠ΄Ρƒ Π½Π°ΠΏΡ€Π°Π²Π»Π΅Π½Π½Ρ‹ΠΌΠΈ Π²ΠΏΡ€Π°Π²ΠΎ ΠΈ Π²Π»Π΅Π²ΠΎ силами ΠΈ Ρ€Π°Π·Π½ΠΈΡ†Ρ‹ Π² 0,2 Н ΠΌΠ΅ΠΆΠ΄Ρƒ силами, Π½Π°ΠΏΡ€Π°Π²Π»Π΅Π½Π½Ρ‹ΠΌΠΈ Π²Π²Π΅Ρ€Ρ… ΠΈ Π²Π½ΠΈΠ·? ΠŸΠΎΡ‡Π΅ΠΌΡƒ ΠΊΠΎΠΌΠΏΠΎΠ½Π΅Π½Ρ‚Ρ‹ силы Ρ‚ΠΎΠ»ΡŒΠΊΠΎ ΠΏΠΎΡ‡Ρ‚ΠΈ ΡƒΡ€Π°Π²Π½ΠΎΠ²Π΅ΡˆΠΈΠ²Π°ΡŽΡ‚? Π”Π°Π½Π½Ρ‹Π΅ ΠΎΠ±Ρ€Π°Π·Ρ†Π°, ΠΈΡΠΏΠΎΠ»ΡŒΠ·ΡƒΠ΅ΠΌΡ‹Π΅ Π² этом Π°Π½Π°Π»ΠΈΠ·Π΅, ΡΠ²Π»ΡΡŽΡ‚ΡΡ Ρ€Π΅Π·ΡƒΠ»ΡŒΡ‚Π°Ρ‚ΠΎΠΌ Π΄Π°Π½Π½Ρ‹Ρ… ΠΈΠ·ΠΌΠ΅Ρ€Π΅Π½ΠΈΠΉ Π½Π° Ρ€Π΅Π°Π»ΡŒΠ½ΠΎΠΉ ΡΠΊΡΠΏΠ΅Ρ€ΠΈΠΌΠ΅Π½Ρ‚Π°Π»ΡŒΠ½ΠΎΠΉ установкС.Π Π°Π·Π½ΠΈΡ†Π° ΠΌΠ΅ΠΆΠ΄Ρƒ фактичСскими Ρ€Π΅Π·ΡƒΠ»ΡŒΡ‚Π°Ρ‚Π°ΠΌΠΈ ΠΈ ΠΎΠΆΠΈΠ΄Π°Π΅ΠΌΡ‹ΠΌΠΈ Ρ€Π΅Π·ΡƒΠ»ΡŒΡ‚Π°Ρ‚Π°ΠΌΠΈ связана с ошибкой, возникшСй ΠΏΡ€ΠΈ ΠΈΠ·ΠΌΠ΅Ρ€Π΅Π½ΠΈΠΈ силы A ΠΈ силы B. ΠœΡ‹ Π΄ΠΎΠ»ΠΆΠ½Ρ‹ ΡΠ΄Π΅Π»Π°Ρ‚ΡŒ Π²Ρ‹Π²ΠΎΠ΄, Ρ‡Ρ‚ΠΎ этот Π½ΠΈΠ·ΠΊΠΈΠΉ ΠΏΡ€Π΅Π΄Π΅Π» ΡΠΊΡΠΏΠ΅Ρ€ΠΈΠΌΠ΅Π½Ρ‚Π°Π»ΡŒΠ½ΠΎΠΉ ошибки ΠΎΡ‚Ρ€Π°ΠΆΠ°Π΅Ρ‚ экспСримСнт с прСвосходными Ρ€Π΅Π·ΡƒΠ»ΡŒΡ‚Π°Ρ‚Π°ΠΌΠΈ. МоТно ΡΠΊΠ°Π·Π°Ρ‚ΡŒ, Ρ‡Ρ‚ΠΎ это «достаточно Π±Π»ΠΈΠ·ΠΊΠΎ для Ρ€Π°Π±ΠΎΡ‚Ρ‹ ΠΏΡ€Π°Π²ΠΈΡ‚Π΅Π»ΡŒΡΡ‚Π²Π°Β».


Анализ висящСго Π·Π½Π°ΠΊΠ°

ΠŸΡ€ΠΈΠ²Π΅Π΄Π΅Π½Π½Ρ‹ΠΉ Π²Ρ‹ΡˆΠ΅ Π°Π½Π°Π»ΠΈΠ· сил, Π΄Π΅ΠΉΡΡ‚Π²ΡƒΡŽΡ‰ΠΈΡ… Π½Π° ΠΎΠ±ΡŠΠ΅ΠΊΡ‚ Π² состоянии равновСсия, ΠΎΠ±Ρ‹Ρ‡Π½ΠΎ ΠΈΡΠΏΠΎΠ»ΡŒΠ·ΡƒΠ΅Ρ‚ΡΡ для Π°Π½Π°Π»ΠΈΠ·Π° ситуаций с ΠΎΠ±ΡŠΠ΅ΠΊΡ‚Π°ΠΌΠΈ Π² состоянии статичСского равновСсия.НаиболСС распространСнноС ΠΏΡ€ΠΈΠΌΠ΅Π½Π΅Π½ΠΈΠ΅ Π²ΠΊΠ»ΡŽΡ‡Π°Π΅Ρ‚ Π°Π½Π°Π»ΠΈΠ· сил, Π΄Π΅ΠΉΡΡ‚Π²ΡƒΡŽΡ‰ΠΈΡ… Π½Π° Π·Π½Π°ΠΊ, ΠΊΠΎΡ‚ΠΎΡ€Ρ‹ΠΉ находится Π² состоянии покоя. НапримСр, рассмотритС ΠΊΠ°Ρ€Ρ‚ΠΈΠ½Ρƒ справа, Π²ΠΈΡΡΡ‰ΡƒΡŽ Π½Π° стСнС. ΠšΠ°Ρ€Ρ‚ΠΈΠ½Π° находится Π² состоянии равновСсия, ΠΈ поэтому всС силы, Π΄Π΅ΠΉΡΡ‚Π²ΡƒΡŽΡ‰ΠΈΠ΅ Π½Π° ΠΊΠ°Ρ€Ρ‚ΠΈΠ½Ρƒ, Π΄ΠΎΠ»ΠΆΠ½Ρ‹ Π±Ρ‹Ρ‚ΡŒ ΡƒΡ€Π°Π²Π½ΠΎΠ²Π΅ΡˆΠ΅Π½Ρ‹. Π’ΠΎ Π΅ΡΡ‚ΡŒ всС Π³ΠΎΡ€ΠΈΠ·ΠΎΠ½Ρ‚Π°Π»ΡŒΠ½Ρ‹Π΅ ΠΊΠΎΠΌΠΏΠΎΠ½Π΅Π½Ρ‚Ρ‹ Π΄ΠΎΠ»ΠΆΠ½Ρ‹ ΡΠΎΡΡ‚Π°Π²Π»ΡΡ‚ΡŒ 0 ΠΡŒΡŽΡ‚ΠΎΠ½ΠΎΠ², Π° всС Π²Π΅Ρ€Ρ‚ΠΈΠΊΠ°Π»ΡŒΠ½Ρ‹Π΅ ΠΊΠΎΠΌΠΏΠΎΠ½Π΅Π½Ρ‚Ρ‹ Π΄ΠΎΠ»ΠΆΠ½Ρ‹ ΡΠΎΡΡ‚Π°Π²Π»ΡΡ‚ΡŒ 0 ΠΡŒΡŽΡ‚ΠΎΠ½ΠΎΠ². НатяТСниС троса А Π²Π»Π΅Π²ΠΎ Π΄ΠΎΠ»ΠΆΠ½ΠΎ ΡƒΡ€Π°Π²Π½ΠΎΠ²Π΅ΡˆΠΈΠ²Π°Ρ‚ΡŒ натяТСниС троса Π’ Π²ΠΏΡ€Π°Π²ΠΎ, Π° сумма натяТСния троса А ΠΈ троса Π’ Π²Π²Π΅Ρ€Ρ… Π΄ΠΎΠ»ΠΆΠ½Π° ΡƒΡ€Π°Π²Π½ΠΎΠ²Π΅ΡˆΠΈΠ²Π°Ρ‚ΡŒ вСс Π·Π½Π°ΠΊΠ°.

ΠŸΡ€Π΅Π΄ΠΏΠΎΠ»ΠΎΠΆΠΈΠΌ, Ρ‡Ρ‚ΠΎ ΠΈΠ·ΠΌΠ΅Ρ€Π΅Π½Π½ΠΎΠ΅ натяТСниС ΠΎΠ±ΠΎΠΈΡ… ΠΊΠ°Π±Π΅Π»Π΅ΠΉ составляСт 50 Н, Π° ΡƒΠ³ΠΎΠ», ΠΊΠΎΡ‚ΠΎΡ€Ρ‹ΠΉ ΠΊΠ°ΠΆΠ΄Ρ‹ΠΉ кабСль ΠΎΠ±Ρ€Π°Π·ΡƒΠ΅Ρ‚ с Π³ΠΎΡ€ΠΈΠ·ΠΎΠ½Ρ‚Π°Π»ΡŒΡŽ, составляСт 30 градусов. Какой вСс Ρƒ Π·Π½Π°ΠΊΠ°? На этот вопрос ΠΌΠΎΠΆΠ½ΠΎ ΠΎΡ‚Π²Π΅Ρ‚ΠΈΡ‚ΡŒ, провСдя силовой Π°Π½Π°Π»ΠΈΠ· с использованиСм тригономСтричСских Ρ„ΡƒΠ½ΠΊΡ†ΠΈΠΉ. ВСс Π·Π½Π°ΠΊΠ° Ρ€Π°Π²Π΅Π½ суммС восходящих ΠΊΠΎΠΌΠΏΠΎΠ½Π΅Π½Ρ‚ΠΎΠ² натяТСния Π΄Π²ΡƒΡ… тросов. Π’Π°ΠΊΠΈΠΌ ΠΎΠ±Ρ€Π°Π·ΠΎΠΌ, для опрСдСлСния этой Π²Π΅Ρ€Ρ‚ΠΈΠΊΠ°Π»ΡŒΠ½ΠΎΠΉ ΡΠΎΡΡ‚Π°Π²Π»ΡΡŽΡ‰Π΅ΠΉ ΠΌΠΎΠΆΠ½ΠΎ ΠΈΡΠΏΠΎΠ»ΡŒΠ·ΠΎΠ²Π°Ρ‚ΡŒ Ρ‚Ρ€ΠΈΠ³ΠΎΠ½ΠΎΠΌΠ΅Ρ‚Ρ€ΠΈΡ‡Π΅ΡΠΊΡƒΡŽ Ρ„ΡƒΠ½ΠΊΡ†ΠΈΡŽ. Π‘Ρ…Π΅ΠΌΠ° ΠΈ ΡΠΎΠΏΡ€ΠΎΠ²ΠΎΠ΄ΠΈΡ‚Π΅Π»ΡŒΠ½Ρ‹Π΅ Ρ€Π°Π±ΠΎΡ‚Ρ‹ ΠΏΠΎΠΊΠ°Π·Π°Π½Ρ‹ Π½ΠΈΠΆΠ΅.


ΠŸΠΎΡΠΊΠΎΠ»ΡŒΠΊΡƒ ΠΊΠ°ΠΆΠ΄Ρ‹ΠΉ трос тянСт Π²Π²Π΅Ρ€Ρ… с силой 25 Н, общая сила тяги Π·Π½Π°ΠΊΠ° Π²Π²Π΅Ρ€Ρ… составляСт 50 Н. Π‘Π»Π΅Π΄ΠΎΠ²Π°Ρ‚Π΅Π»ΡŒΠ½ΠΎ, сила тяТСсти (Ρ‚Π°ΠΊΠΆΠ΅ извСстная ΠΊΠ°ΠΊ вСс) составляСт 50 Н Π²Π½ΠΈΠ·. Π—Π½Π°ΠΊ вСсит 50 Н.

Π’ ΠΏΡ€ΠΈΠ²Π΅Π΄Π΅Π½Π½ΠΎΠΉ Π²Ρ‹ΡˆΠ΅ Π·Π°Π΄Π°Ρ‡Π΅ натяТСниС Π² тросС ΠΈ ΡƒΠ³ΠΎΠ» , ΠΊΠΎΡ‚ΠΎΡ€Ρ‹ΠΉ трос ΠΎΠ±Ρ€Π°Π·ΡƒΠ΅Ρ‚ с Π³ΠΎΡ€ΠΈΠ·ΠΎΠ½Ρ‚Π°Π»ΡŒΡŽ, ΠΈΡΠΏΠΎΠ»ΡŒΠ·ΡƒΡŽΡ‚ΡΡ для опрСдСлСния вСса Π·Π½Π°ΠΊΠ°.ИдСя Π² Ρ‚ΠΎΠΌ, Ρ‡Ρ‚ΠΎ натяТСниС, ΡƒΠ³ΠΎΠ» ΠΈ вСс связаны. Если извСстны Π»ΡŽΠ±Ρ‹Π΅ Π΄Π²Π° ΠΈΠ· этих Ρ‚Ρ€Π΅Ρ…, Ρ‚ΠΎ Ρ‚Ρ€Π΅Ρ‚ΡŒΡ Π²Π΅Π»ΠΈΡ‡ΠΈΠ½Π° ΠΌΠΎΠΆΠ΅Ρ‚ Π±Ρ‹Ρ‚ΡŒ ΠΎΠΏΡ€Π΅Π΄Π΅Π»Π΅Π½Π° с ΠΏΠΎΠΌΠΎΡ‰ΡŒΡŽ тригономСтричСских Ρ„ΡƒΠ½ΠΊΡ†ΠΈΠΉ.

Π’ качСствС Π΅Ρ‰Π΅ ΠΎΠ΄Π½ΠΎΠ³ΠΎ ΠΏΡ€ΠΈΠΌΠ΅Ρ€Π°, ΠΈΠ»Π»ΡŽΡΡ‚Ρ€ΠΈΡ€ΡƒΡŽΡ‰Π΅Π³ΠΎ эту идСю, рассмотрим симмСтричноС Ρ€Π°Π·Π²Π΅ΡˆΠΈΠ²Π°Π½ΠΈΠ΅ Π·Π½Π°ΠΊΠ°, ΠΊΠ°ΠΊ ΠΏΠΎΠΊΠ°Π·Π°Π½ΠΎ справа. Если извСстно, Ρ‡Ρ‚ΠΎ Π·Π½Π°ΠΊ ΠΈΠΌΠ΅Π΅Ρ‚ массу 5 ΠΊΠ³ ΠΈ Ссли ΡƒΠ³ΠΎΠ» ΠΌΠ΅ΠΆΠ΄Ρƒ двумя тросами составляСт 100 градусов, Ρ‚ΠΎ ΠΌΠΎΠΆΠ½ΠΎ ΠΎΠΏΡ€Π΅Π΄Π΅Π»ΠΈΡ‚ΡŒ натяТСниС троса. ΠŸΡ€Π΅Π΄ΠΏΠΎΠ»Π°Π³Π°Ρ, Ρ‡Ρ‚ΠΎ Π·Π½Π°ΠΊ находится Π² состоянии равновСсия (Ρ…ΠΎΡ€ΠΎΡˆΠ΅Π΅ ΠΏΡ€Π΅Π΄ΠΏΠΎΠ»ΠΎΠΆΠ΅Π½ΠΈΠ΅, Ссли ΠΎΠ½ остаСтся Π² состоянии покоя), Π΄Π²Π° троса Π΄ΠΎΠ»ΠΆΠ½Ρ‹ ΠΎΠ±Π΅ΡΠΏΠ΅Ρ‡ΠΈΠ²Π°Ρ‚ΡŒ Π΄ΠΎΡΡ‚Π°Ρ‚ΠΎΡ‡Π½ΡƒΡŽ Π²ΠΎΡΡ…ΠΎΠ΄ΡΡ‰ΡƒΡŽ силу, Ρ‡Ρ‚ΠΎΠ±Ρ‹ ΡƒΡ€Π°Π²Π½ΠΎΠ²Π΅ΡΠΈΡ‚ΡŒ Π½ΠΈΡΡ…ΠΎΠ΄ΡΡ‰ΡƒΡŽ силу тяТСсти.Π‘ΠΈΠ»Π° тяТСсти (Ρ‚Π°ΠΊΠΆΠ΅ извСстная ΠΊΠ°ΠΊ вСс) составляСт 49 Н (Fgrav = m * g), поэтому ΠΊΠ°ΠΆΠ΄Ρ‹ΠΉ ΠΈΠ· Π΄Π²ΡƒΡ… тросов Π΄ΠΎΠ»ΠΆΠ΅Π½ Ρ‚ΡΠ½ΡƒΡ‚ΡŒ Π²Π²Π΅Ρ€Ρ… с силой 24,5 Н. ΠŸΠΎΡΠΊΠΎΠ»ΡŒΠΊΡƒ ΡƒΠ³ΠΎΠ» ΠΌΠ΅ΠΆΠ΄Ρƒ кабСлями составляСт 100 градусов, ΠΊΠ°ΠΆΠ΄Ρ‹ΠΉ кабСль Π΄ΠΎΠ»ΠΆΠ΅Π½ ΡΠΎΡΡ‚Π°Π²Π»ΡΡ‚ΡŒ 50 градусов с Π²Π΅Ρ€Ρ‚ΠΈΠΊΠ°Π»ΡŒΡŽ ΠΈ 40 градусов с Π³ΠΎΡ€ΠΈΠ·ΠΎΠ½Ρ‚Π°Π»ΡŒΡŽ. Набросок этой ситуации (см. Π”ΠΈΠ°Π³Ρ€Π°ΠΌΠΌΡƒ Π½ΠΈΠΆΠ΅) ΠΏΠΎΠΊΠ°Π·Ρ‹Π²Π°Π΅Ρ‚, Ρ‡Ρ‚ΠΎ натяТСниС кабСля ΠΌΠΎΠΆΠ½ΠΎ ΠΎΠΏΡ€Π΅Π΄Π΅Π»ΠΈΡ‚ΡŒ с ΠΏΠΎΠΌΠΎΡ‰ΡŒΡŽ ΡΠΈΠ½ΡƒΡΠΎΠΈΠ΄Π°Π»ΡŒΠ½ΠΎΠΉ Ρ„ΡƒΠ½ΠΊΡ†ΠΈΠΈ. Π’Ρ€Π΅ΡƒΠ³ΠΎΠ»ΡŒΠ½ΠΈΠΊ Π½ΠΈΠΆΠ΅ ΠΈΠ»Π»ΡŽΡΡ‚Ρ€ΠΈΡ€ΡƒΠ΅Ρ‚ эти ΠΎΡ‚Π½ΠΎΡˆΠ΅Π½ΠΈΡ.

ΠœΡ‹ΡΠ»ΠΈΡ‚ΡŒ ΠΊΠΎΠ½Ρ†Π΅ΠΏΡ‚ΡƒΠ°Π»ΡŒΠ½ΠΎ

БущСствуСт Π²Π°ΠΆΠ½Ρ‹ΠΉ ΠΏΡ€ΠΈΠ½Ρ†ΠΈΠΏ, ΠΊΠΎΡ‚ΠΎΡ€Ρ‹ΠΉ Π²Ρ‹Ρ‚Π΅ΠΊΠ°Π΅Ρ‚ ΠΈΠ· Π½Π΅ΠΊΠΎΡ‚ΠΎΡ€Ρ‹Ρ… ΠΈΠ· Π²Ρ‹ΠΏΠΎΠ»Π½Π΅Π½Π½Ρ‹Ρ… Π²Ρ‹ΡˆΠ΅ тригономСтричСских вычислСний .ΠŸΡ€ΠΈΠ½Ρ†ΠΈΠΏ состоит Π² Ρ‚ΠΎΠΌ, Ρ‡Ρ‚ΠΎ ΠΏΠΎ ΠΌΠ΅Ρ€Π΅ увСличСния ΡƒΠ³Π»Π° ΠΊ Π³ΠΎΡ€ΠΈΠ·ΠΎΠ½Ρ‚Π°Π»ΠΈ Π²Π΅Π»ΠΈΡ‡ΠΈΠ½Π° силы натяТСния, нСобходимая для удСрТания Π·Π½Π°ΠΊΠ° Π² состоянии равновСсия, ΡƒΠΌΠ΅Π½ΡŒΡˆΠ°Π΅Ρ‚ΡΡ. Π§Ρ‚ΠΎΠ±Ρ‹ ΠΏΡ€ΠΎΠΈΠ»Π»ΡŽΡΡ‚Ρ€ΠΈΡ€ΠΎΠ²Π°Ρ‚ΡŒ это, рассмотрим ΠΊΠ°Ρ€Ρ‚ΠΈΠ½ΠΊΡƒ с напряТСниСм 10 ΠΡŒΡŽΡ‚ΠΎΠ½, ΡƒΠ΄Π΅Ρ€ΠΆΠΈΠ²Π°Π΅ΠΌΡƒΡŽ трСмя Ρ€Π°Π·Π½Ρ‹ΠΌΠΈ ориСнтациями ΠΏΡ€ΠΎΠ²ΠΎΠ΄ΠΎΠ², ΠΊΠ°ΠΊ ΠΏΠΎΠΊΠ°Π·Π°Π½ΠΎ Π½Π° схСмах Π½ΠΈΠΆΠ΅. Π’ ΠΊΠ°ΠΆΠ΄ΠΎΠΌ случаС для ΠΏΠΎΠ΄Π΄Π΅Ρ€ΠΆΠΊΠΈ изобраТСния ΠΈΡΠΏΠΎΠ»ΡŒΠ·ΡƒΡŽΡ‚ΡΡ Π΄Π²Π° ΠΏΡ€ΠΎΠ²ΠΎΠ΄Π°; ΠΊΠ°ΠΆΠ΄Ρ‹ΠΉ ΠΏΡ€ΠΎΠ²ΠΎΠ΄ Π΄ΠΎΠ»ΠΆΠ΅Π½ Π²Ρ‹Π΄Π΅Ρ€ΠΆΠΈΠ²Π°Ρ‚ΡŒ ΠΏΠΎΠ»ΠΎΠ²ΠΈΠ½Ρƒ вСса Π·Π½Π°ΠΊΠ° (5 Н). Π£Π³ΠΎΠ» ΠΌΠ΅ΠΆΠ΄Ρƒ ΠΏΡ€ΠΎΠ²ΠΎΠ΄Π°ΠΌΠΈ ΠΈ Π³ΠΎΡ€ΠΈΠ·ΠΎΠ½Ρ‚ΠΎΠΌ Π²Π°Ρ€ΡŒΠΈΡ€ΡƒΠ΅Ρ‚ΡΡ ΠΎΡ‚ 60 Π΄ΠΎ 15 градусов. Π˜ΡΠΏΠΎΠ»ΡŒΠ·ΡƒΠΉΡ‚Π΅ эту ΠΈΠ½Ρ„ΠΎΡ€ΠΌΠ°Ρ†ΠΈΡŽ ΠΈ ΠΏΡ€ΠΈΠ²Π΅Π΄Π΅Π½Π½ΡƒΡŽ Π½ΠΈΠΆΠ΅ Π΄ΠΈΠ°Π³Ρ€Π°ΠΌΠΌΡƒ, Ρ‡Ρ‚ΠΎΠ±Ρ‹ ΠΎΠΏΡ€Π΅Π΄Π΅Π»ΠΈΡ‚ΡŒ натяТСниС ΠΏΡ€ΠΎΠ²ΠΎΠ»ΠΎΠΊΠΈ для ΠΊΠ°ΠΆΠ΄ΠΎΠΉ ΠΎΡ€ΠΈΠ΅Π½Ρ‚Π°Ρ†ΠΈΠΈ.По Π·Π°Π²Π΅Ρ€ΡˆΠ΅Π½ΠΈΠΈ Π½Π°ΠΆΠΌΠΈΡ‚Π΅ ΠΊΠ½ΠΎΠΏΠΊΡƒ, Ρ‡Ρ‚ΠΎΠ±Ρ‹ ΠΏΡ€ΠΎΡΠΌΠΎΡ‚Ρ€Π΅Ρ‚ΡŒ ΠΎΡ‚Π²Π΅Ρ‚Ρ‹.


Π’ Π·Π°ΠΊΠ»ΡŽΡ‡Π΅Π½ΠΈΠ΅, равновСсиС — это состояниС ΠΎΠ±ΡŠΠ΅ΠΊΡ‚Π°, Π² ΠΊΠΎΡ‚ΠΎΡ€ΠΎΠΌ всС силы, Π΄Π΅ΠΉΡΡ‚Π²ΡƒΡŽΡ‰ΠΈΠ΅ Π½Π° Π½Π΅Π³ΠΎ, ΡƒΡ€Π°Π²Π½ΠΎΠ²Π΅ΡˆΠ΅Π½Ρ‹. Π’ Ρ‚Π°ΠΊΠΈΡ… случаях чистая сила Ρ€Π°Π²Π½Π° 0 ΠΡŒΡŽΡ‚ΠΎΠ½Π°ΠΌ. Зная силы, Π΄Π΅ΠΉΡΡ‚Π²ΡƒΡŽΡ‰ΠΈΠ΅ Π½Π° ΠΎΠ±ΡŠΠ΅ΠΊΡ‚, тригономСтричСскиС Ρ„ΡƒΠ½ΠΊΡ†ΠΈΠΈ ΠΌΠΎΠ³ΡƒΡ‚ ΠΈΡΠΏΠΎΠ»ΡŒΠ·ΠΎΠ²Π°Ρ‚ΡŒΡΡ для опрСдСлСния Π³ΠΎΡ€ΠΈΠ·ΠΎΠ½Ρ‚Π°Π»ΡŒΠ½Ρ‹Ρ… ΠΈ Π²Π΅Ρ€Ρ‚ΠΈΠΊΠ°Π»ΡŒΠ½Ρ‹Ρ… ΠΊΠΎΠΌΠΏΠΎΠ½Π΅Π½Ρ‚ΠΎΠ² ΠΊΠ°ΠΆΠ΄ΠΎΠΉ силы. Π’ случаС равновСсия всС Π²Π΅Ρ€Ρ‚ΠΈΠΊΠ°Π»ΡŒΠ½Ρ‹Π΅ ΠΊΠΎΠΌΠΏΠΎΠ½Π΅Π½Ρ‚Ρ‹ Π΄ΠΎΠ»ΠΆΠ½Ρ‹ ΡƒΡ€Π°Π²Π½ΠΎΠ²Π΅ΡˆΠΈΠ²Π°Ρ‚ΡŒΡΡ, Π° всС Π³ΠΎΡ€ΠΈΠ·ΠΎΠ½Ρ‚Π°Π»ΡŒΠ½Ρ‹Π΅ ΠΊΠΎΠΌΠΏΠΎΠ½Π΅Π½Ρ‚Ρ‹ Π΄ΠΎΠ»ΠΆΠ½Ρ‹ ΡƒΡ€Π°Π²Π½ΠΎΠ²Π΅ΡˆΠΈΠ²Π°Ρ‚ΡŒΡΡ.


ΠœΡ‹ Ρ…ΠΎΡ‚Π΅Π»ΠΈ Π±Ρ‹ ΠΏΡ€Π΅Π΄Π»ΠΎΠΆΠΈΡ‚ΡŒ … Иногда просто ΠΏΡ€ΠΎΡ‡ΠΈΡ‚Π°Ρ‚ΡŒ ΠΎΠ± этом нСдостаточно. Π’Ρ‹ Π΄ΠΎΠ»ΠΆΠ½Ρ‹ с Π½ΠΈΠΌ Π²Π·Π°ΠΈΠΌΠΎΠ΄Π΅ΠΉΡΡ‚Π²ΠΎΠ²Π°Ρ‚ΡŒ! И это ΠΈΠΌΠ΅Π½Π½ΠΎ Ρ‚ΠΎ, Ρ‡Ρ‚ΠΎ Π²Ρ‹ Π΄Π΅Π»Π°Π΅Ρ‚Π΅, ΠΊΠΎΠ³Π΄Π° ΠΈΡΠΏΠΎΠ»ΡŒΠ·ΡƒΠ΅Ρ‚Π΅ ΠΎΠ΄ΠΈΠ½ ΠΈΠ· ΠΈΠ½Ρ‚Π΅Ρ€Π°ΠΊΡ‚ΠΈΠ²Π½Ρ‹Ρ… ΠΌΠ°Ρ‚Π΅Ρ€ΠΈΠ°Π»ΠΎΠ² The Physics Classroom. ΠœΡ‹ Ρ…ΠΎΡ‚Π΅Π»ΠΈ Π±Ρ‹ ΠΏΡ€Π΅Π΄Π»ΠΎΠΆΠΈΡ‚ΡŒ Π²Π°ΠΌ ΡΠΎΠ²ΠΌΠ΅ΡΡ‚ΠΈΡ‚ΡŒ Ρ‡Ρ‚Π΅Π½ΠΈΠ΅ этой страницы с использованиСм нашСго ΠΈΠ½Ρ‚Π΅Ρ€Π°ΠΊΡ‚ΠΈΠ²Π½ΠΎΠ³ΠΎ прилоТСния «Назови этот Π²Π΅ΠΊΡ‚ΠΎΡ€Β», ΠΈΠ½Ρ‚Π΅Ρ€Π°ΠΊΡ‚ΠΈΠ²Π½ΠΎΠ³ΠΎ элСмСнта Β«Π‘Π»ΠΎΠΆΠ΅Π½ΠΈΠ΅ Π²Π΅ΠΊΡ‚ΠΎΡ€ΠΎΠ²Β» ΠΈΠ»ΠΈ Β«Π˜Π½Ρ‚Π΅Ρ€Π°ΠΊΡ‚ΠΈΠ²Π½ΠΎΠΉ ΠΈΠ³Ρ€Ρ‹ ΠΏΠΎ ΡƒΠ³Π°Π΄Ρ‹Π²Π°Π½ΠΈΡŽ Π²Π΅ΠΊΡ‚ΠΎΡ€ΠΎΠ²Β».ВсС Ρ‚Ρ€ΠΈ ΠΈΠ½Ρ‚Π΅Ρ€Π°ΠΊΡ‚ΠΈΠ²Π½Ρ‹Ρ… элСмСнта ΠΌΠΎΠΆΠ½ΠΎ Π½Π°ΠΉΡ‚ΠΈ Π² Ρ€Π°Π·Π΄Π΅Π»Π΅ Β«Π˜Π½Ρ‚Π΅Ρ€Π°ΠΊΡ‚ΠΈΠ²Π½Π°Ρ Ρ„ΠΈΠ·ΠΈΠΊΠ°Β» нашСго Π²Π΅Π±-сайта ΠΈ ΠΎΠ±Π΅ΡΠΏΠ΅Ρ‡ΠΈΡ‚ΡŒ ΠΈΠ½Ρ‚Π΅Ρ€Π°ΠΊΡ‚ΠΈΠ²Π½Ρ‹ΠΉ ΠΎΠΏΡ‹Ρ‚ с Π½Π°Π²Ρ‹ΠΊΠΎΠΌ добавлСния Π²Π΅ΠΊΡ‚ΠΎΡ€ΠΎΠ².


ΠŸΡ€ΠΎΠ²Π΅Ρ€ΡŒΡ‚Π΅ своС ΠΏΠΎΠ½ΠΈΠΌΠ°Π½ΠΈΠ΅

Π‘Π»Π΅Π΄ΡƒΡŽΡ‰ΠΈΠ΅ вопросы ΠΏΡ€Π΅Π΄Π½Π°Π·Π½Π°Ρ‡Π΅Π½Ρ‹ для ΠΏΡ€ΠΎΠ²Π΅Ρ€ΠΊΠΈ вашСго понимания ситуаций равновСсия. НаТмитС ΠΊΠ½ΠΎΠΏΠΊΡƒ, Ρ‡Ρ‚ΠΎΠ±Ρ‹ ΠΏΡ€ΠΎΡΠΌΠΎΡ‚Ρ€Π΅Ρ‚ΡŒ ΠΎΡ‚Π²Π΅Ρ‚Ρ‹ Π½Π° эти вопросы.

1.На стСнС висит ΡΠ»Π΅Π΄ΡƒΡŽΡ‰Π°Ρ ΠΊΠ°Ρ€Ρ‚ΠΈΠ½Π°. Π˜ΡΠΏΠΎΠ»ΡŒΠ·ΡƒΠΉΡ‚Π΅ тригономСтричСскиС Ρ„ΡƒΠ½ΠΊΡ†ΠΈΠΈ, Ρ‡Ρ‚ΠΎΠ±Ρ‹ ΠΎΠΏΡ€Π΅Π΄Π΅Π»ΠΈΡ‚ΡŒ вСс изобраТСния.

2. Π’Π°Π±Π»ΠΈΡ‡ΠΊΠ° Π²Π½ΠΈΠ·Ρƒ висит снаруТи класса Ρ„ΠΈΠ·ΠΈΠΊΠΈ, рСкламируя ΡΠ°ΠΌΡƒΡŽ Π²Π°ΠΆΠ½ΡƒΡŽ истину, ΠΊΠΎΡ‚ΠΎΡ€ΡƒΡŽ ΠΌΠΎΠΆΠ½ΠΎ Π½Π°ΠΉΡ‚ΠΈ Π²Π½ΡƒΡ‚Ρ€ΠΈ. Π—Π½Π°ΠΊ опираСтся Π½Π° Π΄ΠΈΠ°Π³ΠΎΠ½Π°Π»ΡŒΠ½Ρ‹ΠΉ трос ΠΈ ТСсткий Ρ‚ΡƒΡ€Π½ΠΈΠΊ. Если вывСска ΠΈΠΌΠ΅Π΅Ρ‚ массу 50 ΠΊΠ³, ΠΎΠΏΡ€Π΅Π΄Π΅Π»ΠΈΡ‚Π΅ натяТСниС диагонального троса, ΠΏΠΎΠ΄Π΄Π΅Ρ€ΠΆΠΈΠ²Π°ΡŽΡ‰Π΅Π³ΠΎ Π΅Π³ΠΎ вСс.

3. Π‘Π»Π΅Π΄ΡƒΡŽΡ‰ΠΈΠΉ Π·Π½Π°ΠΊ ΠΌΠΎΠΆΠ½ΠΎ Π½Π°ΠΉΡ‚ΠΈ Π² Π“Π»Π΅Π½Π²ΡŒΡŽ. Π—Π½Π°ΠΊ ΠΈΠΌΠ΅Π΅Ρ‚ массу 50 ΠΊΠ³. ΠžΠΏΡ€Π΅Π΄Π΅Π»ΠΈΡ‚Π΅ натяТСниС тросов.

4. ПослС самой послСднСй доставки ΠΏΠ΅Ρ‡Π°Π»ΡŒΠ½ΠΎ извСстный аист ΠΎΠ±ΡŠΡΠ²Π»ΡΠ΅Ρ‚ Ρ…ΠΎΡ€ΠΎΡˆΠΈΠ΅ новости. Если Π·Π½Π°ΠΊ ΠΈΠΌΠ΅Π΅Ρ‚ массу 10 ΠΊΠ³, Ρ‚ΠΎ ΠΊΠ°ΠΊΠΎΠ²Π° сила натяТСния Π² ΠΊΠ°ΠΆΠ΄ΠΎΠΌ тросС? Π˜ΡΠΏΠΎΠ»ΡŒΠ·ΡƒΠΉΡ‚Π΅ тригономСтричСскиС Ρ„ΡƒΠ½ΠΊΡ†ΠΈΠΈ ΠΈ эскиз, Ρ‡Ρ‚ΠΎΠ±Ρ‹ ΠΏΠΎΠΌΠΎΡ‡ΡŒ Π² Ρ€Π΅ΡˆΠ΅Π½ΠΈΠΈ.

5. ΠŸΡ€Π΅Π΄ΠΏΠΎΠ»ΠΎΠΆΠΈΠΌ, Ρ‡Ρ‚ΠΎ ΡƒΡ‡Π΅Π½ΠΈΠΊ тянСт с двумя большими силами (F 1 ΠΈ F 2 ), Ρ‡Ρ‚ΠΎΠ±Ρ‹ ΠΏΠΎΠ΄Π½ΡΡ‚ΡŒ ΠΊΠ½ΠΈΠ³Ρƒ вСсом 1 ΠΊΠ³ Π·Π° Π΄Π²Π° троса. Если ΠΊΠ°Π±Π΅Π»ΠΈ ΠΎΠ±Ρ€Π°Π·ΡƒΡŽΡ‚ ΡƒΠ³ΠΎΠ» Π² 1 градус с Π³ΠΎΡ€ΠΈΠ·ΠΎΠ½Ρ‚Π°Π»ΡŒΡŽ, Ρ‚ΠΎ ΠΊΠ°ΠΊΠΎΠ²ΠΎ натяТСниС кабСля?

12.1 Условия статичСского равновСсия — University Physics Volume 1

Π¦Π΅Π»ΠΈ обучСния

К ΠΊΠΎΠ½Ρ†Ρƒ этого Ρ€Π°Π·Π΄Π΅Π»Π° Π²Ρ‹ смоТСтС:

  • ΠžΠΏΡ€Π΅Π΄Π΅Π»ΠΈΡ‚Π΅ физичСскиС условия статичСского равновСсия.
  • НарисуйтС Π΄ΠΈΠ°Π³Ρ€Π°ΠΌΠΌΡƒ свободного Ρ‚Π΅Π»Π° для Ρ‚Π²Π΅Ρ€Π΄ΠΎΠ³ΠΎ Ρ‚Π΅Π»Π°, Π½Π° ΠΊΠΎΡ‚ΠΎΡ€ΠΎΠ΅ Π΄Π΅ΠΉΡΡ‚Π²ΡƒΡŽΡ‚ силы.
  • ΠžΠ±ΡŠΡΡΠ½ΠΈΡ‚Π΅, ΠΊΠ°ΠΊ условия равновСсия ΠΏΠΎΠ·Π²ΠΎΠ»ΡΡŽΡ‚ Π½Π°ΠΌ Ρ€Π΅ΡˆΠ°Ρ‚ΡŒ Π·Π°Π΄Π°Ρ‡ΠΈ статики.

ΠœΡ‹ Π³ΠΎΠ²ΠΎΡ€ΠΈΠΌ, Ρ‡Ρ‚ΠΎ Ρ‚Π²Π΅Ρ€Π΄ΠΎΠ΅ Ρ‚Π΅Π»ΠΎ находится Π² состоянии равновСсия , ΠΊΠΎΠ³Π΄Π° Π΅Π³ΠΎ Π»ΠΈΠ½Π΅ΠΉΠ½ΠΎΠ΅ ΠΈ ΡƒΠ³Π»ΠΎΠ²ΠΎΠ΅ ускорСниС Ρ€Π°Π²Π½Ρ‹ Π½ΡƒΠ»ΡŽ ΠΎΡ‚Π½ΠΎΡΠΈΡ‚Π΅Π»ΡŒΠ½ΠΎ ΠΈΠ½Π΅Ρ€Ρ†ΠΈΠ°Π»ΡŒΠ½ΠΎΠΉ систСмы отсчСта. Π­Ρ‚ΠΎ ΠΎΠ·Π½Π°Ρ‡Π°Π΅Ρ‚, Ρ‡Ρ‚ΠΎ Ρ‚Π΅Π»ΠΎ Π² состоянии равновСсия ΠΌΠΎΠΆΠ΅Ρ‚ Π΄Π²ΠΈΠ³Π°Ρ‚ΡŒΡΡ, Π½ΠΎ Π² этом случаС Π΅Π³ΠΎ линСйная ΠΈ угловая скорости Π΄ΠΎΠ»ΠΆΠ½Ρ‹ Π±Ρ‹Ρ‚ΡŒ постоянными. ΠœΡ‹ Π³ΠΎΠ²ΠΎΡ€ΠΈΠΌ, Ρ‡Ρ‚ΠΎ Ρ‚Π²Π΅Ρ€Π΄ΠΎΠ΅ Ρ‚Π΅Π»ΠΎ находится Π² состоянии статичСского равновСсия , ΠΊΠΎΠ³Π΄Π° ΠΎΠ½ΠΎ находится Π² состоянии покоя Π² нашСй Π²Ρ‹Π±Ρ€Π°Π½Π½ΠΎΠΉ систСмС ΠΊΠΎΠΎΡ€Π΄ΠΈΠ½Π°Ρ‚ .ΠžΠ±Ρ€Π°Ρ‚ΠΈΡ‚Π΅ Π²Π½ΠΈΠΌΠ°Π½ΠΈΠ΅, Ρ‡Ρ‚ΠΎ Ρ€Π°Π·Π»ΠΈΡ‡ΠΈΠ΅ ΠΌΠ΅ΠΆΠ΄Ρƒ состояниСм покоя ΠΈ состояниСм Ρ€Π°Π²Π½ΠΎΠΌΠ΅Ρ€Π½ΠΎΠ³ΠΎ двиТСния являСтся искусствСнным, Ρ‚ΠΎ Π΅ΡΡ‚ΡŒ ΠΎΠ±ΡŠΠ΅ΠΊΡ‚ ΠΌΠΎΠΆΠ΅Ρ‚ Π½Π°Ρ…ΠΎΠ΄ΠΈΡ‚ΡŒΡΡ Π² состоянии покоя Π² Π²Ρ‹Π±Ρ€Π°Π½Π½ΠΎΠΉ Π½Π°ΠΌΠΈ систСмС отсчСта, Π½ΠΎ для Π½Π°Π±Π»ΡŽΠ΄Π°Ρ‚Π΅Π»Ρ, двиТущСгося с постоянной ΡΠΊΠΎΡ€ΠΎΡΡ‚ΡŒΡŽ ΠΎΡ‚Π½ΠΎΡΠΈΡ‚Π΅Π»ΡŒΠ½ΠΎ нашСй систСмы ΠΊΠΎΠΎΡ€Π΄ΠΈΠ½Π°Ρ‚, Ρ‚ΠΎΡ‚ ΠΆΠ΅ ΠΎΠ±ΡŠΠ΅ΠΊΡ‚ каТСтся, находится Π² Ρ€Π°Π²Π½ΠΎΠΌΠ΅Ρ€Π½ΠΎΠΌ Π΄Π²ΠΈΠΆΠ΅Π½ΠΈΠΈ с постоянной ΡΠΊΠΎΡ€ΠΎΡΡ‚ΡŒΡŽ. ΠŸΠΎΡΠΊΠΎΠ»ΡŒΠΊΡƒ Π΄Π²ΠΈΠΆΠ΅Π½ΠΈΠ΅ составляСт ΠΎΡ‚Π½ΠΎΡΠΈΡ‚Π΅Π»ΡŒΠ½ΠΎ , Ρ‚ΠΎ, Ρ‡Ρ‚ΠΎ для нас находится Π² статичСском равновСсии, находится Π² динамичСском равновСсии для двиТущСгося Π½Π°Π±Π»ΡŽΠ΄Π°Ρ‚Π΅Π»Ρ, ΠΈ Π½Π°ΠΎΠ±ΠΎΡ€ΠΎΡ‚. ΠŸΠΎΡΠΊΠΎΠ»ΡŒΠΊΡƒ Π·Π°ΠΊΠΎΠ½Ρ‹ Ρ„ΠΈΠ·ΠΈΠΊΠΈ ΠΈΠ΄Π΅Π½Ρ‚ΠΈΡ‡Π½Ρ‹ для всСх ΠΈΠ½Π΅Ρ€Ρ†ΠΈΠ°Π»ΡŒΠ½Ρ‹Ρ… систСм отсчСта, Π² ΠΈΠ½Π΅Ρ€Ρ†ΠΈΠ°Π»ΡŒΠ½ΠΎΠΉ систСмС отсчСта Π½Π΅Ρ‚ различия ΠΌΠ΅ΠΆΠ΄Ρƒ статичСским равновСсиСм ΠΈ равновСсиСм.

Богласно Π²Ρ‚ΠΎΡ€ΠΎΠΌΡƒ Π·Π°ΠΊΠΎΠ½Ρƒ двиТСния ΠΡŒΡŽΡ‚ΠΎΠ½Π°, Π»ΠΈΠ½Π΅ΠΉΠ½ΠΎΠ΅ ускорСниС Ρ‚Π²Π΅Ρ€Π΄ΠΎΠ³ΠΎ Ρ‚Π΅Π»Π° вызываСтся Π΄Π΅ΠΉΡΡ‚Π²ΡƒΡŽΡ‰Π΅ΠΉ Π½Π° Π½Π΅Π³ΠΎ чистой силой, ΠΈΠ»ΠΈ

[латСкс] \ sum _ {k} {\ mathbf {\ overset {\ to} {F}}} _ {k} = m {\ mathbf {\ overset {\ to} {a}}} _ {\ text {CM}}. [/ Latex]

Π—Π΄Π΅ΡΡŒ сумма всСх Π²Π½Π΅ΡˆΠ½ΠΈΡ… сил, Π΄Π΅ΠΉΡΡ‚Π²ΡƒΡŽΡ‰ΠΈΡ… Π½Π° Ρ‚Π΅Π»ΠΎ, Π³Π΄Π΅ ΠΌ — это Π΅Π³ΠΎ масса ΠΈ [латСкс] {\ mathbf {\ overset {\ to} {a}}} _ {\ text {CM}} [ / latex] — это Π»ΠΈΠ½Π΅ΠΉΠ½ΠΎΠ΅ ускорСниС Π΅Π³ΠΎ Ρ†Π΅Π½Ρ‚Ρ€Π° масс (концСпция, ΠΊΠΎΡ‚ΠΎΡ€ΡƒΡŽ ΠΌΡ‹ обсуТдали Π² ΡΡ‚Π°Ρ‚ΡŒΡΡ… Β«Π›ΠΈΠ½Π΅ΠΉΠ½Ρ‹ΠΉ ΠΈΠΌΠΏΡƒΠ»ΡŒΡ ΠΈ столкновСния ΠΏΠΎ Π»ΠΈΠ½Π΅ΠΉΠ½ΠΎΠΌΡƒ ΠΈΠΌΠΏΡƒΠ»ΡŒΡΡƒ ΠΈ столкновСниям»).Π’ состоянии равновСсия Π»ΠΈΠ½Π΅ΠΉΠ½ΠΎΠ΅ ускорСниС Ρ€Π°Π²Π½ΠΎ Π½ΡƒΠ»ΡŽ. Если ΠΌΡ‹ установим ускорСниС Π½Π° ноль Π½Π° рисункС, ΠΌΡ‹ ΠΏΠΎΠ»ΡƒΡ‡ΠΈΠΌ ΡΠ»Π΅Π΄ΡƒΡŽΡ‰Π΅Π΅ ΡƒΡ€Π°Π²Π½Π΅Π½ΠΈΠ΅:

ΠŸΠ΅Ρ€Π²ΠΎΠ΅ условиС равновСсия

ΠŸΠ΅Ρ€Π²ΠΎΠ΅ условиС равновСсия для статичСского равновСсия Ρ‚Π²Π΅Ρ€Π΄ΠΎΠ³ΠΎ Ρ‚Π΅Π»Π° Π²Ρ‹Ρ€Π°ΠΆΠ°Π΅Ρ‚ ΠΏΠΎΡΡ‚ΡƒΠΏΠ°Ρ‚Π΅Π»ΡŒΠ½ΠΎΠ΅ равновСсиС:

[латСкс] \ sum _ {k} {\ mathbf {\ overset {\ to} {F}}} _ {k} = \ mathbf {\ overset {\ to} {0}}. [/ Latex]

ΠŸΠ΅Ρ€Π²ΠΎΠ΅ условиС равновСсия, рисунок, являСтся условиСм равновСсия сил, с ΠΊΠΎΡ‚ΠΎΡ€Ρ‹ΠΌ ΠΌΡ‹ ΡΡ‚ΠΎΠ»ΠΊΠ½ΡƒΠ»ΠΈΡΡŒ ΠΏΡ€ΠΈ ΠΈΠ·ΡƒΡ‡Π΅Π½ΠΈΠΈ ΠΏΡ€ΠΈΠ»ΠΎΠΆΠ΅Π½ΠΈΠΉ Π·Π°ΠΊΠΎΠ½ΠΎΠ² ΠΡŒΡŽΡ‚ΠΎΠ½Π°.

Π­Ρ‚ΠΎ Π²Π΅ΠΊΡ‚ΠΎΡ€Π½ΠΎΠ΅ ΡƒΡ€Π°Π²Π½Π΅Π½ΠΈΠ΅ эквивалСнтно ΡΠ»Π΅Π΄ΡƒΡŽΡ‰ΠΈΠΌ Ρ‚Ρ€Π΅ΠΌ скалярным уравнСниям для ΠΊΠΎΠΌΠΏΠΎΠ½Π΅Π½Ρ‚ΠΎΠ² чистой силы:

[латСкс] \ sum _ {k} {F} _ {kx} = 0, \ quad \ sum _ {k} {F} _ {ky} = 0, \ quad \ sum _ {k} {F} _ {kz} = 0. [/ latex]

Аналогично рисунку, ΠΌΡ‹ ΠΌΠΎΠΆΠ΅ΠΌ ΡƒΡ‚Π²Π΅Ρ€ΠΆΠ΄Π°Ρ‚ΡŒ, Ρ‡Ρ‚ΠΎ Π²Ρ€Π°Ρ‰Π°Ρ‚Π΅Π»ΡŒΠ½ΠΎΠ΅ ускорСниС [latex] \ mathbf {\ overset {\ to} {\ alpha}} [/ latex] Ρ‚Π²Π΅Ρ€Π΄ΠΎΠ³ΠΎ Ρ‚Π΅Π»Π° Π²ΠΎΠΊΡ€ΡƒΠ³ фиксированной оси вращСния вызываСтся Π΄Π΅ΠΉΡΡ‚Π²ΡƒΡŽΡ‰ΠΈΠΌ чистым крутящим ΠΌΠΎΠΌΠ΅Π½Ρ‚ΠΎΠΌ Π½Π° корпусС, ΠΈΠ»ΠΈ

[латСкс] \ sum _ {k} {\ mathbf {\ overset {\ to} {\ tau}}} _ {k} = I \ mathbf {\ overset {\ to} {\ alpha}}.[/ латСкс]

Π—Π΄Π΅ΡΡŒ [латСкс] I [/ latex] — это инСрция вращСния Ρ‚Π΅Π»Π° ΠΏΡ€ΠΈ Π²Ρ€Π°Ρ‰Π΅Π½ΠΈΠΈ Π²ΠΎΠΊΡ€ΡƒΠ³ этой оси, Π° сумма составляСт Π±ΠΎΠ»Π΅Π΅ всСх ΠΌΠΎΠΌΠ΅Π½Ρ‚ΠΎΠ² [латСкс] {\ mathbf {\ overset {\ to} {\ tau}}} _ {k} [/ latex] Π²Π½Π΅ΡˆΠ½ΠΈΡ… сил Π½Π° рис. Π’ состоянии равновСсия ускорСниС вращСния Ρ€Π°Π²Π½ΠΎ Π½ΡƒΠ»ΡŽ. ΠžΠ±Π½ΡƒΠ»ΡΡ ΠΏΡ€Π°Π²ΡƒΡŽ Ρ‡Π°ΡΡ‚ΡŒ рисунка, ΠΌΡ‹ ΠΏΠΎΠ»ΡƒΡ‡Π°Π΅ΠΌ Π²Ρ‚ΠΎΡ€ΠΎΠ΅ условиС равновСсия:

Π’Ρ‚ΠΎΡ€ΠΎΠ΅ состояниС равновСсия

Π’Ρ‚ΠΎΡ€ΠΎΠ΅ условиС равновСсия для статичСского равновСсия Ρ‚Π²Π΅Ρ€Π΄ΠΎΠ³ΠΎ Ρ‚Π΅Π»Π° Π²Ρ‹Ρ€Π°ΠΆΠ°Π΅Ρ‚ Π²Ρ€Π°Ρ‰Π°Ρ‚Π΅Π»ΡŒΠ½ΠΎΠ΅ равновСсиС:

[латСкс] \ sum _ {k} {\ mathbf {\ overset {\ to} {\ tau}}} _ {k} = \ mathbf {\ overset {\ to} {0}}.[/ латСкс]

Π’Ρ‚ΠΎΡ€ΠΎΠ΅ условиС равновСсия, рисунок, являСтся условиСм равновСсия для крутящих ΠΌΠΎΠΌΠ΅Π½Ρ‚ΠΎΠ², с ΠΊΠΎΡ‚ΠΎΡ€Ρ‹ΠΌ ΠΌΡ‹ ΡΡ‚ΠΎΠ»ΠΊΠ½ΡƒΠ»ΠΈΡΡŒ ΠΏΡ€ΠΈ ΠΈΠ·ΡƒΡ‡Π΅Π½ΠΈΠΈ Π΄ΠΈΠ½Π°ΠΌΠΈΠΊΠΈ вращСния. Π‘Ρ‚ΠΎΠΈΡ‚ ΠΎΡ‚ΠΌΠ΅Ρ‚ΠΈΡ‚ΡŒ, Ρ‡Ρ‚ΠΎ это ΡƒΡ€Π°Π²Π½Π΅Π½ΠΈΠ΅ равновСсия ΠΎΠ±Ρ‹Ρ‡Π½ΠΎ справСдливо для Π²Ρ€Π°Ρ‰Π°Ρ‚Π΅Π»ΡŒΠ½ΠΎΠ³ΠΎ равновСсия Π²ΠΎΠΊΡ€ΡƒΠ³ любой оси вращСния (фиксированной ΠΈΠ»ΠΈ ΠΈΠ½ΠΎΠΉ). ΠžΠΏΡΡ‚ΡŒ ΠΆΠ΅, это Π²Π΅ΠΊΡ‚ΠΎΡ€Π½ΠΎΠ΅ ΡƒΡ€Π°Π²Π½Π΅Π½ΠΈΠ΅ эквивалСнтно Ρ‚Ρ€Π΅ΠΌ скалярным уравнСниям для Π²Π΅ΠΊΡ‚ΠΎΡ€Π½Ρ‹Ρ… ΠΊΠΎΠΌΠΏΠΎΠ½Π΅Π½Ρ‚ΠΎΠ² чистого крутящСго ΠΌΠΎΠΌΠ΅Π½Ρ‚Π°:

[латСкс] \ sum _ {k} {\ tau} _ {kx} = 0, \ quad \ sum _ {k} {\ tau} _ {ky} = 0, \ quad \ sum _ {k} {\ Ρ‚Π°Ρƒ} _ {kz} = 0.[/ латСкс]

Π’Ρ‚ΠΎΡ€ΠΎΠ΅ условиС равновСсия ΠΎΠ·Π½Π°Ρ‡Π°Π΅Ρ‚, Ρ‡Ρ‚ΠΎ Π² равновСсии Π½Π΅Ρ‚ чистого внСшнСго крутящСго ΠΌΠΎΠΌΠ΅Π½Ρ‚Π°, Π²Ρ‹Π·Ρ‹Π²Π°ΡŽΡ‰Π΅Π³ΠΎ Π²Ρ€Π°Ρ‰Π΅Π½ΠΈΠ΅ Π²ΠΎΠΊΡ€ΡƒΠ³ любой оси.

ΠŸΠ΅Ρ€Π²ΠΎΠ΅ ΠΈ Π²Ρ‚ΠΎΡ€ΠΎΠ΅ условия равновСсия ΡƒΠΊΠ°Π·Π°Π½Ρ‹ Π² ΠΊΠΎΠ½ΠΊΡ€Π΅Ρ‚Π½ΠΎΠΉ систСмС отсчСта. ΠŸΠ΅Ρ€Π²ΠΎΠ΅ условиС Π²ΠΊΠ»ΡŽΡ‡Π°Π΅Ρ‚ Ρ‚ΠΎΠ»ΡŒΠΊΠΎ силы ΠΈ поэтому Π½Π΅ зависит ΠΎΡ‚ источника систСмы отсчСта. Однако Π²Ρ‚ΠΎΡ€ΠΎΠ΅ условиС Π²ΠΊΠ»ΡŽΡ‡Π°Π΅Ρ‚ крутящий ΠΌΠΎΠΌΠ΅Π½Ρ‚, ΠΊΠΎΡ‚ΠΎΡ€Ρ‹ΠΉ опрСдСляСтся ΠΊΠ°ΠΊ пСрСкрСстноС ΠΏΡ€ΠΎΠΈΠ·Π²Π΅Π΄Π΅Π½ΠΈΠ΅, [латСкс] {\ mathbf {\ overset {\ to} {\ tau}}} _ {k} = {\ mathbf {\ overset {\ to} {r}}} _ {k} \ times {\ mathbf {\ overset {\ to} {F}}} _ {k}, [/ latex] Π³Π΄Π΅ Π²Π΅ΠΊΡ‚ΠΎΡ€ полоТСния [latex] {\ mathbf {\ overset {\ to} {r}}} _ {k} [/ latex] ΠΎΡ‚Π½ΠΎΡΠΈΡ‚Π΅Π»ΡŒΠ½ΠΎ оси вращСния Ρ‚ΠΎΡ‡ΠΊΠΈ прилоТСния силы Π²Ρ…ΠΎΠ΄ΠΈΡ‚ Π² ΡƒΡ€Π°Π²Π½Π΅Π½ΠΈΠ΅.Π‘Π»Π΅Π΄ΠΎΠ²Π°Ρ‚Π΅Π»ΡŒΠ½ΠΎ, крутящий ΠΌΠΎΠΌΠ΅Π½Ρ‚ зависит ΠΎΡ‚ полоТСния оси Π² систСмС отсчСта. Однако, ΠΊΠΎΠ³Π΄Π° условия Π²Ρ€Π°Ρ‰Π°Ρ‚Π΅Π»ΡŒΠ½ΠΎΠ³ΠΎ ΠΈ ΠΏΠΎΡΡ‚ΡƒΠΏΠ°Ρ‚Π΅Π»ΡŒΠ½ΠΎΠ³ΠΎ равновСсия Π²Ρ‹ΠΏΠΎΠ»Π½ΡΡŽΡ‚ΡΡ ΠΎΠ΄Π½ΠΎΠ²Ρ€Π΅ΠΌΠ΅Π½Π½ΠΎ Π² ΠΎΠ΄Π½ΠΎΠΉ систСмС отсчСта, ΠΎΠ½ΠΈ Ρ‚Π°ΠΊΠΆΠ΅ ΡΠΎΡ…Ρ€Π°Π½ΡΡŽΡ‚ΡΡ Π² любой Π΄Ρ€ΡƒΠ³ΠΎΠΉ ΠΈΠ½Π΅Ρ€Ρ†ΠΈΠ°Π»ΡŒΠ½ΠΎΠΉ систСмС отсчСта, Ρ‚Π°ΠΊ Ρ‡Ρ‚ΠΎ чистый крутящий ΠΌΠΎΠΌΠ΅Π½Ρ‚ Π²ΠΎΠΊΡ€ΡƒΠ³ любой оси вращСния ΠΏΠΎ-ΠΏΡ€Π΅ΠΆΠ½Π΅ΠΌΡƒ Ρ€Π°Π²Π΅Π½ Π½ΡƒΠ»ΡŽ. ОбъяснСниС этому довольно простоС.

ΠŸΡ€Π΅Π΄ΠΏΠΎΠ»ΠΎΠΆΠΈΠΌ, Ρ‡Ρ‚ΠΎ Π²Π΅ΠΊΡ‚ΠΎΡ€ [latex] \ mathbf {\ overset {\ to} {R}} [/ latex] — это позиция Π½Π°Ρ‡Π°Π»Π° ΠΊΠΎΠΎΡ€Π΄ΠΈΠ½Π°Ρ‚ Π½ΠΎΠ²ΠΎΠΉ ΠΈΠ½Π΅Ρ€Ρ†ΠΈΠ°Π»ΡŒΠ½ΠΎΠΉ систСмы отсчСта [latex] S \ prime [/ latex] Π² старом ΠΈΠ½Π΅Ρ€Ρ†ΠΈΠ°Π»ΡŒΠ½ΠΎΠΌ БистСма отсчСта S .{\ prime}} _ {k} \ times {\ mathbf {\ overset {\ to} {F}}} _ {k} = \ sum _ {k} ({\ mathbf {\ overset {\ to} {r }}} _ {k} — \ mathbf {\ overset {\ to} {R}}) \ times {\ mathbf {\ overset {\ to} {F}}} _ {k} = \ sum _ {k} {\ mathbf {\ overset {\ to} {r}}} _ {k} \ times {\ mathbf {\ overset {\ to} {F}}} _ {k} — \ sum _ {k} \ mathbf { \ overset {\ to} {R}} \ times {\ mathbf {\ overset {\ to} {F}}} _ {k} = \ sum _ {k} {\ mathbf {\ overset {\ to} {\ tau}}} _ {k} — \ mathbf {\ overset {\ to} {R}} \ times \ sum _ {k} {\ mathbf {\ overset {\ to} {F}}} _ {k} = \ mathbf {\ overset {\ to} {0}}. [/ latex]

На послСднСм этапС этой Ρ†Π΅ΠΏΠΎΡ‡ΠΊΠΈ рассуТдСний ΠΌΡ‹ использовали Ρ‚ΠΎΡ‚ Ρ„Π°ΠΊΡ‚, Ρ‡Ρ‚ΠΎ Π² равновСсии Π² старой систСмС отсчСта S ΠΏΠ΅Ρ€Π²Ρ‹ΠΉ Ρ‡Π»Π΅Π½ исчСзаСт ΠΈΠ·-Π·Π° рисунка, Π° Π²Ρ‚ΠΎΡ€ΠΎΠΉ Ρ‡Π»Π΅Π½ исчСзаСт ΠΈΠ·-Π·Π° рисунка.Π‘Π»Π΅Π΄ΠΎΠ²Π°Ρ‚Π΅Π»ΡŒΠ½ΠΎ, ΠΌΡ‹ Π²ΠΈΠ΄ΠΈΠΌ, Ρ‡Ρ‚ΠΎ чистый крутящий ΠΌΠΎΠΌΠ΅Π½Ρ‚ Π² любой ΠΈΠ½Π΅Ρ€Ρ†ΠΈΠ°Π»ΡŒΠ½ΠΎΠΉ систСмС отсчСта [латСкс] S \ prime [/ latex] Ρ€Π°Π²Π΅Π½ Π½ΡƒΠ»ΡŽ, ΠΏΡ€ΠΈ условии, Ρ‡Ρ‚ΠΎ ΠΎΠ±Π° условия равновСсия Π²Ρ‹ΠΏΠΎΠ»Π½ΡΡŽΡ‚ΡΡ Π² ΠΈΠ½Π΅Ρ€Ρ†ΠΈΠ°Π»ΡŒΠ½ΠΎΠΉ систСмС отсчСта S .

ΠŸΡ€Π°ΠΊΡ‚ΠΈΡ‡Π΅ΡΠΊΠΎΠ΅ Π·Π½Π°Ρ‡Π΅Π½ΠΈΠ΅ этого состоит Π² Ρ‚ΠΎΠΌ, Ρ‡Ρ‚ΠΎ ΠΏΡ€ΠΈ ΠΏΡ€ΠΈΠΌΠ΅Π½Π΅Π½ΠΈΠΈ условий равновСсия для Ρ‚Π²Π΅Ρ€Π΄ΠΎΠ³ΠΎ Ρ‚Π΅Π»Π° ΠΌΡ‹ ΠΌΠΎΠΆΠ΅ΠΌ Π²Ρ‹Π±Ρ€Π°Ρ‚ΡŒ Π»ΡŽΠ±ΡƒΡŽ Ρ‚ΠΎΡ‡ΠΊΡƒ Π² качСствС Π½Π°Ρ‡Π°Π»Π° отсчСта систСмы отсчСта. Наш Π²Ρ‹Π±ΠΎΡ€ систСмы отсчСта ΠΏΡ€ΠΎΠ΄ΠΈΠΊΡ‚ΠΎΠ²Π°Π½ физичСскими особСнностями Ρ€Π΅ΡˆΠ°Π΅ΠΌΠΎΠΉ ΠΏΡ€ΠΎΠ±Π»Π΅ΠΌΡ‹. Π’ ΠΎΠ΄Π½ΠΎΠΉ систСмС отсчСта матСматичСская Ρ„ΠΎΡ€ΠΌΠ° условий равновСсия ΠΌΠΎΠΆΠ΅Ρ‚ Π±Ρ‹Ρ‚ΡŒ довольно слоТной, Ρ‚ΠΎΠ³Π΄Π° ΠΊΠ°ΠΊ Π² Π΄Ρ€ΡƒΠ³ΠΎΠΉ систСмС ΠΊΠΎΠΎΡ€Π΄ΠΈΠ½Π°Ρ‚ Ρ‚Π΅ ΠΆΠ΅ условия ΠΌΠΎΠ³ΡƒΡ‚ ΠΈΠΌΠ΅Ρ‚ΡŒ Π±ΠΎΠ»Π΅Π΅ ΠΏΡ€ΠΎΡΡ‚ΡƒΡŽ ΠΌΠ°Ρ‚Π΅ΠΌΠ°Ρ‚ΠΈΡ‡Π΅ΡΠΊΡƒΡŽ Ρ„ΠΎΡ€ΠΌΡƒ, ΠΊΠΎΡ‚ΠΎΡ€ΡƒΡŽ Π»Π΅Π³ΠΊΠΎ Ρ€Π΅ΡˆΠΈΡ‚ΡŒ.Начало Π²Ρ‹Π±Ρ€Π°Π½Π½ΠΎΠΉ систСмы отсчСта называСтся Ρ‚ΠΎΡ‡ΠΊΠΎΠΉ ΠΏΠΎΠ²ΠΎΡ€ΠΎΡ‚Π° .

Π’ самом ΠΎΠ±Ρ‰Π΅ΠΌ случаС условия равновСсия Π²Ρ‹Ρ€Π°ΠΆΠ°ΡŽΡ‚ΡΡ ΡˆΠ΅ΡΡ‚ΡŒΡŽ скалярными уравнСниями (рисунок ΠΈ рисунок). Для плоских Π·Π°Π΄Π°Ρ‡ равновСсия с Π²Ρ€Π°Ρ‰Π΅Π½ΠΈΠ΅ΠΌ Π²ΠΎΠΊΡ€ΡƒΠ³ фиксированной оси, ΠΊΠΎΡ‚ΠΎΡ€Ρ‹Π΅ ΠΌΡ‹ рассматриваСм Π² этой Π³Π»Π°Π²Π΅, ΠΌΡ‹ ΠΌΠΎΠΆΠ΅ΠΌ ΡΠΎΠΊΡ€Π°Ρ‚ΠΈΡ‚ΡŒ количСство ΡƒΡ€Π°Π²Π½Π΅Π½ΠΈΠΉ Π΄ΠΎ Ρ‚Ρ€Π΅Ρ…. Бтандартная ΠΏΡ€ΠΎΡ†Π΅Π΄ΡƒΡ€Π° состоит Π² Ρ‚ΠΎΠΌ, Ρ‡Ρ‚ΠΎΠ±Ρ‹ ΠΏΡ€ΠΈΠ½ΡΡ‚ΡŒ систСму отсчСта, Π² ΠΊΠΎΡ‚ΠΎΡ€ΠΎΠΉ ось z являСтся осью вращСния. ΠŸΡ€ΠΈ Ρ‚Π°ΠΊΠΎΠΌ Π²Ρ‹Π±ΠΎΡ€Π΅ оси чистый крутящий ΠΌΠΎΠΌΠ΅Π½Ρ‚ ΠΈΠΌΠ΅Π΅Ρ‚ Ρ‚ΠΎΠ»ΡŒΠΊΠΎ ΠΊΠΎΠΌΠΏΠΎΠ½Π΅Π½Ρ‚ z , всС силы, ΠΊΠΎΡ‚ΠΎΡ€Ρ‹Π΅ ΠΈΠΌΠ΅ΡŽΡ‚ Π½Π΅Π½ΡƒΠ»Π΅Π²Ρ‹Π΅ крутящиС ΠΌΠΎΠΌΠ΅Π½Ρ‚Ρ‹, Π»Π΅ΠΆΠ°Ρ‚ Π² плоскости xy , ΠΈ, ΡΠ»Π΅Π΄ΠΎΠ²Π°Ρ‚Π΅Π»ΡŒΠ½ΠΎ, Π²ΠΊΠ»Π°Π΄ Π² чистый крутящий ΠΌΠΎΠΌΠ΅Π½Ρ‚ поступаСт Ρ‚ΠΎΠ»ΡŒΠΊΠΎ ΠΎΡ‚ x — ΠΈ y — ΡΠΎΡΡ‚Π°Π²Π»ΡΡŽΡ‰ΠΈΠ΅ Π²Π½Π΅ΡˆΠ½ΠΈΡ… сил.Π’Π°ΠΊΠΈΠΌ ΠΎΠ±Ρ€Π°Π·ΠΎΠΌ, для плоских Π·Π°Π΄Π°Ρ‡ с осью вращСния, пСрпСндикулярной плоскости xy , ΠΌΡ‹ ΠΈΠΌΠ΅Π΅ΠΌ ΡΠ»Π΅Π΄ΡƒΡŽΡ‰ΠΈΠ΅ Ρ‚Ρ€ΠΈ условия равновСсия для сил ΠΈ ΠΌΠΎΠΌΠ΅Π½Ρ‚ΠΎΠ²:

[латСкс] {F} _ {1x} + {F} _ {2x} + \ cdots + {F} _ {Nx} = 0 [/ латСкс]

[латСкс] {F} _ {1y} + {F} _ {2y} + \ cdots + {F} _ {Ny} = 0 [/ латСкс]

[латСкс] {\ tau} _ {1} + {\ tau} _ {2} + \ cdots + {\ tau} _ {N} = 0 [/ латСкс]

, Π³Π΄Π΅ суммированиС вСдСтся ΠΏΠΎ всСм внСшним силам N , Π΄Π΅ΠΉΡΡ‚Π²ΡƒΡŽΡ‰ΠΈΠΌ Π½Π° Ρ‚Π΅Π»ΠΎ, ΠΈ ΠΈΡ… крутящим ΠΌΠΎΠΌΠ΅Π½Ρ‚Π°ΠΌ. На рисункС ΠΌΡ‹ упростили обозначСния, опустив Π½ΠΈΠΆΠ½ΠΈΠΉ индСкс z , Π½ΠΎ ΠΌΡ‹ ΠΏΠΎΠ½ΠΈΠΌΠ°Π΅ΠΌ, Ρ‡Ρ‚ΠΎ здСсь суммированиС вСдСтся ΠΏΠΎ всСм Π²ΠΊΠ»Π°Π΄Π°ΠΌ вдоль оси z , которая являСтся осью вращСния.На рисункС z -ΠΊΠΎΠΌΠΏΠΎΠ½Π΅Π½Ρ‚ крутящСго ΠΌΠΎΠΌΠ΅Π½Ρ‚Π° [латСкс] {\ mathbf {\ overset {\ to} {\ tau}}} _ {k} [/ latex] ΠΎΡ‚ силы [латСкс] {\ mathbf {\ overset {\ to} {F}}} _ {k} [/ latex] —

[латСкс] {\ tau} _ {k} = {r} _ {k} {F} _ {k} \ text {sin} \, \ theta [/ latex]

Π³Π΄Π΅ [латСкс] {r} _ {k} [/ latex] — Π΄Π»ΠΈΠ½Π° ΠΏΠ»Π΅Ρ‡Π° Ρ€Ρ‹Ρ‡Π°Π³Π° силы, Π° [latex] {F} _ {k} [/ latex] — Π²Π΅Π»ΠΈΡ‡ΠΈΠ½Π° силы (ΠΊΠ°ΠΊ Π²Ρ‹ ΠΏΠΈΠ»Π° Π² Ρ€Π΅ΠΆΠΈΠΌΠ΅ вращСния с фиксированной осью). Π£Π³ΠΎΠ» [latex] \ theta [/ latex] — это ΡƒΠ³ΠΎΠ» ΠΌΠ΅ΠΆΠ΄Ρƒ Π²Π΅ΠΊΡ‚ΠΎΡ€Π°ΠΌΠΈ [latex] {\ mathbf {\ overset {\ to} {r}}} _ {k} [/ latex] ΠΈ [latex] {\ mathbf { \ overset {\ to} {F}}} _ {k}, [/ latex] ΠΈΠ·ΠΌΠ΅Ρ€Π΅Π½ΠΈΠ΅ ΠΎΡ‚ Π²Π΅ΠΊΡ‚ΠΎΡ€Π° [латСкс] {\ mathbf {\ overset {\ to} {r}}} _ {k} [/ latex ] Π² Π²Π΅ΠΊΡ‚ΠΎΡ€ [латСкс] {\ mathbf {\ overset {\ to} {F}}} _ {k} [/ latex] Π² Π½Π°ΠΏΡ€Π°Π²Π»Π΅Π½ΠΈΠΈ ΠΏΡ€ΠΎΡ‚ΠΈΠ² часовой стрСлки (рисунок).ΠŸΡ€ΠΈ использовании рисунка ΠΌΡ‹ часто вычисляСм Π²Π΅Π»ΠΈΡ‡ΠΈΠ½Ρƒ крутящСго ΠΌΠΎΠΌΠ΅Π½Ρ‚Π° ΠΈ Π½Π°Π·Π½Π°Ρ‡Π°Π΅ΠΌ Π΅Π³ΠΎ Π·Π½Π°Ρ‡Π΅Π½ΠΈΠ΅ Π»ΠΈΠ±ΠΎ ΠΏΠΎΠ»ΠΎΠΆΠΈΡ‚Π΅Π»ΡŒΠ½Ρ‹ΠΌ [латСкс] (+) [/ латСкс], Π»ΠΈΠ±ΠΎ ΠΎΡ‚Ρ€ΠΈΡ†Π°Ρ‚Π΅Π»ΡŒΠ½Ρ‹ΠΌ [латСкс] (-), [/ латСкс] Π² зависимости ΠΎΡ‚ направлСния вращСния, Π²Ρ‹Π·Π²Π°Π½Π½ΠΎΠ³ΠΎ этим. Ρ‚ΠΎΠ»ΡŒΠΊΠΎ крутящий ΠΌΠΎΠΌΠ΅Π½Ρ‚. На рисункС чистый крутящий ΠΌΠΎΠΌΠ΅Π½Ρ‚ прСдставляСт собой сумму Ρ‡Π»Π΅Π½ΠΎΠ², ΠΊΠ°ΠΆΠ΄Ρ‹ΠΉ Ρ‡Π»Π΅Π½ вычисляСтся ΠΏΠΎ рисунку, ΠΈ ΠΊΠ°ΠΆΠ΄Ρ‹ΠΉ Ρ‡Π»Π΅Π½ Π΄ΠΎΠ»ΠΆΠ΅Π½ ΠΈΠΌΠ΅Ρ‚ΡŒ ΠΏΡ€Π°Π²ΠΈΠ»ΡŒΠ½ΠΎΠ΅ Π·Π½Π°Ρ‡Π΅Π½ΠΈΠ΅ . Π’ΠΎΡ‡Π½ΠΎ Ρ‚Π°ΠΊ ΠΆΠ΅ Π½Π° рисункС ΠΌΡ‹ Π½Π°Π·Π½Π°Ρ‡Π°Π΅ΠΌ Π·Π½Π°ΠΊ [latex] + [/ latex] ΠΊΠΎΠΌΠΏΠΎΠ½Π΅Π½Ρ‚Π°ΠΌ Π² Π½Π°ΠΏΡ€Π°Π²Π»Π΅Π½ΠΈΠΈ [latex] + [/ latex] x ΠΈ Π·Π½Π°ΠΊ [latex] — [/ latex] ΠΊΠΎΠΌΠΏΠΎΠ½Π΅Π½Ρ‚Π°ΠΌ Π² [латСкс] — [/ латСкс] Ρ… -Π½Π°ΠΏΡ€Π°Π²Π»Π΅Π½ΠΈΠ΅.Π­Ρ‚ΠΎ ΠΆΠ΅ ΠΏΡ€Π°Π²ΠΈΠ»ΠΎ Π΄ΠΎΠ»ΠΆΠ½ΠΎ ΠΏΠΎΡΠ»Π΅Π΄ΠΎΠ²Π°Ρ‚Π΅Π»ΡŒΠ½ΠΎ ΡΠΎΠ±Π»ΡŽΠ΄Π°Ρ‚ΡŒΡΡ Π½Π° рисункС ΠΏΡ€ΠΈ вычислСнии ΠΊΠΎΠΌΠΏΠΎΠ½Π΅Π½Ρ‚ΠΎΠ² силы ΠΏΠΎ оси y .

Рис. 12.2 ΠšΡ€ΡƒΡ‚ΡΡ‰ΠΈΠΉ ΠΌΠΎΠΌΠ΅Π½Ρ‚ силы: (a) Когда крутящий ΠΌΠΎΠΌΠ΅Π½Ρ‚ силы Π²Ρ‹Π·Ρ‹Π²Π°Π΅Ρ‚ Π²Ρ€Π°Ρ‰Π΅Π½ΠΈΠ΅ ΠΏΡ€ΠΎΡ‚ΠΈΠ² часовой стрСлки Π²ΠΎΠΊΡ€ΡƒΠ³ оси вращСния, ΠΌΡ‹ Π³ΠΎΠ²ΠΎΡ€ΠΈΠΌ, Ρ‡Ρ‚ΠΎ Π΅Π³ΠΎ Π½Π°ΠΏΡ€Π°Π²Π»Π΅Π½ΠΈΠ΅ ΠΏΠΎΠ»ΠΎΠΆΠΈΡ‚Π΅Π»ΡŒΠ½ΠΎΠ΅, Ρ‡Ρ‚ΠΎ ΠΎΠ·Π½Π°Ρ‡Π°Π΅Ρ‚, Ρ‡Ρ‚ΠΎ Π²Π΅ΠΊΡ‚ΠΎΡ€ крутящСго ΠΌΠΎΠΌΠ΅Π½Ρ‚Π° ΠΏΠ°Ρ€Π°Π»Π»Π΅Π»Π΅Π½ оси вращСния. (b) Когда крутящий ΠΌΠΎΠΌΠ΅Π½Ρ‚ силы Π²Ρ‹Π·Ρ‹Π²Π°Π΅Ρ‚ Π²Ρ€Π°Ρ‰Π΅Π½ΠΈΠ΅ Π²ΠΎΠΊΡ€ΡƒΠ³ оси ΠΏΠΎ часовой стрСлкС, ΠΌΡ‹ Π³ΠΎΠ²ΠΎΡ€ΠΈΠΌ, Ρ‡Ρ‚ΠΎ Π΅Π³ΠΎ Π½Π°ΠΏΡ€Π°Π²Π»Π΅Π½ΠΈΠ΅ ΠΎΡ‚Ρ€ΠΈΡ†Π°Ρ‚Π΅Π»ΡŒΠ½ΠΎΠ΅, Ρ‡Ρ‚ΠΎ ΠΎΠ·Π½Π°Ρ‡Π°Π΅Ρ‚, Ρ‡Ρ‚ΠΎ Π²Π΅ΠΊΡ‚ΠΎΡ€ крутящСго ΠΌΠΎΠΌΠ΅Π½Ρ‚Π° Π°Π½Ρ‚ΠΈΠΏΠ°Ρ€Π°Π»Π»Π΅Π»Π΅Π½ оси вращСния.

Π’ΠΎ ΠΌΠ½ΠΎΠ³ΠΈΡ… ситуациях равновСсия ΠΎΠ΄Π½ΠΎΠΉ ΠΈΠ· сил, Π΄Π΅ΠΉΡΡ‚Π²ΡƒΡŽΡ‰ΠΈΡ… Π½Π° Ρ‚Π΅Π»ΠΎ, являСтся Π΅Π³ΠΎ вСс. На Π΄ΠΈΠ°Π³Ρ€Π°ΠΌΠΌΠ°Ρ… свободного Ρ‚Π΅Π»Π° Π²Π΅ΠΊΡ‚ΠΎΡ€ вСса привязан ΠΊ Ρ†Π΅Π½Ρ‚Ρ€Ρƒ тяТСсти Ρ‚Π΅Π»Π°. Для всСх практичСских Ρ†Π΅Π»Π΅ΠΉ Ρ†Π΅Π½Ρ‚Ρ€ тяТСсти ΠΈΠ΄Π΅Π½Ρ‚ΠΈΡ‡Π΅Π½ Ρ†Π΅Π½Ρ‚Ρ€Ρƒ масс, ΠΊΠ°ΠΊ Π²Ρ‹ ΡƒΠ·Π½Π°Π»ΠΈ ΠΈΠ· статСй Β«Π›ΠΈΠ½Π΅ΠΉΠ½Ρ‹ΠΉ ΠΈΠΌΠΏΡƒΠ»ΡŒΡΒ» ΠΈ «БтолкновСния» ΠΎ Π»ΠΈΠ½Π΅ΠΉΠ½ΠΎΠΌ ΠΈΠΌΠΏΡƒΠ»ΡŒΡΠ΅ ΠΈ столкновСниях. Волько Π² Ρ‚Π΅Ρ… случаях, ΠΊΠΎΠ³Π΄Π° Ρ‚Π΅Π»ΠΎ ΠΈΠΌΠ΅Π΅Ρ‚ Π±ΠΎΠ»ΡŒΡˆΡƒΡŽ ΠΏΡ€ΠΎΡΡ‚Ρ€Π°Π½ΡΡ‚Π²Π΅Π½Π½ΡƒΡŽ ΠΏΡ€ΠΎΡ‚ΡΠΆΠ΅Π½Π½ΠΎΡΡ‚ΡŒ, Ρ‚Π°ΠΊ Ρ‡Ρ‚ΠΎ Π³Ρ€Π°Π²ΠΈΡ‚Π°Ρ†ΠΈΠΎΠ½Π½ΠΎΠ΅ ΠΏΠΎΠ»Π΅ Π½Π΅ΠΎΠ΄Π½ΠΎΡ€ΠΎΠ΄Π½ΠΎ ΠΏΠΎ всСму Π΅Π³ΠΎ ΠΎΠ±ΡŠΠ΅ΠΌΡƒ, Ρ†Π΅Π½Ρ‚Ρ€ тяТСсти ΠΈ Ρ†Π΅Π½Ρ‚Ρ€ масс располоТСны Π² Ρ€Π°Π·Π½Ρ‹Ρ… Ρ‚ΠΎΡ‡ΠΊΠ°Ρ….{2}. [/ Latex] Π’ этих ситуациях Ρ†Π΅Π½Ρ‚Ρ€ тяТСсти совпадаСт с Ρ†Π΅Π½Ρ‚Ρ€ΠΎΠΌ масс. ΠŸΠΎΡΡ‚ΠΎΠΌΡƒ Π² этой Π³Π»Π°Π²Π΅ ΠΌΡ‹ ΠΈΡΠΏΠΎΠ»ΡŒΠ·ΡƒΠ΅ΠΌ Ρ†Π΅Π½Ρ‚Ρ€ масс (CM) ΠΊΠ°ΠΊ Ρ‚ΠΎΡ‡ΠΊΡƒ, ΠΊ ΠΊΠΎΡ‚ΠΎΡ€ΠΎΠΉ ΠΏΡ€ΠΈΠΊΡ€Π΅ΠΏΠ»Π΅Π½ Π²Π΅ΠΊΡ‚ΠΎΡ€ вСса. Напомним, Ρ‡Ρ‚ΠΎ ЦМ ΠΈΠΌΠ΅Π΅Ρ‚ особый физичСский смысл: ΠΊΠΎΠ³Π΄Π° внСшняя сила ΠΏΡ€ΠΈΠ»ΠΎΠΆΠ΅Π½Π° ΠΊ Ρ‚Π΅Π»Ρƒ Ρ‚ΠΎΡ‡Π½ΠΎ Π² Π΅Π³ΠΎ ЦМ, Ρ‚Π΅Π»ΠΎ Π² Ρ†Π΅Π»ΠΎΠΌ ΡΠΎΠ²Π΅Ρ€ΡˆΠ°Π΅Ρ‚ ΠΏΠΎΡΡ‚ΡƒΠΏΠ°Ρ‚Π΅Π»ΡŒΠ½ΠΎΠ΅ Π΄Π²ΠΈΠΆΠ΅Π½ΠΈΠ΅, ΠΈ такая сила Π½Π΅ Π²Ρ‹Π·Ρ‹Π²Π°Π΅Ρ‚ вращСния.

Когда ЦМ располоТСн Π²Π½Π΅ оси вращСния, Π½Π° ΠΎΠ±ΡŠΠ΅ΠΊΡ‚Π΅ Π²ΠΎΠ·Π½ΠΈΠΊΠ°Π΅Ρ‚ чистый Π³Ρ€Π°Π²ΠΈΡ‚Π°Ρ†ΠΈΠΎΠ½Π½Ρ‹ΠΉ ΠΌΠΎΠΌΠ΅Π½Ρ‚ .Π“Ρ€Π°Π²ΠΈΡ‚Π°Ρ†ΠΈΠΎΠ½Π½Ρ‹ΠΉ ΠΌΠΎΠΌΠ΅Π½Ρ‚ — это крутящий ΠΌΠΎΠΌΠ΅Π½Ρ‚, Π²Ρ‹Π·Π²Π°Π½Π½Ρ‹ΠΉ вСсом. Π­Ρ‚ΠΎΡ‚ Π³Ρ€Π°Π²ΠΈΡ‚Π°Ρ†ΠΈΠΎΠ½Π½Ρ‹ΠΉ ΠΌΠΎΠΌΠ΅Π½Ρ‚ ΠΌΠΎΠΆΠ΅Ρ‚ Π²Ρ€Π°Ρ‰Π°Ρ‚ΡŒ ΠΎΠ±ΡŠΠ΅ΠΊΡ‚, Ссли Π½Π΅Ρ‚ ΠΎΠΏΠΎΡ€Ρ‹ для Π΅Π³ΠΎ балансировки. Π’Π΅Π»ΠΈΡ‡ΠΈΠ½Π° Π³Ρ€Π°Π²ΠΈΡ‚Π°Ρ†ΠΈΠΎΠ½Π½ΠΎΠ³ΠΎ ΠΌΠΎΠΌΠ΅Π½Ρ‚Π° зависит ΠΎΡ‚ Ρ‚ΠΎΠ³ΠΎ, ΠΊΠ°ΠΊ Π΄Π°Π»Π΅ΠΊΠΎ ΠΎΡ‚ оси находится ЦМ. НапримСр, Π² случаС самосвала (рисунок) ось ΠΏΠΎΠ²ΠΎΡ€ΠΎΡ‚Π° располоТСна Π½Π° Π»ΠΈΠ½ΠΈΠΈ, Π³Π΄Π΅ ΡˆΠΈΠ½Ρ‹ ΡΠΎΠΏΡ€ΠΈΠΊΠ°ΡΠ°ΡŽΡ‚ΡΡ с ΠΏΠΎΠ²Π΅Ρ€Ρ…Π½ΠΎΡΡ‚ΡŒΡŽ Π΄ΠΎΡ€ΠΎΠ³ΠΈ. Если CM располоТСн высоко Π½Π°Π΄ ΠΏΠΎΠ²Π΅Ρ€Ρ…Π½ΠΎΡΡ‚ΡŒΡŽ Π΄ΠΎΡ€ΠΎΠ³ΠΈ, Π³Ρ€Π°Π²ΠΈΡ‚Π°Ρ†ΠΈΠΎΠ½Π½Ρ‹ΠΉ ΠΌΠΎΠΌΠ΅Π½Ρ‚ ΠΌΠΎΠΆΠ΅Ρ‚ Π±Ρ‹Ρ‚ΡŒ достаточно большим, Ρ‡Ρ‚ΠΎΠ±Ρ‹ ΠΏΠ΅Ρ€Π΅Π²Π΅Ρ€Π½ΡƒΡ‚ΡŒ Π³Ρ€ΡƒΠ·ΠΎΠ²ΠΈΠΊ.Π›Π΅Π³ΠΊΠΎΠ²Ρ‹Π΅ Π°Π²Ρ‚ΠΎΠΌΠΎΠ±ΠΈΠ»ΠΈ с Π½ΠΈΠ·ΠΊΠΎ располоТСнной КМ, Π±Π»ΠΈΠ·ΠΊΠΎΠΉ ΠΊ Ρ‚Ρ€ΠΎΡ‚ΡƒΠ°Ρ€Ρƒ, Π±ΠΎΠ»Π΅Π΅ устойчивы ΠΊ ΠΎΠΏΡ€ΠΎΠΊΠΈΠ΄Ρ‹Π²Π°Π½ΠΈΡŽ, Ρ‡Π΅ΠΌ Π³Ρ€ΡƒΠ·ΠΎΠ²ΠΈΠΊΠΈ.

Рис. 12.3 РаспрСдСлСниС массы влияСт Π½Π° ΠΏΠΎΠ»ΠΎΠΆΠ΅Π½ΠΈΠ΅ Ρ†Π΅Π½Ρ‚Ρ€Π° масс (CM), ΠΊ ΠΊΠΎΡ‚ΠΎΡ€ΠΎΠΌΡƒ ΠΏΡ€ΠΈΠΊΡ€Π΅ΠΏΠ»Π΅Π½ Π²Π΅ΠΊΡ‚ΠΎΡ€ вСса [latex] \ mathbf {\ overset {\ to} {w}} [/ latex]. Если Ρ†Π΅Π½Ρ‚Ρ€ тяТСсти находится Π² Π·ΠΎΠ½Π΅ ΠΎΠΏΠΎΡ€Ρ‹, ΠΏΠΎΠ³Ρ€ΡƒΠ·Ρ‡ΠΈΠΊ возвращаСтся Π² исходноС ΠΏΠΎΠ»ΠΎΠΆΠ΅Π½ΠΈΠ΅ послС опрокидывания [см. Π›Π΅Π²ΡƒΡŽ панСль Π² (b)]. Но Ссли Ρ†Π΅Π½Ρ‚Ρ€ тяТСсти находится Π·Π° ΠΏΡ€Π΅Π΄Π΅Π»Π°ΠΌΠΈ Π·ΠΎΠ½Ρ‹ ΠΎΠΏΠΎΡ€Ρ‹, Π³Ρ€ΡƒΠ·ΠΎΠ²ΠΈΠΊ пСрСвСрнСтся [см. ΠŸΡ€Π°Π²ΡƒΡŽ панСль Π² (b)].Оба транспортных срСдства Π² (b) находятся Π²Π½Π΅ равновСсия. ΠžΠ±Ρ€Π°Ρ‚ΠΈΡ‚Π΅ Π²Π½ΠΈΠΌΠ°Π½ΠΈΠ΅, Ρ‡Ρ‚ΠΎ Π°Π²Ρ‚ΠΎΠΌΠΎΠ±ΠΈΠ»ΡŒ Π½Π° (Π°) находится Π² равновСсии: Π½ΠΈΠ·ΠΊΠΎΠ΅ располоТСниС Ρ†Π΅Π½Ρ‚Ρ€Π° тяТСсти затрудняСт ΠΎΠΏΡ€ΠΎΠΊΠΈΠ΄Ρ‹Π²Π°Π½ΠΈΠ΅.

ΠŸΡ€ΠΈΠΌΠ΅Ρ€

Π¦Π΅Π½Ρ‚Ρ€ тяТСсти автомобиля

Π›Π΅Π³ΠΊΠΎΠ²ΠΎΠΉ Π°Π²Ρ‚ΠΎΠΌΠΎΠ±ΠΈΠ»ΡŒ с колСсной Π±Π°Π·ΠΎΠΉ 2,5 ΠΌ ΠΈΠΌΠ΅Π΅Ρ‚ 52% вСса Π½Π° ΠΏΠ΅Ρ€Π΅Π΄Π½ΠΈΡ… колСсах Π½Π° Ρ€ΠΎΠ²Π½ΠΎΠΉ повСрхности, ΠΊΠ°ΠΊ ΠΏΠΎΠΊΠ°Π·Π°Π½ΠΎ Π½Π° рисункС. Π“Π΄Π΅ находится ЦМ этого автомобиля ΠΏΠΎ ΠΎΡ‚Π½ΠΎΡˆΠ΅Π½ΠΈΡŽ ΠΊ Π·Π°Π΄Π½Π΅ΠΉ оси?

Рисунок 12.4 РаспрСдСлСниС вСса ΠΌΠ΅ΠΆΠ΄Ρƒ осями автомобиля.Π“Π΄Π΅ находится Ρ†Π΅Π½Ρ‚Ρ€ тяТСсти?
БтратСгия

ВСс автомобиля w Π½Π°ΠΌ нСизвСстСн. ВсС, Ρ‡Ρ‚ΠΎ ΠΌΡ‹ Π·Π½Π°Π΅ΠΌ, это Ρ‚ΠΎ, Ρ‡Ρ‚ΠΎ ΠΊΠΎΠ³Π΄Π° Π°Π²Ρ‚ΠΎΠΌΠΎΠ±ΠΈΠ»ΡŒ стоит Π½Π° Ρ€ΠΎΠ²Π½ΠΎΠΉ повСрхности, 0,52 w Π΄Π°Π²ΠΈΡ‚ Π½Π° ΠΏΠΎΠ²Π΅Ρ€Ρ…Π½ΠΎΡΡ‚ΡŒ Π² Ρ‚ΠΎΡ‡ΠΊΠ°Ρ… ΠΊΠΎΠ½Ρ‚Π°ΠΊΡ‚Π° ΠΏΠ΅Ρ€Π΅Π΄Π½ΠΈΡ… колСс, Π° 0,48 w Ρ‚ΠΎΠ»ΠΊΠ°Π΅Ρ‚ Π²Π½ΠΈΠ· Π½Π° ΠΏΠΎΠ²Π΅Ρ€Ρ…Π½ΠΎΡΡ‚ΡŒ Π² Ρ‚ΠΎΡ‡ΠΊΠ°Ρ… ΠΊΠΎΠ½Ρ‚Π°ΠΊΡ‚Π° Π·Π°Π΄Π½ΠΈΡ… колСс. ΠšΡ€ΠΎΠΌΠ΅ Ρ‚ΠΎΠ³ΠΎ, Ρ‚ΠΎΡ‡ΠΊΠΈ ΠΊΠΎΠ½Ρ‚Π°ΠΊΡ‚Π° ΠΎΡ‚Π΄Π΅Π»Π΅Π½Ρ‹ Π΄Ρ€ΡƒΠ³ ΠΎΡ‚ Π΄Ρ€ΡƒΠ³Π° расстояниСм [латСкс] d = 2,5 \, \ text {m}. [/ Latex] Π’ этих Ρ‚ΠΎΡ‡ΠΊΠ°Ρ… ΠΊΠΎΠ½Ρ‚Π°ΠΊΡ‚Π° Π°Π²Ρ‚ΠΎΠΌΠΎΠ±ΠΈΠ»ΡŒ испытываСт Π½ΠΎΡ€ΠΌΠ°Π»ΡŒΠ½Ρ‹Π΅ силы Ρ€Π΅Π°ΠΊΡ†ΠΈΠΈ Π²Π΅Π»ΠΈΡ‡ΠΈΠ½ΠΎΠΉ [латСкс] {F} _ {\ text {F}} = 0.52 Π’Ρ‚ [/ латСкс] ΠΈ [латСкс] {F} _ {\ text {R}} = 0,48 Π’Ρ‚ [/ латСкс] Π½Π° ΠΏΠ΅Ρ€Π΅Π΄Π½Π΅ΠΉ ΠΈ Π·Π°Π΄Π½Π΅ΠΉ осях соотвСтствСнно. ΠœΡ‹ Ρ‚Π°ΠΊΠΆΠ΅ Π·Π½Π°Π΅ΠΌ, Ρ‡Ρ‚ΠΎ Π°Π²Ρ‚ΠΎΠΌΠΎΠ±ΠΈΠ»ΡŒ являСтся ΠΏΡ€ΠΈΠΌΠ΅Ρ€ΠΎΠΌ Ρ‚Π²Π΅Ρ€Π΄ΠΎΠ³ΠΎ Ρ‚Π΅Π»Π°, находящСгося Π² равновСсии, вСсь вСс ΠΊΠΎΡ‚ΠΎΡ€ΠΎΠ³ΠΎ w дСйствуСт Π½Π° Π΅Π³ΠΎ ЦМ. CM находится Π³Π΄Π΅-Ρ‚ΠΎ ΠΌΠ΅ΠΆΠ΄Ρƒ Ρ‚ΠΎΡ‡ΠΊΠ°ΠΌΠΈ, Π³Π΄Π΅ Π΄Π΅ΠΉΡΡ‚Π²ΡƒΡŽΡ‚ Π½ΠΎΡ€ΠΌΠ°Π»ΡŒΠ½Ρ‹Π΅ силы Ρ€Π΅Π°ΠΊΡ†ΠΈΠΈ, Π³Π΄Π΅-Ρ‚ΠΎ Π½Π° расстоянии x ΠΎΡ‚ Ρ‚ΠΎΡ‡ΠΊΠΈ, Π³Π΄Π΅ дСйствуСт [латСкс] {F} _ {R} [/ латСкс]. Наша Π·Π°Π΄Π°Ρ‡Π° Π½Π°ΠΉΡ‚ΠΈ Ρ… . Π’Π°ΠΊΠΈΠΌ ΠΎΠ±Ρ€Π°Π·ΠΎΠΌ, ΠΌΡ‹ ΠΈΠ΄Π΅Π½Ρ‚ΠΈΡ„ΠΈΡ†ΠΈΡ€ΡƒΠ΅ΠΌ Ρ‚Ρ€ΠΈ силы, Π΄Π΅ΠΉΡΡ‚Π²ΡƒΡŽΡ‰ΠΈΠ΅ Π½Π° ΠΊΡƒΠ·ΠΎΠ² (Π°Π²Ρ‚ΠΎΠΌΠΎΠ±ΠΈΠ»ΡŒ), ΠΈ ΠΌΠΎΠΆΠ΅ΠΌ Π½Π°Ρ€ΠΈΡΠΎΠ²Π°Ρ‚ΡŒ Π΄ΠΈΠ°Π³Ρ€Π°ΠΌΠΌΡƒ свободного Ρ‚Π΅Π»Π° для Ρ€Π°ΡΡˆΠΈΡ€Π΅Π½Π½ΠΎΠ³ΠΎ Ρ‚Π²Π΅Ρ€Π΄ΠΎΠ³ΠΎ Ρ‚Π΅Π»Π°, ΠΊΠ°ΠΊ ΠΏΠΎΠΊΠ°Π·Π°Π½ΠΎ Π½Π° рисункС.

Рис. 12.5 На Π΄ΠΈΠ°Π³Ρ€Π°ΠΌΠΌΠ΅ свободного Ρ‚Π΅Π»Π° для автомобиля Ρ‡Π΅Ρ‚ΠΊΠΎ ΡƒΠΊΠ°Π·Π°Π½Ρ‹ Π²Π΅ΠΊΡ‚ΠΎΡ€Ρ‹ сил, Π΄Π΅ΠΉΡΡ‚Π²ΡƒΡŽΡ‰ΠΈΡ… Π½Π° Π°Π²Ρ‚ΠΎΠΌΠΎΠ±ΠΈΠ»ΡŒ, ΠΈ расстояния Π΄ΠΎ Ρ†Π΅Π½Ρ‚Ρ€Π° масс (CM). Когда CM Π²Ρ‹Π±Ρ€Π°Π½ Π² качСствС Ρ‚ΠΎΡ‡ΠΊΠΈ ΠΏΠΎΠ²ΠΎΡ€ΠΎΡ‚Π°, эти расстояния ΠΏΡ€Π΅Π΄ΡΡ‚Π°Π²Π»ΡΡŽΡ‚ собой ΠΏΠ»Π΅Ρ‡ΠΈ Ρ€Ρ‹Ρ‡Π°Π³Π° Π½ΠΎΡ€ΠΌΠ°Π»ΡŒΠ½Ρ‹Ρ… сил Ρ€Π΅Π°ΠΊΡ†ΠΈΠΈ. ΠžΠ±Ρ€Π°Ρ‚ΠΈΡ‚Π΅ Π²Π½ΠΈΠΌΠ°Π½ΠΈΠ΅, Ρ‡Ρ‚ΠΎ Π²Π΅Π»ΠΈΡ‡ΠΈΠ½Ρ‹ Π²Π΅ΠΊΡ‚ΠΎΡ€ΠΎΠ² ΠΈ Ρ€Ρ‹Ρ‡Π°Π³ΠΈ Π½Π΅ Π½ΡƒΠΆΠ½ΠΎ Ρ€ΠΈΡΠΎΠ²Π°Ρ‚ΡŒ Π² ΠΌΠ°ΡΡˆΡ‚Π°Π±Π΅, Π½ΠΎ всС Ρ€Π΅Π»Π΅Π²Π°Π½Ρ‚Π½Ρ‹Π΅ Π²Π΅Π»ΠΈΡ‡ΠΈΠ½Ρ‹ Π΄ΠΎΠ»ΠΆΠ½Ρ‹ Π±Ρ‹Ρ‚ΡŒ Ρ‡Π΅Ρ‚ΠΊΠΎ ΠΎΠ±ΠΎΠ·Π½Π°Ρ‡Π΅Π½Ρ‹.

ΠœΡ‹ ΠΏΠΎΡ‡Ρ‚ΠΈ Π³ΠΎΡ‚ΠΎΠ²Ρ‹ Π·Π°ΠΏΠΈΡΠ°Ρ‚ΡŒ условия равновСсия с рисунка ΠΏΠΎ рисунок для автомобиля, Π½ΠΎ сначала ΠΌΡ‹ Π΄ΠΎΠ»ΠΆΠ½Ρ‹ ΠΎΠΏΡ€Π΅Π΄Π΅Π»ΠΈΡ‚ΡŒΡΡ с систСмой отсчСта.ΠŸΡ€Π΅Π΄ΠΏΠΎΠ»ΠΎΠΆΠΈΠΌ, ΠΌΡ‹ Π²Ρ‹Π±Ρ€Π°Π»ΠΈ ось x ΠΏΠΎ Π΄Π»ΠΈΠ½Π΅ ΠΊΠ°Π±ΠΈΠ½Ρ‹, ось y — Π²Π΅Ρ€Ρ‚ΠΈΠΊΠ°Π»ΡŒΠ½ΡƒΡŽ, Π° ось z — пСрпСндикулярно этой плоскости xy . ΠŸΡ€ΠΈ Ρ‚Π°ΠΊΠΎΠΌ Π²Ρ‹Π±ΠΎΡ€Π΅ Π½Π°ΠΌ Π½ΡƒΠΆΠ½ΠΎ Π½Π°ΠΏΠΈΡΠ°Ρ‚ΡŒ Ρ‚ΠΎΠ»ΡŒΠΊΠΎ Figure ΠΈ Figure, ΠΏΠΎΡ‚ΠΎΠΌΡƒ Ρ‡Ρ‚ΠΎ всС ΠΊΠΎΠΌΠΏΠΎΠ½Π΅Π½Ρ‚Ρ‹ y тоТдСствСнно Ρ€Π°Π²Π½Ρ‹ Π½ΡƒΠ»ΡŽ. Π’Π΅ΠΏΠ΅Ρ€ΡŒ Π½Π°ΠΌ Π½ΡƒΠΆΠ½ΠΎ ΠΎΠΏΡ€Π΅Π΄Π΅Π»ΠΈΡ‚ΡŒΡΡ с располоТСниСм Ρ‚ΠΎΡ‡ΠΊΠΈ ΠΏΠΎΠ²ΠΎΡ€ΠΎΡ‚Π°. ΠœΡ‹ ΠΌΠΎΠΆΠ΅ΠΌ Π²Ρ‹Π±Ρ€Π°Ρ‚ΡŒ Π»ΡŽΠ±ΡƒΡŽ Ρ‚ΠΎΡ‡ΠΊΡƒ Π² качСствС мСстополоТСния оси вращСния ( z -ось). ΠŸΡ€Π΅Π΄ΠΏΠΎΠ»ΠΎΠΆΠΈΠΌ, ΠΌΡ‹ размСстили ось вращСния Π½Π° CM, ΠΊΠ°ΠΊ ΠΏΠΎΠΊΠ°Π·Π°Π½ΠΎ Π½Π° схСмС свободного Ρ‚Π΅Π»Π° для автомобиля.На этом этапС ΠΌΡ‹ Π³ΠΎΡ‚ΠΎΠ²Ρ‹ Π½Π°ΠΏΠΈΡΠ°Ρ‚ΡŒ условия равновСсия для автомобиля.

РСшСниС

КаТдоС условиС равновСсия содСрТит Ρ‚ΠΎΠ»ΡŒΠΊΠΎ Ρ‚Ρ€ΠΈ Ρ‡Π»Π΅Π½Π°, ΠΏΠΎΡ‚ΠΎΠΌΡƒ Ρ‡Ρ‚ΠΎ Π½Π° Π°Π²Ρ‚ΠΎΠΌΠΎΠ±ΠΈΠ»ΡŒ Π΄Π΅ΠΉΡΡ‚Π²ΡƒΡŽΡ‚ силы [latex] N = 3 [/ latex]. ΠŸΠ΅Ρ€Π²ΠΎΠ΅ условиС равновСсия, рисунок, читаСтся ΠΊΠ°ΠΊ

.

[латСкс] + {F} _ {\ text {F}} — w + {F} _ {\ text {R}} = 0. [/ Latex]

Π­Ρ‚ΠΎ условиС Ρ‚Ρ€ΠΈΠ²ΠΈΠ°Π»ΡŒΠ½ΠΎ выполняСтся, ΠΏΠΎΡ‚ΠΎΠΌΡƒ Ρ‡Ρ‚ΠΎ, ΠΊΠΎΠ³Π΄Π° ΠΌΡ‹ подставляСм Π΄Π°Π½Π½Ρ‹Π΅, Figure становится [latex] + 0,52w-w + 0,48w = 0. [/ Latex] Π’Ρ‚ΠΎΡ€ΠΎΠ΅ условиС равновСсия, Figure, читаСтся ΠΊΠ°ΠΊ

[латСкс] {\ tau} _ {\ text {F}} + {\ tau} _ {w} + {\ tau} _ {\ text {R}} = 0 [/ латСкс]

Π³Π΄Π΅ [латСкс] {\ tau} _ {\ text {F}} [/ latex] — это крутящий ΠΌΠΎΠΌΠ΅Π½Ρ‚ силы [латСкс] {F} _ {\ text {F}}, \, {\ tau} _ {w } [/ latex] — это Π³Ρ€Π°Π²ΠΈΡ‚Π°Ρ†ΠΈΠΎΠ½Π½Ρ‹ΠΉ ΠΌΠΎΠΌΠ΅Π½Ρ‚ силы w , Π° [latex] {\ tau} _ {\ text {R}} [/ latex] — это крутящий ΠΌΠΎΠΌΠ΅Π½Ρ‚ силы [latex] {F} _ {\ text {Π }}.[/ latex] Когда ось располоТСна Π² CM, Π³Ρ€Π°Π²ΠΈΡ‚Π°Ρ†ΠΈΠΎΠ½Π½Ρ‹ΠΉ ΠΌΠΎΠΌΠ΅Π½Ρ‚ ΠΈΠ΄Π΅Π½Ρ‚ΠΈΡ‡Π½ΠΎ Π½ΡƒΠ»ΡŽ, ΠΏΠΎΡ‚ΠΎΠΌΡƒ Ρ‡Ρ‚ΠΎ ΠΏΠ»Π΅Ρ‡ΠΎ Ρ€Ρ‹Ρ‡Π°Π³Π° вСса ΠΎΡ‚Π½ΠΎΡΠΈΡ‚Π΅Π»ΡŒΠ½ΠΎ оси, которая ΠΏΡ€ΠΎΡ…ΠΎΠ΄ΠΈΡ‚ Ρ‡Π΅Ρ€Π΅Π· CM, Ρ€Π°Π²Π½ΠΎ Π½ΡƒΠ»ΡŽ. Π›ΠΈΠ½ΠΈΠΈ дСйствия ΠΎΠ±Π΅ΠΈΡ… Π½ΠΎΡ€ΠΌΠ°Π»ΡŒΠ½Ρ‹Ρ… сил Ρ€Π΅Π°ΠΊΡ†ΠΈΠΈ пСрпСндикулярны ΠΏΠ»Π΅Ρ‡Π°ΠΌ ΠΈΡ… Ρ€Ρ‹Ρ‡Π°Π³ΠΎΠ², поэтому Π½Π° рисункС ΠΌΡ‹ ΠΈΠΌΠ΅Π΅ΠΌ [latex] | \, \ text {sin} \, \ theta | = 1 [/ latex] для ΠΎΠ±Π΅ΠΈΡ… сил. Из Π΄ΠΈΠ°Π³Ρ€Π°ΠΌΠΌΡ‹ свободного Ρ‚Π΅Π»Π° ΠΌΡ‹ Ρ‡ΠΈΡ‚Π°Π΅ΠΌ, Ρ‡Ρ‚ΠΎ крутящий ΠΌΠΎΠΌΠ΅Π½Ρ‚ [латСкс] {\ tau} _ {\ text {F}} [/ latex] Π²Ρ‹Π·Ρ‹Π²Π°Π΅Ρ‚ Π²Ρ€Π°Ρ‰Π΅Π½ΠΈΠ΅ ΠΏΠΎ часовой стрСлкС Π²ΠΎΠΊΡ€ΡƒΠ³ оси Π² CM, поэтому Π΅Π³ΠΎ смысл ΠΎΡ‚Ρ€ΠΈΡ†Π°Ρ‚Π΅Π»ΡŒΠ½Ρ‹ΠΉ; ΠΈ крутящий ΠΌΠΎΠΌΠ΅Π½Ρ‚ [latex] {\ tau} _ {\ text {R}} [/ latex] Π²Ρ‹Π·Ρ‹Π²Π°Π΅Ρ‚ Π²Ρ€Π°Ρ‰Π΅Π½ΠΈΠ΅ ΠΏΡ€ΠΎΡ‚ΠΈΠ² часовой стрСлки Π²ΠΎΠΊΡ€ΡƒΠ³ оси CM, поэтому Π΅Π³ΠΎ Π½Π°ΠΏΡ€Π°Π²Π»Π΅Π½ΠΈΠ΅ ΠΏΠΎΠ»ΠΎΠΆΠΈΡ‚Π΅Π»ΡŒΠ½ΠΎΠ΅.ИмСя эту ΠΈΠ½Ρ„ΠΎΡ€ΠΌΠ°Ρ†ΠΈΡŽ, запишСм Π²Ρ‚ΠΎΡ€ΠΎΠ΅ условиС равновСсия ΠΊΠ°ΠΊ

[латСкс] \ text {-} {r} _ {\ text {F}} {F} _ {\ text {F}} + {r} _ {\ text {R}} {F} _ {\ text {R}} = 0. [/ Latex]

Π‘ ΠΏΠΎΠΌΠΎΡ‰ΡŒΡŽ Π΄ΠΈΠ°Π³Ρ€Π°ΠΌΠΌΡ‹ свободного Ρ‚Π΅Π»Π° ΠΌΡ‹ опрСдСляСм Π²Π΅Π»ΠΈΡ‡ΠΈΠ½Ρ‹ силы [латСкс] {F} _ {\ text {R}} = 0,48w [/ латСкс] ΠΈ [латСкс] {F} _ {\ text {F }} = 0,52w, [/ latex] ΠΈ ΡΠΎΠΎΡ‚Π²Π΅Ρ‚ΡΡ‚Π²ΡƒΡŽΡ‰ΠΈΠ΅ ΠΈΠΌ Ρ€Ρ‹Ρ‡Π°Π³ΠΈ [латСкс] {r} _ {\ text {R}} = x [/ latex] ΠΈ [latex] {r} _ {\ text {F}} = dx. [/ latex] Π’Π΅ΠΏΠ΅Ρ€ΡŒ ΠΌΡ‹ ΠΌΠΎΠΆΠ΅ΠΌ Π·Π°ΠΏΠΈΡΠ°Ρ‚ΡŒ Π²Ρ‚ΠΎΡ€ΠΎΠ΅ условиС равновСсия, рисунок, явно Ρ‡Π΅Ρ€Π΅Π· нСизвСстноС расстояниС x :

[латСкс] -0.52 (d-x) w + 0,48xw = 0. [/ ЛатСкс]

Π—Π΄Π΅ΡΡŒ вСс w отмСняСтся, ΠΈ ΠΌΡ‹ ΠΌΠΎΠΆΠ΅ΠΌ Ρ€Π΅ΡˆΠΈΡ‚ΡŒ ΡƒΡ€Π°Π²Π½Π΅Π½ΠΈΠ΅ для нСизвСстного полоТСния x CM. ΠžΡ‚Π²Π΅Ρ‚: [латСкс] x = 0,52d = 0,52 (2,5 \, \ text {m}) = 1,3 \, \ text {m} \ text {.} [/ Latex]

РСшСниС

Π’Ρ‹Π±ΠΎΡ€ ΡˆΠ°Ρ€Π½ΠΈΡ€Π° Π² ΠΏΠΎΠ»ΠΎΠΆΠ΅Π½ΠΈΠΈ ΠΏΠ΅Ρ€Π΅Π΄Π½Π΅ΠΉ оси Π½Π΅ мСняСт Ρ€Π΅Π·ΡƒΠ»ΡŒΡ‚Π°Ρ‚Π°. Π‘Ρ…Π΅ΠΌΠ° свободного Ρ‚Π΅Π»Π° для этой Ρ‚ΠΎΡ‡ΠΊΠΈ ΠΏΠΎΠ²ΠΎΡ€ΠΎΡ‚Π° прСдставлСна ​​на рисункС. Для этого Π²Ρ‹Π±ΠΎΡ€Π° Ρ‚ΠΎΡ‡ΠΊΠΈ ΠΏΠΎΠ²ΠΎΡ€ΠΎΡ‚Π° Π²Ρ‚ΠΎΡ€ΠΎΠ΅ условиС равновСсия Ρ€Π°Π²Π½ΠΎ

.

[латСкс] \ text {-} {r} _ {w} w + {r} _ {\ text {R}} {F} _ {\ text {R}} = 0.[/ латСкс]

Когда ΠΌΡ‹ подставляСм числа, ΡƒΠΊΠ°Π·Π°Π½Π½Ρ‹Π΅ Π½Π° Π΄ΠΈΠ°Π³Ρ€Π°ΠΌΠΌΠ΅, ΠΏΠΎΠ»ΡƒΡ‡Π°Π΅ΠΌ

[латСкс] \ text {-} (d-x) w + 0,48dw = 0. [/ Latex]

ΠžΡ‚Π²Π΅Ρ‚, ΠΏΠΎΠ»ΡƒΡ‡Π΅Π½Π½Ρ‹ΠΉ ΠΏΡƒΡ‚Π΅ΠΌ Ρ€Π΅ΡˆΠ΅Π½ΠΈΡ рисунка, ΠΎΠΏΡΡ‚ΡŒ ΠΆΠ΅, [латСкс] x = 0,52d = 1,3 \, \ text {m}. [/ Latex]

Рисунок 12.6 ЭквивалСнтная Π΄ΠΈΠ°Π³Ρ€Π°ΠΌΠΌΠ° свободного Ρ‚Π΅Π»Π° для автомобиля; Ρ‚ΠΎΡ‡ΠΊΠ° ΠΏΠΎΠ²ΠΎΡ€ΠΎΡ‚Π° Ρ‡Π΅Ρ‚ΠΊΠΎ ΠΎΠ±ΠΎΠ·Π½Π°Ρ‡Π΅Π½Π°.
Π—Π½Π°Ρ‡Π΅Π½ΠΈΠ΅

Π­Ρ‚ΠΎΡ‚ ΠΏΡ€ΠΈΠΌΠ΅Ρ€ ΠΏΠΎΠΊΠ°Π·Ρ‹Π²Π°Π΅Ρ‚, Ρ‡Ρ‚ΠΎ ΠΏΡ€ΠΈ Ρ€Π΅ΡˆΠ΅Π½ΠΈΠΈ Π·Π°Π΄Π°Ρ‡ статичСского равновСсия ΠΌΡ‹ ΠΌΠΎΠΆΠ΅ΠΌ Π²Ρ‹Π±Ρ€Π°Ρ‚ΡŒ Ρ‚ΠΎΡ‡ΠΊΡƒ ΠΏΠΎΠ²ΠΎΡ€ΠΎΡ‚Π°.Для Ρ€Π°Π·Π»ΠΈΡ‡Π½Ρ‹Ρ… Π²Π°Ρ€ΠΈΠ°Π½Ρ‚ΠΎΠ² Π²Ρ‹Π±ΠΎΡ€Π° Ρ‚ΠΎΡ‡ΠΊΠΈ ΠΏΠΎΠ²ΠΎΡ€ΠΎΡ‚Π° Ρƒ нас Π΅ΡΡ‚ΡŒ Ρ€Π°Π·Π½Ρ‹Π΅ Π½Π°Π±ΠΎΡ€Ρ‹ условий равновСсия, ΠΊΠΎΡ‚ΠΎΡ€Ρ‹Π΅ Π½Π΅ΠΎΠ±Ρ…ΠΎΠ΄ΠΈΠΌΠΎ Ρ€Π΅ΡˆΠΈΡ‚ΡŒ. Однако любой Π²Ρ‹Π±ΠΎΡ€ ΠΏΡ€ΠΈΠ²ΠΎΠ΄ΠΈΡ‚ ΠΊ ΠΎΠ΄Π½ΠΎΠΌΡƒ ΠΈ Ρ‚ΠΎΠΌΡƒ ΠΆΠ΅ Ρ€Π΅ΡˆΠ΅Π½ΠΈΡŽ ΠΏΡ€ΠΎΠ±Π»Π΅ΠΌΡ‹.

ΠŸΡ€ΠΎΠ²Π΅Ρ€ΡŒΡ‚Π΅ своС ΠΏΠΎΠ½ΠΈΠΌΠ°Π½ΠΈΠ΅

Π Π΅ΡˆΠΈΡ‚Π΅ Ρ„ΠΈΠ³ΡƒΡ€Ρƒ, Π²Ρ‹Π±Ρ€Π°Π² ΡˆΠ°Ρ€Π½ΠΈΡ€ Π² мСстС располоТСния Π·Π°Π΄Π½Π΅ΠΉ оси.

ΠŸΠΎΠΊΠ°Π·Π°Ρ‚ΡŒ Ρ€Π΅ΡˆΠ΅Π½ΠΈΠ΅

[латСкс] x = 1,3 \, \ text {m} [/ latex]

ΠŸΡ€ΠΎΠ²Π΅Ρ€ΡŒΡ‚Π΅ своС ΠΏΠΎΠ½ΠΈΠΌΠ°Π½ΠΈΠ΅

ΠžΠ±ΡŠΡΡΠ½ΠΈΡ‚Π΅, какая ΠΈΠ· ΡΠ»Π΅Π΄ΡƒΡŽΡ‰ΠΈΡ… ситуаций удовлСтворяСт ΠΎΠ±ΠΎΠΈΠΌ условиям равновСсия: (Π°) тСннисный мяч, ΠΊΠΎΡ‚ΠΎΡ€Ρ‹ΠΉ Π½Π΅ вращаСтся ΠΏΡ€ΠΈ Π΄Π²ΠΈΠΆΠ΅Π½ΠΈΠΈ Π² Π²ΠΎΠ·Π΄ΡƒΡ…Π΅; (Π±) ΠΏΠ΅Π»ΠΈΠΊΠ°Π½, ΠΊΠΎΡ‚ΠΎΡ€Ρ‹ΠΉ ΠΏΠ°Ρ€ΠΈΡ‚ Π² Π²ΠΎΠ·Π΄ΡƒΡ…Π΅ с постоянной ΡΠΊΠΎΡ€ΠΎΡΡ‚ΡŒΡŽ Π½Π° ΠΎΠ΄Π½ΠΎΠΉ высотС; ΠΈΠ»ΠΈ (c) ΠΊΠΎΠ»Π΅Π½Ρ‡Π°Ρ‚Ρ‹ΠΉ Π²Π°Π» двигатСля ΠΏΡ€ΠΈΠΏΠ°Ρ€ΠΊΠΎΠ²Π°Π½Π½ΠΎΠ³ΠΎ автомобиля.

ΠžΡΠΎΠ±Ρ‹ΠΉ случай статичСского равновСсия Π²ΠΎΠ·Π½ΠΈΠΊΠ°Π΅Ρ‚, ΠΊΠΎΠ³Π΄Π° всС внСшниС силы Π½Π° ΠΎΠ±ΡŠΠ΅ΠΊΡ‚ Π΄Π΅ΠΉΡΡ‚Π²ΡƒΡŽΡ‚ Π½Π° оси вращСния ΠΈΠ»ΠΈ вдоль Π½Π΅Π΅, ΠΈΠ»ΠΈ ΠΊΠΎΠ³Π΄Π° пространствСнноС протяТСниС ΠΎΠ±ΡŠΠ΅ΠΊΡ‚Π° ΠΌΠΎΠΆΠ½ΠΎ Π½Π΅ ΠΏΡ€ΠΈΠ½ΠΈΠΌΠ°Ρ‚ΡŒ Π²ΠΎ Π²Π½ΠΈΠΌΠ°Π½ΠΈΠ΅. Π’ Ρ‚Π°ΠΊΠΎΠΌ случаС ΠΎΠ±ΡŠΠ΅ΠΊΡ‚ ΠΌΠΎΠΆΠ½ΠΎ эффСктивно Ρ€Π°ΡΡΠΌΠ°Ρ‚Ρ€ΠΈΠ²Π°Ρ‚ΡŒ ΠΊΠ°ΠΊ Ρ‚ΠΎΡ‡Π΅Ρ‡Π½ΡƒΡŽ массу. Π’ этом частном случаС Π½Π°ΠΌ Π½Π΅ Π½ΡƒΠΆΠ½ΠΎ Π±Π΅ΡΠΏΠΎΠΊΠΎΠΈΡ‚ΡŒΡΡ ΠΎ Π²Ρ‚ΠΎΡ€ΠΎΠΌ условии равновСсия, рис., ΠŸΠΎΡ‚ΠΎΠΌΡƒ Ρ‡Ρ‚ΠΎ всС крутящиС ΠΌΠΎΠΌΠ΅Π½Ρ‚Ρ‹ тоТдСствСнно Ρ€Π°Π²Π½Ρ‹ Π½ΡƒΠ»ΡŽ, Π° ΠΏΠ΅Ρ€Π²ΠΎΠ΅ условиС равновСсия (для сил) являСтся СдинствСнным условиСм, ΠΊΠΎΡ‚ΠΎΡ€ΠΎΠ΅ Π΄ΠΎΠ»ΠΆΠ½ΠΎ Π²Ρ‹ΠΏΠΎΠ»Π½ΡΡ‚ΡŒΡΡ. Π”ΠΈΠ°Π³Ρ€Π°ΠΌΠΌΠ° свободного Ρ‚Π΅Π»Π° ΠΈ стратСгия Ρ€Π΅ΡˆΠ΅Π½ΠΈΡ Π·Π°Π΄Π°Ρ‡ для этого особого случая Π±Ρ‹Π»ΠΈ ΠΈΠ·Π»ΠΎΠΆΠ΅Π½Ρ‹ Π² Β«Π—Π°ΠΊΠΎΠ½Π°Ρ… ΠΡŒΡŽΡ‚ΠΎΠ½Π°Β» ΠΈ Β«ΠŸΡ€ΠΈΠ»ΠΎΠΆΠ΅Π½ΠΈΡΡ… Π·Π°ΠΊΠΎΠ½ΠΎΠ² ΠΡŒΡŽΡ‚ΠΎΠ½Π°Β».Π’ ΡΠ»Π΅Π΄ΡƒΡŽΡ‰Π΅ΠΌ ΠΏΡ€ΠΈΠΌΠ΅Ρ€Π΅ Π²Ρ‹ ΡƒΠ²ΠΈΠ΄ΠΈΡ‚Π΅ Ρ‚ΠΈΠΏΠΈΡ‡Π½ΡƒΡŽ ΡΠΈΡ‚ΡƒΠ°Ρ†ΠΈΡŽ равновСсия, Π²ΠΊΠ»ΡŽΡ‡Π°ΡŽΡ‰ΡƒΡŽ Ρ‚ΠΎΠ»ΡŒΠΊΠΎ ΠΏΠ΅Ρ€Π²ΠΎΠ΅ условиС равновСсия.

ΠŸΡ€ΠΈΠΌΠ΅Ρ€

Π Π°Π·Ρ€Ρ‹Π²Π½ΠΎΠ΅ напряТСниС

МалСнькая ΠΊΠ°ΡΡ‚Ρ€ΡŽΠ»Ρ массой 42,0 Π³ поддСрТиваСтся двумя струнами, ΠΊΠ°ΠΊ ΠΏΠΎΠΊΠ°Π·Π°Π½ΠΎ Π½Π° рисункС. МаксимальноС натяТСниС, ΠΊΠΎΡ‚ΠΎΡ€ΠΎΠ΅ ΠΌΠΎΠΆΠ΅Ρ‚ Π²Ρ‹Π΄Π΅Ρ€ΠΆΠ°Ρ‚ΡŒ струна, составляСт 2,80 Н. Масса постСпСнно добавляСтся ΠΊ Ρ‡Π°ΡˆΠ΅, ΠΏΠΎΠΊΠ° ΠΎΠ΄Π½Π° ΠΈΠ· струн Π½Π΅ сломаСтся. Какая это струна? ΠšΠ°ΠΊΡƒΡŽ массу Π½ΡƒΠΆΠ½ΠΎ Π΄ΠΎΠ±Π°Π²ΠΈΡ‚ΡŒ, Ρ‡Ρ‚ΠΎΠ±Ρ‹ это ΠΏΡ€ΠΎΠΈΠ·ΠΎΡˆΠ»ΠΎ?

Рисунок 12.7 Масса добавляСтся Π² ΠΊΠ°ΡΡ‚Ρ€ΡŽΠ»ΡŽ постСпСнно, ΠΏΠΎΠΊΠ° ΠΎΠ΄Π½Π° ΠΈΠ· струн Π½Π΅ Π»ΠΎΠΏΠ½Π΅Ρ‚.
БтратСгия

Π­Ρ‚Π° мСханичСская систСма, состоящая ΠΈΠ· струн, масс ΠΈ сковороды, находится Π² статичСском равновСсии. Π’ частности, ΡƒΠ·Π΅Π», ΠΊΠΎΡ‚ΠΎΡ€Ρ‹ΠΉ привязываСт струны ΠΊ ΠΊΠ°ΡΡ‚Ρ€ΡŽΠ»Π΅, находится Π² статичСском равновСсии. Π£Π·Π΅Π» ΠΌΠΎΠΆΠ½ΠΎ Ρ€Π°ΡΡΠΌΠ°Ρ‚Ρ€ΠΈΠ²Π°Ρ‚ΡŒ ΠΊΠ°ΠΊ Ρ‚ΠΎΡ‡ΠΊΡƒ; ΡΠ»Π΅Π΄ΠΎΠ²Π°Ρ‚Π΅Π»ΡŒΠ½ΠΎ, Π½Π°ΠΌ Π½ΡƒΠΆΠ½ΠΎ Ρ‚ΠΎΠ»ΡŒΠΊΠΎ ΠΏΠ΅Ρ€Π²ΠΎΠ΅ условиС равновСсия. Π’Ρ€ΠΈ силы, тянущиС ΡƒΠ·Π΅Π», — это натяТСниС [латСкс] {\ mathbf {\ overset {\ to} {T}}} _ {1} [/ latex] Π² 5,0-сантимСтровой струнС, натяТСниС [латСкс] {\ mathbf {\ overset {\ to} {T}}} _ {2} [/ latex] Π² строкС Π΄Π»ΠΈΠ½ΠΎΠΉ 10,0 см ΠΈ вСс [латСкс] \ mathbf {\ overset {\ to} {w}} [/ latex ] сковороды, ΡƒΠ΄Π΅Ρ€ΠΆΠΈΠ²Π°ΡŽΡ‰Π΅ΠΉ массы.ΠœΡ‹ ΠΏΡ€ΠΈΠ½ΠΈΠΌΠ°Π΅ΠΌ ΠΏΡ€ΡΠΌΠΎΡƒΠ³ΠΎΠ»ΡŒΠ½ΡƒΡŽ систСму ΠΊΠΎΠΎΡ€Π΄ΠΈΠ½Π°Ρ‚ с осью y , Π½Π°ΠΏΡ€Π°Π²Π»Π΅Π½Π½ΠΎΠΉ ΠΏΡ€ΠΎΡ‚ΠΈΠ²ΠΎΠΏΠΎΠ»ΠΎΠΆΠ½ΠΎ Π½Π°ΠΏΡ€Π°Π²Π»Π΅Π½ΠΈΡŽ силы тяТСсти, ΠΈ рисуСм Π΄ΠΈΠ°Π³Ρ€Π°ΠΌΠΌΡƒ свободного Ρ‚Π΅Π»Π° для ΡƒΠ·Π»Π° (см. Рисунок). Π§Ρ‚ΠΎΠ±Ρ‹ Π½Π°ΠΉΡ‚ΠΈ ΠΊΠΎΠΌΠΏΠΎΠ½Π΅Π½Ρ‚Ρ‹ натяТСния, ΠΌΡ‹ Π΄ΠΎΠ»ΠΆΠ½Ρ‹ ΠΎΠΏΡ€Π΅Π΄Π΅Π»ΠΈΡ‚ΡŒ ΡƒΠ³Π»Ρ‹ направлСния [латСкс] {\ alpha} _ {1} [/ latex] ΠΈ [latex] {\ alpha} _ {2} [/ latex], ΠΊΠΎΡ‚ΠΎΡ€Ρ‹Π΅ ΠΎΠ±Ρ€Π°Π·ΡƒΡŽΡ‚ струны с Π³ΠΎΡ€ΠΈΠ·ΠΎΠ½Ρ‚Π°Π»ΡŒΠ½Ρ‹ΠΌ Π½Π°ΠΏΡ€Π°Π²Π»Π΅Π½ΠΈΠ΅ оси x . Как Π²Ρ‹ ΠΌΠΎΠΆΠ΅Ρ‚Π΅ Π²ΠΈΠ΄Π΅Ρ‚ΡŒ Π½Π° рисункС, струны ΡΠΎΡΡ‚Π°Π²Π»ΡΡŽΡ‚ Π΄Π²Π΅ стороны ΠΏΡ€ΡΠΌΠΎΡƒΠ³ΠΎΠ»ΡŒΠ½ΠΎΠ³ΠΎ Ρ‚Ρ€Π΅ΡƒΠ³ΠΎΠ»ΡŒΠ½ΠΈΠΊΠ°. ΠœΡ‹ ΠΌΠΎΠΆΠ΅ΠΌ ΠΈΡΠΏΠΎΠ»ΡŒΠ·ΠΎΠ²Π°Ρ‚ΡŒ Ρ‚Π΅ΠΎΡ€Π΅ΠΌΡƒ ΠŸΠΈΡ„Π°Π³ΠΎΡ€Π°, Ρ‡Ρ‚ΠΎΠ±Ρ‹ Ρ€Π΅ΡˆΠΈΡ‚ΡŒ этот Ρ‚Ρ€Π΅ΡƒΠ³ΠΎΠ»ΡŒΠ½ΠΈΠΊ, ΠΏΠΎΠΊΠ°Π·Π°Π½Π½Ρ‹ΠΉ Π½Π° рисункС, ΠΈ Π½Π°ΠΉΡ‚ΠΈ синус ΠΈ косинус ΡƒΠ³Π»ΠΎΠ² [latex] {\ alpha} _ {1} [/ latex] ΠΈ [latex] {\ alpha} _ {2 }.[/ latex] Π—Π°Ρ‚Π΅ΠΌ ΠΌΡ‹ ΠΌΠΎΠΆΠ΅ΠΌ Ρ€Π°Π·Π»ΠΎΠΆΠΈΡ‚ΡŒ натяТСния Π½Π° ΠΈΡ… ΠΏΡ€ΡΠΌΠΎΡƒΠ³ΠΎΠ»ΡŒΠ½Ρ‹Π΅ ΡΠΎΡΡ‚Π°Π²Π»ΡΡŽΡ‰ΠΈΠ΅, ΠΏΠΎΠ΄ΡΡ‚Π°Π²ΠΈΡ‚ΡŒ Π² ΠΏΠ΅Ρ€Π²ΠΎΠ΅ условиС равновСсия (рисунок ΠΈ рисунок) ΠΈ Π½Π°ΠΉΡ‚ΠΈ натяТСния Π² струнах. ΠŸΠ΅Ρ€Π²ΠΎΠΉ порвСтся струна с большим натяТСниСм.

Рисунок 12.8 Π‘Ρ…Π΅ΠΌΠ° свободного Ρ‚Π΅Π»Π° для ΡƒΠ·Π»Π° Π½Π° рисункС.

РСшСниС

ВСс w , Π½Π°Ρ‚ΡΠ³ΠΈΠ²Π°ΡŽΡ‰ΠΈΠΉ ΡƒΠ·Π΅Π», обусловлСн массой M ΠΊΠ°ΡΡ‚Ρ€ΡŽΠ»ΠΈ ΠΈ массой m , Π΄ΠΎΠ±Π°Π²Π»Π΅Π½Π½ΠΎΠΉ ΠΊ ΠΊΠ°ΡΡ‚Ρ€ΡŽΠ»Π΅, ΠΈΠ»ΠΈ [латСкс] w = (M + m) g.[/ latex] Π‘ ΠΏΠΎΠΌΠΎΡ‰ΡŒΡŽ Π΄ΠΈΠ°Π³Ρ€Π°ΠΌΠΌΡ‹ свободного Ρ‚Π΅Π»Π° Π½Π° рисункС, ΠΌΡ‹ ΠΌΠΎΠΆΠ΅ΠΌ ΡƒΡΡ‚Π°Π½ΠΎΠ²ΠΈΡ‚ΡŒ условия равновСсия для ΡƒΠ·Π»Π°:

[латСкс] \ begin {array} {ccccc} \ text {Π² Π½Π°ΠΏΡ€Π°Π²Π»Π΅Π½ΠΈΠΈ} \, x \ text {-direction,} \ hfill & & \ hfill \ text {-} {T} _ {1x} + {T} _ {2x} & = \ hfill & 0 \ hfill \\ \ text {Π² Π½Π°ΠΏΡ€Π°Π²Π»Π΅Π½ΠΈΠΈ} \, y \ text {,} \ hfill & & \ hfill \ text {+} {T} _ {1y} + { T} _ {2y} -w & = \ hfill & 0. \ hfill \ end {array} [/ latex]

Из Π΄ΠΈΠ°Π³Ρ€Π°ΠΌΠΌΡ‹ свободного Ρ‚Π΅Π»Π°, Π²Π΅Π»ΠΈΡ‡ΠΈΠ½Ρ‹ ΠΊΠΎΠΌΠΏΠΎΠ½Π΅Π½Ρ‚ΠΎΠ² Π² этих уравнСниях Ρ€Π°Π²Π½Ρ‹

.

[латСкс] \ begin {array} {ccc} {T} _ {1x} = {T} _ {1} \ text {cos} \, {\ alpha} _ {1} = {T} _ {1} \ text {/} \ sqrt {5}, \ hfill & & {T} _ {1y} = {T} _ {1} \ text {sin} \, {\ alpha} _ {1} = 2 {T} _ {1} \ text {/} \ sqrt {5} \ hfill \\ {T} _ {2x} = {T} _ {2} \ text {cos} \, {\ alpha} _ {2} = 2 {T} _ {2} \ text {/} \ sqrt {5}, \ hfill & & {T} _ {2y} = {T} _ {2} \ text {sin} \, {\ alpha} _ { 2} = {T} _ {2} \ text {/} \ sqrt {5}.\ hfill \ end {array} [/ latex]

ΠŸΠΎΠ΄ΡΡ‚Π°Π²Π»ΡΠ΅ΠΌ эти ΠΊΠΎΠΌΠΏΠΎΠ½Π΅Π½Ρ‚Ρ‹ Π² условия равновСсия ΠΈ ΡƒΠΏΡ€ΠΎΡ‰Π°Π΅ΠΌ. Π—Π°Ρ‚Π΅ΠΌ ΠΌΡ‹ ΠΏΠΎΠ»ΡƒΡ‡Π°Π΅ΠΌ Π΄Π²Π° уравнСния равновСсия для натяТСний:

[латСкс] \ begin {array} {ccccc} \ text {in} \, x \ text {-direction,} \ hfill & & \ hfill {T} _ {1} & = \ hfill & 2 {T} _ {2} \ hfill \\ \ text {in} \, y \ text {-direction,} \ hfill & & \ hfill \ frac {2 {T} _ {1}} {\ sqrt {5}} + \ frac {{T} _ {2}} {\ sqrt {5}} & = \ hfill & (M + m) g. \ Hfill \ end {array} [/ latex]

Π£Ρ€Π°Π²Π½Π΅Π½ΠΈΠ΅ равновСсия для направлСния x Π³ΠΎΠ²ΠΎΡ€ΠΈΡ‚ Π½Π°ΠΌ, Ρ‡Ρ‚ΠΎ натяТСниС [латСкс] {T} _ {1} [/ латСкс] Π² 5.0-сантимСтровая струна Π²Π΄Π²ΠΎΠ΅ ΠΏΡ€Π΅Π²Ρ‹ΡˆΠ°Π΅Ρ‚ натяТСниС [латСкс] {T} _ {2} [/ latex] Π² 10-сантимСтровой струнС. Π’Π°ΠΊΠΈΠΌ ΠΎΠ±Ρ€Π°Π·ΠΎΠΌ, Π±ΠΎΠ»Π΅Π΅ короткая струна порвСтся. Когда ΠΌΡ‹ ΠΈΡΠΏΠΎΠ»ΡŒΠ·ΡƒΠ΅ΠΌ ΠΏΠ΅Ρ€Π²ΠΎΠ΅ ΡƒΡ€Π°Π²Π½Π΅Π½ΠΈΠ΅, Ρ‡Ρ‚ΠΎΠ±Ρ‹ ΠΈΡΠΊΠ»ΡŽΡ‡ΠΈΡ‚ΡŒ [латСкс] {T} _ {2} [/ latex] ΠΈΠ· Π²Ρ‚ΠΎΡ€ΠΎΠ³ΠΎ уравнСния, ΠΌΡ‹ ΠΏΠΎΠ»ΡƒΡ‡Π°Π΅ΠΌ ΡΠΎΠΎΡ‚Π½ΠΎΡˆΠ΅Π½ΠΈΠ΅ ΠΌΠ΅ΠΆΠ΄Ρƒ массой [латСкс] ΠΌ [/ латСкс] Π½Π° сковородС ΠΈ натяТСниСм [латСкс ] {T} _ {1} [/ latex] Π² Π±ΠΎΠ»Π΅Π΅ ΠΊΠΎΡ€ΠΎΡ‚ΠΊΠΎΠΉ строкС:

[латСкс] 2,5 {T} _ {1} \ text {/} \ sqrt {5} = (M + m) Π³. [/ латСкс]

Π‘Ρ‚Ρ€ΡƒΠ½Π° разрываСтся, ΠΊΠΎΠ³Π΄Π° натяТСниС достигаСт критичСского значСния [латСкс] {T} _ {1} = 2.{2}} — 0,042 \, \ text {kg} = 0,277 \, \ text {kg} = 277,0 \, \ text {g.} [/ Latex]

Π—Π½Π°Ρ‡Π΅Π½ΠΈΠ΅

ΠŸΡ€Π΅Π΄ΠΏΠΎΠ»ΠΎΠΆΠΈΠΌ, Ρ‡Ρ‚ΠΎ мСханичСская систСма, рассматриваСмая Π² этом ΠΏΡ€ΠΈΠΌΠ΅Ρ€Π΅, ΠΏΡ€ΠΈΠΊΡ€Π΅ΠΏΠ»Π΅Π½Π° ΠΊ ΠΏΠΎΡ‚ΠΎΠ»ΠΊΡƒ Π²Π½ΡƒΡ‚Ρ€ΠΈ Π»ΠΈΡ„Ρ‚Π°, ΠΏΠΎΠ΄Π½ΠΈΠΌΠ°ΡŽΡ‰Π΅Π³ΠΎΡΡ Π²Π²Π΅Ρ€Ρ…. Пока Π»ΠΈΡ„Ρ‚ двиТСтся Π²Π²Π΅Ρ€Ρ… с постоянной ΡΠΊΠΎΡ€ΠΎΡΡ‚ΡŒΡŽ, Ρ€Π΅Π·ΡƒΠ»ΡŒΡ‚Π°Ρ‚ остаСтся Π½Π΅ΠΈΠ·ΠΌΠ΅Π½Π½Ρ‹ΠΌ, ΠΏΠΎΡ‚ΠΎΠΌΡƒ Ρ‡Ρ‚ΠΎ вСс [латСкс] w [/ латСкс] Π½Π΅ мСняСтся. Если Π»ΠΈΡ„Ρ‚ двиТСтся Π²Π²Π΅Ρ€Ρ… с ускорСниСм, критичСская масса мСньшС, ΠΏΠΎΡ‚ΠΎΠΌΡƒ Ρ‡Ρ‚ΠΎ вСс [латСкса] M + m [/ latex] становится большС Π½Π° каТущийся вСс ΠΈΠ·-Π·Π° ускорСния Π»ΠΈΡ„Ρ‚Π°.Π’Π΅ΠΌ Π½Π΅ ΠΌΠ΅Π½Π΅Π΅, Π²ΠΎ всСх случаях Π±ΠΎΠ»Π΅Π΅ короткая струна рвСтся ΠΏΠ΅Ρ€Π²ΠΎΠΉ.

Π‘Π²ΠΎΠ΄ΠΊΠ°

  • Π’Π΅Π»ΠΎ находится Π² равновСсии, ΠΊΠΎΠ³Π΄Π° ΠΎΠ½ΠΎ остаСтся Π»ΠΈΠ±ΠΎ Π² Ρ€Π°Π²Π½ΠΎΠΌΠ΅Ρ€Π½ΠΎΠΌ Π΄Π²ΠΈΠΆΠ΅Π½ΠΈΠΈ (ΠΏΠΎΡΡ‚ΡƒΠΏΠ°Ρ‚Π΅Π»ΡŒΠ½ΠΎΠΌ ΠΈ Π²Ρ€Π°Ρ‰Π°Ρ‚Π΅Π»ΡŒΠ½ΠΎΠΌ), Π»ΠΈΠ±ΠΎ Π² состоянии покоя. Когда Ρ‚Π΅Π»ΠΎ Π² Π²Ρ‹Π±Ρ€Π°Π½Π½ΠΎΠΉ ΠΈΠ½Π΅Ρ€Ρ†ΠΈΠ°Π»ΡŒΠ½ΠΎΠΉ систСмС отсчСта Π½Π΅ вращаСтся ΠΈ Π½Π΅ двиТСтся Π² ΠΏΠΎΡΡ‚ΡƒΠΏΠ°Ρ‚Π΅Π»ΡŒΠ½ΠΎΠΌ Π΄Π²ΠΈΠΆΠ΅Π½ΠΈΠΈ, ΠΌΡ‹ Π³ΠΎΠ²ΠΎΡ€ΠΈΠΌ, Ρ‡Ρ‚ΠΎ Ρ‚Π΅Π»ΠΎ находится Π² статичСском равновСсии Π² этой систСмС отсчСта.
  • Условия равновСсия Ρ‚Ρ€Π΅Π±ΡƒΡŽΡ‚, Ρ‡Ρ‚ΠΎΠ±Ρ‹ сумма всСх Π²Π½Π΅ΡˆΠ½ΠΈΡ… сил, Π΄Π΅ΠΉΡΡ‚Π²ΡƒΡŽΡ‰ΠΈΡ… Π½Π° Ρ‚Π΅Π»ΠΎ, Π±Ρ‹Π»Π° Ρ€Π°Π²Π½Π° Π½ΡƒΠ»ΡŽ (ΠΏΠ΅Ρ€Π²ΠΎΠ΅ условиС равновСсия), Π° сумма всСх Π²Π½Π΅ΡˆΠ½ΠΈΡ… ΠΌΠΎΠΌΠ΅Π½Ρ‚ΠΎΠ² ΠΎΡ‚ Π²Π½Π΅ΡˆΠ½ΠΈΡ… сил Π±Ρ‹Π»Π° Ρ€Π°Π²Π½Π° Π½ΡƒΠ»ΡŽ (Π²Ρ‚ΠΎΡ€ΠΎΠ΅ условиС равновСсия).Π­Ρ‚ΠΈ Π΄Π²Π° условия Π΄ΠΎΠ»ΠΆΠ½Ρ‹ ΠΎΠ΄Π½ΠΎΠ²Ρ€Π΅ΠΌΠ΅Π½Π½ΠΎ Π²Ρ‹ΠΏΠΎΠ»Π½ΡΡ‚ΡŒΡΡ Π² состоянии равновСсия. Если ΠΎΠ΄ΠΈΠ½ ΠΈΠ· Π½ΠΈΡ… Π½Π΅ ΡƒΠ΄ΠΎΠ²Π»Π΅Ρ‚Π²ΠΎΡ€Π΅Π½, Ρ‚Π΅Π»ΠΎ Π½Π΅ находится Π² равновСсии.
  • Π”ΠΈΠ°Π³Ρ€Π°ΠΌΠΌΠ° свободного Ρ‚Π΅Π»Π° для Ρ‚Π΅Π»Π° — ΠΏΠΎΠ»Π΅Π·Π½Ρ‹ΠΉ инструмСнт, ΠΊΠΎΡ‚ΠΎΡ€Ρ‹ΠΉ позволяСт Π½Π°ΠΌ ΠΏΡ€Π°Π²ΠΈΠ»ΡŒΠ½ΠΎ ΠΏΠΎΠ΄ΡΡ‡ΠΈΡ‚Π°Ρ‚ΡŒ всС Π²ΠΊΠ»Π°Π΄Ρ‹ ΠΎΡ‚ всСх Π²Π½Π΅ΡˆΠ½ΠΈΡ… сил ΠΈ ΠΌΠΎΠΌΠ΅Π½Ρ‚ΠΎΠ², Π΄Π΅ΠΉΡΡ‚Π²ΡƒΡŽΡ‰ΠΈΡ… Π½Π° Ρ‚Π΅Π»ΠΎ. Π”ΠΈΠ°Π³Ρ€Π°ΠΌΠΌΡ‹ свободного Ρ‚Π΅Π»Π° для равновСсия вытянутого Ρ‚Π²Π΅Ρ€Π΄ΠΎΠ³ΠΎ Ρ‚Π΅Π»Π° Π΄ΠΎΠ»ΠΆΠ½Ρ‹ ΡƒΠΊΠ°Π·Ρ‹Π²Π°Ρ‚ΡŒ Ρ‚ΠΎΡ‡ΠΊΡƒ ΠΏΠΎΠ²ΠΎΡ€ΠΎΡ‚Π° ΠΈ ΠΏΠ»Π΅Ρ‡ΠΈ Ρ€Ρ‹Ρ‡Π°Π³Π° Π΄Π΅ΠΉΡΡ‚Π²ΡƒΡŽΡ‰ΠΈΡ… сил ΠΏΠΎ ΠΎΡ‚Π½ΠΎΡˆΠ΅Π½ΠΈΡŽ ΠΊ оси.

ΠšΠΎΠ½Ρ†Π΅ΠΏΡ‚ΡƒΠ°Π»ΡŒΠ½Ρ‹Π΅ вопросы

Π§Ρ‚ΠΎ Π²Ρ‹ ΠΌΠΎΠΆΠ΅Ρ‚Π΅ ΡΠΊΠ°Π·Π°Ρ‚ΡŒ ΠΎ скорости двиТущСгося Ρ‚Π΅Π»Π°, находящСгося Π² динамичСском равновСсии?

ΠŸΡ€ΠΈ ΠΊΠ°ΠΊΠΈΡ… условиях Π²Ρ€Π°Ρ‰Π°ΡŽΡ‰Π΅Π΅ΡΡ Ρ‚Π΅Π»ΠΎ ΠΌΠΎΠΆΠ΅Ρ‚ Π½Π°Ρ…ΠΎΠ΄ΠΈΡ‚ΡŒΡΡ Π² равновСсии? ΠŸΡ€ΠΈΠ²Π΅Π΄ΠΈΡ‚Π΅ ΠΏΡ€ΠΈΠΌΠ΅Ρ€.

КакиС Ρ‚Ρ€ΠΈ Ρ„Π°ΠΊΡ‚ΠΎΡ€Π° Π²Π»ΠΈΡΡŽΡ‚ Π½Π° крутящий ΠΌΠΎΠΌΠ΅Π½Ρ‚, создаваСмый силой ΠΎΡ‚Π½ΠΎΡΠΈΡ‚Π΅Π»ΡŒΠ½ΠΎ ΠΎΠΏΡ€Π΅Π΄Π΅Π»Π΅Π½Π½ΠΎΠΉ Ρ‚ΠΎΡ‡ΠΊΠΈ ΠΏΠΎΠ²ΠΎΡ€ΠΎΡ‚Π°?

ΠŸΠΎΠΊΠ°Π·Π°Ρ‚ΡŒ Ρ€Π΅ΡˆΠ΅Π½ΠΈΠ΅

Π’Π΅Π»ΠΈΡ‡ΠΈΠ½Π° ΠΈ Π½Π°ΠΏΡ€Π°Π²Π»Π΅Π½ΠΈΠ΅ силы ΠΈ Π΅Π΅ ΠΏΠ»Π΅Ρ‡ΠΎ Ρ€Ρ‹Ρ‡Π°Π³Π°

ΠœΠ΅Ρ…Π°Π½ΠΈΠΊΠΈ ΠΈΠ½ΠΎΠ³Π΄Π° ΠΊΠ»Π°Π΄ΡƒΡ‚ кусок Ρ‚Ρ€ΡƒΠ±Ρ‹ Π½Π° рукоятку Π³Π°Π΅Ρ‡Π½ΠΎΠ³ΠΎ ΠΊΠ»ΡŽΡ‡Π°, ΠΏΡ‹Ρ‚Π°ΡΡΡŒ ΠΎΡ‚ΠΊΡ€ΡƒΡ‚ΠΈΡ‚ΡŒ ΠΎΡ‡Π΅Π½ΡŒ Ρ‚ΡƒΠ³ΠΎΠΉ Π±ΠΎΠ»Ρ‚. Как это ΠΏΠΎΠΌΠΎΠ³Π°Π΅Ρ‚?

Для ΡΠ»Π΅Π΄ΡƒΡŽΡ‰ΠΈΡ… Ρ‡Π΅Ρ‚Ρ‹Ρ€Π΅Ρ… Π·Π°Π΄Π°Ρ‡ ΠΎΡ†Π΅Π½ΠΈΡ‚Π΅ ΡƒΡ‚Π²Π΅Ρ€ΠΆΠ΄Π΅Π½ΠΈΠ΅ ΠΊΠ°ΠΊ истинноС ΠΈΠ»ΠΈ Π»ΠΎΠΆΠ½ΠΎΠ΅ ΠΈ ΠΎΠ±ΡŠΡΡΠ½ΠΈΡ‚Π΅ свой ΠΎΡ‚Π²Π΅Ρ‚.

Если Π½Π° ΠΎΠ±ΡŠΠ΅ΠΊΡ‚ дСйствуСт Ρ‚ΠΎΠ»ΡŒΠΊΠΎ ΠΎΠ΄Π½Π° внСшняя сила (ΠΈΠ»ΠΈ крутящий ΠΌΠΎΠΌΠ΅Π½Ρ‚), ΠΎΠ½ Π½Π΅ ΠΌΠΎΠΆΠ΅Ρ‚ Π½Π°Ρ…ΠΎΠ΄ΠΈΡ‚ΡŒΡΡ Π² равновСсии.

ΠŸΠΎΠΊΠ°Π·Π°Ρ‚ΡŒ Ρ€Π΅ΡˆΠ΅Π½ΠΈΠ΅

Π’Π΅Ρ€Π½ΠΎ, ΠΏΠΎΡΠΊΠΎΠ»ΡŒΠΊΡƒ Π² этом случаС сумма сил Π½Π΅ ΠΌΠΎΠΆΠ΅Ρ‚ Π±Ρ‹Ρ‚ΡŒ Ρ€Π°Π²Π½Π° Π½ΡƒΠ»ΡŽ, Ссли сама сила Π½Π΅ Ρ€Π°Π²Π½Π° Π½ΡƒΠ»ΡŽ.

Если ΠΎΠ±ΡŠΠ΅ΠΊΡ‚ находится Π² равновСсии, Π½Π° Π½Π΅Π³ΠΎ Π΄ΠΎΠ»ΠΆΠ½ΠΎ Π΄Π΅ΠΉΡΡ‚Π²ΠΎΠ²Π°Ρ‚ΡŒ Ρ‡Π΅Ρ‚Π½ΠΎΠ΅ число сил.

Если Π½Π° ΠΎΠ±ΡŠΠ΅ΠΊΡ‚ дСйствуСт Π½Π΅Ρ‡Π΅Ρ‚Π½ΠΎΠ΅ количСство сил, ΠΎΠ±ΡŠΠ΅ΠΊΡ‚ Π½Π΅ ΠΌΠΎΠΆΠ΅Ρ‚ Π½Π°Ρ…ΠΎΠ΄ΠΈΡ‚ΡŒΡΡ Π² равновСсии.

ΠŸΠΎΠΊΠ°Π·Π°Ρ‚ΡŒ Ρ€Π΅ΡˆΠ΅Π½ΠΈΠ΅

НСвСрно, Ссли силы Π΄ΠΎΠ±Π°Π²Π»ΡΡŽΡ‚ΡΡ ΠΊ Π½ΡƒΠ»ΡŽ ΠΊΠ°ΠΊ Π²Π΅ΠΊΡ‚ΠΎΡ€Ρ‹, Ρ‚ΠΎΠ³Π΄Π° равновСсиС ΠΌΠΎΠΆΠ΅Ρ‚ Π±Ρ‹Ρ‚ΡŒ достигнуто.

Π’Π΅Π»ΠΎ, двиТущССся ΠΏΠΎ ΠΊΡ€ΡƒΠ³Ρƒ с постоянной ΡΠΊΠΎΡ€ΠΎΡΡ‚ΡŒΡŽ, находится Π² равновСсии вращСния.

Для Ρ‡Π΅Π³ΠΎ Π½ΡƒΠΆΠ΅Π½ Π΄Π»ΠΈΠ½Π½Ρ‹ΠΉ ΠΈ Π³ΠΈΠ±ΠΊΠΈΠΉ ΡˆΠ΅ΡΡ‚, ΠΊΠΎΡ‚ΠΎΡ€Ρ‹ΠΉ пСрСносят ΠΊΠ°Π½Π°Ρ‚ΠΎΡ…ΠΎΠ΄Ρ†Ρ‹?

ΠŸΠΎΠΊΠ°Π·Π°Ρ‚ΡŒ Ρ€Π΅ΡˆΠ΅Π½ΠΈΠ΅

ΠŸΠΎΠΌΠΎΠ³Π°Π΅Ρ‚ ΠΊΠ°Π½Π°Ρ‚ΠΎΡ…ΠΎΠ΄Ρ†Ρƒ ΡΠΎΡ…Ρ€Π°Π½ΡΡ‚ΡŒ равновСсиС.

ΠŸΡ€ΠΎΠ±Π»Π΅ΠΌΡ‹

ΠŸΡ€ΠΈ затяТкС Π±ΠΎΠ»Ρ‚Π° Π²Ρ‹ Π½Π°ΠΆΠΈΠΌΠ°Π΅Ρ‚Π΅ Π³Π°Π΅Ρ‡Π½Ρ‹ΠΉ ΠΊΠ»ΡŽΡ‡ пСрпСндикулярно с усилиСм 165 Н Π½Π° расстоянии 0,140 ΠΌ ΠΎΡ‚ Ρ†Π΅Π½Ρ‚Ρ€Π° Π±ΠΎΠ»Ρ‚Π°. Какой крутящий ΠΌΠΎΠΌΠ΅Π½Ρ‚ Π²Ρ‹ ΠΏΡ€ΠΈΠΊΠ»Π°Π΄Ρ‹Π²Π°Π΅Ρ‚Π΅ ΠΎΡ‚Π½ΠΎΡΠΈΡ‚Π΅Π»ΡŒΠ½ΠΎ Ρ†Π΅Π½Ρ‚Ρ€Π° Π±ΠΎΠ»Ρ‚Π°?

ΠŸΡ€ΠΈ ΠΎΡ‚ΠΊΡ€Ρ‹Ρ‚ΠΈΠΈ Π΄Π²Π΅Ρ€ΠΈ Π²Ρ‹ Π½Π°ΠΆΠΈΠΌΠ°Π΅Ρ‚Π΅ Π½Π° Π½Π΅Π΅ пСрпСндикулярно с силой 55.0 Н Π½Π° расстоянии 0,850 ΠΌ ΠΎΡ‚ ΠΏΠ΅Ρ‚Π΅Π»ΡŒ. Какой крутящий ΠΌΠΎΠΌΠ΅Π½Ρ‚ Π²Ρ‹ ΠΏΡ€ΠΈΠΊΠ»Π°Π΄Ρ‹Π²Π°Π΅Ρ‚Π΅ ΠΊ пСтлям?

ΠŸΠΎΠΊΠ°Π·Π°Ρ‚ΡŒ Ρ€Π΅ΡˆΠ΅Π½ΠΈΠ΅

[латСкс] 46.8 \, \ text {N} \ cdot \ text {m} [/ latex]

НайдитС Π²Π΅Π»ΠΈΡ‡ΠΈΠ½Ρƒ натяТСния ΠΊΠ°ΠΆΠ΄ΠΎΠ³ΠΎ ΠΏΠΎΠ΄Π΄Π΅Ρ€ΠΆΠΈΠ²Π°ΡŽΡ‰Π΅Π³ΠΎ троса, ΠΏΠΎΠΊΠ°Π·Π°Π½Π½ΠΎΠ³ΠΎ Π½ΠΈΠΆΠ΅. Π’ ΠΊΠ°ΠΆΠ΄ΠΎΠΌ случаС вСс подвСшСнного Ρ‚Π΅Π»Π° составляСт 100,0 Н, Π° массой ΠΊΠ°Π±Π΅Π»Π΅ΠΉ ΠΌΠΎΠΆΠ½ΠΎ ΠΏΡ€Π΅Π½Π΅Π±Ρ€Π΅Ρ‡ΡŒ.

Какая сила Π΄ΠΎΠ»ΠΆΠ½Π° Π±Ρ‹Ρ‚ΡŒ ΠΏΡ€ΠΈΠ»ΠΎΠΆΠ΅Π½Π° Π² Ρ‚ΠΎΡ‡ΠΊΠ΅ P , Ρ‡Ρ‚ΠΎΠ±Ρ‹ ΡƒΠ΄Π΅Ρ€ΠΆΠΈΠ²Π°Ρ‚ΡŒ ΠΏΠΎΠΊΠ°Π·Π°Π½Π½ΡƒΡŽ ΠΊΠΎΠ½ΡΡ‚Ρ€ΡƒΠΊΡ†ΠΈΡŽ Π² равновСсии? ВСс конструкции Π½Π΅Π·Π½Π°Ρ‡ΠΈΡ‚Π΅Π»ΡŒΠ½Ρ‹ΠΉ.

МоТно Π»ΠΈ ΠΏΡ€ΠΈΠ»ΠΎΠΆΠΈΡ‚ΡŒ силу ΠΊ P , Ρ‡Ρ‚ΠΎΠ±Ρ‹ ΡƒΠ΄Π΅Ρ€ΠΆΠΈΠ²Π°Ρ‚ΡŒ ΠΏΠΎΠΊΠ°Π·Π°Π½Π½ΡƒΡŽ ΠΊΠΎΠ½ΡΡ‚Ρ€ΡƒΠΊΡ†ΠΈΡŽ Π² равновСсии? ВСс конструкции Π½Π΅Π·Π½Π°Ρ‡ΠΈΡ‚Π΅Π»ΡŒΠ½Ρ‹ΠΉ.

Π”Π²ΠΎΠ΅ Π΄Π΅Ρ‚Π΅ΠΉ Ρ‚ΠΎΠ»ΠΊΠ°ΡŽΡ‚ ΠΏΡ€ΠΎΡ‚ΠΈΠ²ΠΎΠΏΠΎΠ»ΠΎΠΆΠ½Ρ‹Π΅ стороны Π΄Π²Π΅Ρ€ΠΈ Π²ΠΎ врСмя ΠΈΠ³Ρ€Ρ‹. Оба Ρ‚ΠΎΠ»ΠΊΠ°ΡŽΡ‚ΡΡ Π³ΠΎΡ€ΠΈΠ·ΠΎΠ½Ρ‚Π°Π»ΡŒΠ½ΠΎ ΠΈ пСрпСндикулярно Π΄Π²Π΅Ρ€ΠΈ. Один Ρ€Π΅Π±Π΅Π½ΠΎΠΊ Ρ‚ΠΎΠ»ΠΊΠ°Π΅Ρ‚ с силой 17,5 Н Π½Π° расстоянии 0,600 ΠΌ ΠΎΡ‚ ΠΏΠ΅Ρ‚Π΅Π»ΡŒ, Π° Π²Ρ‚ΠΎΡ€ΠΎΠΉ Ρ€Π΅Π±Π΅Π½ΠΎΠΊ Ρ‚ΠΎΠ»ΠΊΠ°Π΅Ρ‚ Π½Π° расстоянии 0,450 ΠΌ. ΠšΠ°ΠΊΡƒΡŽ силу Π΄ΠΎΠ»ΠΆΠ΅Π½ ΠΏΡ€ΠΈΠ»ΠΎΠΆΠΈΡ‚ΡŒ Π²Ρ‚ΠΎΡ€ΠΎΠΉ Ρ€Π΅Π±Π΅Π½ΠΎΠΊ, Ρ‡Ρ‚ΠΎΠ±Ρ‹ Π΄Π²Π΅Ρ€ΡŒ Π½Π΅ двигалась? ΠŸΡ€Π΅Π΄ΠΏΠΎΠ»ΠΎΠΆΠΈΠΌ, Ρ‚Ρ€Π΅Π½ΠΈΠ΅ Π½Π΅Π·Π½Π°Ρ‡ΠΈΡ‚Π΅Π»ΡŒΠ½ΠΎ.

НСбольшой Π²Π½Π΅Π΄ΠΎΡ€ΠΎΠΆΠ½ΠΈΠΊ массой 1000 ΠΊΠ³ ΠΈΠΌΠ΅Π΅Ρ‚ ΠΊΠΎΠ»Π΅ΡΠ½ΡƒΡŽ Π±Π°Π·Ρƒ 3,0 ΠΌ. Если 60%, Ссли Π΅Π³ΠΎ вСс приходится Π½Π° ΠΏΠ΅Ρ€Π΅Π΄Π½ΠΈΠ΅ колСса, насколько Π΄Π°Π»Π΅ΠΊΠΎ ΠΏΠΎΠ·Π°Π΄ΠΈ ΠΏΠ΅Ρ€Π΅Π΄Π½ΠΈΡ… колСс находится Ρ†Π΅Π½Ρ‚Ρ€ масс Ρ„ΡƒΡ€Π³ΠΎΠ½Π°?

Π£Π½ΠΈΡ„ΠΈΡ†ΠΈΡ€ΠΎΠ²Π°Π½Π½Ρ‹Π΅ ΠΊΠ°Ρ‡Π΅Π»ΠΈ сбалансированы Π² Ρ†Π΅Π½Ρ‚Ρ€Π΅ масс, ΠΊΠ°ΠΊ ΠΏΠΎΠΊΠ°Π·Π°Π½ΠΎ Π½ΠΈΠΆΠ΅. МалСнький ΠΌΠ°Π»ΡŒΡ‡ΠΈΠΊ справа ΠΈΠΌΠ΅Π΅Ρ‚ массу 40,0 ΠΊΠ³. Какая масса Ρƒ Π΅Π³ΠΎ Π΄Ρ€ΡƒΠ³Π°?

Глоссарий

Ρ†Π΅Π½Ρ‚Ρ€ тяТСсти
Ρ‚ΠΎΡ‡ΠΊΠ°, ΠΊ ΠΊΠΎΡ‚ΠΎΡ€ΠΎΠΉ ΠΏΡ€ΠΈΠΊΡ€Π΅ΠΏΠ»Π΅Π½ Π²Π΅ΠΊΡ‚ΠΎΡ€ вСсов
равновСсиС
Π’Π΅Π»ΠΎ
находится Π² равновСсии, ΠΊΠΎΠ³Π΄Π° Π΅Π³ΠΎ Π»ΠΈΠ½Π΅ΠΉΠ½ΠΎΠ΅ ΠΈ ΡƒΠ³Π»ΠΎΠ²ΠΎΠ΅ ускорСния Ρ€Π°Π²Π½Ρ‹ Π½ΡƒΠ»ΡŽ ΠΎΡ‚Π½ΠΎΡΠΈΡ‚Π΅Π»ΡŒΠ½ΠΎ ΠΈΠ½Π΅Ρ€Ρ†ΠΈΠ°Π»ΡŒΠ½ΠΎΠΉ систСмы отсчСта
ΠΏΠ΅Ρ€Π²ΠΎΠ΅ условиС равновСсия
Π²Ρ‹Ρ€Π°ΠΆΠ°Π΅Ρ‚ ΠΏΠΎΡΡ‚ΡƒΠΏΠ°Ρ‚Π΅Π»ΡŒΠ½ΠΎΠ΅ равновСсиС; всС внСшниС силы, Π΄Π΅ΠΉΡΡ‚Π²ΡƒΡŽΡ‰ΠΈΠ΅ Π½Π° Ρ‚Π΅Π»ΠΎ, ΡƒΡ€Π°Π²Π½ΠΎΠ²Π΅ΡˆΠΈΠ²Π°ΡŽΡ‚ΡΡ ΠΈ ΠΈΡ… вСкторная сумма Ρ€Π°Π²Π½Π° Π½ΡƒΠ»ΡŽ
Π³Ρ€Π°Π²ΠΈΡ‚Π°Ρ†ΠΈΠΎΠ½Π½Ρ‹ΠΉ ΠΌΠΎΠΌΠ΅Π½Ρ‚
крутящий ΠΌΠΎΠΌΠ΅Π½Ρ‚ Π½Π° корпусС, Π²Ρ‹Π·Π²Π°Π½Π½Ρ‹ΠΉ Π΅Π³ΠΎ вСсом; Π²ΠΎΠ·Π½ΠΈΠΊΠ°Π΅Ρ‚, ΠΊΠΎΠ³Π΄Π° Ρ†Π΅Π½Ρ‚Ρ€ тяТСсти Ρ‚Π΅Π»Π° Π½Π΅ располоТСн Π½Π° оси вращСния
Π²Ρ‚ΠΎΡ€ΠΎΠ΅ состояниС равновСсия
Π²Ρ‹Ρ€Π°ΠΆΠ°Π΅Ρ‚ Π²Ρ€Π°Ρ‰Π°Ρ‚Π΅Π»ΡŒΠ½ΠΎΠ΅ равновСсиС; всС крутящиС ΠΌΠΎΠΌΠ΅Π½Ρ‚Ρ‹ ΠΎΡ‚ Π²Π½Π΅ΡˆΠ½ΠΈΡ… сил, Π΄Π΅ΠΉΡΡ‚Π²ΡƒΡŽΡ‰ΠΈΡ… Π½Π° Ρ‚Π΅Π»ΠΎ, ΡƒΡ€Π°Π²Π½ΠΎΠ²Π΅ΡˆΠΈΠ²Π°ΡŽΡ‚ΡΡ ΠΈ ΠΈΡ… вСкторная сумма Ρ€Π°Π²Π½Π° Π½ΡƒΠ»ΡŽ
статичСскоС равновСсиС
Π’Π΅Π»ΠΎ находится Π² статичСском равновСсии, ΠΊΠΎΠ³Π΄Π° ΠΎΠ½ΠΎ покоится Π² Π²Ρ‹Π±Ρ€Π°Π½Π½ΠΎΠΉ Π½Π°ΠΌΠΈ ΠΈΠ½Π΅Ρ€Ρ†ΠΈΠ°Π»ΡŒΠ½ΠΎΠΉ систСмС отсчСта

ЀСрмСрских построСк… — Ch5 Π‘Ρ‚Ρ€ΡƒΠΊΡ‚ΡƒΡ€Π½ΠΎΠ΅ ΠΏΡ€ΠΎΠ΅ΠΊΡ‚ΠΈΡ€ΠΎΠ²Π°Π½ΠΈΠ΅: ΠžΡΠ½ΠΎΠ²Π½Ρ‹Π΅ ΠΏΡ€ΠΈΠ½Ρ†ΠΈΠΏΡ‹ статики

ЀСрмСрскиС конструкции … — Ch5 Π‘Ρ‚Ρ€ΡƒΠΊΡ‚ΡƒΡ€Π½ΠΎΠ΅ ΠΏΡ€ΠΎΠ΅ΠΊΡ‚ΠΈΡ€ΠΎΠ²Π°Π½ΠΈΠ΅: ΠžΡΠ½ΠΎΠ²Π½Ρ‹Π΅ ΠΏΡ€ΠΈΠ½Ρ†ΠΈΠΏΡ‹ статики
Π‘Π°Π·ΠΎΠ²Ρ‹ΠΉ ΠΏΡ€ΠΈΠ½Ρ†ΠΈΠΏΡ‹ статики

Π‘ΠΎΠ΄Π΅Ρ€ΠΆΠ°Π½ΠΈΠ΅ Назад Π’ΠΏΠ΅Ρ€Π΅Π΄

ΠšΠΎΠ½ΡΡ‚Ρ€ΡƒΠΊΡ‚ΠΈΠ²Π½ΠΎΠ΅ ΠΏΡ€ΠΎΠ΅ΠΊΡ‚ΠΈΡ€ΠΎΠ²Π°Π½ΠΈΠ΅ связано с ΠΏΡ€ΠΎΡ‡Π½ΠΎΡΡ‚ΡŒΡŽ, ΠΆΠ΅ΡΡ‚ΠΊΠΎΡΡ‚ΡŒ ΠΈ ΡƒΡΡ‚ΠΎΠΉΡ‡ΠΈΠ²ΠΎΡΡ‚ΡŒ Ρ‚Π°ΠΊΠΈΡ… конструкций, ΠΊΠ°ΠΊ здания, ΠΏΠ»ΠΎΡ‚ΠΈΠ½Ρ‹, мосты ΠΈ ΠΏΠΎΠ΄ΠΏΠΎΡ€Π½Ρ‹Π΅ стСны.Π₯отя Π·Π΄Π°Π½ΠΈΠ΅ построСно ΠΎΡ‚ основания Π²Π²Π΅Ρ€Ρ… Π΄ΠΈΠ·Π°ΠΉΠ½Π΅Ρ€ Π΄ΠΎΠ»ΠΆΠ΅Π½ Π½Π°Ρ‡ΠΈΠ½Π°Ρ‚ΡŒ свСрху с ΠΊΡ€Ρ‹ΡˆΠ΅ΠΉ ΠΈ ΠΏΡ€ΠΎΠ΄Π²ΠΈΠ³Π°ΠΉΡ‚Π΅ΡΡŒ Π²Π½ΠΈΠ·. Π•ΡΡ‚ΡŒ Π΄Π²Π° Ρ€Π°Π·Π½Ρ‹Ρ… этапы проСктирования конструкций. Π‘Π½Π°Ρ‡Π°Π»Π° ΠΈΠ½ΠΆΠ΅Π½Π΅Ρ€-ΡΡ‚Ρ€ΠΎΠΈΡ‚Π΅Π»ΡŒ с Π΅Π³ΠΎ ΠΎΠΏΡ‹Ρ‚, интуиция ΠΈ знания ΡΠΎΠ·Π΄Π°ΡŽΡ‚ ΠΎΠ±Ρ€Π°Π·Π½ΠΎΠ΅ Π²Ρ‹Π±ΠΎΡ€ эскизного ΠΏΡ€ΠΎΠ΅ΠΊΡ‚Π° ΠΏΠΎ ΠΏΠ»Π°Π½ΠΈΡ€ΠΎΠ²ΠΊΠ΅, ΠΌΠ°Ρ‚Π΅Ρ€ΠΈΠ°Π»Π°ΠΌ ΠΈ ΠΌΠ΅Ρ‚ΠΎΠ΄Ρ‹ эрСкции. ΠŸΡ€ΠΈΠ²Π΅Π΄Π΅Π½Ρ‹ ΠΎΡ†Π΅Π½ΠΊΠΈ Ρ€Π°Π·Π»ΠΈΡ‡Π½Ρ‹Ρ… Ρ„ΠΎΡ€ΠΌ Π½Π°Π³Ρ€ΡƒΠ·ΠΊΠΈ. сдСлано, Π° Π·Π°Ρ‚Π΅ΠΌ Π²Ρ‹Π±Ρ€Π°Π½Π½Ρ‹ΠΉ Π΄ΠΈΠ·Π°ΠΉΠ½ подвСргаСтся Π΄Π΅Ρ‚Π°Π»ΡŒΠ½ΠΎΠΌΡƒ Π°Π½Π°Π»ΠΈΠ·Ρƒ основанный Π½Π° ΠΏΡ€ΠΈΠ½Ρ†ΠΈΠΏΠ°Ρ… статики.Π‘Ρ‚Π°Ρ‚ΠΈΠΊΠ° — ΠΎΠ΄Π½Π° ΠΈΠ· основных Π²Π΅Ρ‚Π²Π΅ΠΉ ΠΌΠ΅Ρ…Π°Π½ΠΈΠΊΠ° ΠΈ ΠΈΠΌΠ΅Π΅Ρ‚ Π΄Π΅Π»ΠΎ с силами, Π΄Π΅ΠΉΡΡ‚Π²ΡƒΡŽΡ‰ΠΈΠΌΠΈ Π½Π° Ρ‚Π΅Π»Π°, находящиСся Π² состоянии покоя. (статичСскоС равновСсиС). Другая основная Π²Π΅Ρ‚Π²ΡŒ, Π΄ΠΈΠ½Π°ΠΌΠΈΠΊΠ°, занимаСтся двиТущиСся Ρ‚Π΅Π»Π°, Π½Π°ΠΏΡ€ΠΈΠΌΠ΅Ρ€, части машин.

БтатичСскоС равновСсиС

Π‘ΠΈΠ»Ρ‹, Π΄Π΅ΠΉΡΡ‚Π²ΡƒΡŽΡ‰ΠΈΠ΅ Π² ΠΎΠ΄Π½ΠΎΠΉ плоскости (Ρ‚. Π•. Π’ ΠΎΠ΄Π½ΠΎΠΉ плоскости) ΠΈ Π² равновСсии Π΄ΠΎΠ»ΠΆΠ΅Π½ ΡƒΠ΄ΠΎΠ²Π»Π΅Ρ‚Π²ΠΎΡ€ΡΡ‚ΡŒ ΠΎΠ΄Π½ΠΎΠΌΡƒ ΠΈΠ· ΡΠ»Π΅Π΄ΡƒΡŽΡ‰ΠΈΡ… Π½Π°Π±ΠΎΡ€ΠΎΠ² условий:

S F x = 0 S F x = 0 S F y = 0 S M a = 0

S F y = 0 ΠΈΠ»ΠΈ S M a = 0 ΠΈΠ»ΠΈ S M a = 0 ΠΈΠ»ΠΈ S M b = 0

S M a = 0 S M b = 0 S M b = 0 S M c = 0

, Π³Π΄Π΅ F относится ΠΊ силам, Π° M относится ΠΊ ΠΌΠΎΠΌΠ΅Π½Ρ‚Π°ΠΌ сил.

БтатичСская ΠΎΠΏΡ€Π΅Π΄Π΅Π»Π΅Π½Π½ΠΎΡΡ‚ΡŒ

Если Ρ‚Π΅Π»ΠΎ находится Π² равновСсии ΠΏΠΎΠ΄ дСйствиСм ΠΊΠΎΠΌΠΏΠ»Π°Π½Π°Ρ€Π½ΠΎΠ³ΠΎ сил, Π΄ΠΎΠ»ΠΆΠ½Ρ‹ ΠΏΡ€ΠΈΠΌΠ΅Π½ΡΡ‚ΡŒΡΡ ΠΏΡ€ΠΈΠ²Π΅Π΄Π΅Π½Π½Ρ‹Π΅ Π²Ρ‹ΡˆΠ΅ уравнСния статики. Π’ Ρ†Π΅Π»ΠΎΠΌ Ρ‚ΠΎΠ³Π΄Π° Ρ‚Ρ€ΠΈ нСзависимых нСизвСстных ΠΌΠΎΠ³ΡƒΡ‚ Π±Ρ‹Ρ‚ΡŒ ΠΎΠΏΡ€Π΅Π΄Π΅Π»Π΅Π½Ρ‹ ΠΈΠ· Ρ‚Ρ€Π΅Ρ… уравнСния. ΠžΠ±Ρ€Π°Ρ‚ΠΈΡ‚Π΅ Π²Π½ΠΈΠΌΠ°Π½ΠΈΠ΅: Ссли ΠΏΡ€ΠΈΠ»ΠΎΠΆΠ΅Π½Π½Ρ‹Π΅ ΠΈ ΠΏΡ€ΠΎΡ‚ΠΈΠ²ΠΎΠ΄Π΅ΠΉΡΡ‚Π²ΡƒΡŽΡ‰ΠΈΠ΅ силы ΠΏΠ°Ρ€Π°Π»Π»Π΅Π»ΡŒΠ½Ρ‹ (Ρ‚.Π΅. Ρ‚ΠΎΠ»ΡŒΠΊΠΎ Π² ΠΎΠ΄Π½ΠΎΠΌ Π½Π°ΠΏΡ€Π°Π²Π»Π΅Π½ΠΈΠΈ) Ρ‚ΠΎΠ»ΡŒΠΊΠΎ Π΄Π²Π° ΠΎΡ‚Π΄Π΅Π»ΡŒΠ½Ρ‹Ρ… уравнСния Π΄Π°ΡŽΡ‚ ΠΈ Ρ‚ΠΎΠ³Π΄Π° ΠΌΠΎΠΆΠ½ΠΎ ΠΎΠΏΡ€Π΅Π΄Π΅Π»ΠΈΡ‚ΡŒ Ρ‚ΠΎΠ»ΡŒΠΊΠΎ Π΄Π²Π° нСизвСстных. Π’Π°ΠΊΠΈΠ΅ систСмы силы Π½Π°Π·Ρ‹Π²Π°ΡŽΡ‚ΡΡ статичСски ΠΎΠΏΡ€Π΅Π΄Π΅Π»Π΅Π½Π½Ρ‹ΠΌΠΈ.

УсилиС

Π‘ΠΈΠ»Π° опрСдСляСтся ΠΊΠ°ΠΊ любая ΠΏΡ€ΠΈΡ‡ΠΈΠ½Π°, которая ΠΈΠΌΠ΅Π΅Ρ‚ Ρ‚Π΅Π½Π΄Π΅Π½Ρ†ΠΈΡŽ ΠΈΠ·ΠΌΠ΅Π½ΡΡ‚ΡŒ состояниС ΠΈΠ»ΠΈ ΠΏΠΎΠΊΠΎΠΉ Ρ‚Π΅Π»Π° ΠΈΠ»ΠΈ Π΅Π³ΠΎ состояниС Ρ€Π°Π²Π½ΠΎΠΌΠ΅Ρ€Π½ΠΎΠ³ΠΎ двиТСния ΠΏΠΎ прямой линия. Π‘ΠΈΠ»Π° ΠΌΠΎΠΆΠ΅Ρ‚ Π±Ρ‹Ρ‚ΡŒ ΠΎΠΏΡ€Π΅Π΄Π΅Π»Π΅Π½Π° количСствСнно ΠΊΠ°ΠΊ ΠΏΡ€ΠΎΠΈΠ·Π²Π΅Π΄Π΅Π½ΠΈΠ΅ масса Ρ‚Π΅Π»Π°, Π½Π° ΠΊΠΎΡ‚ΠΎΡ€ΠΎΠ΅ дСйствуСт сила, ΠΈ ускорСниС силы.

P = ma, Π³Π΄Π΅
P = прилоТСнная сила
m = масса Ρ‚Π΅Π»Π° (ΠΊΠ³)
a = ускорСниС, Π²Ρ‹Π·Π²Π°Π½Π½ΠΎΠ΅ силой (ΠΌ / с 2 )

Π’Π°ΠΊΠΈΠΌ ΠΎΠ±Ρ€Π°Π·ΠΎΠΌ, Π΅Π΄ΠΈΠ½ΠΈΡ†Π΅ΠΉ измСрСния силы Sl являСтся ΠΊΠ³ ΠΌ / с 2 , Ρ‡Ρ‚ΠΎ обозначаСтся ΠΡŒΡŽΡ‚ΠΎΠ½ΠΎΠΌ (N).Π‘Π»Π΅Π΄ΡƒΡŽΡ‰ΠΈΠ΅ ΠΊΡ€Π°Ρ‚Π½Ρ‹Π΅ числа часто ΠΈΡΠΏΠΎΠ»ΡŒΠ·ΡƒΠ΅ΠΌΡ‹ΠΉ:

1 кН = 1 000 Н, 1 МН = 1 000 000 Н

ВсС ΠΎΠ±ΡŠΠ΅ΠΊΡ‚Ρ‹ Π½Π° Π—Π΅ΠΌΠ»Π΅ стрСмятся ΡƒΡΠΊΠΎΡ€ΡΡ‚ΡŒΡΡ ΠΊ Ρ†Π΅Π½Ρ‚Ρ€Ρƒ ЗСмля ΠΈΠ·-Π·Π° Π³Ρ€Π°Π²ΠΈΡ‚Π°Ρ†ΠΈΠΎΠ½Π½ΠΎΠ³ΠΎ притяТСния, ΡΠ»Π΅Π΄ΠΎΠ²Π°Ρ‚Π΅Π»ΡŒΠ½ΠΎ, сила гравитация, Π΄Π΅ΠΉΡΡ‚Π²ΡƒΡŽΡ‰Π°Ρ Π½Π° Ρ‚Π΅Π»ΠΎ массой (ΠΌ), являСтся ΠΏΡ€ΠΎΠ΄ΡƒΠΊΡ‚ΠΎΠΌ масса ΠΈ ускорСниС свободного падСния (g), ΠΊΠΎΡ‚ΠΎΡ€ΠΎΠ΅ ΠΈΠΌΠ΅Π΅Ρ‚ ΠΌΠ°Π³Π½ΠΈΡ‚ΡƒΠ΄ΠΎΠΉ 9,81 ΠΌ / с2.

F = mg = vr g Π³Π΄Π΅:
F = сила (Н)
m = масса (кг)
g = ускорСниС свободного падСния (9.8 ΠΌ / с2)
v = объСм (м)
r = ΠΏΠ»ΠΎΡ‚Π½ΠΎΡΡ‚ΡŒ (ΠΊΠ³ / ΠΌ)

Π’Π΅ΠΊΡ‚ΠΎΡ€

Π‘ΠΎΠ»ΡŒΡˆΠΈΠ½ΡΡ‚Π²ΠΎ сил ΠΈΠΌΠ΅ΡŽΡ‚ Π²Π΅Π»ΠΈΡ‡ΠΈΠ½Ρƒ ΠΈ Π½Π°ΠΏΡ€Π°Π²Π»Π΅Π½ΠΈΠ΅ ΠΈ ΠΌΠΎΠ³ΡƒΡ‚ Π±Ρ‹Ρ‚ΡŒ ΠΏΠΎΠΊΠ°Π·Π°Π½Ρ‹ ΠΊΠ°ΠΊ Π²Π΅ΠΊΡ‚ΠΎΡ€. Π’Π°ΠΊΠΆΠ΅ Π½Π΅ΠΎΠ±Ρ…ΠΎΠ΄ΠΈΠΌΠΎ ΡƒΠΊΠ°Π·Π°Ρ‚ΡŒ Π΅Π³ΠΎ Ρ‚ΠΎΡ‡ΠΊΡƒ примСнСния. Π’Π΅ΠΊΡ‚ΠΎΡ€ изобраТаСтся Π»ΠΈΠ½ΠΈΠ΅ΠΉ, Π΄Π»ΠΈΠ½Π° ΠΊΠΎΡ‚ΠΎΡ€ΠΎΠΉ ΠΏΡ€ΠΎΠΏΠΎΡ€Ρ†ΠΈΠΎΠ½Π°Π»ΡŒΠ½Π° Π²Π΅Π»ΠΈΡ‡ΠΈΠ½Π° Π² Π½Π΅ΠΊΠΎΡ‚ΠΎΡ€ΠΎΠΉ шкалС ΠΈ стрСлка, ΠΏΠΎΠΊΠ°Π·Ρ‹Π²Π°ΡŽΡ‰Π°Ρ Π½Π°ΠΏΡ€Π°Π²Π»Π΅Π½ΠΈΠ΅.

Π‘Π»ΠΎΠΆΠ΅Π½ΠΈΠ΅ Π²Π΅ΠΊΡ‚ΠΎΡ€ΠΎΠ²

Π‘ΡƒΠΌΠΌΠ° Π΄Π²ΡƒΡ… ΠΈΠ»ΠΈ Π±ΠΎΠ»Π΅Π΅ Π²Π΅ΠΊΡ‚ΠΎΡ€ΠΎΠ² называСтся Ρ€Π΅Π·ΡƒΠ»ΡŒΡ‚ΠΈΡ€ΡƒΡŽΡ‰Π΅ΠΉ.Π’ Ρ€Π°Π²Π½ΠΎΠ΄Π΅ΠΉΡΡ‚Π²ΡƒΡŽΡ‰Π°Ρ Π΄Π²ΡƒΡ… ΠΏΠ°Ρ€Π°Π»Π»Π΅Π»ΡŒΠ½Ρ‹Ρ… Π²Π΅ΠΊΡ‚ΠΎΡ€ΠΎΠ² получаСтся ΠΏΡƒΡ‚Π΅ΠΌ построСния вСкторная Π΄ΠΈΠ°Π³Ρ€Π°ΠΌΠΌΠ° Π΄Π²ΡƒΡ… Π²Π΅ΠΊΡ‚ΠΎΡ€ΠΎΠ².

ДобавляСмыС Π²Π΅ΠΊΡ‚ΠΎΡ€Ρ‹ Ρ€Π°ΡΠΏΠΎΠ»Π°Π³Π°ΡŽΡ‚ΡΡ ΠΏΠΎ схСмС Β«ΠΊΠΎΠ½Ρ‡ΠΈΠΊ ΠΊ хвосту». Если Π½ΡƒΠΆΠ½ΠΎ Π΄ΠΎΠ±Π°Π²ΠΈΡ‚ΡŒ Ρ‚Ρ€ΠΈ ΠΈΠ»ΠΈ Π±ΠΎΠ»Π΅Π΅ Π²Π΅ΠΊΡ‚ΠΎΡ€Π°, ΠΈΡ… ΠΌΠΎΠΆΠ½ΠΎ Ρ€Π°ΡΠΏΠΎΠ»ΠΎΠΆΠΈΡ‚ΡŒ Ρ‚Π°ΠΊΠΈΠΌ ΠΆΠ΅ ΠΎΠ±Ρ€Π°Π·ΠΎΠΌ, ΠΈ это называСтся ΠΌΠ½ΠΎΠ³ΠΎΡƒΠ³ΠΎΠ»ΡŒΠ½ΠΈΠΊΠΎΠΌ. Линия, провСдСнная ΠΊ Π·Π°ΠΊΡ€Ρ‹Ρ‚ΡŒ Ρ‚Ρ€Π΅ΡƒΠ³ΠΎΠ»ΡŒΠ½ΠΈΠΊ ΠΈΠ»ΠΈ ΠΌΠ½ΠΎΠ³ΠΎΡƒΠ³ΠΎΠ»ΡŒΠ½ΠΈΠΊ (ΠΎΡ‚ Π½Π°Ρ‡Π°Π»Π° Π΄ΠΎ ΠΊΠΎΠ½Π΅Ρ‡Π½ΠΎΠΉ Ρ‚ΠΎΡ‡ΠΊΠΈ) Ρ„ΠΎΡ€ΠΌΠΈΡ€ΡƒΠ΅Ρ‚ Ρ€Π΅Π·ΡƒΠ»ΡŒΡ‚ΠΈΡ€ΡƒΡŽΡ‰ΠΈΠΉ Π²Π΅ΠΊΡ‚ΠΎΡ€.

Π’Ρ‹Ρ‡ΠΈΡ‚Π°Π½ΠΈΠ΅ Π²Π΅ΠΊΡ‚ΠΎΡ€Π° опрСдСляСтся ΠΊΠ°ΠΊ слоТСниС ΡΠΎΠΎΡ‚Π²Π΅Ρ‚ΡΡ‚Π²ΡƒΡŽΡ‰ΠΈΠΉ ΠΎΡ‚Ρ€ΠΈΡ†Π°Ρ‚Π΅Π»ΡŒΠ½Ρ‹ΠΉ Π²Π΅ΠΊΡ‚ΠΎΡ€.

Π Π°Π·Ρ€Π΅ΡˆΠ΅Π½ΠΈΠ΅ силы

ΠŸΡ€ΠΈ Π°Π½Π°Π»ΠΈΠ·Π΅ ΠΈ расчСтах часто ΡƒΠ΄ΠΎΠ±Π½ΠΎ ΡƒΡ‡ΠΈΡ‚Ρ‹Π²Π°Ρ‚ΡŒ эффСкты силы Π² Π΄Ρ€ΡƒΠ³ΠΈΡ… направлСниях, ΠΊΡ€ΠΎΠΌΠ΅ силы сам, особСнно ΠΏΠΎ Π΄Π΅ΠΊΠ°Ρ€Ρ‚ΠΎΠ²ΠΎΠΉ оси (xx-yy). Π‘ΠΈΠ»Π° эффСкты вдоль этих осСй Π½Π°Π·Ρ‹Π²Π°ΡŽΡ‚ΡΡ Π²Π΅ΠΊΡ‚ΠΎΡ€Π½Ρ‹ΠΌΠΈ ΠΊΠΎΠΌΠΏΠΎΠ½Π΅Π½Ρ‚Π°ΠΌΠΈ ΠΈ ΠΏΠΎΠ»ΡƒΡ‡Π΅Π½Π½Ρ‹ΠΉ ΠΎΠ±Ρ€Π°Ρ‰Π΅Π½ΠΈΠ΅ΠΌ ΠΌΠ΅Ρ‚ΠΎΠ΄Π° слоТСния Π²Π΅ΠΊΡ‚ΠΎΡ€ΠΎΠ².

F y — ΠΊΠΎΠΌΠΏΠΎΠ½Π΅Π½Ρ‚ F Π² Π½Π°ΠΏΡ€Π°Π²Π»Π΅Π½ΠΈΠΈ y F y = F sin q

F x — ΠΊΠΎΠΌΠΏΠΎΠ½Π΅Π½Ρ‚ F Π² Π½Π°ΠΏΡ€Π°Π²Π»Π΅Π½ΠΈΠΈ x F x = F cos q

ΠŸΠ°Ρ€Π°Π»Π»Π΅Π»ΡŒΠ½Ρ‹Π΅ ΠΊΠΎΠΏΠ»Π°Π½Π°Ρ€Π½Ρ‹Π΅ силы

Π‘ΠΈΠ»Ρ‹, линия дСйствия ΠΊΠΎΡ‚ΠΎΡ€Ρ‹Ρ… встрСчаСтся Π² ΠΎΠ΄Π½ΠΎΠΉ Ρ‚ΠΎΡ‡ΠΊΠ΅, Π½Π°Π·Ρ‹Π²Π°ΡŽΡ‚ΡΡ ΠΎΠ΄Π½ΠΎΠ²Ρ€Π΅ΠΌΠ΅Π½Π½Ρ‹ΠΉ.ΠšΠΎΠΏΠ»Π°Π½Π°Ρ€Π½Ρ‹Π΅ силы Π»Π΅ΠΆΠ°Ρ‚ Π² ΠΎΠ΄Π½ΠΎΠΉ плоскости, Ρ‚ΠΎΠ³Π΄Π° ΠΊΠ°ΠΊ Π½Π΅ΠΊΠΎΠΏΠ»Π°Π½Π°Ρ€Π½Ρ‹Π΅ силы Π΄ΠΎΠ»ΠΆΠ½Ρ‹ Π±Ρ‹Ρ‚ΡŒ связаны с Ρ‚Ρ€Π΅Ρ…ΠΌΠ΅Ρ€Π½Ρ‹ΠΌ пространство ΠΈ Ρ‚Ρ€Π΅Π±ΡƒΡŽΡ‚ Π΄Π²ΡƒΡ… элСмСнтов Π΄Π°Π½Π½Ρ‹Ρ… направлСния вмСстС с Π²Π΅Π»ΠΈΡ‡ΠΈΠ½Π°. Π”Π²Π΅ ΠΊΠΎΠΏΠ»Π°Π½Π°Ρ€Π½Ρ‹Π΅ Π½Π΅ΠΏΠ°Ρ€Π°Π»Π»Π΅Π»ΡŒΠ½Ρ‹Π΅ силы всСгда Π±ΡƒΠ΄ΡƒΡ‚ ΠΎΠ΄Π½ΠΎΠ²Ρ€Π΅ΠΌΠ΅Π½Π½Ρ‹ΠΉ.

РавновСсиС частицы

Когда Ρ€Π°Π²Π½ΠΎΠ΄Π΅ΠΉΡΡ‚Π²ΡƒΡŽΡ‰Π°Ρ всСх сил, Π΄Π΅ΠΉΡΡ‚Π²ΡƒΡŽΡ‰ΠΈΡ… Π½Π° частицу, Ρ€Π°Π²Π½Π° Π½ΡƒΠ»ΡŽ, частица находится Π² равновСсии, Ρ‚. Π΅. Π½Π΅ Π½Π°Ρ€ΡƒΡˆΠ΅Π½Π° Π΅Π³ΠΎ ΡΡƒΡ‰Π΅ΡΡ‚Π²ΡƒΡŽΡ‰Π΅Π΅ состояниС покоя (ΠΈΠ»ΠΈ Ρ€Π°Π²Π½ΠΎΠΌΠ΅Ρ€Π½ΠΎΠ³ΠΎ двиТСния).

Π—Π°ΠΌΠΊΠ½ΡƒΡ‚Ρ‹ΠΉ Ρ‚Ρ€Π΅ΡƒΠ³ΠΎΠ»ΡŒΠ½ΠΈΠΊ ΠΈΠ»ΠΈ ΠΌΠ½ΠΎΠ³ΠΎΡƒΠ³ΠΎΠ»ΡŒΠ½ΠΈΠΊ являСтся графичСским Π²Ρ‹Ρ€Π°ΠΆΠ΅Π½ΠΈΠ΅ΠΌ равновСсиС частицы.

РавновСсиС частицы, ΠΊ ΠΊΠΎΡ‚ΠΎΡ€ΠΎΠΉ ΠΏΡ€ΠΈΠ»ΠΎΠΆΠ΅Π½Π° СдинствСнная сила. примСняСтся ΠΌΠΎΠΆΠ΅Ρ‚ ΠΏΠΎΠ΄Π΄Π΅Ρ€ΠΆΠΈΠ²Π°Ρ‚ΡŒΡΡ ΠΏΡ€ΠΈΠΌΠ΅Π½Π΅Π½ΠΈΠ΅ΠΌ Π²Ρ‚ΠΎΡ€ΠΎΠΉ силы, Ρ€Π°Π²Π½ΠΎΠΉ ΠΏΠΎ Π²Π΅Π»ΠΈΡ‡ΠΈΠ½Π΅ ΠΈ Π½Π°ΠΏΡ€Π°Π²Π»Π΅Π½ΠΈΡŽ, Π½ΠΎ ΠΏΡ€ΠΎΡ‚ΠΈΠ²ΠΎΠΏΠΎΠ»ΠΎΠΆΠ½ΠΎΠΉ ΠΏΠΎ смыслу, ΠΊ ΠΏΠ΅Ρ€Π²ΠΎΠΉ силС. Π­Ρ‚Π° вторая сила, ΠΏΠΎΡΠΊΠΎΠ»ΡŒΠΊΡƒ ΠΎΠ½Π° восстанавливаСт равновСсиС, называСтся равновСсным. Когда частица дСйствуСт ΠΏΡ€ΠΈ дСйствии Π΄Π²ΡƒΡ… ΠΈΠ»ΠΈ Π±ΠΎΠ»Π΅Π΅ сил равновСсиС Π΄ΠΎΠ»ΠΆΠ½ΠΎ Π±Ρ‹Ρ‚ΡŒ Ρ€Π°Π²Π½Ρ‹ΠΌ ΠΈ ΠΏΡ€ΠΎΡ‚ΠΈΠ²ΠΎΠΏΠΎΠ»ΠΎΠΆΠ½ΠΎ Ρ€Π°Π²Π½ΠΎΠ΄Π΅ΠΉΡΡ‚Π²ΡƒΡŽΡ‰Π΅ΠΉ систСмы.Π’Π°ΠΊΠΈΠΌ ΠΎΠ±Ρ€Π°Π·ΠΎΠΌ, равновСсиС Π²Π΅ΠΊΡ‚ΠΎΡ€, нарисованный Π·Π°ΠΊΡ€Ρ‹Π²Π°ΡŽΡ‰ΠΈΠΉ Π²Π΅ΠΊΡ‚ΠΎΡ€Π½ΡƒΡŽ Π΄ΠΈΠ°Π³Ρ€Π°ΠΌΠΌΡƒ ΠΈ ΡΠΎΠ΅Π΄ΠΈΠ½ΡΡŽΡ‰ΠΈΠΉ ΠΎΡ‚ ΠΊΠΎΠ½Π΅Ρ‡Π½ΠΎΠΉ Ρ‚ΠΎΡ‡ΠΊΠΈ Π΄ΠΎ ΠΎΡ‚ΠΏΡ€Π°Π²Π½ΠΎΠΉ Ρ‚ΠΎΡ‡ΠΊΠΈ.

Π‘Ρ…Π΅ΠΌΠ° свободного Ρ‚Π΅Π»Π° Частица

Π˜Π·Π²Π΅ΡΡ‚Π΅Π½ эскиз, ΠΏΠΎΠΊΠ°Π·Ρ‹Π²Π°ΡŽΡ‰ΠΈΠΉ физичСскиС условия ΠΏΡ€ΠΎΠ±Π»Π΅ΠΌΡ‹. ΠΊΠ°ΠΊ пространствСнная Π΄ΠΈΠ°Π³Ρ€Π°ΠΌΠΌΠ°. Π­Ρ‚ΠΎ ΠΌΠΎΠΆΠ½ΠΎ свСсти ΠΊ Π΄ΠΈΠ°Π³Ρ€Π°ΠΌΠΌΠ΅, ΠΏΠΎΠΊΠ°Π·Ρ‹Π²Π°ΡŽΡ‰Π΅ΠΉ частица ΠΈ всС силы, Π΄Π΅ΠΉΡΡ‚Π²ΡƒΡŽΡ‰ΠΈΠ΅ Π½Π° Π½Π΅Π΅. Вакая Π΄ΠΈΠ°Π³Ρ€Π°ΠΌΠΌΠ° называСтся Π΄ΠΈΠ°Π³Ρ€Π°ΠΌΠΌΠΎΠΉ свободного Ρ‚Π΅Π»Π°.

ΠŸΡ€ΠΈΠΌΠ΅Ρ€ 1 ΠžΠΏΡ€Π΅Π΄Π΅Π»ΠΈΡ‚Π΅ натяТСниС ΠΊΠ°ΠΆΠ΄ΠΎΠ³ΠΎ ΠΈΠ· ΠΊΠ°Π½Π°Ρ‚ΠΎΠ² AB ΠΈ AC

ΠŸΡ€ΠΈΠΌΠ΅Ρ€ 2 ЖСсткий ΡΡ‚Π΅Ρ€ΠΆΠ΅Π½ΡŒ ΡˆΠ°Ρ€Π½ΠΈΡ€Π½ΠΎ ΠΏΡ€ΠΈΠΊΡ€Π΅ΠΏΠ»Π΅Π½ ΠΊ Π²Π΅Ρ€Ρ‚ΠΈΠΊΠ°Π»ΡŒΠ½ΠΎΠΉ ΠΎΠΏΠΎΡ€Π΅ ΠΈ удСрТиваСтся Π½Π° 50 ΠΊ Π³ΠΎΡ€ΠΈΠ·ΠΎΠ½Ρ‚Π°Π»ΠΈ с ΠΏΠΎΠΌΠΎΡ‰ΡŒΡŽ троса, ΠΊΠΎΠ³Π΄Π° вСс 250N подвСшСн, ΠΊΠ°ΠΊ ΠΏΠΎΠΊΠ°Π·Π°Π½ΠΎ Π½Π° рисункС.ΠžΠΏΡ€Π΅Π΄Π΅Π»ΠΈΡ‚Π΅ напряТСниС Π² тросС ΠΈ сТатиС Π² стСрТнС Π±Π΅Π· ΡƒΡ‡Π΅Ρ‚Π° вСса стСрТня.

Π‘Ρ…Π΅ΠΌΠ° пространства

Π‘Ρ…Π΅ΠΌΠ° свободного Ρ‚Π΅Π»Π°

Π’Ρ€Π΅ΡƒΠ³ΠΎΠ»ΡŒΠ½ΠΈΠΊ силы

Π‘ΠΈΠ»Ρ‹ ΠΌΠΎΠΆΠ½ΠΎ Ρ‚Π°ΠΊΠΆΠ΅ Ρ€Π°ΡΡΡ‡ΠΈΡ‚Π°Ρ‚ΡŒ ΠΏΠΎ Π·Π°ΠΊΠΎΠ½Ρƒ синусов:

(сТатиС Π² стСрТнС / sin 45) = (натяТСниС Π² ΠΊΠ°Π±Π΅Π»Π΅ / ​​sin 40) = (250Н / Π³Ρ€Π΅Ρ… 65)

Π’ΠΎΡ‡ΠΊΠ° ΠΏΠ°Ρ€Π°Π»Π»Π΅Π»ΠΈΠ·ΠΌΠ°

Π’Ρ€ΠΈ ΠΊΠΎΠΏΠ»Π°Π½Π°Ρ€Π½Ρ‹Ρ… силы, ΠΊΠΎΡ‚ΠΎΡ€Ρ‹Π΅ находятся Π² равновСсии, Π΄ΠΎΠ»ΠΆΠ½Ρ‹ ΠΏΡ€ΠΎΠΉΡ‚ΠΈ всС Ρ‡Π΅Ρ€Π΅Π· Ρ‚Ρƒ ΠΆΠ΅ Ρ‚ΠΎΡ‡ΠΊΡƒ.Π­Ρ‚ΠΎ Π½Π΅ ΠΎΠ±ΡΠ·Π°Ρ‚Π΅Π»ΡŒΠ½ΠΎ относится ΠΊ Π±ΠΎΠ»ΡŒΡˆΠ΅ΠΌΡƒ количСству Ρ‡Π΅ΠΌ Ρ‚Ρ€ΠΈ силы.

Если Π΄Π²Π΅ силы (Π½Π΅ ΠΏΠ°Ρ€Π°Π»Π»Π΅Π»ΡŒΠ½Ρ‹Π΅) Π½Π΅ Π²ΡΡ‚Ρ€Π΅Ρ‡Π°ΡŽΡ‚ΡΡ Π½Π° своих Ρ‚ΠΎΡ‡ΠΊΠΈ соприкосновСния с Ρ‚Π΅Π»ΠΎΠΌ, Π½Π°ΠΏΡ€ΠΈΠΌΠ΅Ρ€, конструктивным элСмСнтом, ΠΈΡ… Π»ΠΈΠ½ΠΈΠΈ дСйствия ΠΌΠΎΠ³ΡƒΡ‚ Π±Ρ‹Ρ‚ΡŒ ΠΏΡ€ΠΎΠ΄Π»Π΅Π½Ρ‹ Π΄ΠΎ Ρ‚Π΅Ρ… ΠΏΠΎΡ€, ΠΏΠΎΠΊΠ° ΠΎΠ½ΠΈ Π½Π΅ встрСтятся.

ΠšΠΎΠ»Π»ΠΈΠ½Π΅Π°Ρ€Π½Ρ‹Π΅ силы

ΠšΠΎΠ»Π»ΠΈΠ½Π΅Π°Ρ€Π½Ρ‹Π΅ силы ΠΏΠ°Ρ€Π°Π»Π»Π΅Π»ΡŒΠ½Ρ‹ ΠΈ ΡΠΎΠ²ΠΏΠ°Π΄Π°ΡŽΡ‚. Π‘ΡƒΠΌΠΌΠ° силы Π΄ΠΎΠ»ΠΆΠ½Ρ‹ Π±Ρ‹Ρ‚ΡŒ Ρ€Π°Π²Π½Ρ‹ Π½ΡƒΠ»ΡŽ, Ρ‡Ρ‚ΠΎΠ±Ρ‹ систСма Π½Π°Ρ…ΠΎΠ΄ΠΈΠ»Π°ΡΡŒ Π² равновСсии.

ΠšΠΎΠΏΠ»Π°Π½Π°Ρ€Π½Ρ‹Π΅, Π½Π΅ΠΏΠ°Ρ€Π°Π»Π»Π΅Π»ΡŒΠ½Ρ‹Π΅, ΠΏΠ°Ρ€Π°Π»Π»Π΅Π»ΡŒΠ½Ρ‹Π΅ силы Π’Ρ€ΠΈ ΠΈΠ»ΠΈ Π±ΠΎΠ»Π΅Π΅ Ρ‚Ρ€Π΅Π±ΡƒΡŽΡ‚ΡΡ ΠΏΠ°Ρ€Π°Π»Π»Π΅Π»ΡŒΠ½Ρ‹Π΅ силы.Они Π±ΡƒΠ΄ΡƒΡ‚ Π² равновСсии, Ссли сумма сил Ρ€Π°Π²Π½Π° Π½ΡƒΠ»ΡŽ, Π° сумма ΠΌΠΎΠΌΠ΅Π½Ρ‚ΠΎΠ² Π²ΠΎΠΊΡ€ΡƒΠ³ Ρ‚ΠΎΡ‡ΠΊΠ° Π½Π° плоскости Ρ€Π°Π²Π½Π° Π½ΡƒΠ»ΡŽ. РавновСсиС Ρ‚Π°ΠΊΠΆΠ΅ обозначаСтся Π΄Π²Π΅ суммы ΠΌΠΎΠΌΠ΅Π½Ρ‚ΠΎΠ² Ρ€Π°Π²Π½Ρ‹ Π½ΡƒΠ»ΡŽ.

Π’Π°Π±Π»ΠΈΡ†Π° 4 1 ДСйствия ΠΈ РСакция

РСакция

Π‘Ρ‚Ρ€ΡƒΠΊΡ‚ΡƒΡ€Π½Ρ‹Π΅ ΠΊΠΎΠΌΠΏΠΎΠ½Π΅Π½Ρ‚Ρ‹ ΠΎΠ±Ρ‹Ρ‡Π½ΠΎ ΠΏΠΎΠ΄Π΄Π΅Ρ€ΠΆΠΈΠ²Π°ΡŽΡ‚ΡΡ Π² равновСсии Π·Π° счСт крСпится ΠΊ ТСстким Ρ‚ΠΎΡ‡ΠΊΠ°ΠΌ крСплСния; это часто Π΄Ρ€ΡƒΠ³ΠΈΠ΅ части Ρ‚Π° ΠΆΠ΅ структура.Π’ΠΎΡ‡ΠΊΠΈ крСплСния ΠΈΠ»ΠΈ ΠΎΠΏΠΎΡ€Ρ‹ Π±ΡƒΠ΄ΡƒΡ‚ Ρ€Π΅Π°Π³ΠΈΡ€ΠΎΠ²Π°Ρ‚ΡŒ ΠΏΡ€ΠΎΡ‚ΠΈΠ² Ρ‚Π΅Π½Π΄Π΅Π½Ρ†ΠΈΠΈ ΠΏΡ€ΠΈΠ»ΠΎΠΆΠ΅Π½Π½Ρ‹Ρ… сил (Π½Π°Π³Ρ€ΡƒΠ·ΠΎΠΊ) Π²Ρ‹Π·Ρ‹Π²Π°Ρ‚ΡŒ Ρ‡Π»Π΅Π½ Π΄Π²ΠΈΠ³Π°Ρ‚ΡŒΡΡ. Π’Π°ΠΊΠΈΠΌ ΠΎΠ±Ρ€Π°Π·ΠΎΠΌ, силы, Π²ΠΎΠ·Π½ΠΈΠΊΠ°ΡŽΡ‰ΠΈΠ΅ Π² ΠΎΠΏΠΎΡ€Π°Ρ…, Ρ€Π°Π²Π½Ρ‹ называСтся Ρ€Π΅Π°ΠΊΡ†ΠΈΠ΅ΠΉ.

Как ΠΏΡ€Π°Π²ΠΈΠ»ΠΎ, конструктивный элСмСнт Π΄ΠΎΠ»ΠΆΠ΅Π½ ΡƒΠ΄Π΅Ρ€ΠΆΠΈΠ²Π°Ρ‚ΡŒΡΡ ΠΈΠ»ΠΈ ΠΏΠΎΠ΄Π΄Π΅Ρ€ΠΆΠΈΠ²Π°Ρ‚ΡŒΡΡ Π½Π° ΠΌΠΈΠ½ΠΈΠΌΡƒΠΌ Π΄Π²Π΅ Ρ‚ΠΎΡ‡ΠΊΠΈ (ΠΈΡΠΊΠ»ΡŽΡ‡Π΅Π½ΠΈΠ΅ — консоль). Π›ΡŽΠ±ΠΎΠΉ, ΠΊΡ‚ΠΎ ΠΏΡ€ΠΎΠ±ΠΎΠ²Π°Π» Β«ΡƒΡ€Π°Π²Π½ΠΎΠ²Π΅ΡΠΈΡ‚ΡŒΒ» Π΄Π»ΠΈΠ½Π½Ρ‹ΠΉ ΡˆΠ΅ΡΡ‚ ΠΈΠ»ΠΈ Ρ‡Ρ‚ΠΎ-Ρ‚ΠΎ ΠΏΠΎΠ΄ΠΎΠ±Π½ΠΎΠ΅ ΠΏΠΎΠΉΠΌΠ΅Ρ‚Π΅, Ρ‡Ρ‚ΠΎ хотя тСорСтичСски Ρ‚ΠΎΠ»ΡŒΠΊΠΎ ΠΎΠ΄Π½Π° ΠΎΠΏΠΎΡ€Π° Π½Π΅ΠΎΠ±Ρ…ΠΎΠ΄ΠΈΠΌΡ‹Π΅ Π΄Π²Π° ΠΈΡΠΏΠΎΠ»ΡŒΠ·ΡƒΡŽΡ‚ΡΡ для обСспСчСния ΡƒΠ΄ΠΎΠ²Π»Π΅Ρ‚Π²ΠΎΡ€ΠΈΡ‚Π΅Π»ΡŒΠ½ΠΎΠΉ ΡΡ‚Π°Π±ΠΈΠ»ΡŒΠ½ΠΎΡΡ‚ΠΈ.

Π Π΅Π·ΡƒΠ»ΡŒΡ‚Π°Ρ‚ сил Π³Ρ€Π°Π²ΠΈΡ‚Π°Ρ†ΠΈΠΈ

МоТно ΠΏΡ€Π΅Π΄ΠΏΠΎΠ»ΠΎΠΆΠΈΡ‚ΡŒ, Ρ‡Ρ‚ΠΎ вСсь вСс Ρ‚Π΅Π»Π° воздСйствуСт Π½Π° Ρ†Π΅Π½Ρ‚Ρ€. силы тяТСсти Ρ‚Π΅Π»Π° с Ρ†Π΅Π»ΡŒΡŽ опрСдСлСния ΠΏΠΎΠ΄Π΄Π΅Ρ€ΠΆΠΊΠΈ Ρ€Π΅Π°ΠΊΡ†ΠΈΠΈ систСмы сил, находящихся Π² равновСсии. ΠŸΡ€ΠΈΠΌΠ΅Ρ‡Π°Π½ΠΈΠ΅ Ρ‡Ρ‚ΠΎ для Π΄Ρ€ΡƒΠ³ΠΈΡ… Ρ†Π΅Π»Π΅ΠΉ силы Π³Ρ€Π°Π²ΠΈΡ‚Π°Ρ†ΠΈΠΈ Π½Π΅ всСгда ΠΌΠΎΠ³ΡƒΡ‚ Π±Ρ‹Ρ‚ΡŒ Π»Π΅Ρ‡ΠΈΠ»ΠΈ Ρ‚Π°ΠΊΠΈΠΌ ΠΎΠ±Ρ€Π°Π·ΠΎΠΌ.

ΠŸΡ€ΠΈΠΌΠ΅Ρ€ 3

ЛСстница упираСтся Π² Π³Π»Π°Π΄ΠΊΡƒΡŽ стСну ΠΈ Ρ‡Π΅Π»ΠΎΠ²Π΅ΠΊ вСсом 900Н стоит Π½Π° Π½Π΅ΠΌ посСрСдинС.ВСс лСстницы 100Н. ΠžΠΏΡ€Π΅Π΄Π΅Π»ΠΈΡ‚Π΅ ΠΎΠΏΠΎΡ€Π½Ρ‹Π΅ Ρ€Π΅Π°ΠΊΡ†ΠΈΠΈ Ρƒ стСны (RW) ΠΈ Ρƒ зСмля (RG)

Π‘Ρ…Π΅ΠΌΠ° свободного Ρ…ΠΎΠ΄Π° лСстницы

Π”ΠΈΠ°Π³Ρ€Π°ΠΌΠΌΠ° сил

Π’Π°ΠΊ ΠΊΠ°ΠΊ стСна гладкая, рСакция RW Π΄ΠΎΠ»ΠΆΠ½Π° Π±Ρ‹Ρ‚ΡŒ справа. Π½Π°ΠΊΠ»ΠΎΠ½Π΅Π½ ΠΊ повСрхности стСны ΠΈ, ΡΠ»Π΅Π΄ΠΎΠ²Π°Ρ‚Π΅Π»ΡŒΠ½ΠΎ, располоТСн Π³ΠΎΡ€ΠΈΠ·ΠΎΠ½Ρ‚Π°Π»ΡŒΠ½ΠΎ. А Π²Π΅Ρ€Ρ‚ΠΈΠΊΠ°Π»ΡŒΠ½Ρ‹ΠΉ ΠΊΠΎΠΌΠΏΠΎΠ½Π΅Π½Ρ‚ ΡƒΠΊΠ°Π·Ρ‹Π²Π°Π» Π±Ρ‹ Π½Π° силу трСния ΠΌΠ΅ΠΆΠ΄Ρƒ лСстница ΠΈ стСна. Π’Π½ΠΈΠ·Ρƒ лСстница опираСтся Π½Π° Π³Ρ€ΡƒΠ½Ρ‚ Π½Π΅ Ρ€ΠΎΠ²Π½Ρ‹ΠΉ, ΠΈ поэтому рСакция RG Π΄ΠΎΠ»ΠΆΠ΅Π½ ΠΈΠΌΠ΅Ρ‚ΡŒ ΠΊΠ°ΠΊ Π²Π΅Ρ€Ρ‚ΠΈΠΊΠ°Π»ΡŒΠ½ΡƒΡŽ, Ρ‚Π°ΠΊ ΠΈ Π³ΠΎΡ€ΠΈΠ·ΠΎΠ½Ρ‚Π°Π»ΡŒΠ½ΡƒΡŽ ΡΠΎΡΡ‚Π°Π²Π»ΡΡŽΡ‰ΡƒΡŽ.

ΠŸΠΎΡΠΊΠΎΠ»ΡŒΠΊΡƒ Π΄Π²Π΅ силы вСса Π² этом ΠΏΡ€ΠΈΠΌΠ΅Ρ€Π΅ ΠΈΠΌΠ΅ΡŽΡ‚ ΠΎΠ΄Π½Ρƒ ΠΈ Ρ‚Ρƒ ΠΆΠ΅ линию дСйствия, ΠΎΠ½ΠΈ ΠΌΠΎΠ³ΡƒΡ‚ Π±Ρ‹Ρ‚ΡŒ ΠΎΠ±ΡŠΠ΅Π΄ΠΈΠ½Π΅Π½Ρ‹ Π² Π΅Π΄ΠΈΠ½ΡƒΡŽ силу, ΡƒΠΌΠ΅Π½ΡŒΡˆΠ°ΡŽΡ‰ΡƒΡŽ ΠΏΡ€ΠΎΠ±Π»Π΅ΠΌΠ° ΠΎΡ‚ ΠΎΠ΄Π½ΠΎΠ³ΠΎ, ΠΈΠΌΠ΅ΡŽΡ‰Π΅Π³ΠΎ Ρ‡Π΅Ρ‚Ρ‹Ρ€Π΅ силы, Π΄ΠΎ ΠΎΠ΄Π½ΠΎΠ³ΠΎ, ΠΈΠΌΠ΅ΡŽΡ‰Π΅Π³ΠΎ Ρ‚ΠΎΠ»ΡŒΠΊΠΎ Ρ‚Ρ€ΠΈ силы. Π—Π°Ρ‚Π΅ΠΌ ΠΌΠΎΠΆΠ½ΠΎ Π½Π°ΠΉΡ‚ΠΈ Ρ‚ΠΎΡ‡ΠΊΡƒ ΠΏΠ°Ρ€Π°Π»Π»Π΅Π»ΠΈΠ·ΠΌΠ° (A), давая Π½Π°ΠΏΡ€Π°Π²Π»Π΅Π½ΠΈΠ΅ силы Ρ€Π΅Π°ΠΊΡ†ΠΈΠΈ Π·Π΅ΠΌΠ»ΠΈ. Π­Ρ‚ΠΎ, Π² свою ΠΎΡ‡Π΅Ρ€Π΅Π΄ΡŒ, позволяСт Π΄ΠΈΠ°Π³Ρ€Π°ΠΌΠΌΡƒ Π²Π΅ΠΊΡ‚ΠΎΡ€Π° силы, ΠΊΠΎΡ‚ΠΎΡ€ΡƒΡŽ Π½ΡƒΠΆΠ½ΠΎ Π½Π°Ρ€ΠΈΡΠΎΠ²Π°Ρ‚ΡŒ, ΠΈ, ΡΠ»Π΅Π΄ΠΎΠ²Π°Ρ‚Π΅Π»ΡŒΠ½ΠΎ, стСну ΠΈ Ρ€Π΅Π°ΠΊΡ†ΠΈΠΈ Π³Ρ€ΡƒΠ½Ρ‚Π° ΠΎΠΏΡ€Π΅Π΄Π΅Π»Π΅Π½Ρ‹.

ΠŸΡ€ΠΈΠΌΠ΅Ρ€ 4

ΠšΠ°Ρ€ΠΊΠ°Ρ (Ρ„Π΅Ρ€ΠΌΠ°) с ΡˆΠ°Ρ€Π½ΠΈΡ€Π½Ρ‹ΠΌ соСдинСниСм воспринимаСт Π΄Π²Π΅ Π½Π°Π³Ρ€ΡƒΠ·ΠΊΠΈ, ΠΊΠ°ΠΊ ΠΏΠΎΠΊΠ°Π·Π°Π½ΠΎ.ΠšΠΎΠ½Π΅Ρ† A ΠΏΡ€ΠΈΠΊΡ€Π΅ΠΏΠ»Π΅Π½ ΠΊ ТСсткой ΠΎΠΏΠΎΡ€Π΅, Π² Ρ‚ΠΎ врСмя ΠΊΠ°ΠΊ ΠΊΠΎΠ½Π΅Ρ† B ΠΈΠΌΠ΅Π΅Ρ‚ роликовая ΠΎΠΏΠΎΡ€Π°. ΠžΠΏΡ€Π΅Π΄Π΅Π»ΠΈΡ‚Π΅ ΠΎΠΏΠΎΡ€Π½Ρ‹Π΅ Ρ€Π΅Π°ΠΊΡ†ΠΈΠΈ графичСски:

  • 1 ΠžΠ±ΡŠΠ΅Π΄ΠΈΠ½ΠΈΡ‚Π΅ Π΄Π²Π΅ ΠΏΡ€ΠΈΠ»ΠΎΠΆΠ΅Π½Π½Ρ‹Π΅ силы Π² ΠΎΠ΄Π½Ρƒ ΠΈ Π½Π°ΠΉΠ΄ΠΈΡ‚Π΅ линию дСйствия.

1 ΠžΠ±ΡŠΠ΅Π΄ΠΈΠ½ΠΈΡ‚Π΅ Π΄Π²Π° примСняСмых силы Π² ΠΎΠ΄Π½Ρƒ ΠΈ Π½Π°ΠΉΠ΄ΠΈΡ‚Π΅ линию дСйствия.

  • 2 Из-Π·Π° ΠΎΠΏΠΎΡ€Ρ‹ Ρ€ΠΎΠ»ΠΈΠΊΠ° рСакция RB Π±ΡƒΠ΄Π΅Ρ‚ Π²Π΅Ρ€Ρ‚ΠΈΠΊΠ°Π»ΡŒΠ½Ρ‹ΠΉ. Π‘Π»Π΅Π΄ΠΎΠ²Π°Ρ‚Π΅Π»ΡŒΠ½ΠΎ, Ρ€Π΅Π·ΡƒΠ»ΡŒΡ‚ΠΈΡ€ΡƒΡŽΡ‰Π°Ρ линия (RL) Π΄ΠΎΠ»ΠΆΠ½Π° Π±Ρ‹Ρ‚ΡŒ вытянут, Ρ‡Ρ‚ΠΎΠ±Ρ‹ ΠΏΠ΅Ρ€Π΅ΡΠ΅Ρ‡ΡŒ Π²Π΅Ρ€Ρ‚ΠΈΠΊΠ°Π»ΡŒΠ½ΡƒΡŽ Ρ€Π΅Π°ΠΊΡ†ΠΈΡŽ ΠΎΠΏΠΎΡ€Ρ‹ B.Π­Ρ‚Π° Ρ‚ΠΎΡ‡ΠΊΠ° являСтся Ρ‚ΠΎΡ‡ΠΊΠΎΠΉ ΠΏΠ°Ρ€Π°Π»Π»Π΅Π»ΠΈΠ·ΠΌΠ° для Ρ€Π΅Π·ΡƒΠ»ΡŒΡ‚ΠΈΡ€ΡƒΡŽΡ‰Π΅Π³ΠΎ Π½Π°Π³Ρ€ΡƒΠ·ΠΊΠ°, рСакция Π² Ρ‚ΠΎΡ‡ΠΊΠ΅ B ΠΈ рСакция Π² Ρ‚ΠΎΡ‡ΠΊΠ΅ A.
  • 3 ΠžΡ‚ этой Ρ‚ΠΎΡ‡ΠΊΠΈ ΠΏΠ°Ρ€Π°Π»Π»Π΅Π»ΠΈΠ·ΠΌΠ° ΠΏΡ€ΠΎΠ²Π΅Π΄ΠΈΡ‚Π΅ линию Ρ‡Π΅Ρ€Π΅Π· ΠΎΠΏΠΎΡ€Π½Ρ‹ΠΉ ΡˆΡ‚ΠΈΡ„Ρ‚ Π² A. Π­Ρ‚ΠΎ Π΄Π°Π΅Ρ‚ линию дСйствия рСакция Π½Π° А.
  • 4 Π˜ΡΠΏΠΎΠ»ΡŒΠ·ΡƒΠΉΡ‚Π΅ эти Ρ‚Ρ€ΠΈ направлСния силы ΠΈ Π²Π΅Π»ΠΈΡ‡ΠΈΠ½Ρƒ RL, Ρ‡Ρ‚ΠΎΠ±Ρ‹ Π½Π°Ρ€ΠΈΡΠΎΠ²Π°Ρ‚ΡŒ ΡΠΈΠ»ΠΎΠ²ΡƒΡŽ Π΄ΠΈΠ°Π³Ρ€Π°ΠΌΠΌΡƒ, ΠΈΠ· ΠΊΠΎΡ‚ΠΎΡ€ΠΎΠΉ ΠΌΠΎΠΆΠ½ΠΎ ΠΎΠΏΡ€Π΅Π΄Π΅Π»ΠΈΡ‚ΡŒ RA ΠΈ RB. Π½Π°ΠΉΠ΄Π΅Π½Π½Ρ‹ΠΉ.

ΠžΡ‚Π²Π΅Ρ‚: RA = 12.2 кН Π½Π° расстоянии 21 ΠΎΡ‚ Π³ΠΎΡ€ΠΈΠ·ΠΎΠ½Ρ‚Π°Π»ΠΈ. RB = 12,7 кН Π²Π΅Ρ€Ρ‚ΠΈΠΊΠ°Π»ΡŒΠ½Ρ‹ΠΉ.

ΠœΠ½ΠΎΠ³ΠΎΡƒΠ³ΠΎΠ»ΡŒΠ½ΠΈΠΊ связи (см. Π˜Π½ΠΆΠ΅Π½Π΅Ρ€Π½ΠΎΠ΅ руководство) Ρ‚Π°ΠΊΠΆΠ΅ ΠΌΠΎΠΆΠ΅Ρ‚ Π±Ρ‹Ρ‚ΡŒ ΠΈΡΠΏΠΎΠ»ΡŒΠ·ΡƒΠ΅Ρ‚ΡΡ для опрСдСлСния Ρ€Π΅Π°ΠΊΡ†ΠΈΠΈ Π½Π° Π±Π°Π»ΠΊΡƒ ΠΈΠ»ΠΈ Ρ„Π΅Ρ€ΠΌΡƒ, хотя ΠΎΠ±Ρ‹Ρ‡Π½ΠΎ быстрСС ΠΈ Π»Π΅Π³Ρ‡Π΅ ΠΏΠΎΠ»ΡƒΡ‡ΠΈΡ‚ΡŒ Ρ€Π΅Π°ΠΊΡ†ΠΈΡŽ расчСт, ΠΌΠ΅Ρ‚ΠΎΠ΄, ΠΏΠΎΠΊΠ°Π·Π°Π½Π½Ρ‹ΠΉ Π² ΠŸΡ€ΠΈΠΌΠ΅Ρ€Π΅ 4, ΠΈΠ»ΠΈ комбинация расчСт ΠΈ Ρ‡Π΅Ρ€Ρ‡Π΅Π½ΠΈΠ΅.

Однако Π΄ΠΎΠ»ΠΆΠ½Ρ‹ Π±Ρ‹Ρ‚ΡŒ Π²Ρ‹ΠΏΠΎΠ»Π½Π΅Π½Ρ‹ ΡΠ»Π΅Π΄ΡƒΡŽΡ‰ΠΈΠ΅ условия.

  • 1 ВсС силы (ΠΊΡ€ΠΎΠΌΠ΅ Π΄Π²ΡƒΡ… Ρ€Π΅Π°ΠΊΡ†ΠΈΠΉ) Π΄ΠΎΠ»ΠΆΠ½Ρ‹ Π±Ρ‹Ρ‚ΡŒ извСстны ΠΏΠΎΠ»Π½ΠΎΡΡ‚ΡŒΡŽ, Ρ‚.Π΅.Π΅., ΠΌΠ°ΡΡˆΡ‚Π°Π±Ρ‹, Π½Π°ΠΏΡ€Π°Π²Π»Π΅Π½ΠΈΠ΅ дСйствий ΠΈ Π½Π°ΠΏΡ€Π°Π²Π»Π΅Π½ΠΈΠ΅.
  • 2 Линия дСйствия ΠΎΠ΄Π½ΠΎΠΉ ΠΈΠ· Ρ€Π΅Π°ΠΊΡ†ΠΈΠΉ Π΄ΠΎΠ»ΠΆΠ½Π° Π±Ρ‹Ρ‚ΡŒ извСстный.
  • 3 По ΠΊΡ€Π°ΠΉΠ½Π΅ΠΉ ΠΌΠ΅Ρ€Π΅, ΠΎΠ΄Π½Π° Ρ‚ΠΎΡ‡ΠΊΠ° Π½Π° Π»ΠΈΠ½ΠΈΠΈ дСйствий для Π΄Ρ€ΡƒΠ³ΠΎΠΉ рСакция Π΄ΠΎΠ»ΠΆΠ½Π° Π±Ρ‹Ρ‚ΡŒ извСстна. (2 ΠΈ 3 ΡƒΠΌΠ΅Π½ΡŒΡˆΠ°ΡŽΡ‚ количСство нСизвСстных, связанных с уравнСниями равновСсия ΠΏΡ€ΠΈΠ΅ΠΌΠ»Π΅ΠΌΡ‹ΠΉ ΡƒΡ€ΠΎΠ²Π΅Π½ΡŒ.)

ΠœΠΎΠΌΠ΅Π½Ρ‚Ρ‹ сил

ДСйствиС силы Π½Π° Ρ‚Π²Π΅Ρ€Π΄ΠΎΠ΅ Ρ‚Π΅Π»ΠΎ зависит ΠΎΡ‚ Π΅Π³ΠΎ Ρ‚ΠΎΡ‡ΠΊΠΈ ΠΏΡ€ΠΈΠΌΠ΅Π½Π΅Π½ΠΈΠ΅, Π° Ρ‚Π°ΠΊΠΆΠ΅ Π΅Π³ΠΎ ΠΌΠ°ΡΡˆΡ‚Π°Π±Ρ‹ ΠΈ Π½Π°ΠΏΡ€Π°Π²Π»Π΅Π½ΠΈΠ΅.Π­Ρ‚ΠΎ ΠΎΠ±Ρ‹Ρ‡Π½ΠΎ Π·Π½Π°Π½ΠΈΠ΅ Ρ‚ΠΎΠ³ΠΎ, Ρ‡Ρ‚ΠΎ нСбольшоС усилиС ΠΌΠΎΠΆΠ΅Ρ‚ ΠΈΠΌΠ΅Ρ‚ΡŒ большой эффСкт ΠΏΠΎΠ²ΠΎΡ€ΠΎΡ‚Π° ΠΈΠ»ΠΈ ΠΈΡΠΏΠΎΠ»ΡŒΠ·ΠΎΠ²Π°Ρ‚ΡŒ. Π’ ΠΌΠ΅Ρ…Π°Π½ΠΈΠΊΠ΅ Ρ‚Π΅Ρ€ΠΌΠΈΠ½ ΠΌΠΎΠΌΠ΅Π½Ρ‚ ΠΈΡΠΏΠΎΠ»ΡŒΠ·ΡƒΠ΅Ρ‚ΡΡ вмСсто эффСкт ΠΏΠΎΠ²ΠΎΡ€ΠΎΡ‚Π°.

ΠœΠΎΠΌΠ΅Π½Ρ‚ силы с Π²Π΅Π»ΠΈΡ‡ΠΈΠ½ΠΎΠΉ (F) ΠΎΡ‚Π½ΠΎΡΠΈΡ‚Π΅Π»ΡŒΠ½ΠΎ Ρ‚ΠΎΡ‡ΠΊΠΈ ΠΏΠΎΠ²ΠΎΡ€ΠΎΡ‚Π° (O) опрСдСляСтся ΠΊΠ°ΠΊ: M = F x d, Π³Π΄Π΅ d — пСрпСндикуляр расстояниС ΠΎΡ‚ O Π΄ΠΎ Π»ΠΈΠ½ΠΈΠΈ дСйствия силы F. РасстояниС d часто Π½Π°Π·Ρ‹Π²Π°ΡŽΡ‚ Ρ€Ρ‹Ρ‡Π°Π³ΠΎΠΌ. ΠœΠΎΠΌΠ΅Π½Ρ‚ ΠΈΠΌΠ΅Π΅Ρ‚ Ρ€Π°Π·ΠΌΠ΅Ρ€Ρ‹ силы Ρ€Π°Π· Π΄Π»ΠΈΠ½Π° (Нм). НаправлСниС ΠΌΠΎΠΌΠ΅Π½Ρ‚Π° ΠΎΡ‚Π½ΠΎΡΠΈΡ‚Π΅Π»ΡŒΠ½ΠΎ Ρ‚ΠΎΡ‡ΠΊΠΈ ΠΈΠ»ΠΈ оси Ρ€Π°Π²Π½ΠΎ опрСдСляСтся Π½Π°ΠΏΡ€Π°Π²Π»Π΅Π½ΠΈΠ΅ΠΌ вращСния, ΠΊ ΠΊΠΎΡ‚ΠΎΡ€ΠΎΠΌΡƒ стрСмится сила Π΄Π°Ρ‚ΡŒ Ρ‚Π΅Π»Ρƒ.ΠœΠΎΠΌΠ΅Π½Ρ‚ ΠΏΠΎ часовой стрСлкС ΠΎΠ±Ρ‹Ρ‡Π½ΠΎ рассматриваСтся ΠΊΠ°ΠΊ ΠΈΠΌΠ΅ΡŽΡ‰ΠΈΠΉ ΠΏΠΎΠ»ΠΎΠΆΠΈΡ‚Π΅Π»ΡŒΠ½Ρ‹ΠΉ Π·Π½Π°ΠΊ ΠΈ ΠΌΠΎΠΌΠ΅Π½Ρ‚ ΠΏΡ€ΠΎΡ‚ΠΈΠ² часовой стрСлки ΠΎΡ‚Ρ€ΠΈΡ†Π°Ρ‚Π΅Π»ΡŒΠ½Ρ‹ΠΉ Π·Π½Π°ΠΊ.

ΠžΠΏΡ€Π΅Π΄Π΅Π»Π΅Π½ΠΈΠ΅ ΠΌΠΎΠΌΠ΅Π½Ρ‚Π° силы Π² ΠΊΠΎΠΌΠΏΠ»Π°Π½Π°Ρ€Π½ΠΎΠΌ систСма Π±ΡƒΠ΄Π΅Ρ‚ ΡƒΠΏΡ€ΠΎΡ‰Π΅Π½Π°, Ссли сила ΠΈ Π΅Π΅ Ρ‚ΠΎΡ‡ΠΊΠ° ΠΏΡ€ΠΈΠ»ΠΎΠΆΠ΅Π½ΠΈΠ΅ Ρ€Π°Π·Π»ΠΎΠΆΠ΅Π½ΠΎ Π½Π° Π³ΠΎΡ€ΠΈΠ·ΠΎΠ½Ρ‚Π°Π»ΡŒΠ½ΠΎΠ΅ ΠΈ Π²Π΅Ρ€Ρ‚ΠΈΠΊΠ°Π»ΡŒΠ½ΠΎΠ΅ составныС части.

Π‘Ρ…Π΅ΠΌΠ° свободного Ρ‚Π΅Π»Π° для ТСсткого Ρ‚Π΅Π»Π°

ΠŸΡ€ΠΈ Ρ€Π΅ΡˆΠ΅Π½ΠΈΠΈ ΠΏΡ€ΠΎΠ±Π»Π΅ΠΌΡ‹ Π½Π΅ΠΎΠ±Ρ…ΠΎΠ΄ΠΈΠΌΠΎ ΡƒΡ‡ΠΈΡ‚Ρ‹Π²Π°Ρ‚ΡŒ всС силы. воздСйствуя Π½Π° Ρ‚Π΅Π»ΠΎ, ΠΈ ΠΈΡΠΊΠ»ΡŽΡ‡ΠΈΡ‚ΡŒ Π»ΡŽΠ±ΡƒΡŽ силу, которая нСпосрСдствСнно Π½Π΅ наносится Π½Π° Ρ‚Π΅Π»ΠΎ.ΠŸΠ΅Ρ€Π²Ρ‹ΠΉ шаг Π² Ρ€Π΅ΡˆΠ΅Π½ΠΈΠΈ ΠΏΡ€ΠΎΠ±Π»Π΅ΠΌΡ‹ ΠŸΠΎΡΡ‚ΠΎΠΌΡƒ слСдуСт Ρ€ΠΈΡΠΎΠ²Π°Ρ‚ΡŒ Π΄ΠΈΠ°Π³Ρ€Π°ΠΌΠΌΡƒ свободного Ρ‚Π΅Π»Π°.

  • 1 Π’Ρ‹Π±Π΅Ρ€ΠΈΡ‚Π΅ свободноС Ρ‚Π΅Π»ΠΎ, ΠΊΠΎΡ‚ΠΎΡ€ΠΎΠ΅ Π±ΡƒΠ΄Π΅Ρ‚ ΠΈΡΠΏΠΎΠ»ΡŒΠ·ΠΎΠ²Π°Ρ‚ΡŒΡΡ, ΠΈΠ·ΠΎΠ»ΠΈΡ€ΡƒΠΉΡ‚Π΅ Π΅Π³ΠΎ ΠΎΡ‚ Π»ΡŽΠ±Ρ‹Ρ… Π΄Ρ€ΡƒΠ³ΠΎΠ΅ Ρ‚Π΅Π»ΠΎ ΠΈ нарисуйтС Π΅Π³ΠΎ ΠΊΠΎΠ½Ρ‚ΡƒΡ€.
  • 2 НайдитС всС внСшниС силы Π½Π° свободном Ρ‚Π΅Π»Π΅ ΠΈ Ρ‡Π΅Ρ‚ΠΊΠΎ ΠΎΡ‚ΠΌΠ΅Ρ‚ΡŒΡ‚Π΅ ΠΈΡ… Π²Π΅Π»ΠΈΡ‡ΠΈΠ½Ρƒ ΠΈ Π½Π°ΠΏΡ€Π°Π²Π»Π΅Π½ΠΈΠ΅. Π­Ρ‚ΠΎ Π΄ΠΎΠ»ΠΆΠ½ΠΎ Π²ΠΊΠ»ΡŽΡ‡Π°Ρ‚ΡŒ вСс свободного Ρ‚Π΅Π»Π°, ΠΏΡ€ΠΈΠ»ΠΎΠΆΠ΅Π½Π½Ρ‹ΠΉ ΠΊ Ρ†Π΅Π½Ρ‚Ρ€ тяТСсти.
  • 3 НайдитС ΠΈ ΠΎΡ‚ΠΌΠ΅Ρ‚ΡŒΡ‚Π΅ нСизвСстныС внСшниС силы ΠΈ Ρ€Π΅Π°ΠΊΡ†ΠΈΠΈ, Π½Π° Π΄ΠΈΠ°Π³Ρ€Π°ΠΌΠΌΠ΅ свободного Ρ‚Π΅Π»Π°.
  • 4 Π’ΠΊΠ»ΡŽΡ‡ΠΈΡ‚Π΅ всС Ρ€Π°Π·ΠΌΠ΅Ρ€Ρ‹, ΡƒΠΊΠ°Π·Ρ‹Π²Π°ΡŽΡ‰ΠΈΠ΅ мСстополоТСниС, ΠΈ Π½Π°ΠΏΡ€Π°Π²Π»Π΅Π½ΠΈΠ΅ сил.

ΠŸΡ€ΠΎΠ΄ΠΎΠ»ΠΆΠ΅Π½ΠΈΠ΅ ΠΏΡ€ΠΈΠΌΠ΅Ρ€Π° 3

ΠŸΠΎΡΠΊΠΎΠ»ΡŒΠΊΡƒ лСстница Π² ΠŸΡ€ΠΈΠΌΠ΅Ρ€Π΅ 3 находится Π² состоянии покоя, условия равновСсиС для Ρ‚Π²Π΅Ρ€Π΄ΠΎΠ³ΠΎ Ρ‚Π΅Π»Π° ΠΌΠΎΠΆΠ½ΠΎ ΠΈΡΠΏΠΎΠ»ΡŒΠ·ΠΎΠ²Π°Ρ‚ΡŒ для расчСта Ρ€Π΅Π°ΠΊΡ†ΠΈΠΈ. ΠžΠ±Ρ€Π°Ρ‰Π°Ρ Π²Π½ΠΈΠΌΠ°Π½ΠΈΠ΅ Π½Π° Ρ‚ΠΎ мСсто, Π³Π΄Π΅ лСстница упираСтся Π² зСмлю, ΠΌΠΎΠΌΠ΅Π½Ρ‚ Ρ€Π΅Π°ΠΊΡ†ΠΈΠΈ Π Π“ ΠΌΠΎΠΆΠ½ΠΎ Π½Π΅ ΡƒΡ‡ΠΈΡ‚Ρ‹Π²Π°Ρ‚ΡŒ ΠΏΠΎΡΠΊΠΎΠ»ΡŒΠΊΡƒ Ρƒ Π½Π΅Π³ΠΎ Π½Π΅Ρ‚ ΠΏΠ»Π΅Ρ‡Π° Ρ€Ρ‹Ρ‡Π°Π³Π° (ΠΌΠΎΠΌΠ΅Π½Ρ‚ Ρ€Π°Π²Π΅Π½ Π½ΡƒΠ»ΡŽ).Богласно Ρ‚Ρ€Π΅Ρ‚ΡŒΠ΅ условиС равновСсия, сумма ΠΌΠΎΠΌΠ΅Π½Ρ‚ΠΎΠ² Π΄ΠΎΠ»ΠΆΠ½Π° Ρ€Π°Π²Π½ΡΡ‚ΡŒΡΡ ноль, ΡΠ»Π΅Π΄ΠΎΠ²Π°Ρ‚Π΅Π»ΡŒΠ½ΠΎ:

(6 x R W ) — (900N x 1,5 ΠΌ) — (100N x 1,5 ΠΌ. = 0

R W = 250N

Π’Π΅Ρ€Ρ‚ΠΈΠΊΠ°Π»ΡŒΠ½Π°Ρ ΡΠΎΡΡ‚Π°Π²Π»ΡΡŽΡ‰Π°Ρ RG Π΄ΠΎΠ»ΠΆΠ½Π°, согласно Π²Ρ‚ΠΎΡ€ΠΎΠΌΡƒ условиС, Π±Ρ‹Ρ‚ΡŒ Ρ€Π°Π²Π½Ρ‹ΠΌ, Π½ΠΎ ΠΏΡ€ΠΎΡ‚ΠΈΠ²ΠΎΠΏΠΎΠ»ΠΎΠΆΠ½Ρ‹ΠΌ суммС вСса лСстницС ΠΈ вСс Ρ‡Π΅Π»ΠΎΠ²Π΅ΠΊΠ° Π½Π° лСстницС, Ρ‚Π°ΠΊ ΠΊΠ°ΠΊ Ρ‚Π΅ Π΄Π²Π° — это СдинствСнныС Π²Π΅Ρ€Ρ‚ΠΈΠΊΠ°Π»ΡŒΠ½Ρ‹Π΅ силы ΠΈ сумма Π²Π΅Ρ€Ρ‚ΠΈΠΊΠ°Π»ΡŒΠ½Ρ‹Ρ… силы Π΄ΠΎΠ»ΠΆΠ½Ρ‹ Ρ€Π°Π²Π½ΡΡ‚ΡŒΡΡ Π½ΡƒΠ»ΡŽ.Ρ‚.Π΅.

R Gy = 1000N

Π˜ΡΠΏΠΎΠ»ΡŒΠ·ΡƒΡ ΠΏΠ΅Ρ€Π²ΠΎΠ΅ условиС равновСсия, ΠΌΠΎΠΆΠ½ΠΎ ΡƒΠ²ΠΈΠ΄Π΅Ρ‚ΡŒ, Ρ‡Ρ‚ΠΎ Π³ΠΎΡ€ΠΈΠ·ΠΎΠ½Ρ‚Π°Π»ΡŒΠ½Π°Ρ ΡΠΎΡΡ‚Π°Π²Π»ΡΡŽΡ‰Π°Ρ RG Π΄ΠΎΠ»ΠΆΠ½Π° Π±Ρ‹Ρ‚ΡŒ ΠΎΠ΄ΠΈΠ½Π°ΠΊΠΎΠ²ΠΎΠΉ, Π½ΠΎ ΠΏΡ€ΠΎΡ‚ΠΈΠ²ΠΎΠΏΠΎΠ»ΠΎΠΆΠ½ΠΎΠΉ Π² Π½Π°ΠΏΡ€Π°Π²Π»Π΅Π½ΠΈΠ΅ Π½Π° RW Ρ‚.Π΅ .;

R GX = 250N

Π’Π°ΠΊ ΠΊΠ°ΠΊ RG — это Ρ‚Ρ€Π΅Ρ‚ΡŒΡ сторона силового Ρ‚Ρ€Π΅ΡƒΠ³ΠΎΠ»ΡŒΠ½ΠΈΠΊΠ°, Π³Π΄Π΅ Π΄Π²Π΅ Π΄Ρ€ΡƒΠ³ΠΈΠ΅ стороны — это Π³ΠΎΡ€ΠΈΠ·ΠΎΠ½Ρ‚Π°Π»ΡŒΠ½Π°Ρ ΠΈ Π²Π΅Ρ€Ρ‚ΠΈΠΊΠ°Π»ΡŒΠ½Π°Ρ ΡΠΎΡΡ‚Π°Π²Π»ΡΡŽΡ‰ΠΈΠ΅, Π’Π΅Π»ΠΈΡ‡ΠΈΠ½Π° RG ΠΌΠΎΠΆΠ΅Ρ‚ Π±Ρ‹Ρ‚ΡŒ рассчитана ΠΊΠ°ΠΊ:

1000 2 + 250 2 = 1030N

Π Π΅Π·ΡƒΠ»ΡŒΡ‚Π°Ρ‚ ΠΏΠ°Ρ€Π°Π»Π»Π΅Π»ΡŒΠ½Ρ‹Ρ… сил

Если Π΄Π²Π΅ ΠΈΠ»ΠΈ Π±ΠΎΠ»Π΅Π΅ ΠΏΠ°Ρ€Π°Π»Π»Π΅Π»ΡŒΠ½Ρ‹Ρ… силы ΠΏΡ€ΠΈΠ»ΠΎΠΆΠ΅Π½Ρ‹ ΠΊ Π³ΠΎΡ€ΠΈΠ·ΠΎΠ½Ρ‚Π°Π»ΡŒΠ½ΠΎΠΌΡƒ Π±Π°Π»ΠΊΠ°, Ρ‚ΠΎ тСорСтичСски Π±Π°Π»ΠΊΡƒ ΠΌΠΎΠΆΠ½ΠΎ ΡƒΠ΄Π΅Ρ€ΠΆΠΈΠ²Π°Ρ‚ΡŒ Π² равновСсии с ΠΏΠΎΠΌΠΎΡ‰ΡŒΡŽ ΠΏΡ€ΠΈΠ»ΠΎΠΆΠ΅Π½ΠΈΠ΅ Π΅Π΄ΠΈΠ½ΠΎΠΉ силы (Ρ€Π΅Π°ΠΊΡ†ΠΈΠΈ), которая Ρ€Π°Π²Π½Π° ΠΈ Π½Π°ΠΏΡ€ΠΎΡ‚ΠΈΠ² Ρ€Π΅Π·ΡƒΠ»ΡŒΡ‚ΠΈΡ€ΡƒΡŽΡ‰Π΅Π³ΠΎ, R.РавновСсиС нисходящСго силы Π΄ΠΎΠ»ΠΆΠ½Ρ‹ Π±Ρ‹Ρ‚ΡŒ Ρ€Π°Π²Π½Ρ‹ ΠΈ ΠΏΡ€ΠΎΡ‚ΠΈΠ²ΠΎΠΏΠΎΠ»ΠΎΠΆΠ½Ρ‹ ΠΈΡ… Ρ€Π°Π²Π½ΠΎΠ΄Π΅ΠΉΡΡ‚Π²ΡƒΡŽΡ‰Π΅ΠΉ. Π­Ρ‚ΠΎΡ‚ обСспСчиваСт ΠΌΠ΅Ρ‚ΠΎΠ΄ вычислСния Ρ€Π°Π²Π½ΠΎΠ΄Π΅ΠΉΡΡ‚Π²ΡƒΡŽΡ‰Π΅ΠΉ систСмы ΠΏΠ°Ρ€Π°Π»Π»Π΅Π»ΡŒΠ½Ρ‹Π΅ силы. Однако Π½Π΅ΠΎΠ±Ρ…ΠΎΠ΄ΠΈΠΌΡ‹ Π΄Π²Π΅ Ρ€Π΅Π°ΠΊΡ†ΠΈΠΈ, Ρ‡Ρ‚ΠΎΠ±Ρ‹ Π³Π°Ρ€Π°Π½Ρ‚ΠΈΡ€ΠΎΠ²Π°Ρ‚ΡŒ нСобходимая ΡΡ‚Π°Π±ΠΈΠ»ΡŒΠ½ΠΎΡΡ‚ΡŒ ΠΈ Π±ΠΎΠ»Π΅Π΅ вСроятноС располоТСниС Π±ΡƒΠ΄Π΅Ρ‚ ΠΈΠΌΠ΅Ρ‚ΡŒ Π΄Π²Π΅ ΠΈ Π±ΠΎΠ»Π΅Π΅ ΠΎΠΏΠΎΡ€Ρ‹.

ОбС Ρ€Π΅Π°ΠΊΡ†ΠΈΠΈ RA ΠΈ RB Π΄ΠΎΠ»ΠΆΠ½Ρ‹ Π±Ρ‹Ρ‚ΡŒ Π²Π΅Ρ€Ρ‚ΠΈΠΊΠ°Π»ΡŒΠ½Ρ‹ΠΌΠΈ, Ρ‚Π°ΠΊ ΠΊΠ°ΠΊ Π΅ΡΡ‚ΡŒ Π½Π΅Ρ‚ Π³ΠΎΡ€ΠΈΠ·ΠΎΠ½Ρ‚Π°Π»ΡŒΠ½ΠΎΠΉ ΡΠΎΡΡ‚Π°Π²Π»ΡΡŽΡ‰Π΅ΠΉ силы. ΠšΡ€ΠΎΠΌΠ΅ Ρ‚ΠΎΠ³ΠΎ, сумма силы Ρ€Π΅Π°ΠΊΡ†ΠΈΠΈ RA ΠΈ RB Π΄ΠΎΠ»ΠΆΠ½Ρ‹ Π±Ρ‹Ρ‚ΡŒ Ρ€Π°Π²Π½Ρ‹ суммС силы, Π΄Π΅ΠΉΡΡ‚Π²ΡƒΡŽΡ‰ΠΈΠ΅ Π²Π½ΠΈΠ·.

Π Π΅Π°ΠΊΡ†ΠΈΠΈ ΠΏΡƒΡ‡ΠΊΠ°

Π’Π΅Π»ΠΈΡ‡ΠΈΠ½Ρƒ Ρ€Π΅Π°ΠΊΡ†ΠΈΠΈ ΠΌΠΎΠΆΠ½ΠΎ ΡƒΠ·Π½Π°Ρ‚ΡŒ ΠΈΠ· прилоТСния Ρ‚Ρ€Π΅Ρ‚ΡŒΠ΅Π³ΠΎ условия равновСсия, Ρ‚. Π΅. алгСбраичСской суммы ΠΌΠΎΠΌΠ΅Π½Ρ‚ΠΎΠ² сил ΠΎΡ‚Π½ΠΎΡΠΈΡ‚Π΅Π»ΡŒΠ½ΠΎ любой Ρ‚ΠΎΡ‡ΠΊΠΈ Π΄ΠΎΠ»ΠΆΠ½Ρ‹ Π±Ρ‹Ρ‚ΡŒ Ρ€Π°Π²Π½Ρ‹ Π½ΡƒΠ»ΡŽ.

Π’ΠΎΠ·ΡŒΠΌΠΈΡ‚Π΅ ΠΌΠΎΠΌΠ΅Π½Ρ‚Ρ‹ Π²ΠΎΠΊΡ€ΡƒΠ³ Ρ‚ΠΎΡ‡ΠΊΠΈ A, Π·Π°Ρ‚Π΅ΠΌ:

(80 x 2) + (70 x 4) + (100 x 7) + (30 x 10) — (R B Ρ… 12) = 0;

R B = 120 кН

RA Ρ‚Π΅ΠΏΠ΅Ρ€ΡŒ Π»Π΅Π³ΠΊΠΎ Π½Π°ΠΉΡ‚ΠΈ с ΠΏΡ€ΠΈΠΌΠ΅Π½Π΅Π½ΠΈΠ΅ΠΌ Π²Ρ‚ΠΎΡ€ΠΎΠ³ΠΎ условиС равновСсия.

R A = 75-70-100-30+ R B = 0; RB = 120 кН Π΄Π°Π΅Ρ‚:

R A = 160 кН.

ΠŸΠ°Ρ€Ρ‹

Π”Π²Π΅ Ρ€Π°Π²Π½Ρ‹Π΅, ΠΏΠ°Ρ€Π°Π»Π»Π΅Π»ΡŒΠ½Ρ‹Π΅ ΠΈ ΠΏΡ€ΠΎΡ‚ΠΈΠ²ΠΎΠΏΠΎΠ»ΠΎΠΆΠ½Ρ‹Π΅, Π½ΠΎ Π½Π΅ ΠΊΠΎΠ»Π»ΠΈΠ½Π΅Π°Ρ€Π½Ρ‹Π΅ силы Π΄Π΅ΠΉΡΡ‚Π²ΡƒΡŽΡ‚. говорят, Ρ‡Ρ‚ΠΎ это ΠΏΠ°Ρ€Π°.

ΠŸΠ°Ρ€Π°, Π΄Π΅ΠΉΡΡ‚Π²ΡƒΡŽΡ‰Π°Ρ Π½Π° Ρ‚Π΅Π»ΠΎ, ΠΏΡ€ΠΎΠΈΠ·Π²ΠΎΠ΄ΠΈΡ‚ Π²Ρ€Π°Ρ‰Π΅Π½ΠΈΠ΅. ΠžΠ±Ρ€Π°Ρ‚ΠΈΡ‚Π΅ Π²Π½ΠΈΠΌΠ°Π½ΠΈΠ΅, Ρ‡Ρ‚ΠΎ ΠΏΠ°Ρ€Ρƒ нСльзя ΡƒΡ€Π°Π²Π½ΠΎΠ²Π΅ΡΠΈΡ‚ΡŒ ΠΎΠ΄Π½ΠΎΠΉ силой. ΠŸΡ€ΠΎΠΈΠ·Π²ΠΎΠ΄ΠΈΡ‚ΡŒ равновСсиС другая ΠΏΠ°Ρ€Π° Ρ€Π°Π²Π½ΠΎΠ³ΠΎ ΠΈ ΠΏΡ€ΠΎΡ‚ΠΈΠ²ΠΎΠΏΠΎΠ»ΠΎΠΆΠ½ΠΎΠ³ΠΎ ΠΌΠΎΠΌΠ΅Π½Ρ‚Π° ΠΎΠ±ΡΠ·Π°Ρ‚Π΅Π»ΡŒΠ½Ρ‹ΠΉ.

ΠŸΠ°Ρ€Ρ‹

ΠŸΠΎΠ³Ρ€ΡƒΠ·ΠΎΡ‡Π½Ρ‹Π΅ систСмы

ΠŸΠ΅Ρ€Π΅Π΄ Π»ΡŽΠ±Ρ‹ΠΌ ΠΈΠ· Ρ€Π°Π·Π»ΠΈΡ‡Π½Ρ‹Ρ… воздСйствий Π½Π°Π³Ρ€ΡƒΠ·ΠΊΠΈ (растяТСниС, сТатиС, ΠΈΠ·Π³ΠΈΠ± ΠΈ Ρ‚. Π΄.), ΠΏΡ€ΠΈΠ»ΠΎΠΆΠ΅Π½Π½Ρ‹Π΅ Π½Π°Π³Ρ€ΡƒΠ·ΠΊΠΈ Π΄ΠΎΠ»ΠΆΠ½Ρ‹ Π±Ρ‹Ρ‚ΡŒ Ρ€Π°Ρ†ΠΈΠΎΠ½Π°Π»ΠΈΠ·ΠΈΡ€ΠΎΠ²Π°Π½ Π² ряд упорядочСнных систСм. НСравномСрная Π·Π°Π³Ρ€ΡƒΠ·ΠΊΠ° Ρ‚Ρ€ΡƒΠ΄Π½ΠΎ ΡΠΏΡ€Π°Π²ΠΈΡ‚ΡŒΡΡ Ρ‚ΠΎΡ‡Π½ΠΎ, Π½ΠΎ Π΄Π°ΠΆΠ΅ самыС нСстандартныС Π½Π°Π³Ρ€ΡƒΠ·ΠΊΠΈ ΠΌΠΎΠ³ΡƒΡ‚ Π±Ρ‹Ρ‚ΡŒ ΡƒΠΌΠ΅Π½ΡŒΡˆΠ΅Π½Ρ‹ ΠΈ ΠΏΡ€ΠΈΠ±Π»ΠΈΠΆΠ΅Π½Ρ‹ ΠΊ количСству ΠΎΠ±Ρ‹Ρ‡Π½Ρ‹Ρ… систСмы. Π—Π°Ρ‚Π΅ΠΌ с Π½ΠΈΠΌΠΈ ΠΌΠΎΠΆΠ½ΠΎ ΡΠΏΡ€Π°Π²ΠΈΡ‚ΡŒΡΡ матСматичСски, ΠΈΡΠΏΠΎΠ»ΡŒΠ·ΡƒΡ ΠΏΡ€ΠΈΠ½Ρ†ΠΈΠΏ налоТСния для ΠΎΡ†Π΅Π½ΠΊΠΈ ΠΎΠ±Ρ‰Π΅ΠΉ ΠΊΠΎΠΌΠ±ΠΈΠ½ΠΈΡ€ΠΎΠ²Π°Π½Π½ΠΎΠΉ эффСкт.

БосрСдоточСнныС Π½Π°Π³Ρ€ΡƒΠ·ΠΊΠΈ — это Π½Π°Π³Ρ€ΡƒΠ·ΠΊΠΈ, ΠΊΠΎΡ‚ΠΎΡ€Ρ‹Π΅, ΠΊΠ°ΠΊ ΠΌΠΎΠΆΠ½ΠΎ ΠΏΡ€Π΅Π΄ΠΏΠΎΠ»ΠΎΠΆΠΈΡ‚ΡŒ, Π΄Π΅ΠΉΡΡ‚Π²ΡƒΡŽΡ‚ Π½Π° СдинствСнная Ρ‚ΠΎΡ‡ΠΊΠ°, Π½Π°ΠΏΡ€ΠΈΠΌΠ΅Ρ€, Π³Ρ€ΡƒΠ·, ΡΠ²ΠΈΡΠ°ΡŽΡ‰ΠΈΠΉ с ΠΏΠΎΡ‚ΠΎΠ»ΠΊΠ°, ΠΈΠ»ΠΈ Ρ‡Π΅Π»ΠΎΠ²Π΅ΠΊ толкая ΠΊΠΎΡ€ΠΎΠ±ΠΊΡƒ.

ΠšΠΎΠ½Ρ†Π΅Π½Ρ‚Ρ€ΠΈΡ€ΠΎΠ²Π°Π½Π½Ρ‹Π΅ Π½Π°Π³Ρ€ΡƒΠ·ΠΊΠΈ прСдставлСны ΠΎΠ΄Π½ΠΎΠΉ стрСлкой, нарисованной Π½Π° Π½Π°ΠΏΡ€Π°Π²Π»Π΅Π½ΠΈΠ΅ ΠΈ Ρ‡Π΅Ρ€Π΅Π· Ρ‚ΠΎΡ‡ΠΊΡƒ дСйствия силы. Π’ всСгда указываСтся Π²Π΅Π»ΠΈΡ‡ΠΈΠ½Π° силы.

Π Π°Π²Π½ΠΎΠΌΠ΅Ρ€Π½ΠΎ распрСдСлСнныС Π½Π°Π³Ρ€ΡƒΠ·ΠΊΠΈ, ΠΎΠ±ΠΎΠ·Π½Π°Ρ‡Π°Π΅ΠΌΡ‹Π΅ ΠΊΠ°ΠΊ u.d.l. Ρ‚Π΅, ΠΊΠΎΡ‚ΠΎΡ€Ρ‹Π΅ ΠΌΠΎΠΆΠ½ΠΎ ΠΏΡ€Π΅Π΄ΠΏΠΎΠ»ΠΎΠΆΠΈΡ‚ΡŒ, Ρ‡Ρ‚ΠΎ дСйствуСт Ρ€Π°Π²Π½ΠΎΠΌΠ΅Ρ€Π½ΠΎ ΠΏΠΎ ΠΏΠ»ΠΎΡ‰Π°Π΄ΠΈ ΠΈΠ»ΠΈ ΠΏΠΎ Π΄Π»ΠΈΠ½Π΅ конструктивного элСмСнта, e.Π³., Π½Π°Π³Ρ€ΡƒΠ·ΠΊΠΈ Π½Π° ΠΊΡ€Ρ‹ΡˆΡƒ, Π²Π΅Ρ‚Ρ€ΠΎΠ²Ρ‹Π΅ Π½Π°Π³Ρ€ΡƒΠ·ΠΊΠΈ, влияниС вСса Π²ΠΎΠ΄Ρ‹ Π½Π° Π³ΠΎΡ€ΠΈΠ·ΠΎΠ½Ρ‚Π°Π»ΡŒΠ½ΠΎΠΉ повСрхности ΠΈ Ρ‚. Π΄.

Для Ρ†Π΅Π»Π΅ΠΉ расчСта u.d.l. ΠΎΠ±Ρ‹Ρ‡Π½ΠΎ рассматриваСтся Π² плоскости ΠΈ прСдставляСтся, ΠΊΠ°ΠΊ ΠΏΠΎΠΊΠ°Π·Π°Π½ΠΎ.

ΠŸΡ€ΠΈ расчСтС Ρ€Π΅Π°ΠΊΡ†ΠΈΠΉ Ρ€Π°Π²Π½ΠΎΠΌΠ΅Ρ€Π½ΠΎ распрСдСлСнныС Π½Π°Π³Ρ€ΡƒΠ·ΠΊΠΈ ΠΌΠΎΠ³ΡƒΡ‚ Π±ΠΎΠ»ΡŒΡˆΠΈΠ½ΡΡ‚Π²ΠΎ, Π½ΠΎ Π½Π΅ всС случаи ΠΌΠΎΠ³ΡƒΡ‚ Π±Ρ‹Ρ‚ΡŒ прСдставлСны сосрСдоточСнной Π½Π°Π³Ρ€ΡƒΠ·ΠΊΠΎΠΉ Ρ€Π°Π²Π½ΠΎΠΉ ΠΎΠ±Ρ‰Π΅ΠΉ распрСдСлСнной Π½Π°Π³Ρ€ΡƒΠ·ΠΊΠ΅ ΠΈ проходящСй Ρ‡Π΅Ρ€Π΅Π· Ρ†Π΅Π½Ρ‚Ρ€ тяТСсти распрСдСлСнной Π½Π°Π³Ρ€ΡƒΠ·ΠΊΠΈ.

Π­Ρ‚ΠΎΡ‚ ΠΌΠ΅Ρ‚ΠΎΠ΄ нСльзя ΠΈΡΠΏΠΎΠ»ΡŒΠ·ΠΎΠ²Π°Ρ‚ΡŒ для расчСта сдвига. сила, ΠΈΠ·Π³ΠΈΠ±Π°ΡŽΡ‰ΠΈΠΉ ΠΌΠΎΠΌΠ΅Π½Ρ‚ ΠΈΠ»ΠΈ ΠΏΡ€ΠΎΠ³ΠΈΠ±.

ΠŸΡ€ΠΈΠΌΠ΅Ρ€ 5

Рассмотрим подвСсной ΠΏΠΎΠ», Π² ΠΊΠΎΡ‚ΠΎΡ€ΠΎΠΌ Π½Π°Π³Ρ€ΡƒΠ·ΠΊΠΈ ΠΏΠΎΠ΄Π΄Π΅Ρ€ΠΆΠΈΠ²Π°ΡŽΡ‚ΡΡ Π½Π°Π±ΠΎΡ€ Π½Π΅Ρ€Π°Π²Π½ΠΎΠΌΠ΅Ρ€Π½ΠΎ располоТСнных Π±Π°Π»ΠΎΠΊ. ΠŸΡƒΡΡ‚ΡŒ Π½Π°Π³Ρ€ΡƒΠ·ΠΊΠ°, Π²ΠΎΠ·Π½ΠΈΠΊΠ°ΡŽΡ‰Π°Ρ ΠΎΡ‚ вСс самого ΠΏΠΎΠ»Π° ΠΈ вСс любого ΡƒΠ»ΠΎΠΆΠ΅Π½Π½ΠΎΠ³ΠΎ ΠΌΠ°Ρ‚Π΅Ρ€ΠΈΠ°Π»Π° ΠΏΠΎΠ²Π΅Ρ€Ρ… Π½Π΅Π³ΠΎ (Π½Π°ΠΏΡ€ΠΈΠΌΠ΅Ρ€, Ρ…Ρ€Π°Π½ΠΈΠΌΠΎΠ³ΠΎ Π·Π΅Ρ€Π½Π°) Π΄ΠΎΠ»ΠΆΠ½ΠΎ Π±Ρ‹Ρ‚ΡŒ 10 кН / ΠΌ. ΠžΠ±ΠΎΠ·Π½Π°Ρ‡ΠΈΡ‚ΡŒ u.d.l. воздСйствуСт Π½Π° Π»ΡƒΡ‡ A ΠΈ Π»ΡƒΡ‡ C.

ЭВАЖНАЯ ЧАБВЬ

Из рисунка Π½ΠΈΠΆΠ΅ Π²ΠΈΠ΄Π½ΠΎ, Ρ‡Ρ‚ΠΎ Π±Π°Π»ΠΊΠ° A нСсСт Π½Π°Π³Ρ€ΡƒΠ·ΠΊΠΈ Π½Π° ΠΏΠ΅Ρ€Π΅ΠΊΡ€Ρ‹Ρ‚ΠΈΠ΅, создаваСмыС ΠΏΠΎΠ»ΠΎΠ²ΠΈΠ½ΠΎΠΉ ΠΏΠ»ΠΎΡ‰Π°Π΄ΠΈ ΠΌΠ΅ΠΆΠ΄Ρƒ Π±Π°Π»ΠΊΠ°ΠΌΠΈ А ΠΈ B i.Π΅., Π·Π°ΡˆΡ‚Ρ€ΠΈΡ…ΠΎΠ²Π°Π½Π½Π°Ρ ΠΎΠ±Π»Π°ΡΡ‚ΡŒ L. Π‘Π°Π»ΠΊΠ° C нСсСт ΠΏΠΎ Π·Π°ΡˆΡ‚Ρ€ΠΈΡ…ΠΎΠ²Π°Π½Π½ΠΎΠΉ Π·ΠΎΠ½Π΅ ΠΌ.

БСкция ΠΏΠΎΠ»Π°, Ρ‡Π°ΡΡ‚ΡŒ 2

Π‘Π»Π΅Π΄ΠΎΠ²Π°Ρ‚Π΅Π»ΡŒΠ½ΠΎ, Π±Π°Π»ΠΊΠ° A нСсСт ΠΎΠ±Ρ‰ΡƒΡŽ Π½Π°Π³Ρ€ΡƒΠ·ΠΊΡƒ:

1 м x 4 м x 10 кН / м = 40 кН или 40 кН / 4 = 10 кН / м.

Π’Π°ΠΊΠΈΠΌ ΠΆΠ΅ ΠΎΠ±Ρ€Π°Π·ΠΎΠΌ ΠΌΠΎΠΆΠ½ΠΎ Ρ€Π°ΡΡΡ‡ΠΈΡ‚Π°Ρ‚ΡŒ Π½Π°Π³Ρ€ΡƒΠ·ΠΊΡƒ Π½Π° Π±Π°Π»ΠΊΡƒ C 25кН / ΠΌ. Π—Π°Ρ‚Π΅ΠΌ ΠΌΠΎΠΆΠ½ΠΎ ΠΈΡΠΏΠΎΠ»ΡŒΠ·ΠΎΠ²Π°Ρ‚ΡŒ Π½Π°Π³Ρ€ΡƒΠ·ΠΊΡƒ Π½Π° ΠΌΠ΅Ρ‚Ρ€ ΠΏΡ€ΠΎΠ±Π΅Π³Π° для расчСта Π½Π΅ΠΎΠ±Ρ…ΠΎΠ΄ΠΈΠΌΡ‹ΠΉ Ρ€Π°Π·ΠΌΠ΅Ρ€ Π±Π°Π»ΠΎΠΊ.

РаспрСдСлСнныС Π½Π°Π³Ρ€ΡƒΠ·ΠΊΠΈ с Π»ΠΈΠ½Π΅ΠΉΠ½Ρ‹ΠΌ ΠΈΠ·ΠΌΠ΅Π½Π΅Π½ΠΈΠ΅ΠΌ — Π΅Ρ‰Π΅ ΠΎΠ΄Π½Π° распространСнная Π½Π°Π³Ρ€ΡƒΠ·ΠΊΠ°. ситуация.

Нагрузочная Ρ„ΠΎΡ€ΠΌΠ° Ρ‚Ρ€Π΅ΡƒΠ³ΠΎΠ»ΡŒΠ½Π°Ρ ΠΈ являСтся Ρ€Π΅Π·ΡƒΠ»ΡŒΡ‚Π°Ρ‚ΠΎΠΌ Ρ‚Π°ΠΊΠΎΠ³ΠΎ дСйствия ΠΊΠ°ΠΊ Π΄Π°Π²Π»Π΅Π½ΠΈΠ΅ Π²ΠΎΠ΄Ρ‹ Π½Π° ΠΏΠΎΠ΄ΠΏΠΎΡ€Π½Ρ‹Π΅ стСны ΠΈ ΠΏΠ»ΠΎΡ‚ΠΈΠ½Ρ‹.

Π—Π°Π³Ρ€ΡƒΠ·ΠΊΠ° Π±Π°Π»ΠΊΠΈ C

Π‘ΠΈΠ»Π° сдвига ΠΈ ΠΈΠ·Π³ΠΈΠ±Π°ΡŽΡ‰ΠΈΠΉ ΠΌΠΎΠΌΠ΅Π½Ρ‚

Π‘Π°Π»ΠΊΠ° — это элСмСнт конструкции, ΠΏΠΎΠ΄Π²Π΅Ρ€ΠΆΠ΅Π½Π½Ρ‹ΠΉ Π±ΠΎΠΊΠΎΠ²ΠΎΠΉ Π½Π°Π³Ρ€ΡƒΠ·ΠΊΠ΅ Π² ΠΊΠΎΡ‚ΠΎΡ€ΠΎΠ΅ проявлСнноС сопротивлСниС Π΄Π΅Ρ„ΠΎΡ€ΠΌΠ°Ρ†ΠΈΠΈ являСтся ΠΈΠ·Π³ΠΈΠ±Π½Ρ‹ΠΌ пСрсонаТ. Π­Ρ„Ρ„Π΅ΠΊΡ‚ ΠΏΠ΅Ρ€Π²ΠΈΡ‡Π½ΠΎΠΉ Π½Π°Π³Ρ€ΡƒΠ·ΠΊΠΈ, Π½Π° ΠΊΠΎΡ‚ΠΎΡ€Ρ‹ΠΉ рассчитана Π±Π°Π»ΠΊΠ°. сопротивлСниС — это ΠΈΠ·Π³ΠΈΠ±Π°ΡŽΡ‰ΠΈΠ΅ ΠΌΠΎΠΌΠ΅Π½Ρ‚Ρ‹, Π½ΠΎ, ΠΊΡ€ΠΎΠΌΠ΅ Ρ‚ΠΎΠ³ΠΎ, Π½Π΅ΠΎΠ±Ρ…ΠΎΠ΄ΠΈΠΌΠΎ ΡƒΡ‡ΠΈΡ‚Ρ‹Π²Π°Ρ‚ΡŒ ΠΏΠΎΠΏΠ΅Ρ€Π΅Ρ‡Π½Ρ‹Π΅ ΠΈΠ»ΠΈ Π²Π΅Ρ€Ρ‚ΠΈΠΊΠ°Π»ΡŒΠ½Ρ‹Π΅ силы сдвига.

Рассмотрим консоль AB, ΠΏΠΎΠΊΠ°Π·Π°Π½Π½ΡƒΡŽ Π½Π° (Π°). Для равновСсия сила Ρ€Π΅Π°ΠΊΡ†ΠΈΠΈ Π² Ρ‚ΠΎΡ‡ΠΊΠ΅ A Π΄ΠΎΠ»ΠΆΠ½Π° Π±Ρ‹Ρ‚ΡŒ Π²Π΅Ρ€Ρ‚ΠΈΠΊΠ°Π»ΡŒΠ½ΠΎΠΉ ΠΈ Ρ€Π°Π²Π½ΠΎΠΉ Π½Π°Π³Ρ€ΡƒΠ·ΠΊΠ΅ W.

Π‘Π»Π΅Π΄ΠΎΠ²Π°Ρ‚Π΅Π»ΡŒΠ½ΠΎ, консоль Π΄ΠΎΠ»ΠΆΠ½Π° ΠΏΠ΅Ρ€Π΅Π΄Π°Π²Π°Ρ‚ΡŒ воздСйствиС Π½Π°Π³Ρ€ΡƒΠ·ΠΊΠΈ W Π½Π° ΠΏΠΎΠ΄Π΄Π΅Ρ€ΠΆΠΊΠ° Π² Ρ‚ΠΎΡ‡ΠΊΠ΅ A ΠΏΡƒΡ‚Π΅ΠΌ развития сопротивлСния (ΠΏΠΎ Π²Π΅Ρ€Ρ‚ΠΈΠΊΠ°Π»ΠΈ плоскости сСчСния ΠΌΠ΅ΠΆΠ΄Ρƒ Π³Ρ€ΡƒΠ·ΠΎΠΌ ΠΈ ΠΎΠΏΠΎΡ€ΠΎΠΉ) ΠΊ Π­Ρ„Ρ„Π΅ΠΊΡ‚ Π½Π°Π³Ρ€ΡƒΠ·ΠΊΠΈ называСтся силой сдвига. ΠΠ΅ΡΠΏΠΎΡΠΎΠ±Π½ΠΎΡΡ‚ΡŒ ΠΏΠ΅Ρ€Π΅Π΄Π°Ρ‚ΡŒ сила сдвига Π² любом Π·Π°Π΄Π°Π½Π½ΠΎΠΌ сСчСнии, Π½Π°ΠΏΡ€ΠΈΠΌΠ΅Ρ€, сСчСнии x-x, Π±ΡƒΠ΄Π΅Ρ‚ привСсти ΠΊ Ρ€Π°Π·Ρ€ΡƒΡˆΠ΅Π½ΠΈΡŽ Π±Π°Π»ΠΊΠΈ, ΠΊΠ°ΠΊ ΠΏΠΎΠΊΠ°Π·Π°Π½ΠΎ Π½Π° рисункС (b).Π˜Π·Π³ΠΈΠ±Π°ΡŽΡ‰ΠΈΠΉ эффСкт Π½Π°Π³Ρ€ΡƒΠ·ΠΊΠ° Π²Ρ‹Π·ΠΎΠ²Π΅Ρ‚ Π΄Π΅Ρ„ΠΎΡ€ΠΌΠ°Ρ†ΠΈΡŽ Π±Π°Π»ΠΊΠΈ, ΠΊΠ°ΠΊ ΠΏΠΎΠΊΠ°Π·Π°Π½ΠΎ Π² (c). Π§Ρ‚ΠΎΠ±Ρ‹ ΠΏΡ€Π΅Π΄ΠΎΡ‚Π²Ρ€Π°Ρ‚ΠΈΡ‚ΡŒ Π²Ρ€Π°Ρ‰Π΅Π½ΠΈΠ΅ Π±Π°Π»ΠΊΠΈ Π½Π° ΠΎΠΏΠΎΡ€Π΅ A Π΄ΠΎΠ»ΠΆΠ΅Π½ Π±Ρ‹Ρ‚ΡŒ ΠΌΠΎΠΌΠ΅Π½Ρ‚ Ρ€Π΅Π°ΠΊΡ†ΠΈΠΈ ΠΏΡ€ΠΈ A, ΠΎΠ±ΠΎΠ·Π½Π°Ρ‡Π΅Π½Π½Ρ‹ΠΉ ΠΊΠ°ΠΊ M A , Ρ‡Ρ‚ΠΎ Ρ€Π°Π²Π½ΠΎ ΠΏΡ€ΠΎΠΈΠ·Π²Π΅Π΄Π΅Π½ΠΈΡŽ Π½Π°Π³Ρ€ΡƒΠ·ΠΊΠΈ W ΠΈ расстояниС ΠΎΡ‚ W Π΄ΠΎ Ρ‚ΠΎΡ‡ΠΊΠΈ A.

УсилиС сдвига ΠΈ ΠΈΠ·Π³ΠΈΠ±Π°ΡŽΡ‰ΠΈΠΉ ΠΌΠΎΠΌΠ΅Π½Ρ‚, ΠΏΠ΅Ρ€Π΅Π΄Π°Π²Π°Π΅ΠΌΡ‹Π΅ Ρ‡Π΅Ρ€Π΅Π· сСчСниС x-x ΠΌΠΎΠΆΠ½ΠΎ Ρ€Π°ΡΡΠΌΠ°Ρ‚Ρ€ΠΈΠ²Π°Ρ‚ΡŒ ΠΊΠ°ΠΊ силу ΠΈ ΠΌΠΎΠΌΠ΅Π½Ρ‚ соотвСтствСнно, ΠΊΠΎΡ‚ΠΎΡ€Ρ‹Π΅ Π½Π΅ΠΎΠ±Ρ…ΠΎΠ΄ΠΈΠΌΡ‹ для поддСрТания равновСсия, Ссли Ρ€Π°Π·Ρ€Π΅Π· производится рассСчСниС Π±Π°Π»ΠΊΠΈ Π² Ρ‚ΠΎΡ‡ΠΊΠ°Ρ… x-x.Π”ΠΈΠ°Π³Ρ€Π°ΠΌΠΌΡ‹ свободного Ρ‚Π΅Π»Π° Π΄Π²Π΅ части Π±Π°Π»ΠΊΠΈ ΠΏΠΎΠΊΠ°Π·Π°Π½Ρ‹ Π½Π° (d).

Π’ΠΎΠ³Π΄Π° сила сдвига ΠΌΠ΅ΠΆΠ΄Ρƒ A ΠΈ C = Q x = W

ΠΈ ΠΈΠ·Π³ΠΈΠ±Π°ΡŽΡ‰ΠΈΠΉ ΠΌΠΎΠΌΠ΅Π½Ρ‚ ΠΌΠ΅ΠΆΠ΄Ρƒ A ΠΈ C = M x = W x

ΠŸΡ€ΠΈΠΌΠ΅Ρ‡Π°Π½ΠΈΠ΅: сила сдвига ΠΈ ΠΈΠ·Π³ΠΈΠ±Π°ΡŽΡ‰ΠΈΠΉ ΠΌΠΎΠΌΠ΅Π½Ρ‚ Π±ΡƒΠ΄ΡƒΡ‚ ноль ΠΌΠ΅ΠΆΠ΄Ρƒ C ΠΈ B.

Π‘ΠΈΠ»Π° сдвига, Ρ‡Π°ΡΡ‚ΡŒ 1

Π‘ΠΈΠ»Π° сдвига, Ρ‡Π°ΡΡ‚ΡŒ 2

ΠžΠΏΡ€Π΅Π΄Π΅Π»Π΅Π½ΠΈΡ

Π‘ΠΈΠ»Π° сдвига (Q) — это алгСбраичСская сумма всСх ΠΏΠΎΠΏΠ΅Ρ€Π΅Ρ‡Π½Ρ‹Ρ… силы, Π΄Π΅ΠΉΡΡ‚Π²ΡƒΡŽΡ‰ΠΈΠ΅ слСва ΠΈΠ»ΠΈ справа ΠΎΡ‚ Π²Ρ‹Π±Ρ€Π°Π½Π½ΠΎΠ³ΠΎ участка.

Π˜Π·Π³ΠΈΠ±Π°ΡŽΡ‰ΠΈΠΉ ΠΌΠΎΠΌΠ΅Π½Ρ‚ (M) Π½Π° любом ΠΏΠΎΠΏΠ΅Ρ€Π΅Ρ‡Π½ΠΎΠΌ сСчСнии прямая Π±Π°Π»ΠΊΠ° — это алгСбраичСская сумма ΠΌΠΎΠΌΠ΅Π½Ρ‚ΠΎΠ², взятых ΠΎΠΊΠΎΠ»ΠΎ ось, проходящая Ρ‡Π΅Ρ€Π΅Π· Ρ†Π΅Π½Ρ‚Ρ€ тяТСсти ΠΏΠΎΠΏΠ΅Ρ€Π΅Ρ‡Π½ΠΎΠ³ΠΎ сСчСния, всСх силы, ΠΏΡ€ΠΈΠ»ΠΎΠΆΠ΅Π½Π½Ρ‹Π΅ ΠΊ Π±Π°Π»ΠΊΠ΅ ΠΏΠΎ ΠΎΠ±Π΅ стороны ΠΎΡ‚ Π²Ρ‹Π±Ρ€Π°Π½Π½ΠΎΠ³ΠΎ крСста Ρ€Π°Π·Π΄Π΅Π».

Π’Π°Π±Π»ΠΈΡ†Π° 4.2. Π˜Π·Π³ΠΈΠ±Π°ΡŽΡ‰ΠΈΠ΅ силы

ИзмСнСниС силы сдвига

ΠšΠΎΠ½Ρ†Π΅Π½Ρ‚Ρ€ΠΈΡ€ΠΎΠ²Π°Π½Π½Ρ‹Π΅ Π½Π°Π³Ρ€ΡƒΠ·ΠΊΠΈ ΠΈΠ·ΠΌΠ΅Π½ΡΡŽΡ‚ Π·Π½Π°Ρ‡Π΅Π½ΠΈΠ΅ ΠΏΠΎΠΏΠ΅Ρ€Π΅Ρ‡Π½ΠΎΠΉ силы Ρ‚ΠΎΠ»ΡŒΠΊΠΎ Π² Ρ‚Π΅Ρ… мСстах, Π³Π΄Π΅ ΠΎΠ½ΠΈ Π²ΠΎΠ·Π½ΠΈΠΊΠ°ΡŽΡ‚, Ρ‚.Π΅.Π΅., попСрСчная сила остаСтся постоянная ΠΌΠ΅ΠΆΠ΄Ρƒ Π½ΠΈΠΌΠΈ. Когда Π½Π°Π³Ρ€ΡƒΠ·ΠΊΠ° распрСдСлСна Ρ€Π°Π²Π½ΠΎΠΌΠ΅Ρ€Π½ΠΎ, ΠΎΠ΄Π½Π°ΠΊΠΎ сила сдвига Π±ΡƒΠ΄Π΅Ρ‚ ΠΈΠ·ΠΌΠ΅Π½ΡΡ‚ΡŒΡΡ с постоянной ΡΠΊΠΎΡ€ΠΎΡΡ‚ΡŒΡŽ. Π’Π°ΠΊΠΈΠΌ ΠΎΠ±Ρ€Π°Π·ΠΎΠΌ Π±ΡƒΠ΄Π΅Ρ‚ Π²ΠΈΠ΄Π½ΠΎ, Ρ‡Ρ‚ΠΎ Ρ€Π°Π²Π½ΠΎΠΌΠ΅Ρ€Π½Ρ‹Π΅ Π½Π°Π³Ρ€ΡƒΠ·ΠΊΠΈ Π²Ρ‹Π·Ρ‹Π²Π°ΡŽΡ‚ постСпСнноС ΠΈ Ρ€Π°Π²Π½ΠΎΠΌΠ΅Ρ€Π½ΠΎΠ΅ ΠΈΠ·ΠΌΠ΅Π½Π΅Π½ΠΈΠ΅ сдвига, Π² Ρ‚ΠΎ врСмя ΠΊΠ°ΠΊ сосрСдоточСнныС Π½Π°Π³Ρ€ΡƒΠ·ΠΊΠΈ приводят ΠΊ Π²Π½Π΅Π·Π°ΠΏΠ½ΠΎΠΌΡƒ измСнСнию Π·Π½Π°Ρ‡Π΅Π½ΠΈΠ΅ ΠΏΠΎΠΏΠ΅Ρ€Π΅Ρ‡Π½ΠΎΠΉ силы.

Вариация ΠΈΠ·Π³ΠΈΠ±Π°ΡŽΡ‰Π΅Π³ΠΎ ΠΌΠΎΠΌΠ΅Π½Ρ‚Π°

БосрСдоточСнныС Π½Π°Π³Ρ€ΡƒΠ·ΠΊΠΈ Π²Ρ‹Π·Ρ‹Π²Π°ΡŽΡ‚ Ρ€Π°Π²Π½ΠΎΠΌΠ΅Ρ€Π½ΠΎΠ΅ ΠΈΠ·ΠΌΠ΅Π½Π΅Π½ΠΈΠ΅ ΠΈΠ·Π³ΠΈΠ±Π° ΠΌΠΎΠΌΠ΅Π½Ρ‚ ΠΌΠ΅ΠΆΠ΄Ρƒ Ρ‚ΠΎΡ‡ΠΊΠ°ΠΌΠΈ дСйствия Π½Π°Π³Ρ€ΡƒΠ·ΠΎΠΊ.На случай, Ссли Ρ€Π°Π²Π½ΠΎΠΌΠ΅Ρ€Π½ΠΎ распрСдСлСнныС Π½Π°Π³Ρ€ΡƒΠ·ΠΊΠΈ, ΡΠΊΠΎΡ€ΠΎΡΡ‚ΡŒ измСнСния ΠΈΠ·Π³ΠΈΠ±Π° ΠΌΠΎΠΌΠ΅Π½Ρ‚ Π±ΡƒΠ΄Π΅Ρ‚ параболичСским. ΠœΠ°ΠΊΡΠΈΠΌΠ°Π»ΡŒΠ½Ρ‹Π΅ значСния ΠΈΠ·Π³ΠΈΠ±Π°ΡŽΡ‰Π΅Π³ΠΎ ΠΌΠΎΠΌΠ΅Π½Ρ‚Π° Π±ΡƒΠ΄ΡƒΡ‚ Π²ΠΎΠ·Π½ΠΈΠΊΠ°ΡŽΡ‚ Ρ‚Π°ΠΌ, Π³Π΄Π΅ сила сдвига Ρ€Π°Π²Π½Π° Π½ΡƒΠ»ΡŽ ΠΈΠ»ΠΈ ΠΊΠΎΠ³Π΄Π° ΠΎΠ½Π° мСняСт Π·Π½Π°ΠΊ.

Π”ΠΈΠ°Π³Ρ€Π°ΠΌΠΌΡ‹ ΠΏΠΎΠΏΠ΅Ρ€Π΅Ρ‡Π½Ρ‹Ρ… сил ΠΈ ΠΈΠ·Π³ΠΈΠ±Π°ΡŽΡ‰ΠΈΡ… ΠΌΠΎΠΌΠ΅Π½Ρ‚ΠΎΠ²

Π’ΠΈΠΏΠΈΡ‡Π½Ρ‹Π΅ Π΄ΠΈΠ°Π³Ρ€Π°ΠΌΠΌΡ‹ распрСдСлСния усилия сдвига ΠΈ ΠΈΠ·Π³ΠΈΠ±Π°ΡŽΡ‰ΠΈΠΉ ΠΌΠΎΠΌΠ΅Π½Ρ‚ часто Ρ‚Ρ€Π΅Π±ΡƒΡŽΡ‚ΡΡ Π½Π° Π½Π΅ΡΠΊΠΎΠ»ΡŒΠΊΠΈΡ… этапах процСсс проСктирования. Π­Ρ‚ΠΈ Π΄ΠΈΠ°Π³Ρ€Π°ΠΌΠΌΡ‹ ΠΏΠΎΠ»ΡƒΡ‡Π΅Π½Ρ‹ ΠΏΡƒΡ‚Π΅ΠΌ построСния Π³Ρ€Π°Ρ„ΠΈΠΊΠΎΠ². с Π±Π°Π»ΠΊΠ°ΠΌΠΈ Π² качСствС основы ΠΈ значСниями ΠΊΠΎΠ½ΠΊΡ€Π΅Ρ‚Π½Ρ‹Ρ… эффСкт Π² Π²ΠΈΠ΄Π΅ ΠΎΡ€Π΄ΠΈΠ½Π°Ρ‚.Π­Ρ‚ΠΈ Π΄ΠΈΠ°Π³Ρ€Π°ΠΌΠΌΡ‹ ΠΎΠ±Ρ‹Ρ‡Π½ΠΎ строят Π² Π½Π°Π±ΠΎΡ€Ρ‹ ΠΈΠ· Ρ‚Ρ€Π΅Ρ…, ΠΏΡ€Π΅Π΄ΡΡ‚Π°Π²Π»ΡΡŽΡ‰ΠΈΠ΅ распрСдСлСниС Π½Π°Π³Ρ€ΡƒΠ·ΠΎΠΊ, сдвиг силы ΠΈ ΠΈΠ·Π³ΠΈΠ±Π°ΡŽΡ‰ΠΈΠ΅ ΠΌΠΎΠΌΠ΅Π½Ρ‚Ρ‹ соотвСтствСнно. Π­Ρ‚ΠΈ графичСскиС ΠŸΡ€Π΅Π΄ΡΡ‚Π°Π²ΠΈΡ‚Π΅Π»ΡŒΡΡ‚Π²Π° ΠΏΡ€Π΅Π΄ΠΎΡΡ‚Π°Π²Π»ΡΡŽΡ‚ ΠΏΠΎΠ»Π΅Π·Π½ΡƒΡŽ ΠΈΠ½Ρ„ΠΎΡ€ΠΌΠ°Ρ†ΠΈΡŽ ΠΎΡ‚Π½ΠΎΡΠΈΡ‚Π΅Π»ΡŒΠ½ΠΎ:

  • a Π½Π°ΠΈΠ±ΠΎΠ»Π΅Π΅ вСроятный участок, Π³Π΄Π΅ Π±Π°Π»ΠΊΠ° ΠΌΠΎΠΆΠ΅Ρ‚ Ρ€Π°Π·Ρ€ΡƒΡˆΠΈΡ‚ΡŒΡΡ ΠΏΡ€ΠΈ сдвигС ΠΈΠ»ΠΈ Π² Π³ΠΈΠ±ΠΊΠ΅
  • b, Π³Π΄Π΅ усилСниС ΠΌΠΎΠΆΠ΅Ρ‚ ΠΏΠΎΡ‚Ρ€Π΅Π±ΠΎΠ²Π°Ρ‚ΡŒΡΡ Π² ΠΎΠΏΡ€Π΅Π΄Π΅Π»Π΅Π½Π½Ρ‹Ρ… Ρ‚ΠΈΠΏΠ°Ρ… Π±Π°Π»ΠΊΠΈ, Π½Π°ΠΏΡ€ΠΈΠΌΠ΅Ρ€, Π±Π΅Ρ‚ΠΎΠ½Π½Ρ‹Π΅ Π±Π°Π»ΠΊΠΈ
  • c Π΄ΠΈΠ°Π³Ρ€Π°ΠΌΠΌΠ° ΠΏΠΎΠΏΠ΅Ρ€Π΅Ρ‡Π½ΠΎΠ³ΠΎ усилия прСдоставит ΠΏΠΎΠ»Π΅Π·Π½ΡƒΡŽ ΠΈΠ½Ρ„ΠΎΡ€ΠΌΠ°Ρ†ΠΈΡŽ ΠΎ ΠΈΠ·Π³ΠΈΠ±Π°ΡŽΡ‰Π΅ΠΌ ΠΌΠΎΠΌΠ΅Π½Ρ‚Π΅ Π² любой Ρ‚ΠΎΡ‡ΠΊΠ΅
  • d Π΄ΠΈΠ°Π³Ρ€Π°ΠΌΠΌΠ° ΠΈΠ·Π³ΠΈΠ±Π°ΡŽΡ‰Π΅Π³ΠΎ ΠΌΠΎΠΌΠ΅Π½Ρ‚Π° Π΄Π°Π΅Ρ‚ ΠΏΠΎΠ»Π΅Π·Π½ΡƒΡŽ ΠΈΠ½Ρ„ΠΎΡ€ΠΌΠ°Ρ†ΠΈΡŽ ΠΎ отклонСнная Ρ„ΠΎΡ€ΠΌΠ° Π±Π°Π»ΠΊΠΈ.

Π’ ΡΠ»Π΅Π΄ΡƒΡŽΡ‰Π΅ΠΌ ΠΏΡ€ΠΈΠΌΠ΅Ρ€Π΅ ΠΏΠΎΠΊΠ°Π·Π°Π½ΠΎ, ΠΊΠ°ΠΊ выглядят Ρ‚Ρ€ΠΈ Π΄ΠΈΠ°Π³Ρ€Π°ΠΌΠΌΡ‹. построСно:

ΠŸΡ€ΠΈΠΌΠ΅Ρ€ 6.1

НарисуйтС Π΄ΠΈΠ°Π³Ρ€Π°ΠΌΠΌΡƒ Π½Π°Π³Ρ€ΡƒΠ·ΠΊΠΈ Π½Π° Π±Π°Π»ΠΊΡƒ, ΠΏΠΎΠΊΠ°Π·Ρ‹Π²Π°ΡŽΡ‰ΡƒΡŽ всС Π½Π°Π³Ρ€ΡƒΠ·ΠΊΠΈ ΠΈ ΡΠΎΠΎΡ‚Π²Π΅Ρ‚ΡΡ‚Π²ΡƒΡŽΡ‰ΠΈΠ΅ Π“Π°Π±Π°Ρ€ΠΈΡ‚Π½Ρ‹Π΅ Ρ€Π°Π·ΠΌΠ΅Ρ€Ρ‹. Π­Ρ‚ΠΎ просто Π΄ΠΈΠ°Π³Ρ€Π°ΠΌΠΌΠ° свободного Ρ‚Π΅Π»Π° Π±Π°Π»ΠΊΠΈ.

2 ΠžΠΏΡ€Π΅Π΄Π΅Π»ΠΈΡ‚Π΅ Ρ€Π΅Π°ΠΊΡ†ΠΈΠΈ Π½Π° ΠΎΠΏΠΎΡ€Π°Ρ…. Π‘Π½Π°Ρ‡Π°Π»Π° ΠΈΡΠΏΠΎΠ»ΡŒΠ·ΡƒΠΉΡ‚Π΅ условиС равновСсия ΠΌΠΎΠΌΠ΅Π½Ρ‚ΠΎΠ² ΠΎΠΊΠΎΠ»ΠΎ Ρ‚ΠΎΡ‡ΠΊΠΈ

S M E = 0
M E = (P x a) + (w 1 x b x b 2 ) + w 2 x c (b + c / 2) — R G (b + c) = 0
M E = — (10 x 10) + (2 x 10 x 5) + 4 x 10 x (15) — R G (20) = 0
R G = 30 кН
S F y = 0, ΡΠ»Π΅Π΄ΠΎΠ²Π°Ρ‚Π΅Π»ΡŒΠ½ΠΎ,
S F y = R E + R G -P- (w 1 x b) — (w 2 x c) = 0
S F y = R E + 30-10 — (2 x 10) — (4 x 10) = 0
R E = 40 кН

3 НарисуйтС Π΄ΠΈΠ°Π³Ρ€Π°ΠΌΠΌΡƒ ΠΏΠΎΠΏΠ΅Ρ€Π΅Ρ‡Π½Ρ‹Ρ… сил (SFD) нСпосрСдствСнно ΠΏΠΎΠ΄ Π΄ΠΈΠ°Π³Ρ€Π°ΠΌΠΌΡƒ Π½Π°Π³Ρ€ΡƒΠ·ΠΊΠΈ ΠΈ Π²Ρ‹Π±Π΅Ρ€ΠΈΡ‚Π΅ ΡƒΠ΄ΠΎΠ±Π½Ρ‹ΠΉ ΠΌΠ°ΡΡˆΡ‚Π°Π± для прСдставлСния ΡΠ΄Π²ΠΈΠ³Π°ΡŽΡ‰Π°Ρ сила.

РассчитайтС значСния ΠΏΠΎΠΏΠ΅Ρ€Π΅Ρ‡Π½ΠΎΠΉ силы слСва ΠΈ справа ΠΎΡ‚ всСх критичСских Ρ‚ΠΎΡ‡Π΅ΠΊ. ΠšΡ€ΠΈΡ‚ΠΈΡ‡Π΅ΡΠΊΠΈΠ΅ Ρ‚ΠΎΡ‡ΠΊΠΈ:

  • ΠΏΡ€ΠΈ сосрСдоточСнных Π½Π°Π³Ρ€ΡƒΠ·ΠΊΠ°Ρ…
  • ΠΏΡ€ΠΈ Ρ€Π΅Π°ΠΊΡ†ΠΈΠΈ
  • Π² Ρ‚ΠΎΡ‡ΠΊΠ°Ρ…, Π³Π΄Π΅ Π²Π΅Π»ΠΈΡ‡ΠΈΠ½Π° распрСдСлСнной Π½Π°Π³Ρ€ΡƒΠ·ΠΊΠΈ измСнСния.
  • a Рассмотрим Ρ€Π°Π·Ρ€Π΅Π· Π±Π°Π»ΠΊΠΈ слСва ΠΎΡ‚ D, ΠΈ Π½Π°ΠΉΡ‚ΠΈ Π°Π»Π³Π΅Π±Ρ€Π°ΠΈΡ‡Π΅ΡΠΊΡƒΡŽ сумму всСх Π²Π΅Ρ€Ρ‚ΠΈΠΊΠ°Π»ΡŒΠ½Ρ‹Ρ… сил для слСва ΠΎΡ‚ этого Ρ€Π°Π·Π΄Π΅Π»Π°.S F y = 0:. попСрСчная сила слСва ΠΎΡ‚ D Ρ€Π°Π²Π½Π° Π½ΡƒΠ»ΡŽ
  • b Рассмотрим участок справа ΠΎΡ‚ D, алгСбраичСский сумма сил слСва ΠΎΡ‚ этой сСкции составляСт 10 кН Π΄ΠΎ Π»Π΅Π²Ρ‹ΠΉ. Π‘Π»Π΅Π΄ΠΎΠ²Π°Ρ‚Π΅Π»ΡŒΠ½ΠΎ, ΠΏΠΎΠΏΠ΅Ρ€Π΅Ρ‡Π½ΠΎΠ΅ усилиС справа ΠΎΡ‚ D составляСт 10 кН (ΠΎΡ‚Ρ€ΠΈΡ†Π°Ρ‚Π΅Π»ΡŒΠ½ΠΎΠ΅)
  • c Π’ΠΎΡ‚ ΠΆΠ΅ Ρ€Π΅Π·ΡƒΠ»ΡŒΡ‚Π°Ρ‚, Ρ‡Ρ‚ΠΎ ΠΈ Π² 2) Π²Ρ‹ΡˆΠ΅, Π±ΡƒΠ΄Π΅Ρ‚ Π½Π°ΠΉΠ΄Π΅Π½ для любого Ρ‚Π°ΠΊΠΎΠ΅ сСчСниС ΠΌΠ΅ΠΆΠ΄Ρƒ D ΠΈ E. Π”ΠΈΠ°Π³Ρ€Π°ΠΌΠΌΠ° ΠΏΠΎΠΏΠ΅Ρ€Π΅Ρ‡Π½ΠΎΠΉ силы Π’Π°ΠΊΠΈΠΌ ΠΎΠ±Ρ€Π°Π·ΠΎΠΌ, ΠΌΠ΅ΠΆΠ΄Ρƒ D ΠΈ E ΠΏΡ€ΠΎΡ…ΠΎΠ΄ΠΈΡ‚ Π³ΠΎΡ€ΠΈΠ·ΠΎΠ½Ρ‚Π°Π»ΡŒΠ½Π°Ρ линия -10 кН.
  • d Рассмотрим участок справа ΠΎΡ‚ E, алгСбраичСский Π‘ΡƒΠΌΠΌΠ° сил слСва ΠΎΡ‚ этого участка складываСтся ΠΈΠ· P ΠΈ RE, попСрСчная сила Ρ€Π°Π²Π½Π° (-10 + 40) кН = + 30 кН, Ρ‚ΠΎ Π΅ΡΡ‚ΡŒ слСва ΠΎΡ‚ Ρ€Π°Π·Π΄Π΅Π»Π°. Π’Π°ΠΊΠΈΠΌ ΠΎΠ±Ρ€Π°Π·ΠΎΠΌ, ΠΏΡ€ΠΈ силС сдвига E Π΄ΠΈΠ°Π³Ρ€Π°ΠΌΠΌΠ° измСняСтся ΠΎΡ‚ -10кН Π΄ΠΎ + 30кН.
  • По ΠΌΠ΅Ρ€Π΅ приблиТСния ΠΊ ΠΏΡ€Π°Π²ΠΎΠΌΡƒ ΠΊΠΎΠ½Ρ†Ρƒ Π±Π°Π»ΠΊΠΈ Π½Π°Ρ…ΠΎΠ΄ΠΈΠΌ ΠΌΠ°Ρ‚Π΅ΠΌΠ°Ρ‚ΠΈΠΊΡƒ Π»Π΅Π³Ρ‡Π΅ ΡΡ‡ΠΈΡ‚Π°Ρ‚ΡŒ ΠΏΡ€Π°Π²ΠΎΠΉ Ρ‡Π°ΡΡ‚ΡŒΡŽ любого Ρ€Π°Π·Π΄Π΅Π».
  • e Π‘Π΅Ρ‡Π΅Π½ΠΈΠ΅ слСва ΠΎΡ‚ F. ΠΏΠΎΠΏΠ΅Ρ€Π΅Ρ‡Π½ΠΎΠ΅ усилиС = (4 кН / ΠΌ x 10 ΠΌ) — (30 кН) с использованиСм Π·Π½Π°ΠΊΠΎΠ²ΠΎΠ³ΠΎ соглашСния для опрСдСлСния ΠΏΠΎΠ»ΠΎΠΆΠΈΡ‚Π΅Π»ΡŒΠ½ΠΎΠ³ΠΎ ΠΈΠ»ΠΈ ΠΎΡ‚Ρ€ΠΈΡ†Π°Ρ‚Π΅Π»ΡŒΠ½Ρ‹ΠΉ. Π‘Π΄Π²ΠΈΠ³Π°ΡŽΡ‰Π°Ρ сила здСсь составляСт + 40 — 30 = + 10 кН.
  • f Π‘Π΅Ρ‡Π΅Π½ΠΈΠ΅ справа ΠΎΡ‚ F. ΠΏΠΎΠΏΠ΅Ρ€Π΅Ρ‡Π½ΠΎΠ΅ усилиС = + 40-30 = + 10 кН (Ρ‚.Π΅. Π±Π΅Π· Ρ€Π΅Π·ΠΊΠΈΡ… ΠΈΠ·ΠΌΠ΅Π½Π΅Π½ΠΈΠΉ ΠΏΡ€ΠΈ F). g Π Π°Π·Π΄Π΅Π» Ρ‚ΠΎΠ»ΡŒΠΊΠΎ для слСва ΠΎΡ‚ G. попСрСчная сила = -30 кН Ρ‡ ИзмСнСниС сдвига ΠΏΠΎΠ΄ a u.d.l. Π΄ΠΎΠ»ΠΆΠ΅Π½ Π±Ρ‹Ρ‚ΡŒ Π»ΠΈΠ½Π΅ΠΉΠ½Ρ‹ΠΌ.

ΠŸΡ€ΠΈΠΌΠ΅Ρ€ 6.1

ΠžΠ±Ρ€Π°Ρ‚ΠΈΡ‚Π΅ Π²Π½ΠΈΠΌΠ°Π½ΠΈΠ΅ Π½Π° ΡΠ»Π΅Π΄ΡƒΡŽΡ‰Π΅Π΅ Π½Π° Π΄ΠΈΠ°Π³Ρ€Π°ΠΌΠΌΠ΅ ΠΏΠΎΠΏΠ΅Ρ€Π΅Ρ‡Π½ΠΎΠ³ΠΎ усилия:

  • МаксимальноС усилиС сдвига Π²ΠΎΠ·Π½ΠΈΠΊΠ°Π΅Ρ‚ Π² Ρ‚ΠΎΡ‡ΠΊΠ°Ρ… E ΠΈ G, Π³Π΄Π΅ значСния ΡΠΎΡΡ‚Π°Π²Π»ΡΡŽΡ‚ + 30кН ΠΈ 30кН соотвСтствСнно. Π­Ρ‚ΠΈ Π΄Π²Π° ΠΏΠΎΠΏΠ΅Ρ€Π΅Ρ‡Π½Ρ‹Ρ… Ρ€Π°Π·Π΄Π΅Π»Ρ‹ — Π΄Π²Π΅ Π½Π°ΠΈΠ±ΠΎΠ»Π΅Π΅ вСроятныС Ρ‚ΠΎΡ‡ΠΊΠΈ ΠΎΡ‚ΠΊΠ°Π·Π° Π² сдвиг.
  • ΠœΠ°ΠΊΡΠΈΠΌΠ°Π»ΡŒΠ½Ρ‹ΠΉ ΠΈΠ·Π³ΠΈΠ±Π°ΡŽΡ‰ΠΈΠΉ ΠΌΠΎΠΌΠ΅Π½Ρ‚ Π²ΠΎΠ·Π½ΠΈΠΊΠ°Π΅Ρ‚ Ρ‚Π°ΠΌ, Π³Π΄Π΅ сдвиг сила Ρ€Π°Π²Π½Π° Π½ΡƒΠ»ΡŽ ΠΈΠ»ΠΈ Π³Π΄Π΅ сила сдвига мСняСт Π·Π½Π°ΠΊ. Однако ΡƒΡ‡Ρ‚ΠΈΡ‚Π΅, Ρ‡Ρ‚ΠΎ ΠΊΠΎΠ½ΡΠΎΠ»ΡŒΠ½Ρ‹Π΅ Π±Π°Π»ΠΊΠΈ всСгда Π±ΡƒΠ΄ΡƒΡ‚ ΠΈΠΌΠ΅Ρ‚ΡŒ ΠΌΠ°ΠΊΡΠΈΠΌΠ°Π»ΡŒΠ½Ρ‹ΠΉ ΠΈΠ·Π³ΠΈΠ± Π½Π° фиксированном ΠΊΠΎΠ½Ρ†Π΅.

SFD Π² ΠΏΡ€ΠΈΠ²Π΅Π΄Π΅Π½Π½ΠΎΠΌ Π²Ρ‹ΡˆΠ΅ ΠΏΡ€ΠΈΠΌΠ΅Ρ€Π΅ ΠΈΠΌΠ΅Π΅Ρ‚ Π΄Π²Π΅ Ρ‚ΠΎΡ‡ΠΊΠΈ, Π³Π΄Π΅ сдвиг сила Ρ€Π°Π²Π½Π° Π½ΡƒΠ»ΡŽ. Один находится Π² E, Π° Π΄Ρ€ΡƒΠ³ΠΎΠΉ — H ΠΌΠ΅ΠΆΠ΄Ρƒ ΠΈ G. ΠΏΠΎΠ»ΠΎΠΆΠ΅Π½ΠΈΠ΅ H ΠΌΠΎΠΆΠ½ΠΎ Π²Ρ‹Ρ‡ΠΈΡΠ»ΠΈΡ‚ΡŒ ΠΈΠ· Ρ‚ΠΎΠ³ΠΎ Ρ„Π°ΠΊΡ‚Π°, Ρ‡Ρ‚ΠΎ ΠΏΡ€ΠΈ F сдвиг усилиС 10 кН ΠΈ ΠΏΠΎΠ΄ дСйствиСм Ρƒ.Π΄. 1. справа ΠΎΡ‚ F сниТаСтся со ΡΠΊΠΎΡ€ΠΎΡΡ‚ΡŒΡŽ 4 кН / ΠΌ. Он Π±ΡƒΠ΄Π΅Ρ‚ Ρ‡ΠΈΡ‚Π°Ρ‚ΡŒ Π·Π½Π°Ρ‡Π΅Π½ΠΈΠ΅ ноль Ρ‡Π΅Ρ€Π΅Π· 2,5 ΠΌ, Ρ‚.Π΅. Ρ‚ΠΎΡ‡ΠΊΠ° H находится Π½Π° 2,5 ΠΌ ΠΏΡ€Π°Π²Π΅Π΅ F.

4 ΠŸΠΎΡΡ‚Ρ€ΠΎΠΉΡ‚Π΅ Π΄ΠΈΠ°Π³Ρ€Π°ΠΌΠΌΡƒ ΠΈΠ·Π³ΠΈΠ±Π°ΡŽΡ‰Π΅Π³ΠΎ ΠΌΠΎΠΌΠ΅Π½Ρ‚Π° (BMD) нСпосрСдствСнно ΠΏΠΎΠ΄ SFD. ΠΈ Π²Ρ‹Π±Π΅Ρ€ΠΈΡ‚Π΅ ΡƒΠ΄ΠΎΠ±Π½Ρ‹ΠΉ ΠΌΠ°ΡΡˆΡ‚Π°Π± для прСдставлСния ΠΈΠ·Π³ΠΈΠ±Π°ΡŽΡ‰Π΅Π³ΠΎ ΠΌΠΎΠΌΠ΅Π½Ρ‚Π°.РассчитайтС значСния ΠΈΠ·Π³ΠΈΠ±Π°ΡŽΡ‰Π΅Π³ΠΎ ΠΌΠΎΠΌΠ΅Π½Ρ‚Π° Π²ΠΎ всСх критичСских Ρ‚ΠΎΡ‡ΠΊΠ°Ρ…. ΠšΡ€ΠΈΡ‚ΠΈΡ‡Π΅ΡΠΊΠΈΠ΅ Ρ‚ΠΎΡ‡ΠΊΠΈ для ΠΈΠ·Π³ΠΈΠ±Π°ΡŽΡ‰Π΅Π³ΠΎ ΠΌΠΎΠΌΠ΅Π½Ρ‚Π°:

  • ΠΊΠΎΠ½Ρ†Ρ‹ Π±Π°Π»ΠΊΠΈ
  • , Π³Π΄Π΅ сила сдвига Ρ€Π°Π²Π½Π° Π½ΡƒΠ»ΡŽ ΠΈΠ»ΠΈ мСняСт Π·Π½Π°ΠΊ
  • Π΄Ρ€ΡƒΠ³ΠΈΡ… Ρ‚ΠΎΡ‡Π΅ΠΊ, ΠΊΠΎΡ‚ΠΎΡ€Ρ‹Π΅ ΠΏΠΎ ΠΎΠΏΡ‹Ρ‚Ρƒ извСстны критичСский.

ЗначСния ΠΈΠ·Π³ΠΈΠ±Π°ΡŽΡ‰Π΅Π³ΠΎ ΠΌΠΎΠΌΠ΅Π½Ρ‚Π° Ρ€Π°ΡΡΡ‡ΠΈΡ‚Ρ‹Π²Π°ΡŽΡ‚ΡΡ с использованиСм опрСдСлСния ΠΈ ΠΏΠΎΠ΄ΠΏΠΈΡΠ°Ρ‚ΡŒ соглашСниС, учитывая ΠΊΠ°ΠΆΠ΄ΡƒΡŽ Π½Π°Π³Ρ€ΡƒΠ·ΠΊΡƒ (с ΠΎΠ΄Π½ΠΎΠΉ стороны Ρ‚ΠΎΡ‡ΠΊΠ°) ΠΎΡ‚Π΄Π΅Π»ΡŒΠ½ΠΎ.Π­Ρ‚ΠΎ эффСкт, ΠΊΠΎΡ‚ΠΎΡ€Ρ‹ΠΉ ΠΎΠ΄Π½Π° Π½Π°Π³Ρ€ΡƒΠ·ΠΊΠ° ΠΎΠΊΠ°Π·Π°Π»Π° Π±Ρ‹ Π½Π° изогнутая Ρ„ΠΎΡ€ΠΌΠ° Π² Π²Ρ‹Π±Ρ€Π°Π½Π½ΠΎΠΉ Ρ‚ΠΎΡ‡ΠΊΠ΅, ΠΎΠΏΡ€Π΅Π΄Π΅Π»ΡΡŽΡ‰Π°Ρ Π·Π½Π°ΠΊ.

a Для B.M. Π² D рассмотрим Π»Π΅Π²ΡƒΡŽ Ρ‡Π°ΡΡ‚ΡŒ этой Ρ‚ΠΎΡ‡ΠΊΠΈ M p = 0

b Для B. M. Π² E рассмотрим Π»Π΅Π²ΡƒΡŽ Ρ‡Π°ΡΡ‚ΡŒ этой Ρ‚ΠΎΡ‡ΠΊΠΈ M E = P x a, ΠΈ Π±Π°Π»ΠΊΠ° ΠΏΡ€ΠΈΠΌΠ΅Ρ‚ Ρ„ΠΎΡ€ΠΌΡƒ ΠΈΠ·Π»ΠΎΠΌΠ°;

M F = — (10 x 10) = -100 кНм

c Для B. M. ΠΏΡ€ΠΈ F ΡƒΡ‡ΠΈΡ‚Ρ‹Π²Π°Ρ‚ΡŒ Π½Π°Π³Ρ€ΡƒΠ·ΠΊΠΈ справа ΠΎΡ‚ Ρ‚ΠΎΡ‡ΠΊΠΈ, провисаниС Π±Π°Π»ΠΎΡ‡Π½Ρ‹Π΅ Ρ€Π΅Π·ΡƒΠ»ΡŒΡ‚Π°Ρ‚Ρ‹ ΠΈ:

M F = — (4 x 10 x 10/2) + (30 x 10) = 100 кНм

Π΄ Π‘.М. Ρƒ G явно ноль

e Π’ Ρ‚ΠΎΡ‡ΠΊΠ΅ H ΠΌΡ‹ ΠΈΠΌΠ΅Π΅ΠΌ ΠΌΠ°ΠΊΡΠΈΠΌΠ°Π»ΡŒΠ½Ρ‹ΠΉ ΠΈΠ·Π³ΠΈΠ±Π°ΡŽΡ‰ΠΈΠΉ ΠΌΠΎΠΌΠ΅Π½Ρ‚: учитывая силы справа ΠΎΡ‚ этой Ρ‚ΠΎΡ‡ΠΊΠΈ Π΄Π°Π΅Ρ‚

MH = — (4 x 7 512 x 7 5) + (30 x 7 5) = 112,5 (провисаниС)

f ИзмСнСниС ΠΈΠ·Π³ΠΈΠ±Π°ΡŽΡ‰Π΅Π³ΠΎ ΠΌΠΎΠΌΠ΅Π½Ρ‚Π° ΠΏΠΎΠ΄ дСйствиСм u.d.l. параболичСский

Π³ Если Π²ΠΊΠ»ΡŽΡ‡Π΅Π½ΠΈΠ΅ Π΄Ρ€ΡƒΠ³ΠΈΡ… Ρ‚ΠΎΡ‡Π΅ΠΊ Π±Ρ‹Π»ΠΎ Π±Ρ‹ ΠΏΠΎΠ»Π΅Π·Π½ΠΎ ΠΏΡ€ΠΈ рисовании ΠΊΡ€ΠΈΠ²ΡƒΡŽ, ΠΈΡ… Ρ‚Π°ΠΊΠΆΠ΅ слСдуСт нанСсти Π½Π° Π³Ρ€Π°Ρ„ΠΈΠΊ.

ΠžΠ±Ρ€Π°Ρ‚ΠΈΡ‚Π΅ Π²Π½ΠΈΠΌΠ°Π½ΠΈΠ΅ Π½Π° ΡΠ»Π΅Π΄ΡƒΡŽΡ‰Π΅Π΅ Π½Π° Π΄ΠΈΠ°Π³Ρ€Π°ΠΌΠΌΠ΅ ΠΈΠ·Π³ΠΈΠ±Π°ΡŽΡ‰Π΅Π³ΠΎ ΠΌΠΎΠΌΠ΅Π½Ρ‚Π°:

  • Π’ΠΎΠ·Π½ΠΈΠΊΠ°Π΅Ρ‚ ΠΌΠ°ΠΊΡΠΈΠΌΠ°Π»ΡŒΠ½Ρ‹ΠΉ ΠΎΡ‚Ρ€ΠΈΡ†Π°Ρ‚Π΅Π»ΡŒΠ½Ρ‹ΠΉ ΠΈΠ·Π³ΠΈΠ±Π°ΡŽΡ‰ΠΈΠΉ ΠΌΠΎΠΌΠ΅Π½Ρ‚ (100 кНм) ΠΏΡ€ΠΈ E ΠΈ максимальном ΠΏΠΎΠ»ΠΎΠΆΠΈΡ‚Π΅Π»ΡŒΠ½ΠΎΠΌ ΠΏΡ€ΠΎΠ³ΠΈΠ±Π΅ ΠΈΠ·Π³ΠΈΠ±Π°ΡŽΡ‰Π΅Π³ΠΎ ΠΌΠΎΠΌΠ΅Π½Ρ‚Π° ( 112.5 кНм) Π²ΠΎΠ·Π½ΠΈΠΊΠ°Π΅Ρ‚ Π² Ρ‚ΠΎΡ‡ΠΊΠ΅ ΠΌΠ΅ΠΆΠ΄Ρƒ F ΠΈ G. ΠΏΡ€ΠΎΠ΅ΠΊΡ‚ΠΈΡ€ΠΎΠ²Π°Π½ΠΈΠ΅ Π±Π°Π»ΠΎΠΊ ΠΈΠ· Ρ‚Π°ΠΊΠΈΡ… ΠΌΠ°Ρ‚Π΅Ρ€ΠΈΠ°Π»ΠΎΠ², ΠΊΠ°ΠΊ Π±Π΅Ρ‚ΠΎΠ½, ΡΡ‚Π°Π»ΡŒ Π°Ρ€ΠΌΠ°Ρ‚ΡƒΡ€Π° Π΄ΠΎΠ»ΠΆΠ½Π° Π±Ρ‹Ρ‚ΡŒ Ρ€Π°Π·ΠΌΠ΅Ρ‰Π΅Π½Π° Π² соотвСтствии с этими ΠΌΠΎΠΌΠ΅Π½Ρ‚Ρ‹.

Рисунок

  • BMD Ρ‚Π°ΠΊΠΆΠ΅ ΠΏΠΎΠΊΠ°ΠΆΠ΅Ρ‚, ΠΊΠ°ΠΊ Π·Π°Π³Ρ€ΡƒΠΆΠ΅Π½ Π»ΡƒΡ‡ отклонится. ΠŸΠΎΠ»ΠΎΠΆΠΈΡ‚Π΅Π»ΡŒΠ½Ρ‹Π΅ ΠΈΠ·Π³ΠΈΠ±Π°ΡŽΡ‰ΠΈΠ΅ ΠΌΠΎΠΌΠ΅Π½Ρ‚Ρ‹ (провисаниС) Π²Ρ‹Π·Ρ‹Π²Π°ΡŽΡ‚ сТатиС Π² Π²Π΅Ρ€Ρ…Π½ΠΈΡ… Π²ΠΎΠ»ΠΎΠΊΠ½Π°Ρ… Π±Π°Π»ΠΊΠΈ, ΡΠ»Π΅Π΄ΠΎΠ²Π°Ρ‚Π΅Π»ΡŒΠ½ΠΎ ΠΎΠ½ΠΈ склонны Π³Π½ΡƒΡ‚ΡŒ Π±Π°Π»ΠΊΡƒ Π²ΠΎΠ³Π½ΡƒΡ‚ΠΎΠΉ стороной Π²Π½ΠΈΠ·.
  • На ΠΎΠΏΠΎΡ€Π½Ρ‹Ρ… ΠΊΠΎΠ½Ρ†Π°Ρ… простой Π±Π°Π»ΠΊΠΈ ΠΈ Π½Π° свободных ΠΊΠΎΠ½Π΅Ρ† консольной Π±Π°Π»ΠΊΠΈ, Π³Π΄Π΅ Π½Π΅ ΠΌΠΎΠΆΠ΅Ρ‚ Π±Ρ‹Ρ‚ΡŒ сопротивлСниС ΠΈΠ·Π³ΠΈΠ±Ρƒ, ΠΈΠ·Π³ΠΈΠ±Π°ΡŽΡ‰ΠΈΠΉ ΠΌΠΎΠΌΠ΅Π½Ρ‚ всСгда Ρ€Π°Π²Π΅Π½ Π½ΡƒΠ»ΡŽ.

УсилиС Π² Ρ€Π°ΠΌΠ°Ρ… с ΡˆΠ°Ρ€Π½ΠΈΡ€Π½Ρ‹ΠΌ соСдинСниСм

ΠŸΡ€ΠΈ ΠΏΡ€ΠΎΠ΅ΠΊΡ‚ΠΈΡ€ΠΎΠ²Π°Π½ΠΈΠΈ каркаса Π½Π΅ΠΎΠ±Ρ…ΠΎΠ΄ΠΈΠΌΠΎ Π½Π°ΠΉΡ‚ΠΈ силы Π² участники. Для расчСта ΠΏΠ΅Ρ€Π²ΠΈΡ‡Π½Ρ‹Ρ… напряТСний ΠΊΠ°ΠΆΠ΄Ρ‹ΠΉ элСмСнт считаСтся соСдинСнным ΡˆΡ‚ΠΈΡ„Ρ‚ΠΎΠΌ Π½Π° ΠΊΠ°ΠΆΠ΄ΠΎΠΌ ΠΊΠΎΠ½Ρ†Π΅, Ρ‚Π°ΠΊ Ρ‡Ρ‚ΠΎ ΠΎΠ½ ΠΌΠΎΠΆΠ΅Ρ‚ ΠΏΠ΅Ρ€Π΅Π΄Π°Π²Π°Ρ‚ΡŒ осСвоС усилиС Ρ‚ΠΎΠ»ΡŒΠΊΠΎ Π² Π½Π°ΠΏΡ€Π°Π²Π»Π΅Π½ΠΈΠΈ Π»ΠΈΠ½ΠΈΠΈ соСдиняя ΡˆΡ‚Ρ‹Ρ€Π΅Π²Ρ‹Π΅ соСдинСния Π½Π° ΠΊΠ°ΠΆΠ΄ΠΎΠΌ ΠΊΠΎΠ½Ρ†Π΅.Π‘ΠΈΠ»Π° ΠΌΠΎΠΆΠ΅Ρ‚ Π±Ρ‹Ρ‚ΡŒ чистой напряТСниС (условно ΠΎΠ±ΠΎΠ·Π½Π°Ρ‡Π΅Π½Π½ΠΎΠ΅ ΠΊΠ°ΠΊ ΠΏΠΎΠ»ΠΎΠΆΠΈΡ‚Π΅Π»ΡŒΠ½ΠΎΠ΅), ΠΈ Π² этом случаС Ρ‡Π»Π΅Π½ называСтся стяТкой ΠΈΠ»ΠΈ чистым сТатиСм (условно обозначаСтся ΠΎΡ‚Ρ€ΠΈΡ†Π°Ρ‚Π΅Π»ΡŒΠ½Ρ‹ΠΌ), ΠΊΠΎΠ³Π΄Π° элСмСнт называСтся распоркой.

Π­Ρ‚ΠΎ Π²Π½ΡƒΡ‚Ρ€Π΅Π½Π½ΠΈΠ΅ силы, ΠΊΠΎΡ‚ΠΎΡ€Ρ‹Π΅ Π΄ΠΎΠ»ΠΆΠ½Ρ‹ Π±Ρ‹Ρ‚ΡŒ Π² равновСсии с внСшниС ΠΏΡ€ΠΈΠ»ΠΎΠΆΠ΅Π½Π½Ρ‹Π΅ силы.

Для опрСдСлСния сил Π² стСрТнях ΠΌΠΎΠΆΠ½ΠΎ ΠΈΡΠΏΠΎΠ»ΡŒΠ·ΠΎΠ²Π°Ρ‚ΡŒ ряд Ρ€Π°Π·Π½Ρ‹Π΅ Ρ‚Π΅Ρ…Π½ΠΈΠΊΠΈ.

БовмСстный Π°Π½Π°Π»ΠΈΠ·: ΠΎΠ½ основан Π½Π° рассмотрСнии равновСсия ΠΊΠ°ΠΆΠ΄ΠΎΠ³ΠΎ сустава ΠΏΠΎ ΠΎΡ‡Π΅Ρ€Π΅Π΄ΠΈ ΠΈ ΠΈΡΠΏΠΎΠ»ΡŒΠ·ΡƒΡ Π΄ΠΈΠ°Π³Ρ€Π°ΠΌΠΌΡƒ свободного Ρ‚Π΅Π»Π° для ΠΊΠ°ΠΆΠ΄ΠΎΠ³ΠΎ соСдинСниС.

ΠœΠ΅Ρ‚ΠΎΠ΄ сСчСния: РассматриваСмая Π΄ΠΈΠ°Π³Ρ€Π°ΠΌΠΌΠ° свободного Ρ‚Π΅Π»Π° ΠΏΡ€Π΅Π΄Π½Π°Π·Π½Π°Ρ‡Π΅Π½Π° для Ρ‡Π°ΡΡ‚ΡŒ каркаса Π² Ρ‚Ρƒ ΠΈΠ»ΠΈ ΠΈΠ½ΡƒΡŽ сторону ΠΎΡ‚ Ρ€Π°Π·Ρ€Π΅Π·Π° Ρ€Π°Π·Π΄Π΅Π». Π‘ΠΈΠ»Ρ‹ Π² элСмСнтах, Ρ€Π°Π·Ρ€Π΅Π·Π°Π΅ΠΌΡ‹Ρ… сСкциСй, Ρ€Π°Π²Π½Ρ‹ Π²ΠΊΠ»ΡŽΡ‡Π΅Π½Ρ‹ Π² Π΄ΠΈΠ°Π³Ρ€Π°ΠΌΠΌΡƒ свободного Ρ‚Π΅Π»Π°. ΠŸΡ€ΠΈΠΌΠ΅Π½Π΅Π½ΠΈΠ΅ ΡƒΡ€Π°Π²Π½Π΅Π½ΠΈΠΉ равновСсия Ρ€Π΅ΡˆΠΈΡ‚ нСизвСстныС силы Π² Ρ€Π°Π·Ρ€Π΅Π·Π΅. Π­Ρ‚ΠΎ Π΄Π°Π΅Ρ‚ аналитичСскоС Ρ€Π΅ΡˆΠ΅Π½ΠΈΠ΅ ΠΈ Π½Π°ΠΈΠ±ΠΎΠ»Π΅Π΅ ΠΏΠΎΠ»Π΅Π·Π½ΠΎ, ΠΊΠΎΠ³Π΄Π° трСбуя ΠΎΡ‚Π²Π΅Ρ‚ΠΎΠ² Ρ‚ΠΎΠ»ΡŒΠΊΠΎ для ΠΎΠ΄Π½ΠΎΠ³ΠΎ ΠΈΠ»ΠΈ Π΄Π²ΡƒΡ… участников.

ΠŸΡ€ΠΈΠΌΠ΅Ρ€ 7

НайдитС силы ΠΈ ΠΈΡ… Π½Π°ΠΏΡ€Π°Π²Π»Π΅Π½ΠΈΠ΅ Π² Ρ‡Π»Π΅Π½Π°Ρ… BH ΠΈ HG. ΠΌΠ΅Ρ‚ΠΎΠ΄ΠΎΠΌ сСкций.

FHG ΠΌΠΎΠΆΠ½ΠΎ Π½Π°ΠΉΡ‚ΠΈ, ΠΎΡΡ‚Π°Π½ΠΎΠ²ΠΈΠ²ΡˆΠΈΡΡŒ Π½Π° Ρ‚ΠΎΡ‡ΠΊΠ΅ C, учитывая правая Ρ‡Π°ΡΡ‚ΡŒ Ρ€Π°Π·Ρ€Π΅Π·Π° 1-1 находится Π² равновСсии. Π’ Π‘ΠΈΠ»Ρ‹ FHC ΠΈ FBC Π½Π΅ ΠΈΠΌΠ΅ΡŽΡ‚ Π½ΠΈΠΊΠ°ΠΊΠΎΠ³ΠΎ ΠΎΡ‚Π½ΠΎΡˆΠ΅Π½ΠΈΡ ΠΊ Ρ‚ΠΎΡ‡ΠΊΠ΅ CBL, ΠΏΠΎΡΠΊΠΎΠ»ΡŒΠΊΡƒ ΠΎΠ½ΠΈ ΠΏΠ΅Ρ€Π΅ΡΠ΅ΠΊΠ°ΡŽΡ‚ΡΡ ΠΈ проходят Ρ‡Π΅Ρ€Π΅Π· Ρ‚ΠΎΡ‡ΠΊΡƒ.

S M c = 0 (F HG x CG) + (9 x CD) — (R E x 20) = 0

CG = FX = 10 Π·Π°Π³Π°Ρ€ 30 = 5,774

CD = DE = FE / cos 30

FE = EX / cos 30 = 11.547ΠΌ

CD = 11 547 / cos 30 = 13 333 ΠΌ

RE = (9 + 12 + 12) / 2 = 15 кН

Π‘Π»Π΅Π΄ΠΎΠ²Π°Ρ‚Π΅Π»ΡŒΠ½ΠΎ (F HG x 5,774) + (9 x 13,333) — (15 x 20) = 0

F HG = 31,17

Π’Π·ΡΡ‚ΡŒ Ρ€Π°Π·Π΄Π΅Π» 2-2.

Π’Π°ΠΊ ΠΊΠ°ΠΊ HC = FE = 11,547 (FBH x 11547) + (9 x 13,333) — (15 x 20) = 0 FBH = 15,59 кН

Π’ΠΈΠ΄Π½ΠΎ, Ρ‡Ρ‚ΠΎ FC; H, FBH ΠΈ FBH Π΄ΠΎΠ»ΠΆΠ½Ρ‹ Π±Ρ‹Ρ‚ΡŒ ΠΏΠΎΠ²Π΅Ρ€Π½ΡƒΡ‚Ρ‹ ΠΏΠΎ часовой стрСлкС, Ρ‡Ρ‚ΠΎΠ±Ρ‹ ΠΈΠΌΠ΅ΡŽΡ‚ равновСсиС ΠΎΠΊΠΎΠ»ΠΎ Ρ‚ΠΎΡ‡ΠΊΠΈ C.